Sunteți pe pagina 1din 243

INSURANCE ABOITIZ SHIPPING CORPORATION V NEW INDIA ASSURANCE COMPANY, LTD.

[CITATION] QUISUMBING; May 2, 2006 NATURE Petition for review on certiorari FACTS - Societe Francaise Des Colloides loaded a cargo of textiles and auxiliary chemicals rom France on board a vessel owned by Franco-Belgian Services, Inc. - The cargo was consigned to General Textile, Inc., in Manila and insured by respondent New India Assurance Company, Ltd. - While in Hongkong, the cargo was transferred to M/V P. Aboitiz for transshipment to Manila. - Before departing, the vessel was advised that itwas safe to travel to its destination, but while at sea, the vessel received a report of a typhoon movingwithin its path. - To avoid the typhoon, the vessel changed itscourse. However, it was still at the fringe of thetyphoon when its hull leaked. - On October 31, 1980, the vessel sank, but the captain and his crews were saved. - On November 3, 1980, the captain of M/V P. Aboitiz filed his Marine Protest, stating that the wind force was at 10 to 15 knots at the time the shipfoundered and described the weather as moderatebreeze, small waves, becoming longer, fairly frequentwhite horse - Petitioner notified the consignee of the total loss of the vessel and all of its cargoes. - General Textile, lodged a claim with respondent for the amount of its loss. - Respondent paid General Textile and was subrogated to the rights of the latter. - Respondent hired a surveyor to investigate, and thesame concluded that the cause was the flooding ofthe holds brought about by the vessels questionableseaworthiness. - Respondent filed a complaint for damages againstpetitioner Aboitiz, Franco-Belgian Services and thelatters local agent, F.E. Zuellig, Inc. (Zuellig); - On November 20, 1989, the trial court ruled infavor of respondent andheld petitioner Aboitiz liablefor the total value of the lost cargo plus legal interest- The complaint with respect to Franco and Zuelligwas dismissed - Petitioner elevated the case to the Court ofAppeals, which in turn, affirmed in toto the trialcourts decision. - Petitioner moved for reconsideration but the same was denied. - Hence, this petition for review Petitioners Claim > Petitioner contends that respondents claim fordamages should only be against the insuranceproceeds and limited to its pro-rata share in view ofthe doctrine of limited liability Respondents Comments > Respondent counters that the doctrine of real andhypothecary nature of maritime law is not applicablein the present case because petitioner was found tohave been negligent. Hence, according torespondent, petitioner should be held liable for thetotal value of the lost cargo

ISSUE WON the limited liability doctrine applies in this case HELD NO Ratio Where the shipowner fails to overcome the presumption of negligence, the doctrine of limited liability cannot be applied. Reasoning - From the nature of their business and for reasonsof public policy, common carriers are bound toobserve extraordinary diligence over the goods theytransport according to all the circumstances of eachcase. In the event of loss, destruction ordeterioration of the insured goods, common carriersare responsible, unless they can prove that the loss,destruction or deterioration was brought about bythe causes specified in Article 17341 of the CivilCode. In all other cases, common carriers arepresumed to have been at fault or to have actednegligently, unless they prove that they observedextraordinary diligence. Moreover, where the vesselis found unseaworthy, the shipowner is also presumed to be negligent since it is tasked with themaintenance of its vessel.Though this duty can bedelegated, still, the shipowner must exercise closesupervision over its men. - In the present case, petitioner has the burden ofshowing that it exercised extraordinary diligence inthe transport of the goods it had on board in order toinvoke the limited liability doctrine.Differently put,to limit its liability to the amount of the insuranceproceeds, petitioner has the burden of proving thatthe unseaworthiness of its vessel was not due to itsfault or negligence. - Considering the evidence presented and thecircumstances obtaining in this case, we find thatpetitioner failed to discharge this burden. Both thetrial and the appellate courts, in this case, found thatthe sinking was not due to the typhoon but to itsunseaworthiness.Evidence on record showed thatthe weather was moderate when the vessel sank.These factual findings of the Court of Appeals,affirming those of the trial court are not to bedisturbed on appeal, but must be accorded greatweight.These findings are conclusive not only onthe parties but on this Court as well. DispositionPetition is denied for lack of merit.

PHILAMCARE HEALTH SYSTEMS, INC. V CA (TRINOS) 379 SCRA 357 YNARES-SANTIAGO; March 18, 2002 NATURE Petition for review of CA decision FACTS - Ernani TRINOS, deceased husband of respondent Julita, applied for a health care coverage with Philamcare Health Systems, Inc. In the standard application form, he answered no to the question: Have you or any of your family members ever consulted or been treated for high blood pressure,heart trouble, diabetes, cancer, liver disease, asthmaor peptic ulcer? (If Yes, give details). - The application was approved for period of one year; upon termination, it was extended for another 2 years. Amount of coverage was increased to a maximum sum of P75T per disability. - During this period, Ernani suffered a HEART ATTACK and was confined at the Manila Medical Center (MMC) for one month. While her husband was in the hospital, Julita tried to claim the hospitalization benefits. Petitioner treated the Health Care Agreement (HCA) as void since there was a concealment regarding Ernanis medical history. Doctors at the MMC allegedly discovered at the time of his confinement, he was hypertensive, diabetic and asthmatic. Julita then paid the hospitalization expenses herself, amounting to about P76T. - After her husband died, Julita instituted action for damages against Philamcare and its res. After trial, the lower court ruled in her favor and ordered Philamcare to reimburse medical and hospital coverage amounting to P76T plus interest, until fully paid; pay moral damages of P10T; pay exemplary damages of P10T; attys fees of P20T. - CA affirmed the decision of the trial court butd eleted all awards for damages and absolved petitioner Reverente. Petitioners Claims: (1) Agreement grants living benefits such as medical check-ups and hospitalization which a member may immediately enjoy so long as he is alive upon effectivity of the agreement until its expiration. (2) Only medical and hospitalization benefits are given under the agreement without any indemnification, unlike in an insurance contract where the insured is indemnified for his loss. (3) HCAs are only for a period of one year; therefore, incontestability clause does not apply, as it requires effectivity period of at least 2 yrs. (4) It is not an insurance company, governed by Insurance Commission, but a Health Maintenance Organization under the authority of DOH. (5) Trinos concealed a material fact in his application. (6) Julita was not the legal wife since at the timeof their marriage, the deceased was previously married to another woman who was still alive.* ISSUES 1. WON a health care agreement is an insurance contract (If so, incontestability clause under the Insurance Code is applicable) 2. WON the HCA can be invalidated on the basis of alleged concealment.

HELD YES Ratio: Every person has an insurable interest in the life and health of himself. The health care agreement was in the nature of non-life insurance, which is primarily a contract of indemnity. Once the member incurs hospital, medical or any other expense arising from sickness, injury or other stipulated contingent, the health care provider must pay for the same to the extent agreed upon under the contract. Reasoning - A contract of insurance is an agreement whereby one undertakes for a consideration to indemnify another against loss, damage or liability arising from an unknown or contingent event. - An insurance contract exists where the following elements concur: (a) The insured has an insurable interest; (b) The insured is subject to a risk of loss by the happening of the peril; (c) The insurer assumes the risk; (d) Such assumption of risk is part of a general scheme to distribute actual losses among a large group of persons bearing a similar risk; and (e) In consideration of the insurers promise, the insured pays a premium. 2. NO Ratio: Where matters of opinion or judgment are called for, answers made in good faith and without intent to deceive will not avoid a policy even though they are untrue; since in such case the insurer is not justified in relying upon such statement, but is obligated to make further inquiry. Reasoning - The fraudulent intent on the part of the insured must be established to warrant rescission of the insurance contract. The right to rescind should be exercised previous to the commencement of an action on the contract. No rescission was made. Besides, the cancellation of health care agreements as in insurance policies requires: (a) Prior notice of cancellation to insured; (b) Notice must be based on the occurrence after effective date of the policy of one or more of the grounds mentioned; (c) Must be in writing, mailed or delivered to the insured at the address shown in the policy; (d) Must state the grounds relied upon provided in Section 64 of the Insurance Code and upon requestof insured, to furnish facts on which cancellation is based. - These conditions have not been met. When the terms of insurance contract contain limitations on liability, courts should construe them in such a way as to preclude insurer from non-compliance of obligation. Being a contract of adhesion, terms of an insurance contract are to be construed strictly against the party which prepared it the insurer. - Also, Philamcare had 12 months from the date of issuance of the Agreement within which to contest the membership of the patient if he had previous ailment of asthma, and six months from the issuance of the agreement if the patient was sick of diabetes or hypertension. The health care agreement is in the nature of a contract of indemnity.Hence, payment should be made to the party who incurred the expenses.It is clear that respondent paid all the hospital and medical bills; thus, she is entitled to reimbursement. Disposition: Petition DENIED.

PINEDA V CA (INSULAR LIFE INSURANCE COMPANY) 226 SCRA 755 DAVIDE; September 27, 1993 NATURE Appeal by certiorari for review and set aside theDecision of the public respondent Court of Appealsand its Resolution denying the petitioners' motion forreconsideration FACTS - In 1983, Prime Marine Services, Inc. (PMSI)procured a group policy from Insular Life to provide life insurance coverage to its sea-based employees enrolled under the plan. During the effectivity of the policy, 6 covered employees perished at sea. Theywere survived by complainants-appellees, the beneficiaries under the policy. - Complainants-appellees sought to claim death benefits due them and approached Capt. Roberto Nuval, President and GM of PMSI, then executed Special Powers Of Attorney authorizing Capt. Nuval to follow up, ask, demand, collect and receive fortheir benefit indemnities of sums of money due them - Unknown to the complainants, PMSI filed with Insular Life claims for and in behalf of them through Capt. Nuval even using the 5 special powers of attorney that they executed as documents. Insular Life then released 6 checks, payable to the order of the complainant-appellees, to the treasurer of PMSI (who happened to be Capt. Nuvals son-in-law).Capt. Nuval then endorsed and deposited these checks (which were for the complainants) in his bank account. - 3 years after, the complainants-appellees found out that they were entitled, as beneficiaries, to life insular benefits under a group policy withrespondent-appellant so they sought to recover these benefits from Insular Life. Insular Life denied the claim, saying that their liability to complainants was already extinguished upon delivery to and receipt by PMSI of the 6 checks issued in the complainants names. Complainants filed case withthe Insurance Commission which decided in their favor. - Insurance Commission held that the special powersof attorney executed by complainants in favor of the complainants do not contain in unequivocal and clear terms authority to Capt. Nuval to obtain, receive,receipt from respondent company insurance proceeds arising from the death of the seamaninsured; also, that Insular Life did not convincingly refuted the claim of Mrs. Alarcon that neither she nor her husband executed a special power of authority infavor of Capt. Nuval (and therefore, the company should have not released the check to Capt. Nuval-PMSI); and that it did not observe Sec 180 (3), as repealed by Art. 225 of the Family Code, when it released the benefits due to the minor children of Ayo and Lontok, when the said complainants did not post a bond as required - Insular Life appealed to the CA; CA modified the decision of the Insurance Commission, eliminating the award to the Lontoks and Ayo ISSUES 1. WON Insular Life should be liable to the complainants when they relied on the special powers of attorney, which Capt. Nuval presented as documents, when they released the checks to the latter 2. WON Insular Life should be liable to the complainants when they released the check in favor of Ayo and LOntok, even if no bond was posted as required HELD 1. YES Ratio: Third persons deal with agents at their peril and are bound to inquire as to the extent of the power of the agent with whom they contract.The person dealing with an agent must also act with ordinary prudence and reasonable diligence. Obviously, if he knows or has good reason to believe that the agent is exceeding his authority, he cannot claim protection. So if the suggestions of probable limitations be of such a clear and reasonable quality, or if the character assumed by the agent is of such a suspicious or unreasonable nature, or if the authority which he seeks to exerciseis of such an unusual or improbable character, as would suffice to put an ordinarily prudent man upon his guard, the party dealing with him may not shut his eyes to the real state of the case, but should either refuse to deal with the agent at all, or should ascertain from the principal the true condition of affairs.

Reasoning - The execution by the principals of special powers of attorney, which clearly appeared to be in prepared forms and only had to be filled up with their names, residences, dates of execution, dates of acknowledgement and others, excludes any intent to grant a general power of attorney or to constitute a universal agency. Being special powers of attorney, they must be strictly construed. Insular Life knew that a power of attorney in favor of Capt. Nuval for the collection and receipt of such proceeds was a deviation from its practice with respect to group policies (that the employer-policy holder is the agent of the insurer). - The employer acts as a functionary in the collection and payment of premiums and in performing related duties. Likewise falling within the ambit of administration of a group policy is the disbursement of insurance payments by the employer to the employees. Most policies, such as the one in this case, require an employee to pay a portion of the premium, which the employer deducts from wages while the remainder is paid by the employer. This is known as a contributory plan as compared to a noncontributory plan where the premiums are solely paid by the employer. - The labor of the employees is the true source of the benefits, which are a form of additional compensation to them. - The employer is the agent of the insurer in performing the duties of administering group insurance policies. It cannot be said that the employer acts entirely for its own benefit or for the benefit of its employees in undertaking administrative functions. While a reduced premium may result if the employer relieves the insurer of these tasks, and this, of course, is advantageous to both the employer and the employees, the insurer also enjoys significant advantages from the arrangement. The reduction in the premium which results from employer-administration permits the insurer to realize a larger volume of sales, insurer to realize a larger volume of sales, and at the same time the insurer's own administrative costs are markedly reduced. - The employee has no knowledge of or control over the employer's actions in handling the policy or its administration. An agency relationship is based upon consent by one person that another shall act in his behalf and be subject to his control. It is clear from the evidence regarding procedural techniques here that the insurer-employer relationship meets this agency test with regard to the administration of the policy, whereas that between the employer and its employees fails to reflect true agency. The insurer directs the performance of the employer's administrative acts, and if these duties are not undertaken properly the insurer is in a position to exercise more constricted control over the employer's conduct. - ON GROUP INSURANCE: Group insurance is essentially a single insurance contract that provides coverage for many individuals. In its original and most common form, group insurance provides life or health insurance coverage for the employees of one employer. The coverage terms for group insurance are usually stated in a master agreement or policy that is issued by the insurer to a representative of the group or to an administrator of the insurance program, such as an employer. Although the employer may be the titular or named insured, the insurance is actually related to the life and health of the employee. Indeed, the employee is in the position of a real party to the master policy, and even in a non-contributory plan, the payment by the employer of the entire premium is a part of the total compensation paid for the services of the employee. 2. YES Ratio: Regardless of the value of the unemancipated common child's property, the father and mother ipso jure becomes the legal guardian of the child's property. However, if the market value of the property or the annual income of the child exceeds P50,000,00 a bond has to be posted by the parents concerned to guarantee the performance of the obligations of a general guardian. Reasoning - Sec 180. Insurance Code: 'In the absence of a judicial guardian, the father, or in the latter's absence or incapacity, the mother of any minor, who is an insured or a beneficiary under a contract of life, health or accident insurance, may exercise, in behalfof said minor, any right under the policy, without necessity of court authority or the giving of a bond where the interest of the minor in the particular act involved does not exceed twenty thousand pesos " - repealed by Art. 225, Family Code: "ART. 225. The father and the mother shall jointly exercise legal guardianship over the property of their unemancipated common child without the necessity of a

court appointment. In case of disagreement, the father's decision shall prevail, unless there is judicial order to the contrary. - Where the market value of the property or the annual income of the child exceeds P50,000, the parent concerned shall be required to furnish a bondin such amount as the court may determine, but not less than ten per centum (10%) of the value of the property or annual income, to guarantee the performance of the obligations prescribed for general guardians." -"market value of the property or the annual income of the child": the aggregate of the child's property or annual income; if this exceeds P50,000.00, a bond is required - There is no evidence that the share of each of the minors in the proceeds of the group policy in question is the minor's only property. Without such evidence, it would not be safe to conclude that, indeed, that is his only property. - Disposition: The instant petition is GRANTED. The Decision of 10 October 1991 and the Resolution of 19 May 1992 of the public respondent in CA-G.R. SP No.22950 are SET ASIDE and the Decision of the Insurance Commission in IC Case No. RD-058 is REINSTATED. Costs against the private respondent. SO ORDERED.

CIR v. Lincoln Phil Life - Automatic Increase Clause 379 SCRA 423 (2002) Facts: In the years prior to 1984, Lincoln issued a special kind of life insurance policy known as the "Junior Estate Builder Policy," the distinguishing feature of which is a clause providing for an automatic increase in the amount of life insurance coverage upon attainment of a certain age by the insured without the need of issuing a new policy. The clause was to take effect in the year 1984. Documentary stamp taxes due on the policy were paid to the petitioner only on the initial sum assured. Subsequently, petitioner issued deficiency documentary stamps tax assessment for the year 1984, corresponding to the amount of automatic increase of the sum assured on the policy issued by respondent. Lincoln questioned the deficiency assessments and sought their cancellation in a petition filed in the Court of Tax Appeals. CTA found no basis for the assessment. CA affirmed. Issue: Whether or not the automatic increase of the sum assured on the policy is taxable. Held: YES. CIR claims that the "automatic increase clause" in the subject insurance policy is separate and distinct from the main agreement and involves another transaction; and that, while no new policy was issued, the original policy was essentially re-issued when the additional obligation was assumed upon the effectivity of this "automatic increase clause" in 1984; hence, a deficiency assessment based on the additional insurance not covered in the main policy is in order. The SC agreed with this contention. The subject insurance policy at the time it was issued contained an "automatic increase clause." Although the clause was to take effect only in 1984, it was written into the policy at the time of its issuance. The distinctive feature of the "junior estate builder policy" called the "automatic increase clause" already formed part and parcel of the insurance contract, hence, there was no need for an execution of a separate agreement for the increase in the coverage that took effect in 1984 when the assured reached a certain age. It is clear from Section 173 of the NIRC that the payment of documentary stamp taxes is done at the time the act is done or transaction had and the tax base for the computation of documentary stamp taxes on life insurance policies under Section 183 of NIRC is the amount fixed in policy, unless the interest of a person insured is susceptible of exact pecuniary measurement. Logically, we believe that the amount fixed in the policy is the figure written on its face and whatever increases will take effect in the future by reason of the "automatic increase clause" embodied in the policy without the need of another contract. Here, although the automatic increase in the amount of life insurance coverage was to take effect later on, the date of its effectivity, as well as the amount of the increase, was already definite at the time of the issuance of the policy. Thus, the amount insured by the policy at the time of its issuance necessarily included the additional sum covered by the automatic increase clause because it was already determinable at the time the transaction was entered into and formed part of the policy. The "automatic increase clause" in the policy is in the nature of a conditional obligation under Article 1181, 8 by which the increase of the insurance coverage shall depend upon the happening of the event which constitutes the obligation. In the instant case, the additional insurance that took effect in 1984 was an obligation subject to a suspensive obligation, 9 but still a part of the insurance sold to which private respondent was liable for the payment of the documentary stamp tax.

CEBU SHIPYARD ENGINEERING WORKS, INC. V WILLIAM LINES, INC. and PRUDENTIAL GUARANTEE and ASSURANCE COMPANY, INC. [CITATION] PURISIMA; May 5, 1999 NATURE Petition for review on certiorari FACTS - Cebu Shipyard and Engineering Works, Inc.(CSEW) is engaged in the business of dry-dockingand repairing of marine vessels while the PrudentialGuarantee and Assurance, Inc. (Prudential) is in thenon-life insurance business. -William Lines, Inc. is in the shipping business. Itwas the owner of M/V Manila City, a luxurypassenger-cargo vessel, which caught fire and sankon Feb. 16, 1991. At the time of the unfortunateoccurrence sued upon, subject vessel was insuredwith Prudential for P45M for hull and machinery. TheHull Policy included an Additional Perils(INCHMAREE) Clausecovering loss of or damage tothe vessel through the negligence of, among others,ship repairmen -Petitioner CSEW was also insured by Prudential forthird party liability under a Shiprepairers LegalLiability Insurance Policy. The policy was for P10million only, under the limited liability clause, to wit:-On Feb. 5, 1991, William Lines, Inc. brought itsvessel, M/V Manila City, to the Cebu Shipyard inLapulapu City for annual dry-docking and repair. -On Feb. 6, 1991, an arrival conference was heldbetween representatives of William Lines, Inc. andCSEW to discuss the work to be undertaken on theM/V Manila City. The contracts, denominated as WorkOrders, were signed thereafter, with the followingstipulations: 10. The Contractor shall replace at its own work andat its own cost anywork or material which can beshown tobe defective and which is communicated inwriting within one (1) month of redelivery of thevessel or if the vessel was not in the ContractorsPossession, the withdrawal of the Contractorsworkmen, or at its option to pay a sum equal to thecost of such replacement at its own works. Theseconditions shall apply to any such replacements. 11. Save as provided in Clause 10, the Contractorshall not be under any liability to the Customer eitherin contract or for delict or quasi-delict or otherwiseexcept for negligence and such liability shall itself be subject to the following overriding limitations and exceptions, namely: (a) The total liability of the Contractor to theCustomer (over and above the liability to replaceunderClause 10) or of any sub-contractor shall belimited in respect of any defect or event (and a seriesof accidents arising out ofthe same defect or eventshall constitute one defect or event) to the sum ofPesos Philippine Currency One Million only. 20. The insurance on the vessel should bemaintained by the customer and/or owner of thevessel during the period the contract is in effect. -While the M/V Manila City was undergoing dry-docking and repairs within the premises of CSEW, themaster, officers and crew of M/V Manila City stayedin the vessel, using their cabins as living quarters.Other employees hired by William Lines to do repairsand maintenance work on the vessel were alsopresent during the dry-docking. - On February 16, 1991, after subject vessel wastransferred to the docking quay, it caught fire andsank, resulting to its eventual total loss. - On February 21, 1991, William Lines, Inc. filed acomplaint for damages against CSEW, alleging thatthe fire which broke out in M/V Manila City wascaused by CSEWs negligence and lack of care.

-On July 15, 1991 was filed an Amended Complaintimpleading Prudential as co-plaintiff, after the latterhad paid William Lines, Inc. the value of the hullandmachinery insurance on the M/V Manila City. As aresult of such payment Prudential was subrogatedtothe claim of P45 million, representing the value ofthe saidinsurance it paid. On June 10, 1994, the trial court a quo came out with ajudgment against CSEW: 1. To pay unto plaintiff Prudential Guarantee andAssurance, Inc., the subrogee, the amount of P45M,with interest at the legal rate until full payment ismade; the amount of P56,715,000 representing lossof income of M/V MANILA CITY, with interest at thelegal rate until full payment is made; 2. To pay unto plaintiff, William Lines, Inc. theamount of P11M as payment, in addition to what itreceived fromthe insurance company to fully coverthe injury or loss, in order to replace the M/VMANILA CITY, with interest at the legal rate until fullpayment is made; the sum of P927,039 for the lossof fuel and lub oil on board the vessel when she wascompletely gutted by fire at defendant, CebuShipyards quay, with interest at the legal rate untilfull payment is made; the sum of P3,054,677.95 as payment for the spareparts and materials used inthe M/V MANILA CITY during dry-docking withinterest at the legal rate until full payment is made;P500,000in moral damages;the amount of P10Minattorneys fees; and to pay the costs of this suit. -On September 3, 1997, the Court of Appealsaffirmed the appealed decision of the trial court,ordering CSEW to pay Prudential, the subrogee, thesum of P45 Million, with interest at the legal rateuntil full payment is made. CSEWs version: On Feb. 13, 1991, the CSEW completed thedrydocking of M/V Manila City at its grave dock. Itwas then transferred to the docking quay of CSEWwhere the remaining repair to be donewas thereplating of the top of Water Ballast Tank No. 12which was subcontractedby CSEWto JNB GeneralServices.Tank Top No. 12 was at the rear section ofthe vessel, on level with the flooring of the crewcabins located on the vessels second deck. At around 7AM of Feb. 16, 1991, the JNB workerstrimmed and cleaned the tank top framing whichinvolved minor hotworks (welding/cutting works).The said work was completed at about 10AM. TheJNB workers then proceeded to rig the steel plates,after which they had their lunch break. The riggingwas resumed at 1PM While in the process of rigging the second steelplate, the JNB workers noticed smoke coming fromthe passageway along the crew cabins. When one ofthe workers, Mr. Casas, proceeded to thepassageway to ascertain the origin of the smoke, henoticed that smoke was gathering on the ceiling ofthe passageway but did not see any fire as the crewcabins on either side of the passageway were locked.He immediately sought out the proprietor ofJNB, Mr.Buenavista, and the Safety Officer of CSEW, Mr.Aves, who sounded the fire alarm.CSEWs firebrigade immediately respondedas well as the otherfire fighting units in Metro Cebu. However, therewere no WLI representative, officer or crew to guidethe firemen inside the vessel. - Despite the combined efforts of the firemen of theLapulapu City Fire Dept., Mandaue Fire Dept.,Cordova Fire Dept. Emergency Rescue UnitFoundation, and fire brigade ofCSEW, the fire wasnot controlled until 2AM of the following day. - On the early morning of Feb. 17, 1991, gusty windsrekindled the flames on the vessel and fire againbroke out.Then the huge amounts of water pumpedinto the vessel, coupled with the strong current,caused the vesselto tilt until it capsized and sank - When M/V Manila City capsized, steel and anglebars were noticed to have been newly welded alongthe port side of the hull of the vessel, at the level ofthe crew cabins.William Lines did not previouslyapply for a permit to do hotworks on the said portionof the ship as it should have done pursuant to itswork order with CSEW. Prudentials version

> At around 7AM of Feb. 16, 1991, the Chief Mate ofM/V Manila City was inspectingthe various worksbeing done by CSEW on the vessel, when he sawthat some workers of CSEW were cropping out steelplates on Tank Top No. 12 using acetylene, oxygenand welding torch. He also observed that the rubberinsulation wire coming out of the air-conditioning unitwas already burning, prompting him to scold theworkers. > At 2:45 PM of the same day, witnesses saw smokecomingfrom Tank No. 12. The vessels reefermanreported such occurence to the Chief Mate whoimmediately assembled the crewmembers to putout the fire. When it was too hot for them to stay onboard and seeing that the fire cannot be controlled,the vessels crew were forced to withdraw fromCSEWs docking quay. - In the morning of Feb. 17, 1991, M/V Manila Citysank.As the vessel was insured with PrudentialGuarantee, William Lines filed a claim forconstructive total loss, and after a thoroughinvestigation of the surrounding circumstances of thetragedy, Prudential found the said insurance claim tobe meritorious and issued a checkin favor of WilliamLines in the amount of P45 million pesosrepresenting the total value of M/V Manila Citys hulland machinery insurance. ISSUES 1. WON CSEW had management and supervisorycontrol of the m/v manila city at the time the firebroke out 2. WON the doctrine of res ipsa loquitur applies against the crew 3. WON CSEWS expert evidence is admissible or of probative value 4. WON Prudential has the right of subrogationagainst its own insured THE CONTRACTUAL 5. WON the provisions limiting csews liability fornegligence to a maximum of p1 million arevalid HELD 1. YES - The that factual findings by the CA are conclusiveon the parties and are not reviewableby this Court.They are entitled to great weight and respect, evenfinality, especially when, as in this case, the CAaffirmed the factual findings arrived at by the trialcourt. When supported by sufficient evidence,findings of fact by the CA affirming those of the trialcourt, are not to be disturbed on appeal. Therationale behind this doctrine is that review of thefindings of fact oftheCA is not a function that theSupreme Courtnormally undertakes. - The CA and the Cebu RTC are agreed that the firewhich caused the total loss of subject M/V ManilaCity was due to the negligence of the employees andworkers of CSEW. Both courts found that the M/VManila City was under the custody and control ofpetitioner CSEW, when the ill-fated vessel caughtfire. The decisions of both the lower court and the CAset forth clearly the evidence sustaining their findingof actionable negligence on the part of CSEW.Thisfactual finding is accorded great weight and isconclusive on the parties. The court discerns no basisfor disturbing such finding firmly anchored onenough evidence. - Furthermore, in petitions for review oncertiorari,only questions of law may be put into issue.Questions of fact cannot be entertained. The findingof negligence by the CA is a question which thisCourt cannot look into as it would entail going intofactual matters on which the finding of negligencewas based. Such an approach cannot be allowed bythis Court in the absence of clear showing that thecase falls under any of the exceptions to the well-established principle. The finding by the trial court and the Court ofAppeals that M/V Manila City caught fire and sank byreason of the negligence of the workers of CSEW,when the said vessel was under the exclusivecustody and control of CSEW is accordingly upheld. 2. YES - For the doctrine of res ipsa loquitur to apply to agiven situation, the following conditions must concur:(1) the accidentwas of a kind which does notordinarily occur unless someone is negligent; and

(2)that the instrumentality or agency which caused theinjury was under the exclusive control of the personcharged with negligence. The facts and evidence on record reveal theconcurrence of said conditions in the case underscrutiny. First, the fire that occurred and consumedM/V Manila City would not have happened in theordinary course of things if reasonable care and diligence had been exercised. In other words, somenegligence must have occurred. Second, the agencycharged with negligence, as found by the trial courtand the CA and as shown by the records, is theherein petitioner, CSEW, which had control oversubject vessel when it was docked for annual repairs.So also, as found by the RTC, other responsiblecauses, including the conduct of the plaintiff, andthird persons, are sufficiently eliminated by theevidence. What is more, in the present case the trial courtfound direct evidence to prove that the workersand/or employees of CSEW were remiss in their dutyof exercising due diligence in the care of subjectvessel. The direct evidence substantiates theconclusion that CSEW was really negligent.Thus,even without applying the doctrine of res ipsaloquitur, in light of the direct evidence on record, theineluctable conclusion is that CSEW was negligentand consequently liable for damages to therespondent, William Lines, Inc. 3. NO - Petitioner maintains that the CA erred indisregarding the testimonies of the fire experts,Messrs. David Grey and Gregory Michael Southeard,who testified on the probable origin of the fire in M/VManila City. Petitioner avers that since the said fireexperts were one in their opinion that the fire did notoriginate in the area of Tank Top No. 12 where theJNB workers were doing hotworks but on the crewaccommodation cabins on the portside No. 2 deck,the RTC and the CA should have given weight to suchfinding based on the testimonies of fire experts;petitioner argues. But courts are not bound by the testimonies ofexpert witnesses. Although they may have probativevalue, reception in evidence of expert testimonies iswithin the discretion of the court, underSection 49,Rule 130 of the Revised Rules of Court. It is nevermandatory for judges to give substantial weight toexpert testimonies. If from the facts and evidence onrecord, a conclusion is readily ascertainable, there isno need for the judge to resort to expert opinionevidence. In the case under consideration, thetestimonies of the fire experts were not the onlyavailable evidence on the probable cause and originof the fire. There were witnesses who were actuallyon boardthe vessel when the fire occurred. Betweenthe testimonies of the fire experts who merely basedtheir findings and opinions on interviews and thetestimonies of those present during the fire, thelatter are of more probative value. 4. YES - Petitioner contends that Prudential is not entitled tobe subrogated to the rights of William Lines, Inc.,theorizing that (1) the fire which gutted M/V ManilaCity was an excluded risk and (2) it is a coassuredunder the Marine Hull Insurance Policy. - It is petitioners submission that the loss of M/VManila City or damage thereto is expressly excludedfrom the coverage of the insurance because thesame resulted from want of due diligence by theAssured, Owners or Managers which is not includedin the risks insured against. Again, this theory ofpetitioner is bereft of any factual or legal basis. Itproceeds from a wrong premise that the fire whichgutted subject vessel was caused by the negligenceof the employees of William Lines, Inc.To repeat,the issue of who between the parties was negligenthas already been resolved against CSEW. Upon proofof payment by Prudential to William Lines, Inc., theformer was subrogated to the right of the latter toindemnification from CSEW.As aptly ruled by theCourt of Appeals, the law on the matter is succinctand clear, to wit: - Art. 2207. If the plaintiffs property has beeninsured, and he has received indemnity from theinsurance company for the injury or loss arising outof the wrong or breach of contract complained of,the insurance company shall be subrogated to therights of the insured against the wrongdoer or theperson who has violated the contract. If the amountpaid by the insurance company does not fully coverthe injury or loss, the aggrieved party shall beentitled to recover the deficiencyfrom the personcausing the loss or injury.

- Thus, when Prudential, after due verification of themerit and validity of the insurance claim of WilliamLines, Inc., paid the latter the total amount coveredby its insurance policy, it was subrogated to the rightof the latter to recover the insured loss from theliable party, CSEW. - Petitioner theorizes further that there can be noright of subrogation as it is deemed a coassuredunder the subject insurance policy.To buttress itsstance that it is a co-assured, petitioner placedreliance on Clause 20 ofthe Work Order whichstates: The insurance on the vessel should bemaintained by the customer and/or owner of thevessel during the period the contract is in effect. - According to petitioner, under the aforecited clause,William Lines, Inc., agreed to assume the risk of lossof the vessel while under drydock or repair and tosuch extent, it is benefited and effectivelyconstituted as a co-assured under the policy. - This theory of petitioner is devoid of sustainablemerit. Clause 20 of the Work Order in question isclear in the sense that it requires William Lines tomaintain insurance on the vessel during the period ofdry-docking or repair.Concededly, such a stipulationworks to the benefit of CSEW as the shiprepairer.However, the fact that CSEW benefits from the saidstipulation does not automatically make it as a co-assured of William Lines. The intention of the partiesto make each other a co-assured under an insurancepolicy is to be gleaned principally from the insurancecontract or policy itself and not from any othercontract or agreement because the insurance policydenominates the assured and the beneficiaries of theinsurance. The hull and machinery insuranceprocured by William Lines, Inc. from Prudentialnamed only William Lines, Inc. as the assured.There was no manifestation of any intention ofWilliam Lines, Inc. to constitute CSEW as a co-assured under subject policy.It is axiomatic thatwhen the terms of a contract are clear its stipulationscontrol.i] Thus, when the insurance policy involvednamed only William Lines, Inc. as the assuredthereunder, the claim of CSEW that it is a co-assuredis unfounded. - Then too, in the Additional Perils Clause of the same Marine Insurance Policy, it is provided that: Subject to the conditions of this Policy, thisinsurance also covers loss of or damage to vesseldirectly caused by the following: Negligence of Charterers and/or Repairers, provided such Charterers and/or Repairers are not an Assured hereunder. -As correctly pointed out by respondent Prudential,if CSEW were deemed a co-assured under the policy,it would nullify any claim of William Lines, Inc. fromPrudential for any loss or damage caused by thenegligence of CSEW. Certainly, no shipowner wouldagree to make a shiprepairer a co-assured undersuch insurance policy; otherwise, any claim for lossor damage under the policy would be invalidated.Such result could not have been intended by WilliamLines, Inc. 5. NO - Although in this jurisdiction, contracts of adhesion have been consistently upheld as valid per se; as binding as an ordinary contract, the Court recognizesinstances when reliance on such contracts cannot befavored especially where the facts and circumstanceswarrant that subject stipulations be disregarded.Thus, in ruling on the validity and applicability of thestipulation limiting the liability of CSEW fornegligence to P1M only, the facts and circumstancesvis-a-vis the nature of the provision sought to beenforced should be considered, bearing in mind theprinciples of equity and fair play. - It is worthy to note that M/V Manila City wasinsured with Prudential for P45M.To determine thevalidity and sustainability of the claim of WilliamLines, Inc., for a total loss, Prudential conducted itsown inquiry. Upon thorough investigation by its hullsurveyor, M/V Manila City was found to be beyondeconomical salvage and repair. The evaluation of theaverage adjuster also reported a constructive totalloss. The said claim of William Lines, Inc., was thenfound to be valid and compensable such thatPrudential paid the latter the total value of itsinsurance claim. Furthermore, it was ascertained thatthe replacement cost of the vessel (the price of avessel similar to M/V Manila City), amounts to P55M.- Considering the aforestated circumstances, letalone the fact that negligence on the part ofpetitioner has been sufficiently proven,it wouldindeed be unfair and inequitable to limit

the liabilityof petitioner to One Million Pesos only. As aptly heldby the trial court, it is rather unconscionable if notoverstrained. To allow CSEW to limit its liability toP1M notwithstanding the fact that the total losssuffered by the assured and paid for by Prudentialamounted to P45M would sanction the exercise of adegree of diligence short of what is ordinarilyrequired because, then, it would not be difficult forpetitioner to escape liability by the simple expedientof paying an amount very much lower than theactual damage or loss suffered by William Lines, Inc. DispositionPetition is DENIED. Resolution of the CA is AFFIRMED.

NEW LIFE ENTERPRISES V CA 207 SCRA 669 REGALADO; March 31, 1992 NATURE Appeal by certiorari FACTS - The antecedents of this case show that Julian Sy and Jose Sy Bang have formed a business partnership in the City of Lucena. Under the businessname of New Life Enterprises, the partnershipengaged in the sale of construction materials at itsplace of business, a two storey building situated atIyam, Lucena City. The facts show that Julian Syinsured the stocks in trade of New Life Enterpriseswith Western Guaranty Corporation, Reliance Suretyand Insurance Co. Inc., and Equitable InsuranceCorporation. - On May 15, 1981, Western Guaranty Corporationissued Fire Insurance Policy No. 37201 in the amountof P350,000.00. This policy was renewed on May 13,1982. -On July 30, 1981, Reliance Surety and InsuranceCo., Inc. issued Fire Insurance Policy No. 69135 inthe amount of P300,000.00 (Renewed under RenewalCertificate No. 41997). An additional insurance wasissued by the same company on November 12, 1981under Fire Insurance Policy No. 71547 in the amountof P700,000.00. - On February 8, 1982, Equitable InsuranceCorporation issued Fire Insurance Policy No. 39328 inthe amount of P200,000.00. - Thus when the building occupied by the New LifeEnterprises was gutted by fire at about 2:00 o'clockin the morning of October 19, 1982, the stocks intrade inside said building were insured against fire inthe total amount of P1,550,000.00. According to thecertification issued by the Headquarters, PhilippineConstabulary/Integrated National Police, CampCrame, the cause of fire was electrical in nature.According to the plaintiffs, the building and thestocks inside were burned. After the fire, Julian Sywent to the agent of Reliance Insurance whom heasked to accompany him to the office of thecompany so that he can file his claim. He averredthat in support of his claim, he submitted the fireclearance, the insurance policies and inventory of stocks. He further testified that the three insurancecompanies are sister companies, and as a matter offact when he was following-up his claim withEquitable Insurance, the Claims Manager told him togo first to Reliance Insurance and if said companyagrees to pay, they would also pay. The sametreatment was given him by the other insurancecompanies. Ultimately, the three insurance companies denied plaintiffs' claim for payment. Respondents comments > Western Guaranty Corporation through ClaimsManager Bernard S. Razon told the plaintiff that hisclaim 'is denied for breach of policy conditions.' Reliance Insurance purveyed the same message as well as Equitable Insurance Corporation. - The said policy in question follows: "The insured shall give notice to the Company of anyinsurance or insurances already effected, or whichmay subsequently be effected, covering any of theproperty or properties consisting of stocks in trade,goods in process and/or inventories only herebyinsured, and unless such notice be given and theparticulars of such insurance or insurances be statedtherein or endorsed on this policy pursuant toSection 50 of the Insurance Code, by or on behalf ofthe Company before the occurrence of any loss ordamage, all benefits under this policy shall bedeemed forfeited, provided however, that thiscondition shall not apply when the total insurance orinsurances in force at the time of loss or damage isnot more than P200,000.00." Petitioners comments > Petitioners contend that they are not to be blamedfor the omissions, alleging that insurance agent LeonAlvarez (for Western) and Yap Kam Chuan (forReliance and Equitable) knew about the existence ofthe additional insurance coverage and that they werenot informed about the

requirement that such otheror additional insurance should be stated in the policy,as they have not even read said policies. ISSUE WON New Life Enterprises claim for payment be denied HELD YES Ratio Furthermore, when the words and language of documents are clear and plain or readilyunderstandable by an ordinary reader thereof, thereis absolutely no room for interpretation orconstruction anymore.Courts are not allowed tomake contracts for the parties; rather, they willintervene only when the terms of the policy areambiguous, equivocal, or uncertain. The partiesmust abide by the terms of the contract becausesuch terms constitute the measure of the insurer'sliability and compliance therewith is a conditionprecedent to the insured's right of recovery from theinsurer. - While it is a cardinal principle of insurance law thata policy or contract of insurance is to be construedliberally in favor of the insured and strictly againstthe insurer company, yet contracts of insurance, likeother contracts, are to be construed according to the sense and meaning of the terms which the partiesthemselves have used. If such terms are clear andunambiguous, they must be taken and understood intheir plain, ordinary and popular sense. Moreover,obligations arising from contracts have the force oflaw between the contracting parties and should becomplied with in good faith. Reasoning a. The terms of the contract are clear andunambiguous. The insured is specifically required todisclose to the insurer any other insurance and itsparticulars which he may have effected on the samesubject matter. The knowledge of such insurance bythe insurer's agents, even assuming the acquisitionthereof by the former, is not the "notice" that wouldstop the insurers from denying the claim. Besides,the so-called theory of imputed knowledge, that is,knowledge of the agent is knowledge of the principal,aside from being of dubious applicability here haslikewise been roundly refuted by respondent courtwhose factual findings we find acceptable. b. Petitioners should be aware of the fact that aparty is not relieved of the duty to exercise theordinary care and prudence that would be exacted inrelation to other contracts. The conformity of theinsured to the terms of the policy is implied from hisfailure to express any disagreement with what isprovided for.

FIRST QUEZON CITY INSURANCE CO. v. CA (DE DIOS MARIKINA TRANSPORT CO) 218 SCRA 526 GRINO-AQUINO; February 28, 1993 NATURE PETITION for review of the decision of the Court ofAppeals. FQCIC seeks to limit to P12000, the amountspecified in the insurance contract, its liability toindemnify the respomdemt DMTC, for the damagessuffered by a passenger, who accidentally fell off thebug. FACTS - After sending off certain seamen at the departure area of MIA, Jose V. del Rosario proceeded to thepublic utility bus stop. While at the bus stop, theplaintiff saw a DMTC bus. While moving at a crawlingpace, it was taking several passengers, all of whommanaged to board the bus while it was already at thebus stop; plaintiff was the last one to board the bus.While the plaintiff was still on the bus with his handon the bus door, the slowly moving bus sped forward at a high speed, as a result of which, the plaintiff lostbalance and fell from the bus. As plaintiff clunginstinctively to the handle bar, he was dragged bythe bus along the asphalted road. The bus driver, GilAgpalo, abruptly stopped the bus. Then fled from thescene, leaving the bus and the injured plaintiff behind. - The plaintiff was brought to the Manila Sanitariumand Hospital where the doctors performed 2 majorsurgical operations on plaintiffs right leg. - Plaintiff was confined at the hospital for (40) days,from June 10, 1984 to August 26, 1984. Medicalexpenses totaled the amount of P69,444.41.Plaintiffs medical expenses were advanced by hisemployer Maglines but he was required to reimburseMaglines on a staggered basis by way of salarydeductions. After his release from the hospital, hereturned to the hospital for further treatment andcheckup. The injuries had left plaintiff with a hugescar on his right leg. Also, the plaintiff incurred lostearning by way of unearned salaries amounting toP7,500.00 due to said physical injuries and theconsequent hospital confinement. - Plaintiff filed on June 26, 1985 the complaintagainst DMTC and its driver. Agpalo was laterdropped as a party defendant because he could notbe served with summons. Upon filing its answer,defendant DMTC filed a thirdparty complaint againstFirst Quezon City Insurance Co., Inc. September 17,1985, third-party defendant filed its answer to thethird-party complaint. - TC held DMTC complaint dismissed for lack of meritand as regards the third-party complaint FirstQuezon City Insurance Co., Inc. was to indemnifythird-party plaintiff DMTC in the sum of P12,000.00with interest. There being no satisfactory warrant thecourt dismissed the rest of the claims in thecomplaint and third-party complaint. - The bus company appealed to the CA, whichmodified the dispositive as regards the thirdpartycomplaint, that the third-party defendant FirstQuezon City Insurance Co., Inc. be ordered toindemnify third-party plaintiff DMTC the SUM ofP50,000.00 with legal interest. Insurance companyfiled a MFR which was denied. Hence, this petition for review, assailing theappellate courts' interpretation of the provision of theinsurance contract on the limit of the insurer's liability. ISSUE WON the CA erred in the interpretation of theinsurance contract on the limit of the insurersliability HELD YES - The insurance policy clearly placed the maximumlimit of the petitioner's liability for damages arisingfrom death or bodily injury at P12,000.00 perpassenger and its maximum liability per accident at(P50,000.00. Since only one passenger was injuredin the accident, the insurer's liability for the damagessuffered by said passenger is pegged to the amountof P12,000.00 only.

- The limit of P50,000.00 per accident means thatthe insurer's maximum liability for any singleaccident will not exceed P50,000.00 regardless of thenumber of passengers killed or injured therein. The bus company may not recover from theinsurance company more than P12,000.00 perpassenger killed or injured, or (P50,000.00) peraccident even if under the judgment of the court, theerring bus operator will have to pay more thanP12,000.00 to each injured passenger. The trialcourt's interpretation of the insurance contract wasthe correct interpretation. Disposition petition for review is GRANTED. The decision promulgated by the CA, ordering the thirdparty defendent, First Quezon City Insurance Co.,Inc. to indemnify theI private respondent,(DMTC),the sum of P50,000.00 for the damages of thepassenger, Jose V. Del Rosario, is hereby modified byreducing the award to 12,000.00 only. Costs against the private respondent De Dios Marikina Transportation Co., Inc.

TY V FIRST NATIONAL SURETY 1 SCRA 1324 LABRADOR; April 29, 1961 FACTS - At different times within a period of two monthsprior to 24 December 1953, Diosdado C. Ty,employed as operator mechanic foreman in theBroadway Cotton Factory insured himself in 18 localinsurance companies, among which being the 8above-named defendants, which issued to himpersonal accident policies. Plaintiffs beneficiary washis employer, Broadway Cotton Factory, which paidthe insurance premiums. On 24 December 1953, afire broke out which totally destroyed the Broadway Cotton Factory. Fighting his way out of the factory,plaintiff was injured on the left hand by a heavyobject. He was brought to the Manila CentralUniversity hospital, and after receiving first-aid, hewent to the National Orthopedic Hospital fortreatment of his injuries (fractures in index, middle,fourth, and fifth fingers of left hand).From 26December 1953 to 8 February 1954, he underwentmedical treatment in the hospital. The above-described physical injuries have caused temporarytotal disability of plaintiffs left hand. Plaintiff filed thecorresponding notice of accident and notice of claimwith all of the above-named defendants to recoverindemnity. Defendants rejected plaintiffs claim forindemnity for the reason that there being noseverance of amputation of the left hand, thedisability suffered by him was not covered by his policy. - Plaintiff sued the defendants in the MunicipalityCourt of this City, which dismissed his complaints.Thereafter, the plaintiff appealed to the Court of FirstInstance Manila, presided by Judge Gregorio S.Narvasa, which absolved the defendants from thecomplaints. Hence, the appeal. ISSUE WONDiosdado Ty is entitled to indemnity under the insurance policy for the disability of his left hand HELD - The agreement contained in the insurance policiesis the law between the parties. As the terms of thepolicies are clear, express and specific that onlyamputation of the left hand should be considered asa loss thereof, an interpretation that would includethe mere fracture or other temporary disability notcovered by the policies would certainly beunwarranted. In the case at bar, due to the clarity ofthe stipulation, distinction between temporarydisability and total disability need not be made inrelation to ones occupation means that the conditionof the insurance is such that common prudencerequires him to desist from transacting his businessor renders him incapable of working. While the Courtsympathizes with the plaintiff or his employer, forwhose benefit the policies were issued, it can not gobeyond the clear and express conditions of theinsurance policies, all of which define partial disabilityas loss of either hand by a amputation through thebones of the wrist. There was no such amputation inthe case at bar. - The Supreme Court affirmed the appealed decision, with costs against the plaintiff-appellant.

MISAMIS LUMBER V CAPITAL INSURANCE 17 SCRA 288 REYES; May 20, 1966 NATURE Direct appeal on a point of law from the judgment of the Court of First Instance of Manila FACTS - Misamis Lumber Corporation, under its formername, Lanao Timber Mills, Inc., insured its FordFalcon motor car for the amount of P14,000 withCapital Insurance & Surety Company, Inc. Thepertinent provisions of the policy provided, asfollows: 1. The Company will subject to the Limits ofLiability indemnify the Insured against loss ordamage to the Motor Vehicle and its accessoriesand spare parts whilst thereon. 2. (a) by accidental collision or overturning or collision or overturning consequent whenmechanical breakdown or consequent upon wearand tear. 3. At its option, the Company may pay in cash theamount of the loss or damage or may repair, reinstate or replace the Motor Vehicle or any partthereof or its accessories or spare parts. Theliability of the Company shall not exceed the valueof the parts lost or damaged and the reasonablecost of fitting such parts or the value of the MotorVehicle at the time of the loss or damagewhichever is the loss. The Insured's estimate ofvalue stated in the schedule shall be the maximumamount payable by the Company in respect of anyclaim for loss or damage. 4. The Insured may authorize the repair of theMotor Vehicle necessitated by damage for whichthe Company may be liable under this policyprovided that: (a) the estimated cost of such repair does not exceed the authorized Repair Limit. (b) a detailed estimate of the cost is forwardedto the Company without delay and providing alsothat the authorized repair limit is P150.00. - One night, the insured car, while traveling along inAurora Boulevard, passed over a water hole whichthe driver did not see because an oncoming car didnot dim its light. The crankcase and flywheel housingof the car broke when it hit a hollow block lying alongside the water hole. The car was towed andrepaired by Morosi Motors at a total cost of P302.27. - When the repairs on the car had already beenmade, Misamis made a report of the accident toCapital Insurance. - Since Capital refused to pay for the total cost of towage and repairs, suit was filed in the municipalcourt originally. - The defendant-appellant admits liability in theamount of P150, but not for any excess thereof.Thelower court did not exonerate the said appellant forthe excess because the company's absolution wouldrender the insurance contract one-sided and that thesaid insurer had not shown that the cost of repairs inthe sum of P302.27 is unreasonable, excessive orpadded, nor had it shown that it could haveundertaken the repairs itself at less expense. ISSUE WON Capital Insurance can be made to pay more than P150 HELD NO - The insurance policy stipulated in paragraph 4 thatif the insured authorizes the repair the liability of theinsurer, per its sub-paragraph (a), is limited toP150.00. The literal meaning of this stipulation mustcontrol, it being the actual contract, expressly andplainly provided for in the policy. - Recourse to legal hermeneutics is not called forbecause paragraph 4 of the policy is clear andspecific and leaves no room for interpretation.

- The option to undertake the repairs is accorded tothe insurance company per paragraph 2. The saidcompany was deprived of the option because theinsured took it upon itself to have the repairs made,and only notified the insurer when the repairs weredone. As a consequence, paragraph 4, which limitsthe company's liability to P150.00, applies. - The insurance contract may be rather onerous("one-sided", as the lower court put it), but that initself does not justify the abrogation of its expressterms, terms which the insured accepted or adheredto and which is the law between the contractingparties. - To require the insurer to prove that the cost of therepairs ordered by the insured is unreasonable, whenthe insurer was not given an opportunity to inspectand assess the damage before the repairs weremade, is contrary to elementary justice and equity

SUN INSURANCE OFFICE LTD. V CA (TAN) 195 SCRA 193 PARAS; March 13, 1991 NATURE Petition for certiorari to review the decision of the CA FACTS - Private respondent Emilio Tan took from thepetitioner a Peso 300,000 property insurance policyto cover his interest in the electrical insurance storeof his brother housed in a building in Iloilo City onAugust 15, 1983. Four days after the issuance of thepolicy, the building including the insured store burned. - On August 20, 1983, Tan filed his claim for fire loss.Sun Insurance, on February 29, 1984, wrote theprivate respondent denying the claim. On April 3,1984, private respondent wrote another letter to theinsurance company requesting reconsideration of thedenial. Tans lawyer wrote another letter to theinsurance company inquiring about the April 3 letterwhich sought for a reconsideration of the denial. Inits reply to the lawyers letter, Sun Insurancereiterated its denial of the claim and enclosed thereincopies of the two previous denials dated February29, 1984 and May 17, 1985. - On November 20, 1985, Tan filed a civil case withthe RTC.Petition filed a motion to dismiss on thealleged ground that the action has already prescribedbased on Condition 27 of the Insurance Policy whichstated that the window to file the appropriate actionwith either the Insurance Commission or in any courtof competent jurisdiction is twelve months from therejection of the claim. RTC denied the motion and thesubsequent motion for reconsideration. The CAlikewise denied the petition of Sun Insurance. ISSUE 1. WON the court the filing of a motion forreconsideration interrupts the 12 months prescriptionperiod to contest the denial of the insurance claim 2. WON the rejection of the claim shall be deemedfinal only if it contains words to the effect that thedenial is final HELD 1. NO - The SC held that Condition 27 of the Insurancepolicy is very clear and free from any doubt orambiguity. It has to be taken in its plain, ordinary, and popular sense. The rejection letter of February29, 1984 was clear and plain. The Court noted thatthe one year period is likewise in accord with Section23 of the Insurance Code which states that anycondition which limits the time for commencing anaction to a period of less than one year when thecause of action accrues is void.The right of action,according to the SC, accrues at the time that theclaim is rejected at the first instance. A request forreconsideration of the denial cannot suspend therunning of the prescriptive period. The Court notedthat the rationale for the one year period is to ensurethat the evidence as to the origin and cause of thedestruction have not yet disappeared. 2. NO - The Court clarified its ruling in Eagle Star InsuranceCo. vs Chia Yu where it ruled that the cause ofaction in an insurance contract does not accrue untilthe Insureds claim is finally rejected by the Insurerby stating the use of the word finally cannot beconstrued to mean the rejection of a petition forreconsideration. What the court referred to in effectis the rejection in the first instance as claimed bySun Insurance DispositionThe decision of the CA is reversed and set aside. The case is dismissed

FORTUNE INSURANCE AND SURETY CO. INC.V CA (PRODUCERS BANK OF THE PHILIPPINES) 244 SCRA 308 DAVIDE; May 23, 1995 NATURE Petition for Review on certiorari of CA decision FACTS - Producers Bank of the Philippines filed a complaint against Fortune Insurance and Surety Co., Inc. forrecovery of P725,000.00 under the policy issued byFortune. The sum was allegedly lost on June 29,1987 during a robbery of Producer's armored vehiclewhile it was in transit to transfer the money from itsPasay City Branch to its head office in Makati underthe custody of its teller, Maribeth Alampay.Thearmored car was driven by Benjamin Magalong Y deVera, escorted by Security Guard Saturnino Atiga YRosete. Driver Magalong was assigned by PRCManagement Systems. - After an investigation by the Pasay police, driverMagalong and guard Atiga were charged, togetherwith Batigue , Aquino and John Doe, with violation ofP.D. 532 (Anti-Highway Robbery Law) - Demands were made by the Producers upon theFortune to pay the amount of the loss ofP725,000.00 but the latter refused to pay as the lossis excluded from the coverage of the insurance policyspecifically under "General Exceptions" > The company shall not be liable under thispolicy in respect of x x x (b) any loss caused byany dishonest, fraudulent or criminal act of theinsured or any officer, employee, partner,director, trustee or authorized representative ofthe Insured whether acting alone or inconjunction with others. - Fortune opposes the contention of Producers that Atiga and Magalong are not its "officer, employee, x xx trustee or authorized representative x x x at thetime of the robbery - Trial Court > On being EMPLOYEES Magalong and Atiga were not employees orrepresentatives of Producers as their services asarmored car driver and as security guard havingbeen merely offered by PRC Management and byUnicorn Security and which latter firms assignedthem to plaintiff.The wages and salaries of bothMagalong and Atiga are presumably paid by theirrespective firms, which alone wields the power todismiss them > On being AUTHORIZED REPRESENTATIVE They were merely an assigned armored car driverand security guard for the money transfer. It wasteller Maribeth Alampay who had "custody" of theP725,000.00 cash being transferred along a specifiedmoney route - Court of Appeals > affirmed in toto > A policy or contract of insurance is to be construedliberally in favor of the insured and strictly againstthe insurance company (New Life Enterprises vs.Court of Appeals; Sun Insurance Office, Ltd. vs.Court of Appeals). Contracts of insurance, like othercontracts, are to be construed according to the senseand meaning of the terms which the partiesthemselves have used. If such terms are clear andunambiguous, they must be taken and understood intheir plain, ordinary and popular sense (New LifeEnterprises Case; Sun Insurance Office). > The language used by Fortune in the policy isplain, ordinary and simple. No other interpretation isnecessary. The word "employee" should be taken tomean in the ordinary sense.The Labor Code is aspecial law specifically dealing with/and specificallydesigned to protect labor and therefore its definition as to employer-employee relationships insofar as theapplication/enforcement of said Code is concernedmust necessarily be inapplicable to an insurancecontract. Had it intended to apply the Labor Code indefining what the word "employee" refers to, it must/should have so stated expressly in the insurancepolicy.Said driver and security guard cannot beconsidered as employees of Producers

bank becauseit has no power to hire or to dismiss said driver andsecurity guard under the contracts except only to askfor their replacements from the contractors. - Fortunes Contention > when Producers commissioned a guard and adriver to transfer its funds from one branch toanother, they effectively and necessarily became itsauthorized representatives in the care and custody ofthe money. Assuming that they could not beconsidered authorized representatives, they were, nevertheless, employees of Producers. It asserts thatthe existence of an employer-employee relationship"is determined by law and being such, it cannot bethe subject of agreement." Thus, if there was inreality an employer-employee relationship betweenProducers, on the one hand, and Magalong andAtiga, on the other, the provisions in the contracts ofProducers with PRC Management System forMagalong and with Unicorn Security Services forAtiga which state that Producers is not theiremployer and that it is absolved from any liability asan employer, would not obliterate the relationship. > an employer-employee relationship depends upon four standards: (1) the manner of selection and engagement of the putative employee (2) the mode of payment of wages (3) the presence or absence of a power to dismiss and (4) the presence and absence of a power to control the putative employee's conduct. > Of the four, the right-of-control test has been heldto be the decisive factor. It asserts that the power ofcontrol over Magalong and Atiga was vested in andexercised by Producers. Fortune further insists thatPRC Management System and Unicorn SecurityServices are but "labor-only" contractors underArticle 106 of the Labor Code which provides: Art. 106. Contractor or subcontractor. - There is"labor-only" contracting where the personsupplying workers to an employer does not havesubstantial capital or investment in the form oftools, equipment, machineries, work premises,among others, and the workers recruited and placed by such persons are performing activitieswhich are directly related to the principal businessof such employer. In such cases, the person orintermediary shall be considered merely as anagent of the employer who shall be responsible tothe workers in the same manner and extent as ifthe latter were directly employed by him. > International Timber Corp. vs. NLRC - a "labor-only" contractor is equivalent to a finding that thereis an employer-employee relationship between theowner of the project and the employee of the "laboronly" contractor - Producers Contention > Magalong and Atiga were not its employees since ithad nothing to do with their selection andengagement, the payment of their wages, theirdismissal, and the control of their conduct. > International Timber Corp. is not applicable to allcases but only when it becomes necessary to preventany violation or circumvention of the Labor Code, asocial legislation whose provisions may set asidecontracts entered into by parties in order to giveprotection to the working man. > American President Lines vs. Clave should be applied which stated In determining the existence of employer-employee relationship, the following elements aregenerally considered, namely: (1) the selectionand engagement of the employee; (2) thepayment of wages; (3) the power of dismissal; and(4) the power to control the employee's conduct. - Since under Producers' contract with PRCManagement Systems it is the latter which assignedMagalong as the driver of Producers' armored carand was responsible for his faithful discharge of hisduties and responsibilities, and since Producers paidthe monthly compensation of P1,400.00 per driver toPRC Management Systems and not to Magalong, it isclear that Magalong was not Producers' employee. Asto Atiga, Producers relies on the provision of itscontract with Unicorn Security Services whichprovides that the guards of the latter "are in nosense employees of the CLIENT." ISSUE WON Fortune Insurance and Surety Co. Inc. is liableunder the Money, Security, and Payroll Robberypolicy it issued to Producers Bank of the Philippinesor WON recovery is precluded under the generalexceptions clause of the policy

HELD NO Ratio A contract of insurance is a contract ofadhesion, thus any ambiguity therein should beresolved against the insurer, or it should beconstrued liberally in favor of the insured and strictlyagainst the insurer. Limitations of liability should beregarded with extreme jealousy and must beconstrued in such a way as to preclude the insurerfrom non-compliance with its obligation. It goeswithout saying then that if the terms of the contractare clear and unambiguous, there is no room forconstruction and such terms cannot be enlarged ordiminished by judicial construction. - An insurance contract is a contract of indemnityupon the terms and conditions specified therein. It issettled that the terms of the policy constitute themeasure of the insurer's liability. In the absence ofstatutory prohibition to the contrary, insurancecompanies have the same rights as individuals tolimit their liability and to impose whatever conditionsthey deem best upon their obligations notinconsistent with public policy. Reasoning - It should be noted that the insurance policy enteredinto by the parties is a theft or robbery insurancepolicy which is a form of casualty insurance. Section174 of the Insurance Code provides: Sec. 174. Casualty insurance is insurance coveringloss or liability arising from accident or mishap,excluding certain types of loss which by law orcustom are considered as failing exclusively withinthe scope of insurance such as fire or marine. Itincludes, but is not limited to, employer's liabilityinsurance, public liability insurance, motor vehicleliability insurance, plate glass insurance, burglaryand theft insurance, personal accident and healthinsurance as written by non-life insurancecompanies, and other substantially similar kinds ofinsurance. (italics supplied) - Except with respect to compulsory motor vehicleliability insurance, the Insurance Code contains noother provisions applicable to casualty insurance orto robbery insurance in particular. These contractsare, therefore, governed by the general provisionsapplicable to all types of insurance. Outside of these,the rights and obligations of the parties must bedetermined by the terms of their contract, taking intoconsideration its purpose and always in accordancewith the general principles of insurance law. - With the foregoing principles in mind, it may nowbe asked whether Magalong and Atiga qualify asemployees or authorized representatives has been aptly observed that in burglary, robbery, and theftinsurance, "the opportunity to defraud the insurer -the moral hazard - is so great that insurers havefound it necessary to fill up their policies withcountless restrictions, many designed to reduce thishazard. Seldom does the insurer assume the risk ofall losses due to the hazards insured against."Persons frequently excluded under such provisionsare those in the insured's service and employment.The purpose of the exception is to guard againstliability should the theft be committed by one havingunrestricted access to the property. In such cases,the terms specifying the excluded classes are to begiven their meaning as understood in commonspeech. The terms "service" and "employment" aregenerally associated with the idea of selection,control, and compensation. - There is marked disagreement between the parties on the correct meaning of the terms"employee" and "authorized representatives." It is clear to us that insofar as Fortune is concerned,it was its intention to exclude and exempt fromprotection and coverage losses arising fromdishonest, fraudulent, or criminal acts of personsgranted or having unrestricted access to Producers'money or payroll. When it used then the term"employee," it must have had in mind any personwho qualifies as such as generally and universallyunderstood, or jurisprudentially established in thelight of the four standards in the determination of theemployer-employee relationship or as statutorilydeclared even in a limited sense as in the case ofArticle 106 of the Labor Code which considers theemployees under a "labor-only" contract asemployees ofthe party employing them and not ofthe party who supplied them to the employer. - But even granting for the sake of argument thatthese contracts were not "labor-only" contracts, andPRC Management Systems and Unicorn SecurityServices were truly independent contractors, we aresatisfied that Magalong and Atiga were, in respect ofthe transfer of Producer's money from its Pasay Citybranch to its head office in Makati, its "authorizedrepresentatives" who served as such with its tellerMaribeth Alampay. Howsoever viewed, Producersentrusted the three with the specific duty to

safelytransfer the money to its head office, with Alampayto be responsible for its custody in transit; Magalongto drive the armored vehicle which would carry themoney; and Atiga to provide the needed security forthe money, the vehicle, and his two othercompanions. In short, for these particular tasks, the three acted as agents of Producers. A"representative" is defined as one who represents orstands in the place of another; one who representsothers or another in a special capacity, as an agent,and is interchangeable with "agent." Dispositioninstant petition is hereby GRANTED. CAdecision and RTC Makati decision are REVERSED and SET ASIDE. Civil Case is DISMISSED.

VERENDIA V CA (FIDELITY & SURETY CO. OF THE PHILS) 217 SCRA 417 MELO; January 22, 1993 NATURE Petition to review decision of the CA FACTS - Fidelity Co. issued a Fire Insurance Policy coveringVerendias residential building in the amount ofP385k. Verendia also insured the same building withtwo other companies (Country Bankers Insurance forP56k, and Development Insurance for P400k). - While all 3 policies were in force, the insuredproperty was completely destroyed by fire. Verendiafiled a claim against Fidelity, but the latter refusedpayment, thus a complaint was filed in the RTC. Fidelitys reason for refusal: the policy was avoidedby reason of over -insurance, and that Verendiamaliciously represented that the building was underlease to a Roberto Garcia, when it was actually aMarcelo Garcia who was the lessee. - RTC: policy was violated by Verendia when it failedto inform Fidelity of his other insurance coverages,thus no need to pay. - CA: reversed decision ISSUE (There is a procedural issue involved here, but isirrelevant to our discussion. It concerns the filing of amotion for extension of time to file a motion forreconsideration, where the court said that although itnow prohibits filing of such motion, the instantmotion was filed before the effectivity of this rule,thus allowing the adjudication of the case) WON Fidelity was liable to pay Verendia considering the circumstances HELD 1. NO RatioAs the insurance contract is the law between the parties, Verendia is deemed to have forfeited his right to claim by the misrepresentation he made. Reasoning - the court reviewed the factual findings of the courtsbelow, since it appears that there was amisapprehension of the facts by the CA. - Verendia is found to have concocted the leasecontract to deflect responsibility for the fire towardsan alleged lessee, even making it appear that thealleged lessee had disappeared, inflated the value ofthe property, and insured same property with twoother companies. - An insurance contract is the law between theparties, its terms and conditions constitute themeasure of the insurers liability and compliancetherewith is a condition precedent to the insuredsright to recovery from the insurer. - As it is also a contract of adhesion, an insurancecontract should be liberally construed in favor of theinsured and strictly against the insurer companywhich usually prepares it. - Considering, however, the fact that Verendia used afalse lease contract to support his claim, the terms ofthe policy should be strictly construed against theinsured. Verendia failed to live by the terms of thepolicy, specifically Section 13 thereof which isexpressed in terms that are clear and unambiguous,that all benefits under the policy shall be forfeited Ifthe claim be in any respect fraudulent, or if any falsedeclaration be made or used in support thereof, or ifany fraudulent means or devises are used by theInsured or anyone acting in his behalf to obtain anybenefit under the policy. Verendia, having presenteda false declaration to support his claim for benefits inthe form of a fraudulent lease contract, he forfeitedall benefits therein by virtue of Section 13 of thepolicy in the absence of proof that Fidelity waivedsuch provision. Worse yet, by presenting a falselease contract, Verendia reprehensibly disregardedthe principle that insurance contracts areuberrimae fidae and demand the most abundant good faith. DispositionDecision of CA reversed, and that of RTC is reinstated.

FIELDMEN'S INSURANCE CO. INC V VDA. DE SONGCO 25 SCRA 20 FERNANDO; 1968 FACTS - An insurance firm, petitioner Fieldmen's Insurance Co., Inc., was not allowed to escape liability under a common carrier insurance policy on the pretext thatwhat was insured, not once but twice, was a privatevehicle and not a common carrier, the policy beingissued upon the insistence of its agent whodiscounted fears of the insured that his privatelyowned vehicle might not fall within its terms, theinsured moreover being "a man of scant education,"finishing only the first grade. So it was held in adecision of the lower court thereafter affirmed byrespondent Court of Appeals. Petitioner in seekingthe review of the above decision of respondent Courtof Appeals cannot be so sanguine as to entertain thebelief that a different outcome could be expected. Tobe more explicit, we sustain the Court of Appeals. - The facts as found by respondent Court of Appeals,binding upon us, follow: "This is a peculiar case.Federico Songco of Floridablanca, Pampanga, a manof scant education being only a first grader ..., owneda private jeepney with Plate No. 41-289 for the year1960. On September 15, 1960, as such privatevehicle owner, he was induced by Fieldmen'sInsurance Company Pampanga agent BenjaminSambat to apply for a Common Carrier's LiabilityInsurance Policy covering his motor vehicle ... Uponpaying an annual premium of P16.50, defendantFieldmen's Insurance Company, Inc. issued onSeptember 19, 1960, Common Carriers AccidentInsurance Policy No. 45-HO- 4254 ... the duration ofwhich will be for one (1) year, effective September15, 1960 to September 15, 1961. On September 22,1961, the defendant company, upon payment of thecorresponding premium, renewed the policy byextending the coverage from October 15, 1961 toOctober 15, 1962. This time Federico Songco'sprivate jeepney carried Plate No. J-68136-Pampanga-1961. ... On October 29, 1961, during theeffectivity of the renewed policy, the insured vehiclewhile being driven by Rodolfo Songco, a duly licenseddriver and son of Federico (the vehicle owner)collided with a car in the municipality of Calumpit,province of Bulacan, as a result of which mishapFederico Songco (father) and Rodolfo Songco (son)died, Carlos Songco (another son), the latter's wife,Angelita Songco, and a family friend by the name ofJose Manuel sustained physical injuries of varyingdegree."1 - It was further shown according to the decision ofrespondent Court of Appeals: "Amor Songco, 42year-old son of deceased Federico Songco, testifyingas witness, declared that when insurance agentBenjamin Sambat was inducing his father to insurehis vehicle, he butted in saying: 'That cannot be, Mr. Sambat, because our vehicle is an "owner" privatevehicle and not for passengers,' to which agentSambat replied: 'whether our vehicle was an "owner"type or for passengers it could be insured becausetheir company is not owned by the Government andthe Government has nothing to do with theircompany. So they could do what they pleasewhenever they believe a vehicle is insurable' ... Inspite of the fact that the present case was filed andtried in the CFI of Pampanga, the defendantcompany did not even care to rebut Amor Songco'stestimony by calling on the witnessstand agentBenjamin Sambat, its Pampanga Field Representative."2 - The plaintiffs in the lower court, likewiserespondents here, were the surviving widow andchildren of the deceased Federico Songco as well asthe injured passenger Jose Manuel. On the abovefacts they prevailed, as had been mentioned, in thelower court and in the respondent Court ofAppeals.1awphl.nt - The basis for the favorable judgment is the doctrine announced in Qua Chee Gan v. Law Union and Rock Insurance Co., Ltd.,3 with Justice J. B. L. Reyes speaking for the Court. It is now beyond questionthat where inequitable conduct is shown by aninsurance firm, it is "estopped from enforcingforfeitures in its favor, in order to forestall fraud orimposition on the insured."4 - As much, if not much more so than the Qua Chee Gan decision, this is a case where the doctrine of estoppel undeniably calls for application. Afterpetitioner Fieldmen's Insurance Co., Inc. had led theinsured Federico Songco to believe that he couldqualify under the common carrier liability

insurancepolicy, and to enter into contract of insurance payingthe premiums due, it could not, thereafter, in anylitigation arising out of such representation, bepermitted to change its stand to the detriment of theheirs of the insured. As estoppel is primarily basedon the doctrine of good faith and the avoidance ofharm that will befall the innocent party due to itsinjurious reliance, the failure to apply it in this casewould result in a gross travesty of justice. - That is all that needs be said insofar as the firstalleged error of respondent Court of Appeals isconcerned, petitioner being adamant in its far-from-reasonable plea that estoppel could not be invokedby the heirs of the insured as a bar to the allegedbreach of warranty and condition in the policy. ltwould now rely on the fact that the insured owned aprivate vehicle, not a common carrier, something which it knew all along when not once but twice itsagent, no doubt without any objection in its part,exerted the utmost pressure on the insured, a manof scant education, to enter into such a contract. - Nor is there any merit to the second alleged errorof respondent Court that no legal liability wasincurred under the policy by petitioner. Why liabilityunder the terms of the policy5 was inescapable wasset forth in the decision of respondent Court ofAppeals. Thus: "Since some of the conditionscontained in the policy issued by the defendant-appellant were impossible to comply with under theexisting conditions at the time and 'inconsistent withthe known facts,' the insurer 'is estopped fromasserting breach of such conditions.' From thisjurisprudence, we find no valid reason to deviate andconsequently hold that the decision appealed fromshould be affirmed. The injured parties, to wit, CarlosSongco, Angelito Songco and Jose Manuel, for whosehospital and medical expenses the defendantcompany was being made liable, were passengers ofthe jeepney at the time of the occurrence, andRodolfo Songco, for whose burial expenses thedefendant company was also being made liable wasthe driver of the vehicle in question. Except for thefact, that they were not fare paying passengers, theirstatus as beneficiaries under the policy is recognizedtherein."6 - Even if it be assumed that there was an ambiguity,an excerpt from the Qua Chee Gan decision wouldreveal anew the weakness of petitioner's contention.Thus: "Moreover, taking into account the well knownrule that ambiguities or obscurities must be strictlyinterpreted against the party that caused them, the'memo of warranty' invoked by appellant bars thelatter from questioning the existence of theappliances called for in the insured premises, sinceits initial expression, 'the undernoted appliances forthe extinction of fire being kept on the premises insured hereby, ... it is hereby warranted ...,' admits of interpretation as an admission of the existence ofsuch appliances which appellant cannot nowcontradict, should the parol evidence rule apply."7 - To the same effect is the following citation from thesame leading case: "This rigid application of the ruleon ambiguities has become necessary in view ofcurrent business practices. The courts cannot ignorethat nowadays monopolies, cartels and concentrationof capital, endowed with overwhelming economicpower, manage to impose upon parties dealing withthem cunningly prepared 'agreements' that theweaker party may not change one whit, his participation in the 'agreement' being reduced to thealternative to 'take it or leave it' labelled sinceRaymond Saleilles 'contracts by adherence' (contrats d'adhesion), in contrast to those entered into by parties bargaining on an equal footing, suchcontracts (of which policies of insurance andinternational bills of lading are prime examples)obviously call for greater strictness and vigilance onthe part of courts of justice with a view to protectingthe weaker party from abuses and imposition, andprevent their becoming traps for the unwary (NewCivil Code. Article 24; Sent. of Supreme Court ofSpain, 13 Dec. 1934, 27 February 1942)."8 - The last error assigned which would find fault withthe decision of respondent Court of Appeals insofaras it affirmed the lower court award for exemplarydamages as well as attorney's fees is, on its face, ofno persuasive force at all. - The conclusion that inescapably emerges from theabove is the correctness of the decision ofrespondent Court of Appeals sought to be reviewed.For, to borrow once again from the language of theQua Chee Gan opinion: "The contract of insurance isone of perfect good faith (uberima fides) not for theinsured alone,but equally so for the insurer; in fact,it is more so for the latter, since its dominantbargaining position carries with it stricterresponsibility."9 - This is merely to stress that while the morality ofthe business world is not the morality of institutionsof rectitude like the pulpit and the academe, itcannot descend so low as to be another name forguile or deception. Moreover, should it happen thus,no court of justice should allow itself to lend itsapproval and support.1awphl.nt

- We have no choice but to recognize the monetaryresponsibility of petitioner Fieldmen's Insurance Co.,Inc. It did not succeed in its persistent effort to avoidcomplying with its obligation in the lower court andthe Court of Appeals. Much less should it find anyreceptivity from us for its unwarranted andunjustified plea to escape from its liability.

MALAYAN INSURANCE CORP. V CA (TKC MARKETING CORP.) 270 SCRA 242 ROMERO; March 20, 1997 NATURE Petition for review on certiorari FACTS - TKC Marketing Corp. was the owner/consignee ofsome 3,189.171 metric tons of soya bean mealwhich was loaded on board the ship MV Al Kaziemahfor carriage from the port of Rio del Grande, Brazil,to the port of Manila. Said cargo was insured againstthe risk of loss by petitioner Malayan InsuranceCorporation for which it issued two (2) Marine CargoPolicies. - While the vessel was docked in Durban, SouthAfrica the civil authorities arrested and detained itbecause of a lawsuit on a question of ownership andpossession. TKC Marketing notified Malayan of thearrest of the vessel and made a formal claim for the dollar equivalent on the policies (US$916,886.66) fornon-delivery of the cargo. It likewise sought theassistance of Malayan on what to do with the cargo.- Malayan replied that the arrest of the vessel by civilauthority was not a peril covered by the policies. TKCadvised Malayan that it might tranship the cargo andrequested an extension of the insurance coverageuntil actual transhipment, which extension wasapproved upon payment of additional premium. Theinsurance coverage was extended under the sameterms and conditions embodied in the originalpolicies while in the process of making arrangementsfor the transhipment of the cargo from Durban toManila. However the cargo was sold in Durban,South Africa, for US$154.40 per metric ton or a totalof P10,304,231.75 due to its perishable nature whichcould no longer stand a voyage of twenty days toManila and another twenty days for the dischargethereof. It reduced its claim to US$448,806.09 (or itspeso equivalent of P9,879,928.89 at the exchangerate of P22.0138 per $1.00) representing its lossafter the proceeds of the sale were deducted fromthe original claim.Malayan maintained its positionthat the arrest of the vessel by civil authorities on aquestion of ownership was an excepted risk underthe marine insurance policies. Petitioners Claim - an arrest by civil authority is not compensablesince the term "arrest" refers to "political orexecutive acts" and does not include a loss causedby riot or by ordinary judicial process as in this case- the deletion of the Free from Capture or SeizureClause would leave the assured covered solely forthe perils specified by the wording of the policy itself- the rationale for the exclusion of an arrest pursuantto judicial authorities is to eliminate collusionbetween unscrupulous assured and civil authorities. - any loss which private respondent may have incurred was in the nature and form of unrecovered acquisition value brought about by a voluntarysacrifice sale and not by arrest, detention or seizureof the ship. - its act of rejecting the claim was a result of itshonest belief that the arrest of the vessel was not acompensable risk under the policies issued Respondents Comments - petitioner, being the sole author of the policies,"arrests" should be strictly interpreted against itbecause the rule is that any ambiguity is to be takencontra proferentum. Risk policies should beconstrued reasonably and in a manner as to makeeffective the intentions and expectations of the parties. - the policies clearly stipulate that they cover therisks of non-delivery of an entire package and that itwas petitioner itself that invited and granted theextensions and collected premiums thereon. ISSUES 1. WON the arrest of the vessel was a risk covered under the subject insurance policies 2. WON insurance policies should be strictly construed against the insurer HELD 1.YES - With the incorporation of subsection 1.1 of Section1 of the Institute War Clauses, "arrest" caused byordinary judicial process is deemed included amongthe covered risks. This interpretation becomesinevitable when subsection 1.1 of Section 1 of theInstitute War Clauses provided that "this insurancecovers the risks excluded from the Standard Form ofEnglish Marine Policy by the clause 'Warranted free ofcapture, seizure, arrest, etc. x x x'" or the F.C. & S.Clause. Jurisprudentially, "arrests" caused byordinary judicial process is also a risk excluded fromthe Standard Form of English Marine Policy by theF.C. & S. Clause.

- Petitioner cannot adopt the argument that the"arrest" caused by ordinary judicial process is notincluded in the covered risk simply because the F.C.& S. Clause under the Institute War Clauses can onlybe operative in case of hostilities or warlikeoperations on account of its heading "Institute WarClauses." 2. YES Ratio Insurance Policies should be construedliberally in favor of the insured and strictly againstthe insurer. Reasoning - An insurance contract should be so interpreted asto carry out the purpose for which the partiesentered into the contract which is, to insure againstrisks of loss or damage to the goods. Suchinterpretation should result from the natural andreasonable meaning of language in the policy. Whererestrictive provisions are open to two interpretations,that which is most favorable to the insured is adopted. Indemnity and liability insurance policies areconstrued in accordance with the general rule ofresolving any ambiguity therein in favor of theinsured, where the contract or policy is prepared bythe insurer. A contract of insurance, being a contractof adhesion, par excellence, any ambiguity thereinshould be resolved against the insurer.Limitations ofliability should be regarded with extreme jealousyand must be construed in such a way as to precludethe insurer from noncompliance with its obligations - It must be borne in mind that such contracts areinvariably prepared by the companies and must beaccepted by the insured in the form in which they arewritten. Any construction of a marine policyrendering it void should be avoided. Such policieswill, therefore, be construed strictly against thecompany in order to avoid a forfeiture, unless noother result is possible from the language used. - If a marine insurance company desires to limit orrestrict the operation of the general provisions of itscontract by special proviso, exception, or exemption,it should express such limitation in clear andunmistakable language. Be that as it may, exceptions to the general coverageare construed most strongly against the company.Even an express exception in a policy is to beconstrued against the underwriters by whom thepolicy is framed, and for whose benefit the exceptionis introduced.

WESTERN GUARANTY CORPORATION V CA(RODRIGUEZ, and DE DIOS TRANSPORTATIONCO) 187 SCRA 652 FELICIANO; July 20, 1990 FACTS - At around 4:30 in the afternoon of 27 March 1982,while crossing Airport Road on a pedestrian lane onher way to work, respondent Priscilla E. Rodriguezwas struck by a De Dios passenger bus owned byrespondent De Dios Transportation Co., Inc., thendriven by one Walter Saga y Aspero. The bus driver disregarded the stop signal given by a trafficpoliceman to allow pedestrians to cross the road.Priscilla was thrown to the ground, hitting herforehead. She was treated at the Protacio EmergencyHospital and later on hospitalized at the San Juan DeDios Hospital. Her face was permanently disfigured,causing her serious anxiety and moral distress. - Respondent bus company was insured withpetitioner Western Guaranty Corporation ("Western")under its Master Policy which enumerated specificliabilities of the insurance company and ended with aclause to clarify the limitations of the amount whichcould be granted as indemnity. - Respondent Priscilla Rodriguez filed a complaint fordamages before the Regional Trial Court of Makatiagainst De Dios Transportation Co. and Walter A.Saga. Respondent De Dios Transportation Co., inturn, filed a third-party complaint against itsinsurance carrier, petitioner Western. - On 6 August 1985, the trial court rendered adecision in favor of respondent Priscilla E. Rodriguez,On appeal, the Court of Appeals affirmed in totothe decision of the trial court. Petitioner moved forthe reconsideration of the appellate court's decision.In a Resolution dated 10 January 1990, the Court ofAppeals denied the motion for reconsideration forlack of merit. Petitioner Western is now before us ona Petition for Review alleging that the Court ofAppeals erred in holding petitioner liable to paybeyond the limits set forth in the ScheduleIndemnities and in finding Western liable for loss ofearnings, moral damages and attorney's fees.Succinctly stated, it is petitioner Western's positionthat it cannot be held liable for loss of earnings,moral damages and attorney's fees because theseitems are not among those included in the ScheduleIndemnities set forth in the insurance policy. - Petitioner Western in effect contends before thisCourt, as it did before the Court of Appeals, thatbecause the Schedule of Indemnities limits theamount payable for certain kinds of expenses"hospital room", "surgical expenses", "anaesthesiologists' fee", "operating room" and "medicalexpenses" that Schedule should be read asexcluding liability for any other type of expense ordamage or loss even though actually sustained orincurred by the third party victim. We are notpersuaded by Western's contention. ISSUE WON the Schedule of indemnities as stated in theinsurance policy should be construed strictly toexclude all others not explicitly stated therein HELD NO RatioAn insurance policy being in the nature of an adhesion contract is to be strictly construed against the insurer and liberally in favor of the insured. Reasoning - Firstly, the Schedule of Indemnities does notpurport to restrict the kinds of damages that may beawarded against Western once liability has arisen.Section 1, quoted above, does refer to certain "Limitsof Liability" which in the case of the third partyliability section of the Master Policy, is apparentlyP50,000.00 per person per accident. Within thisover-all quantitative limit, all kinds of damagesallowable by law "actual or compensatorydamages"; "moral damages"; "nominal damages";"temperate or moderate damages"; "liquidateddamages"; and "exemplary damages" may beawarded by a competent court against the insureronce liability is shown to have arisen, and theessential requisites or conditions for grant of eachspecies of damages are present. It appears to usself-evident that the Schedule of Indemnities was notintended to be an enumeration, much less a closedenumeration, of the specific kinds of damages whichmay be awarded under the Master Policy Westernhas issued.

- Secondly, the reading urged by Western of theSchedule of Indemnities comes too close to workingfraud upon both the insured and the third partybeneficiary of Section 1, quoted above. For Western'sreading would drastically and without warning limitthe otherwise unlimited (save for the over-allquantitative limit of liability of P50,000.00 per personper accident) and comprehensive scope of liabilityassumed by the insurer Western under Section 1:"all sums necessary to discharge liability of theinsured in respect of [bodily injury to a third party]".This result which is not essentially different fromtaking away with the left hand what had been givenwith the right handwe must avoid as obviouslyrepugnant to public policy. If what Western nowurges is what Western intended to achieve by itsSchedule of Indemnities, it was incumbent uponWestern to use language far more specific andprecise than that used in fact by Western, so that the insured, and potential purchasers of its Master Policy,and the Office of the Insurance Commissioner, maybe properly informed and act accordingly. - Petitioner Western would have us construe theSchedule of Indemnities as comprising contractuallimitations of liability which, as already noted, iscomprehensively defined in Section 1 "Liability tothe Public" of the Master Policy. It is wellsettled,however, that contractual limitations of liability foundin insurance contracts should be regarded by courtswith a jaundiced eye and extreme care and shouldbe so construed as to preclude the insurer fromevading compliance with its just obligations. - Finally, an insurance contract is a contract ofadhesion. The rule is well entrenched in ourjurisprudence that the terms of such contract are tobe construed strictly against the party whichprepared the contract, which in this case happens tobe petitioner Western.

QUA CHEE GAN V LAW UNION AND ROCK INSURANCE CO., LTD. 96 PHIL 85 REYES; December 17, 1955 NATURE An appeal by defendant insurance company from the decision of CFI in favor of the plaintiff FACTS - before the last war, plaintiff-appellee owned 4warehouses or bodegas in Tabaco, Albay, used forthe storage of stocks of copra and of hemp, baledand loose, in which the appellee dealt extensively.They had been, with their contents, insured with the defendant Company since 1937, and the loose madepayable to the Philippine National Bank as mortgageof the hemp and crops, to the extent of its interest.- Fire of undetermined origin that broke out in theearly morning of July 21, 1940, and lasted almostone week, gutted and completely destroyed BodegasNos. 1, 2 and 4, with the merchandise storedtherein. Plaintiff-appellee informed the insurer bytelegram on the same date; and on the next day, thefire adjusters engaged by appellant insurancecompany arrived and proceeded to examine andphotograph the premises, pored over the books of the insured and conducted an extensiveinvestigation. The plaintiff having submitted thecorresponding fire claims, totalling P398,562.81 (butreduced to the full amount of the insurance,P370,000), the Insurance Company resisted payment, claiming violation of warranties andconditions, filing of fraudulent claims, and that thefire had been deliberately caused by the insured orby other persons in connivance with him. - Que Chee Gan, with his brother, Qua Chee Pao, andsome employees of his, were indicted and tried in1940 for the crime of arson, it being claimed thatthey had set fire to the destroyed warehouses tocollect the insurance. They were, however, acquittedby the trial court. - the civil suit to collect the insurance moneyproceeded to its trial with the CFI holding that:judgment is rendered for the plaintiff and against thedefendant condemning the latter to pay the former (a) Under the first cause of action, the sum ofP146,394.48; (b) Under the second cause of action,the sum of P150,000; (c) Under the third cause ofaction, the sum of P5,000; (d) Under the fourthcause of action, the sum of P15,000; and (e) Underthe fifth cause of action, the sum of P40,000; all ofwhich shall bear interest at the rate of 8% perannum in accordance with Section 91 (b) of theInsurance Act from September 26, 1940, until eachis paid, with costs against the defendant. - In its first assignment of error, the insurancecompany alleges that the trial Court should have heldthat the policies were avoided for breach of warranty,specifically the one appearing on a rider pasted (withother similar riders) on the face of the policies.4 - It is argued that since the bodegas insured had an external wall perimeter of 500 meters or 1,640 feet,the appellee should have 11 fire hydrants in thecompound, and that he actually had only 2, with afurther pair nearby, belonging to the municipality ofTabaco. ISSUES 1. WON the defendant-appellant can claim the policies it had issued as void ab initio 2. WON the insured violated the "Hemp Warranty"provisions of Policy No. 2637165 against the storageof gasoline 3. WON the insured connived at the loss andfraudulently inflated the quantity of the insured stockin the burnt bodegas HELD 1. NO Ratio It is usually held that where the insurer, at the time of the issuance of a policy of insurance, hasknowledge of existing facts which, if insisted on,would invalidate the contract from its very inception,such knowledge constitutes a waiver of conditions inthe contract inconsistent with the facts, and theinsurer is stopped thereafter from asserting thebreach of such conditions. The law is charitableenough to assume, in the absence of any showing tothe contrary, that an insurance company intends toexecuted a valid contract in return for the premiumreceived; and when the policy contains a conditionwhich renders it voidable at its inception, and thisresult is known to the insurer, it will be presumed tohave intended to waive the

conditions and to executea binding contract, rather than to have deceived theinsured into thinking he is insured when in fact he isnot, and to have taken his money withoutconsideration. Reasoning - The appellant is barred estoppel to claim violation of the so-called fire hydrants warranty, for the reasonthat knowing fully all that the number of hydrantsdemanded therein never existed from the verybeginning, the appellant neverthless issued thepolicies in question subject to such warranty, andreceived the corresponding premiums. The insurancecompany was aware, even before the policies wereissued, that in the premises insured there were onlytwo fire hydrants installed by Qua Chee Gan and twoothers nearby, owned by the municipality of Tabaco,contrary to the requirements of the warranty inquestion - The plain, human justice of this doctrine is perfectly apparent. To allow a company toaccept one's money for a policy of insurancewhich it then knows to be void and of no effect,though it knows as it must, that the assuredbelieves it to be valid and binding, is socontrary to the dictates of honesty and fairdealing, and so closely related to positive fraud, as to the abhorrent to fair-minded men. Itwould be to allow the company to treat thepolicy as valid long enough to get the premiumon it, and leave it at liberty to repudiate it thenext moment. This cannot be deemed to be thereal intention of the parties. To hold that aliteral construction of the policy expressed thetrue intention of the company would be toindict it, for fraudulent purposes and designswhich we cannot believe it to be guilty of. - The appellant company so worded the policies thatwhile exacting the greater number of fire hydrantsand appliances, it kept the premium discount at theminimum of 2 1/2%, thereby giving the insurancecompany a double benefit. Such abnormal treatmentof the insured strongly points at an abuse of theinsurance company's selection of the words andterms of the contract, over which it had absolutecontrol. - Receipt of Premiums or Assessments after Cause for Forfeiture Other than Nonpayment. It is a well settled rule of law that an insurer which withknowledge of facts entitling it to treat a policy as nolonger in force, receives and accepts a premium onthe policy, estopped to take advantage of theforfeiture. It cannot treat the policy as void for thepurpose of defense to an action to recover for a lossthereafter occurring and at the same time treat it asvalid for the purpose of earning and collecting furtherpremiums. - Moreover, taking into account the well known rulethat ambiguities or obscurities must be strictlyinterpreted against the party that caused them, the"memo of warranty" invoked by appellant bars thelatter from questioning the existence of theappliances called for in the insured premises On the alleged violations of the plaintiffThe alleged violation of the warranty of 100 feet of firehose for every two hydrants, must be equallyrejected, since the appellant's argument thereon isbased on the assumption that the insured was boundto maintain no less than eleven hydrants, whichrequirement appellant is estopped from enforcing. - As to maintenance of a trained fire brigade of 20men, the record is preponderant that the same wasorganized, and drilled, from time to give, althoughnot maintained as a permanently separate unit,which the warranty did not require. 2. NO Ratio Here, again, by reason of the exclusive control of the insurance company over the terms and phraseology of the contract, the ambiguity must be held strictly against the insurer and liberally in favor of the insured, specially to avoid a forfeiture. Insurance is, in its nature, complex and difficultfor the layman to understand. Policies areprepared by experts who know and can anticipate the hearing and possiblecomplications of every contingency. So long asinsurance companies insist upon the use ofambiguous, intricate and technical provisions,which conceal rather than frankly disclose,their own intentions, the courts must, infairness to those who purchase insurance,construe every ambiguity in favor of theinsured. An insurer should not be allowed, bythe use of obscure phrases and exceptions, todefeat the very purpose for which the policywas procured.

Reasoning - Appellee admitted that there were 36 cans of gasoline in the building designed. It However,gasoline is not specifically mentioned among theprohibited articles listed in the so-called "hempwarranty." The cause relied upon by the insurerspeaks of "oils (animal and/or vegetable and/ormineral and/or their liquid products having a flashpoint below 300 Fahrenheit)", and is decidedlyambiguous and uncertain; for in ordinary parlance,"Oils" mean "lubricants" and not gasoline orkerosene. And how many insured, it may well bewondered, are in a position to understand ordetermine "flash point below 300 Fahrenheit. - If the company intended to rely upon a condition ofthat character, it ought to have been plainlyexpressed in the policy. - The contract of insurance is one of perfect goodfaith not for the insured alone, but equally so for theinsurer; in fact, it is mere so for the latter, since itsdominant bargaining position carries with it stricterresponsibility. - Another point that is in favor of the insured is thatthe gasoline kept in Bodega No. 2 was only incidentalto his business, being no more than a customary 2day's supply for the five or six motor vehicles usedfor transporting of the stored merchandise. "It is wellsettled that the keeping of inflammable oils on thepremises though prohibited by the policy does notvoid it if such keeping is incidental to the business." On the submission of books, voucbers, etc.T he charge that the insured failed or refused to submit tothe examiners of the insurer the books, vouchers,etc. demanded by them was found unsubstantiatedby the trial Court, and no reason has been shown to alter this finding. The insured gave the insuranceexaminer all the date he asked for, and the examinereven kept and photographed some of the examinedbooks in his possession. What does appear to havebeen rejected by the insured was the demand thathe should submit "alist ofall books,vouchers, receipts and other records", but the refusal of the insured in this instance was well justified, since thedemand for a list of all the vouchers (which were notin use by the insured) and receipts was positivelyunreasonable, considering that such listing wassuperfluous because the insurer was not deniedaccess to the records, that the volume of Qua CheeGan's business ran into millions, and that thedemand was made just after the fire wheneverything was in turmoil. That the representativesof the insurance company were able to secure all thedate they needed is proved by the fact that theadjuster Alexander Stewart was able to prepare hisown balance sheet that did not differ from thatsubmitted by the insured except for the valuation ofthe merchandise, as expressly found by the Court inthe criminal case for arson. 3. NO Ratio Both defenses are predicted on the assumption that the insured was in financialdifficulties and set the fire to defraud the insurancecompany, presumably in order to pay off thePhilippine National Bank, to which most of theinsured hemp and copra was pledged. Both defensesare fatally undermined by the established fact that,notwithstanding the insurer's refusal to pay the valueof the policies the extensive resources of the insuredenabled him to pay off the National Bank in a shorttime; and if he was able to do so, no motive appearsfor attempt to defraud the insurer. While theacquittal of the insured in the arson case is notres judicata on the present civil action, the insurer's evidence, to judge from the decision in the criminalcase, is practically identical in both cases and mustlead to the same result, since the proof to establishthe defense of connivance at the fire in order todefraud the insurer "cannot be materially lessconvincing than that required in order to convict theinsured of the crime of arson." - As to the defense that the burned bodegas couldnot possibly have contained the quantities of copraand hemp stated in the fire claims, the insurer's caserests almost exclusively on the estimates, inferencesand conclusions of its adjuster investigator whoexamined the premises during and after the fire. Histestimony, however, was based on inferences from the photographs and traces found after the fire, andmust yield to the contradictory testimony of thosewho actually saw the contents of the bodegas shortlybefore the fire, while inspecting them for themortgagee Bank. DispositionWe find no reversible error in the judgment appealed from, wherefore the same is hereby affirmed.

DEL ROSARIO V EQUITABLE INSURANCE & CASUALTY CO., INC 8 SCRA 343 PAREDES; June 29, 1963 NATURE Appeal from judgment of CFI Rizal FACTS - Francisco del Rosario was insured by EquitableInsurance and Casualty Co. Inc under PersonalAccidentPolicy no. 7136. The Companybound itselfto pay P1000to P3000 as indemnity for the deathof the insured. - Under the policy: Part I.Indemnity for Death If the insured sustains any bodily injurywhich is effected solely through violent,external, visible and accidental means,and whichshall result, independently ofall other causes and within sixty daysfrom the occurrence thereof, in the Deathof the Insured, the Company shall pay theamount set opposite such injury: Section 1. Injury sustained other thanthose specified below unless exceptedhereinafter P1000 Section 2. Injury sustained by thewrecking or disablement of a railroadpassenger car or street railway car in oron which the Insured is traveling as afarepaying passenger P1500 Section 3. Injury sustained by the burningof a church, theatre, public library ormunicipal administration building while the Insured is therein at the commencement of the fireP2000 Section 4. Injury sustained by thewreckingor disablement of a regularpassenger elevator car in which theInsured is being conveyed as a passenger(Elevator in mines exluded) P2500 Section 5. Injury sustainedby a stroke of lightning or by a cycloneP3000 xxxx xxxx xxxx Part VI. Exceptions This policy shall not cover disappearanceof the Insured nor shall it cover Death,Disability, Hospital fees, or Loss of time,caused to the insured: x x x (h) By drowning except as a consequence of the wrecking or disablement in the Philippine waters of a passenger steam or motor vessel inwhich the Insured is traveling as afarepaying passenger; x x x - A rider to the Policy contained the following; IV. DROWNING It is hereby declared and agreed thatexemption clause Letter (h) in PART VI ofthe policy is hereby waived by thecompany, and to form a part of theprovision covered by the policy. - Feb 24, 1957, Francisco del Rosario while on boardthe motor launch ISLAMA, with his beneficiary to thepolicy, Remedios Jayme, were forced to jump off saidlaunch on account of fire which broke out on saidvessel, resulting in the death by drowning of theinsured and his beneficiary. - Simeon del Rosario, the insureds father, filed aclaim for paymentwith the company. The companypaid him P1000 pursuant to section 1 Part I of thepolicy. - On the same date, Atty. Francisco wrote to thecompany acknowledging receipt by his client of theP1000 but informing said company that said amountwas not the correct one. He claimed that the

amountpayable should be P1500 under the provision ofSection 2 Part I, based on the rule of pari materia. - The company referred the matter to the InsuranceCoomissioner, who was of the opinion that theliability of the company was only P1000. thus thecompany refused to pay more that P1000. Atty.Francisco wrote a subsequent letter to companyasking for p3000, which the company refused to pay.- A complaint for recovery of the balance of P2000was instituted with the CFI Rizal, praying for afurther sum of P10000 as attorneys fees, expensesof litigation and costs. - CFI ruled in favor of petitioner, ordering the company to pay P2000 to del Rosario. ISSUE How much should the indemnity be HELD - All the parties agree that indemnity has to be paid, but the conflict centers on how much it should be. - Where there is ambiguity with respect to the termsand conditions of the policy, the same will beresolved against the one responsible thereof.Generally, the insured has little, if any, participationin the preparation of the policy, together with thedrafting of its terms and conditions. Theinterpretation of obscure stipulations in a contractshould not favor the party who caused the obscurity.- SC agreed with the ruling of the lower court: x x x death by drowning is a ground for recoveryapart from the bodily injury because death bybodily injury is covered by Part I of the policywhile death by drowning is covered by Part VIthereof. But while the policy mentions specificamounts that may be recovered for death forbodily injury, yet, there is not specific amountmentioned in the policy for death thru drowningalthough the latter is, under Part VI of the policy,a ground for recovery thereunder. Since thedefendant has bound itself to pay P1000 toP3000 as indemnity for the death of the insuredbut the policy does not positively state anydefinite amount that may be recovered in case ofdeath by drowning, there is an ambiguity in thisrespect in the policy, which ambiguity must beinterpreted in favor of the insured and strictlyagainst the insurer so as to allow a greaterindemnity. x x xplaintiff is therefore entitled torecover P3000. DispositionJudgment appealed from is affirmed.

GEAGONIA v. CA (COUNTRY BANKERS INSURANCE) 8 SCRA 343 DAVIDE; February 6 1995 FACTS -Geagonia is the owner of Norman's Mart located inthe public market of San Francisco, Agusan del Sur.On 22 Dec 1989, he obtained from the privaterespondent fire insurance policy for P100,000.00.The period of the policy was from 22 Dec 1989 to 22Dec 1990 and covered the ff: "Stock-in-tradeconsisting principally of dry goods such as RTW's formen and women wear and other usual to assured'sbusiness. -The policy contained the following condition: "3.The insured shall give notice to the Company ofany insurance or insurances already effected, orwhich may subsequently be effected, covering any ofthe property or properties consisting of stocks intrade, goods in process and/or inventories onlyhereby insured, and unless notice be given and theparticulars of such insurance or insurances be statedtherein or endorsed in this policy pursuant to Section50 of the Insurance Code, by or on behalf of theCompany before the occurrence of any loss ordamage, all benefits under this policy shall bedeemed forfeited, provided however, that thiscondition shall not apply when the total insurance orinsurances in force at the time of the loss or damageis not more than P200,000.00." -On 27 May 1990, fire of accidental origin broke outat around 7:30 p.m. at the public market of SanFrancisco, Agusan del Sur. The petitioner's insuredstocks-in-trade were completely destroyed promptinghim to file w/ the private respondent a claim underthe policy. On 28 Dec 1990, the private respondentdenied the claim because it found that at the time ofthe loss the petitioner's stocks-in-trade were likewisecovered by two fire insurance policies forP100,000.00 each, issued by the Cebu Branch of thePhilippines First Insurance Co., Inc. (PFIC). -The basis of the private respondent's denial was thepetitioner's alleged violation of Condition 3 of the policy. - Geagonia then filed a complaint against the privaterespondent w/ the Insurance Commission for therecovery of P100,000.00 under fire insurance policy,for attorney's fees, and costs of litigation. He claimsthat the time he obtained the private respondent'sfire insurance policy he knew that the two policiesissued by the PFIC were already in existence;however, he had no knowledge of the provision in theprivate respondent's policy requiring him to inform itof the prior policies; this requirement was notmentioned to him by the private respondent's agent;and had it been so mentioned, he would not havewithheld such information. He further asserted thatthe total of the amounts claimed under the threepolicies was below the actual value of his stocks atthe time of loss, w/c was P1M. - The Insurance Commission found that thepetitioner did not violate Condition 3 as he had noknowledge of the existence of the two fire insurancepolicies obtained from the PFIC; that it was CebuTesing Textiles w/c procured the PFIC policies w/oinforming him or securing his consent; and that CebuTesing Textile, as his creditor, had insurable interest on the stocks. These findings were based on thepetitioner's testimony that he came to know of thePFIC policies only when he filed his claim with theprivate respondent and that Cebu Tesing Textileobtained them and paid for their premiums w/oinforming him. The Insurance Commission thenordered the respondent company to pay complainantthe sum of P100,000.00 with legal interest from thetime the complaint was filed until fully satisfied plusthe amount of P10,000.00 as attorney's fees. -CA reversed the decision of the InsuranceCommission because it found that the petitionerknew of the existence of the two other policies issuedby the PFIC ISSUES 1. WON the petitioner had prior knowledge of thetwo insurance policies issued by the PFIC when heobtained the fire insurance policy from the privaterespondent, thereby, for not disclosing such fact,violating Condition 3 of the policy 2.if he had, WON he is precluded from recovering therefrom HELD 1. YES

- We agree w/ the CA that the petitioner knew of theprior policies issued by the PFIC. His letter of 18January 1991 to the private respondent conclusivelyproves this knowledge. His testimony to the contrarybefore the Insurance Commissioner and which thelatter relied upon cannot prevail over a writtenadmission made ante litem motam. It was, indeed,incredible that he did not know about the priorpolicies since these policies were not new or original.2. NO - It must, however, be underscored thatunlike the"other insurance" clauses involved in GeneralInsurance and Surety Corp. vs. Ng Hua or inPioneer Insurance & Surety Corp. vs. Yap,whichread: "The insured shall give notice to the company of anyinsurance or insurances already effected, or whichmay subsequently be effected covering any of theproperty hereby insured, and unless such notice begiven and the particulars of such insurance orinsurances be stated in or endorsed on this Policy byor on behalf of the Company before the occurrenceof any loss or damage, all benefits under this Policyshall be forfeited." or in the 1930 case of Santa Anavs. Commercial Union Assurance Co. which provided "that any outstanding insurance upon the whole or a portion of the objects thereby assuredmust be declared by the insured in writing and hemust cause the company to add or insert it in thepolicy, without which such policy shall be null andvoid, and the insured will not be entitled toindemnity in case of loss," Condition 3 in the private respondent's policy No. F-14622 doesnot absolutely declare void any violationthereof. It expressly provides that thecondition "shall not apply when the totalinsurance or insurances in force at the time ofthe loss or damage is not more thanP200,000.00." - Interpretation: It is a cardinal rule on insurance that a policy or insurance contract is to beinterpreted liberally in favor of the insured andstrictly against the company, the reason being,undoubtedly, to afford the greatest protection whichthe insured was endeavoring to secure when heapplied for insurance. It is also a cardinal principle oflaw that forfeitures are not favored and that anyconstruction which would result in the forfeiture ofthe policy benefits for the person claimingthereunder, will be avoided, if it is possible toconstrue the policy in a manner which would permitrecovery, as, for example, by finding a waiver forsuch forfeiture. Stated differently, provisions, conditions or exceptions in policies which tendto work a forfeiture of insurance policies shouldbe construed most strictly against those forwhose benefits they are inserted, and mostfavorably toward those against whom they areintended to operate.The reason for this is that, except for riders which may later be inserted, theinsured sees the contract already in its final form andhas had no voice in the selection or arrangement ofthe words employed therein. On the other hand, thelanguage of the contract was carefully chosen anddeliberated upon by experts and legal advisers whohad acted exclusively in the interest of the insurersand the technical language employed therein israrely understood by ordinary laymen. - With these principles in mind, we are of the opinion that Condition 3 of the subject policy isnot totally free from ambiguity and must bemeticulously analyzed. Such analysis leads usto conclude that (a) the prohibition applies onlyto double insurance, and (b) the nullity of thepolicy shall only be to the extent exceedingP200,000.00 of the total policies obtained. - Furthermore, by stating within Condition 3 itself that such condition shall not apply if the total insurance in force at the time of loss does not exceedP200,000.00, the private respondent was amenableto assume a co-insurer's liability up to a loss notexceeding P200,000.00. What it had in mind was todiscourage over-insurance. Indeed, the rationalebehind the incorporation of "other insurance" clausein fire policies is to prevent over-insurance and thusavert the perpetration of fraud. When a propertyowner obtains insurance policies from two or moreinsurers in a total amount that exceeds theproperty's value, the insured may have aninducement to destroy the property for the purposeof collecting the insurance. The public as well as theinsurer is interested in preventing a situation inwhich a fire would be profitable to the insured. Disposition Petition granted. The decision of the Court of Appeals in CA-G.R. SP No. 31916 is SETASIDE and the decision of the Insurance Commissionin Case No. 3340 is REINSTATED.

SUN INSURANCE OFFICE, LTD. V CA (LIM) 211 SCRA 554 CRUZ; July 17, 1992 NATURE Petition for review from the decision of the Court of Appeals FACTS - Felix Lim was issued a Personal Accident Policyinsurance with petitioner company with a face valueof P200,000.His beneficiary was his wife Nerissa. - October 6, 1982 Felix accidentally shot himself in - He was playing with the handgun after he had removed the guns magazine (kasi naman). - He pointed the gun at his secretary and onlywitness Pilar Nalagon as a joke and assured herthat the gun was not loaded (are you sure). - He then put the gun to his temple and fired it (haaay, sabi ko na nga ba). - Both parties are in agreement that there was no suicide. - Nerissa claimed as Felixs beneficiary but SunInsurance would not grant her claim, saying that herhusbands death was not an accident. - Nerissa sued Sun Insurance and won the case.SunInsurance was ordered to pay her P200,000representing the face value of the claim along withmoral, exemplary and compensatory damages andattorneys fees.The decision was affirmed by the CA. Petitioners Claim - Sun Insurance cites one of the four exceptions inthe contract of insurance which includes bodily injuryconsequent upon the insured person attempting tocommit suicide or willfully exposing himself toneedless peril in an attempt to save a human life. - There mere act of pointing the gun to his templeshowed that Felix willfully exposed himself to dangerbecause a gun should always be handled with caution. Respondents Comments - Felix believed the gun to be safe because he had removed the magazine. - He repeatedly assured his secretary that the gun was not loaded. ISSUES 1. WON Felix Lims death was an accident, thusmaking his widow Nerissa liable to claim the accidentinsurance 2. WON the award of damages to Nerissa Lim was justified HELD 1. YES, Felix Lims death was an accident. Ratio There is no accident when a deliberate act is performed unless some additional, unexpected,independent and unforeseen happening occurs whichproduces or brings bout their injury or death. Reasoning - An accident has been defined to be that whichhappens by chance or fortuitously without intentionor design and which is unexpected, unusual andunforeseen.It an event that takes pace wit houtones foresight or expectastion an event thatproceeds from an unknown cause or is an unusualeffect of a known case and therefore not expected.It happens without any human agency, an eventwhich, under the circumstances, is unusual to andnot expected by the person to whom it happens. - The firing of the gun was deemed to be theunexpected and independent and unforeseenoccurrence that led to the insured persons death. - There was no willful exposure to needless peril forthe part of Felix.Suicide and exposure to needlessperil are similar in the sense that both signifydisregard for ones life.Suicide imparts a positiveact of ending ones life whereas the latter indicatesrecklessness that is almost suicidal in intent. - Accident insurance policies were never meant to reward the insured for his tendency to show off or for his miscalculations.They were intended to provide for contingencies.

- Lim was unquestionably negligent but it should notprevent his widow from recovering from theinsurance policy he obtained precisely againstaccident. - Insurance contracts are, as a rule, supposed to be interpreted liberally in favor of the assured. 2. NO, the claim for damages should not be granted for being unjust. Ratio A person may be made liable to the payment of moral damages if his act is wrongful.The adverseresult of an action does not per se make the actwrongful and subject the act or to the payment ofmoral damages. Reasoning - Petitioner was acting in good faith when it resistedthe private respondents claim on the ground that thedeath of the insured was covered by the exception. - The issue was debatable and was clearly not raisedonly for the purpose of evading a legitimate obligation.

RIZAL SURETY & INSURANCE COMPANY V CA (TRANSWORLD KNITTING MILLS, INC.) 336 SCRA 12 PURISIMA; July 18, 2000 NATURE Petition for Review onCertiorari under Rule 45 of the Rules of Court FACTS - Rizal Surety & Insurance Company (RizalInsurance) issued Fire Insurance Policy No. 45727 infavor of Transworld Knitting Mills, Inc. (Transworld).- Pertinent portions of subject policy on the buildingsinsured, and location thereof, read: "On stocks of finished and/or unfinishedproducts, raw materials and supplies of everykind and description, the properties of theInsureds and/or held by them in trust, oncommission or on joint account with othersand/or for which they (sic) responsible in caseof loss whilst contained and/or stored duringthe currency of this Policy in the premisesoccupied by them forming part of the buildingssituate (sic) within own Compound atMAGDALO STREET, BARRIO UGONG, PASIG,METRO MANILA, PHILIPPINES, BLOCK NO. 601. x xx............ ...xxx....... ........xxx Said building of four-span lofty one storey inheight with mezzanine portions is constructedof reinforced concrete and hollow blocksand/or concrete under galvanized iron roofand occupied as hosiery mills, garment andlingerie factory, transistor-stereo assemblyplant, offices, warehouse and caretaker'squarters. 'Bounds in front partly by one-storey concretebuilding under galvanized iron roof occupiedas canteen and guardhouse, partly by buildingof two and partly one storey constructed ofconcrete below, timber above undergalvanizediron roof occupied as garage and quarters andpartly by open space and/or tracking/packing, beyond which is the aforementionedMagdalo Street; on its right and left bydriveway, thence open spaces, and at the rearby open spaces.'" - The same pieces of property insured with thepetitioner were also insured with New IndiaAssurance Company, Ltd., (New India). - Fire broke out in the compound of Transworld,razing the middle portion of its four-span buildingand partly gutting the left and right sectionsthereof. A two-storey building (behind said four-span building) where fun and amusementmachines and spare parts were stored, was alsodestroyed by the fire. - Transworld filed its insurance claims with RizalSurety & Insurance Company and New IndiaAssurance Company but to no avail. - Private respondent brought against the saidinsurance companies an action for collection ofsum of money and damages. - Petitioner Rizal Insurance countered that its fireinsurance policy sued upon covered only thecontents of the four-span building, which waspartly burned, and not the damage caused by thefire on the two-storey annex building. - The trial court dismissed the case as against TheNew India Assurance Co., Ltd. but ordereddefendant Rizal Surety And Insurance Company topay Transwrold (sic) Knitting Mills, Inc. - Both the petitioner, Rizal Insurance Company,and private respondent, Transworld Knitting Mills,Inc., went to the Court of Appeals, which requiredNew India Assurance Company to pay plaintiffappellant the amount of P1,818,604.19 while theRizal Surety has to pay the plaintiffappellantP470,328.67. - New India appealed to the Court theorizinginter alia that the private respondent could not be compensated for the loss of the fun andamusement machines and spare parts stored atthe twostorey building because it (Transworld)had no insurable interest in said goods or items. - The Court denied the appeal with finality. - Petitioner Rizal Insurance and private respondent Transworld, interposed a Motion forReconsideration before the Court of Appeals, whichreconsidered its decision of July 15, 1993, asregards the imposition of interest. - Undaunted, petitioner Rizal Surety & Insurance

Company found its way to the Court. ISSUE WON the fire insurance policy litigated uponprotected only the contents of the main building(fourspan), and did not include those stored in thetwo-storey annex building HELD NO - Resolution of the issue posited hinges on the properinterpretation of the stipulation in subject fireinsurance policy regarding its coverage, which reads:"xxx contained and/or stored during the currency ofthis Policy in the premises occupied by them formingpart of the buildings situate (sic) within ownCompound xxx" - It can be gleaned unerringly that the fire insurancepolicy in question did not limit its coverage to whatwere stored in the four-span building. As opined bythe trial court of origin, two requirements mustconcur in order that the said fun and amusementmachines and spare parts would be deemedprotected by the fire insurance policy under scrutiny,to wit: "First, said properties must be contained and/orstored in the areas occupied by Transworld andsecond, said areas must form part of the buildingdescribed in the policy xxx" - Said building of four-span lofty one storey in heightwith mezzanine portions is constructed of reinforcedconcrete and hollow blocks and/or concrete undergalvanized iron roof and occupied as hosiery mills,garment and lingerie factory, transistor-stereoassembly plant, offices, ware house and caretaker'squarter. - The Court is mindful of the well-entrencheddoctrine that factual findings by the Court of Appealsare conclusive on the parties and not reviewable bythis Court, and the same carry even more weightwhen the Court of Appeals has affirmed the findingsof fact arrived at by the lower court. - In the case under consideration, both the trial courtand the Court of Appeals found that the so called"annex " was not an annex building but an integraland inseparable part of the four-span buildingdescribed in the policy and consequently, themachines and spare parts stored therein werecovered by the fire insurance in dispute. - Verily, the two-storey building involved, a permanent structure which adjoins andintercommunicates with the "first right span of thelofty storey building", formed part thereof, andmeets the requisites for compensability under thefire insurance policy sued upon. - So also, considering that the two-storey buildingaforementioned was already existing when subjectfire insurance policy contract was entered into,petitioner should have specifically excluded the saidtwo-storey building from the coverage of the fireinsurance if minded to exclude the same but if didnot, and instead, went on to provide that such fireinsurance policy covers the products, raw materialsand supplies stored within the premises ofrespondent Transworld which was an integral part ofthe four-span building occupied by Transworld,knowing fully well the existence of such buildingadjoining and intercommunicating with the rightsection of the four-span building. - Indeed, the stipulation as to the coverage of thefire insurance policy under controversy has created adoubt regarding the portions of the building insuredthereby. Article 1377 of the New Civil Code provides: "Art.1377. The interpretation of obscure wordsor stipulations in a contract shall not favor theparty who caused the obscurity" - Conformably, it stands to reason that the doubtshould be resolved against the petitioner, RizalSurety Insurance Company, whose lawyer ormanagers drafted the fire insurance policy contractunder scrutiny. Citing the aforecited provision of lawin point, the Court in Landicho vs. Government Service Insurance System, ruled: "This is particularly true as regards insurancepolicies, in respect of which it is settled that the'terms in an insurance policy, which areambiguous, equivocal, or uncertain x x x are tobe construed strictly and most strongly against the insurer, and liberally in favor of the insuredso as to effect the dominant purpose ofindemnity or payment to the insured, especiallywhere forfeiture is involved' and the reason forthis is that the 'insured usually has no voice inthe selection or arrangement of the wordsemployed and that the language of the contractis selected with great care and deliberation byexperts and legal advisers employed by, andacting exclusively in the interest of, theinsurance company.' "

- Equally relevant is the following disquisition of the Court in Fieldmen's Insurance Company, Inc. vs. Vda. De Songco, to wit: "'This rigid application of the rule on ambiguitieshas become necessary in view of currentbusiness practices. The courts cannot ignorethat nowadays monopolies, cartels andconcentration of capital, endowed withoverwhelming economic power, manage toimpose upon parties dealing with themcunningly prepared 'agreements' that theweaker party may not change one whit, hisparticipation in the 'agreement' being reduced tothe alternative to 'take it or leave it' labelled since Raymond Saleilles 'contracts byadherence' (contrats [sic] d'adhesion), incontrast to these entered into by partiesbargaining on an equal footing, such contracts(of which policies of insurance and internationalbills of lading are prime example) obviously callfor greater strictness and vigilance on the partof courts of justice with a view to protecting theweaker party from abuses and imposition, andprevent their becoming traps for the unwary.'" - The issue of whether or not Transworld has aninsurable interest in the fun and amusementmachines and spare parts, which entitles it to beindemnified for the loss thereof, had been settled inG.R. No. L111118, entitled New India Assurance Company, Ltd., vs. Court of Appeals, where the appeal of New India from the decision of the Court ofAppeals under review, was denied with finality bythis Court on February 2, 1994. - The rule on conclusiveness of judgment, whichobtains under the premises, precludes the relitigationof a particular fact or issue in another action betweenthe same parties based on a different claim or causeof action. "xxx the judgment in the prior actionoperates as estoppel only as to those matters inissue or points controverted, upon the determinationof which the finding or judgment was rendered. In fine, the previous judgment is conclusive in thesecond case, only as those matters actually anddirectly controverted and determined and not as tomatters merely involved therein." DispositionDecision, and the Resolution of the CA WERE AFFIRMED intoto. No pronouncement as to costs.

PAN MALAYAN INSURANCE CORPORATIONvs.COURT OF APPEALS (ERLINDA FABIE & HERUNKNOWN DRIVER) 184 SCRA 55; G.R. No. 81026 CORTES; April 3, 1990 FACTS - December 10, 1985: PANMALAY filed a complaintfor damages with the RTC of Makati against privaterespondents Erlinda Fabie and her driver. PANMALAYaverred the following: that it insured a MitsubishiColt Lancer car registered in the name of CanlubangAutomotive Resources Corporation [CANLUBANG];that on May 26, 1985, due to the "carelessness,recklessness, and imprudence" of the unknowndriver of a pick-up, the insured car was hit andsuffered damages in the amount of P42,052.00; thatPANMALAY defrayed the cost of repair of the insuredcar and, therefore, was subrogated to the rights ofCANLUBANG against the driver of the pick-up and hisemployer, Erlinda Fabie; and that, despite repeateddemands, defendants, failed and refused to pay theclaim of PANMALAY. PANMALAY clarified that thedamage caused to the insured car was settled underthe "own damage", coverage of the insurance policy.- Private respondents filed a Motion to Dismissalleging that PANMALAY had no cause of actionagainst them. RTC dismissed PANMALAY's complaintfor no cause of action and denied PANMALAY'smotion for reconsideration. CA affirmed. Hence, thispetition for review. ISSUE WON the insurer PANMALAY may institute an actionto recover the amount it had paid its assured insettlement of an insurance claim against privaterespondents as the parties allegedly responsible forthe damage caused to the insured vehicle HELD YES - Article 2207 of the Civil Code is founded on thewell-settled principle of subrogation. If the insuredproperty is destroyed or damaged through the fault or negligence of a party other than the assured, thenthe insurer, upon payment to the assured, will besubrogated to the rights of the assured to recoverfrom the wrongdoer to the extent that the insurerhas been obligated to pay. - General Rule: Payment by the insurer to theassured operates as an equitable assignment to theformer of all remedies which the latter may haveagainst the third party whose negligence or wrongfulact caused the loss. The right of subrogation is notdependent upon, nor does it grow out of, any privityof contract or upon written assignment of claim. Itaccrues simply upon payment of the insurance claimby the insurer. - Exceptions: a.) if the assured by his own act releases thewrongdoer or third party liable for the loss ordamage, from liability b.) where the insurer pays the assured the value ofthe lost goods without notifying the carrier who hasin good faith settled the assured's claim for loss c.) where the insurer pays the assured for a losswhich is not a risk covered by the policy, therebyeffecting "voluntary payment" - None of the exceptions are availing in the present case. - When PANMALAY utilized the phrase "own damage" a phrase which, incidentally, is not found in theinsurance policy to define the basis for itssettlement of CANLUBANG's claim under the policy, itsimply meant that it had assumed to reimburse thecosts for repairing the damage to the insured vehicle. - It is a basic rule in the interpretation of contractsthat the terms of a contract are to be construedaccording to the sense and meaning of the termswhich the parties thereto have used. In the case ofproperty insurance policies, the evident intention ofthe contracting parties, i . e., the insurer and theassured, determine the import of the various termsand provisions embodied in the policy. It is onlywhen the terms of the policy are ambiguous,equivocal or uncertain, such that the partiesthemselves disagree about the meaning of particularprovisions, that the courts will intervene. In such anevent, the policy will be construed by the courtsliberally in favor of the assured and strictly againstthe insurer. - Considering that the very parties to the policy werenot shown to be in disagreement regarding themeaning and coverage of Section III-1, specificallysub-paragraph (a) thereof, it was improper for the appellate court to indulge in contract construction, toapply the ejusdem generis rule, and to ascribemeaning contrary to the clear intention andunderstanding of these parties.

- Although the terms "accident" or "accidental" asused in insurance contracts have not acquired atechnical meaning, the Court has on severaloccasions defined these terms to mean that whichtakes place "without one's foresight or expectation,an event that proceeds from an unknown cause, or isan unusual effect of a known cause and, therefore,not expected." The concept "accident" is notnecessarily synonymous with the concept of "nofault". It may be utilized simply to distinguishintentional or malicious acts from negligent orcareless acts of man. - Obiter Dicta: Even if under the abovecircumstances PANMALAY could not be deemedsubrogated to the rights of its assured under Article2207 of the Civil Code, PANMALAY would still have acause of action against private respondents. Theinsurer who may have no rights of subrogation dueto "voluntary" payment may nevertheless recoverfrom the third party responsible for the damage tothe insured property under Article 1236 of the CivilCode. Disposition Petition is GRANTED. Petitioner's complaint for damages against private respondents isREINSTATED. Case remanded to the lower court fortrial on the merits.

AMERICAN HOME ASSURANCE COMPANY V TANTUCO ENTERPRISES 366 SCRA 740 PUNO; October 8, 2001 NATURE Petition for Review on Certiorari assailing the Decision of the Court of Appeals. FACTS - Respondent Tantuco Enterprises, Inc. is engaged inthe coconut oil milling and refining industry.It ownstwo oil mills. Both are located at its factory compound at Iyam, Lucena City. Respondentcommenced its business operations with only one oilmill.In 1988, it started operating its second oil mill( the new oil mill). - The two oil mills were separately covered by fireinsurance policies issued by petitioner AmericanHome Assurance Co. The first oil mill was insuredPolicy No. 306-7432324-3 for the period March 1, 1991 to 1992.The new oil mill was insured underPolicy No. 306-7432321-9 for the same term. Officialreceipts indicating payment for the full amount of thepremium were issued by the petitioner's agent. - Policy description: Front:by a driveway thence at 18 meters distance by Bldg. No. 2. Right:by an open space thence by Bldg. No. 4. Left: Adjoining thence an imperfect wall by Bldg. No. 4. Rear: by an open space thence at 8 meters distance. - A fire that broke out in the early morning ofSeptember 30,1991 gutted and consumed the newoil mill.Petitioner rejected respondents claim forthe insurance proceeds on the ground that no policywas issued by it covering the burned oil mill.Itstated that the description of the insuredestablishment referred to another building. Petitioners Claim The policies referred to the old mill, as stated in the description contained in the policy. ISSUE WON new oil mill is insured by fire insurance policy HELD YES, new oil mill is insured. Ratio In construing the words used descriptive of a building insured, the greatest liberality is shown bythe courts in giving effect to the insurance. In viewof the custom of insurance agents to examinebuildings before writing policies upon them, andsince a mistake as to the identity and character ofthe building is extremely unlikely, the courts areinclined to consider that the policy of insurancecovers any building which the parties manifestlyintended to insure, however inaccurate thedescription may be. Reasoning - The parties manifestly intended to insure the newoil mill. On machineries and equipment withcomplete accessories usual to a coconut oil millincluding stocks of copra, copra cake and copra millswhilst contained in the new oil mill building, situate(sic) at UNNO. ALONG NATIONAL HIGH WAY, BO.IYAM, LUCENA CITY UNBLOCKED. - If the parties really intended to protect the first oilmill, then there is no need to specify it as new. Itwould be absurd to assume that respondent wouldprotect its first oil mill for different amounts andleave uncovered its second one. - As may be gleaned from the testimony of thepetitioners employee, the source of the discrepancyhappened during the preparation of the writtencontract. - Respondent is not estopped from claiming that thepolicy description is wrong. Evidence on recordreveals that respondents operating manager, Mr.Edison Tantuco, notified the petitioners agent withwhom respondent negotiated for the contract aboutthe inaccurate description in the

policy.However, Mr.Borja assured Mr. Tantuco that the use of theadjective new will distinguish the insured property. Regarding policy requirements that fireextinguishment appliances should be available and ingood working condition, warranty did not requirerespondent to provide for all the fire extinguishingappliances enumerated therein. Neither did itrequire that the appliances are restricted to thosementioned in the warranty. (Within the vicinity of thenew oil mill can be found the following devices:numerous portable fire extinguishers, two fire hoses,fire hydrant, and an emergency fire engine.) - The object of the court in construing a contract isto ascertain the intent of the parties to the contractand to enforce the agreement which the parties haveentered into. In determining what the partiesintended, the courts will read and construe the policyas a whole and if possible, give effect to all the partsof the contract. DispositionPetition is dismissed.

PERLA COMPANIA DE SEGUROS, INC. v, CA (MILAGROS CAYAS) 185 SCRA 741 FERNAN; May 28, 1990 NATURE Petition for review oncertiorari of a decision of the Court of Appeals FACTS - Private respondent Milagros Cayas was theregistered owner of a Mazda bus, insured with PerlaCompania de Seguros, Inc. (PCSI) under a policyissued on February 3, 1978. - On December 17, 1978, the bus figured in anaccident in Naic, Cavite injuring several of itspassengers. - One of them, 19-year old Edgardo Perea, suedMilagros Cayas for damages in the CFI of Cavite,while three others agreed to a settlement ofP4,000.00 each. - After trial, the court rendered a decision in favor ofPerea, ordering Cayas to compensate him, with anaward of exemplary and moral damages, as well asattorneys fees. ( P32,000 total) - On November 11, 1981, Milagros Cayas filed acomplaint for a sum of money and damages againstPCSI in the Court of First Instance of Cavite. - In view of Milagros Cayas' failure to prosecute thecase, the court motu propio ordered its dismissalwithout prejudice. - Alleging that she had not received a copy of theanswer to the complaint, and that "out ofsportsmanship", she did not file a motion to holdPCSI in default, Milagros Cayas moved for thereconsideration of the dismissal order. Said motionfor reconsideration was acted upon favorably by thecourt. - About two months later, Milagros Cayas filed amotion to declare PCSI in default for its failure to filean answer. - The motion was granted and plaintiff was allowed to adduce evidenceex-parte. - On July 13, 1982, the court rendered judgment bydefault ordering PCSI to pay Milagros Cayas P50,000as compensation for the injured passengers, P5,000as moral damages and P5,000 as attorney's fees. - Said decision was set aside after the PCSI filed a motion therefor. Trial of the case ensued. - In due course, the court promulgated a decisionordering defendant Perla Compania de Seguros, Inc.to pay plaintiff Milagros Cayas the sum of P50,000.00under its maximum liability as provided for in theinsurance policy; and the sum of P5,000.00 asreasonable attorney's fee - PCSI appealed to the Court of Appeals, which affirmed in toto the lower court's decision. - Its motion for reconsideration having been denied by said appellate court, PCSI filed this petition ISSUE WON, as maintained by petitioner, its liability islimited only to the payment made by privaterespondent to Perea and only up to the amount ofP12,000.00 HELD YES - The insurance policy involved explicitly limits petitioner's liability to P12,000.00 per person and to P50,000.00 per accident. - In Stokes vs. Malayan Insurance Co., Inc., the Court held that the terms of the contract constitute the measure of the insurer's liability and compliancetherewith is a condition precedent to the insured'sright of recovery from the insurer. - In the case at bar, the insurance policy clearly andcategorically placed petitioner's liability for alldamages arising out of death or bodily injurysustained by one person as a result of any oneaccident at P12,000.00. - Said amount complied with the minimum fixed bythe law then prevailing, Section 377 of PresidentialDecree No. 612, which provided that the liability of land transportation vehicle operators for bodilyinjuries sustained by a passenger arising out of theuse of their vehicles shall not be less than P12,000.- In other words, under the law, theminimumliability is P12,000 per passenger. Petitioner's liabilityunder the insurance contract not being less

thanP12,000.00, and therefore not contrary to law,morals, good customs, public order or public policy,said stipulation must be upheld as effective, validand binding as between the parties. - In like manner, we rule as valid and binding uponprivate respondent the condition requiring her tosecure the written permission of petitioner beforeeffecting any payment in settlement of any claimagainst her. - There is nothing unreasonable, arbitrary orobjectionable in this stipulation as would warrant itsnullification. The same was obviously designed tosafeguard the insurer's interest against collusionbetween the insured and the claimants. - It being specifically required that petitioner'swritten consent be first secured before any paymentin settlement of any claim could be made, privaterespondent is precluded from seeking reimbursementof the payments made to the three other passangersin view of her failure to comply with the conditioncontained in the insurance policy. - Clearly, the fundamental principle that contractsare respected as the law between the contractingparties finds application in the present case. - It was error on the part of the trial and appellatecourts to have disregarded the stipulations of theparties and to have substituted their owninterpretation of the insurance policy. - In Phil. American General Insurance Co., Inc vs. Mutuc, we ruled that contracts which are the private laws of the contracting parties should be fulfilledaccording to the literal sense of their stipulations, iftheir terms are clear and leave no room for doubt asto the intention of the contracting parties, forcontracts are obligatory, no matter what form they may be, whenever the essential requisites for their validity are present. - In Pacific Oxygen & Acetylene Co. vs. Central Bank," it was stated that the first and fundamental duty of the courts is the application of the lawaccording to its express terms, interpretation beingcalled for only when such literal application isimpossible. - We observe that although Milagros Cayas was ableto prove a total loss of only P44,000.00, petitionerwas made liable for the amount of P50,000.00, themaximum liability per accident stipulated in thepolicy. This is patent error. An insurance indemnity,being merely an assistance or restitution insofar ascan be fairly ascertained, cannot be availed of by anyaccident victim or claimant as an instrument ofenrichment by reason of an accident. DispositionPetition granted. The decision of the Court of Appeals is modified in that petitioner shallpay Milagros Cayas the amount of Twelve ThousandPesos (P12,000. 00) plus legal interest from thepromulgation of the decision of the lower court untilit is fully paid and attorney's fees in the amount ofP5,000.00.

POLTAN v. BPI & JOHN DOE G.R. No. 164307 CHICO-NAZARIO; March 5, 2007 NATURE Petition for review of CA decision FACTS - Petitioners POLTAN obtained a loan evidenced by apromissory note from the MANTRADE Devt Corp.This was secured by a chattel mortgage over a 1-unitNissan Sentra vehicle. - With notice to petitioners, MANTRADE assigned toBPI, by way of a Deed of Assignment, all its rights,title and interest to the promissory note and chattelmortgage. - Petitioners defaulted and so BPI demanded the whole balance of P286.5T including accrued interest, or to return to BPI the possession of the motor vehicle for foreclosure. It is specifically provided inthe promissory note and chattel mortgage thatfailure to pay any installment when due shall makesubsequent installments and the entire balance ofthe obligation due and payable. - After they refused to do so, BPI then filed complaint. - Petitioners claimed that BPI required them toobtain a motor vehicle insurance policy from FGUInsurance Corporation (FGU Insurance). This is asister company of BPI. They had been paying themonthly installments on the vehicle until it figured inan accident where it became a total wreck. Under theterms of the insurance policy from FGU Insurance,the vehicle had to be replaced or its value paid tothem. Due to the failure and refusal of FGUInsurance to replace the vehicle or pay its value,they stopped payment of the monthly installments. - RTC ordered POLTANS to pay BPI the said amount. - CA reversed and remanded case to RTC for trial on the merits. - RTC again ruled in favor of BPI. CA affirmed. ISSUES 1. WON contracts presented in evidence by BPI were unjust and unacceptable contracts of adhesion 2. WON the terms and conditions of thecomprehensive car insurance policy issued by FGUshould be deemed as having automatically operatedin favor of BPI as the assured mortgagee, and if so,it should be deemed as resulting in theextinguishment of petitioners obligation HELD 1. NO RatioA contract of adhesion is one in which one of the parties imposes a ready-made form of contract,which the other party may accept or reject, butwhich the latter cannot modify. It is just as bindingas ordinary contracts. Reasoning - Petitioners failed to show that they were under duress or forced to sign the loan documents. Thenatural presumption is that one does not sign adocument without first informing himself of itscontents and consequences. - Contracts of adhesion are not entirely prohibited even as the courts remain careful in scrutinizing thefactual circumstances underlying each case todetermine the respective claims of contendingparties on their efficacy. 2. NO Reasoning - Petitioners failed to show any provision in the insurance policy or mortgage contract providing thatthe loss of the mortgaged vehicle extinguishes theirprincipal obligation to BPI. - While it is true that the proceeds from the insurance policy over the mortgaged chattel is for the benefit of BPI, this will result in partial or fullsatisfaction of the obligationonly if the insurer paysthe mortgagee, BPI, or if the insurance proceedswere paid to BPI. In this case, upon the loss of thevehicle due to total wreck, the petitioners filed aclaim under the insurance policy, collected

andreceived the proceeds thereof, but did not settle theirobligation with BPI which remained outstandingdespite the loss of the vehicle. Disposition CA decision AFFIRMED with themodification that the interest rate be reduced to 12%per annum from 24 May 1994 until fully paid, andthe award of attorneys fees be reduced to P50T.

FILIPINO MERCHANTS INS. v. CA (CHOA TIEK SENG) 179 SCRA 638 REGALADO; November 28, 1989 NATURE Review of the decision of the CA FACTS - Plaintiff insured said shipment with defendant insurance company under said cargo for the goodsdescribed as 600 metric tons of fishmeal in newgunny bags of 90 kilos each from Bangkok, Thailandto Manila against all risks under warehouse towarehouse terms. - Some of the goods arrived in bad condition. Plaintiffmade a claim against Filipino Merchants InsuranceCompany. The latter refused to pay. Plaintiff broughtan action against them. The defendant insurancecompany presented a third party complaint againstthe vessel and the arrastre contractor. - Judgment was rendered against the insurancecompany. On the third party complaint, the thirdparty defendants were ordered to pay the third partyplaintiffs. The CA affirmed, but modified the samewith regard to the adjudication of the third-partycomplaint ISSUES 1. WON some fortuity, casualty or accidental cause isneeded to be proved despite the all risks policy (asasserted by the insurance company) 2. WON the respondent has an insurable interest HELD 1. NO - The very nature of the term "all risks" must be given a broad and comprehensive meaning as covering any loss other than a willful and fraudulentact of the insured. 7 This is pursuant to the verypurpose of an "all risks" insurance to give protectionto the insured in those cases where difficulties oflogical explanation or some mystery surround theloss or damage to property. - Generally, the burden of proof is upon the insuredto show that a loss arose from a covered peril, butunder an "all risks" policy the burden is not on theinsured to prove the precise cause of loss or damagefor which it seeks compensation. The insured underan "all risks insurance policy" has the initial burdenof proving that the cargo was in good condition whenthe policy attached and that the cargo was damagedwhen unloaded from the vessel; thereafter, theburden then shifts to the insurer to show theexception to the coverage. As we held inParis-Manila Perfumery Co. vs. Phoenix Assurance Co., Ltd.the basic rule is that the insurance company has theburden of proving that the loss is caused by the riskexcepted and for want of such proof, the company isliable. In the present case, there being no showingthat the loss was caused by any of the exceptedperils, the insurer is liable under the policy. 2. YES - Section 13 of the Insurance Code defines insurableinterest in property as every interest in property,whether real or personal, or any relation thereto, orliability in respect thereof, of such nature that acontemplated peril might directly damnify theinsured. In principle, anyone has an insurableinterest in property who derives a benefit from itsexistence or would suffer loss from its destructionwhether he has or has not any title in, or lien uponor possession of the property y.16 Insurable interestin property may consist in (a) an existing interest;(b) an inchoate interest founded on an existinginterest; or (c) an expectancy, coupled with anexisting interest in that out of which the expectancyarises. - Respondents interest over the goods is based onthe perfected contract of sale. The perfected contractof sale between him and the shipper of the goodsoperates to vest in him an equitable title even beforedelivery or before be performed the conditions of thesale. - Further, Article 1523 of the Civil Code provides thatwhere, in pursuance of a contract of sale, the selleris authorized or required to send the goods to thebuyer, delivery of the goods to a carrier, whethernamed by the buyer or not, for, the purpose oftransmission to the buyer is deemed to be a delivery of the goods to the buyer, the exceptions to said rulenot obtaining in the present case. The Court hasheretofore ruled that the delivery of the goods onboard the carrying vessels partake of the nature ofactual delivery since, from that time, the foreignbuyers assumed the risks of loss of the goods andpaid the insurance premium covering them - Moreover, the issue of lack of insurable interest was not raised in petitioners answer. Disposition Petition denied

GAISANO CAGAYAN v. INSURANCE Co. OF NORTH AMERICA 490 SCRA 296 Austria-Martinez; June 8, 2006 NATURE Petition for review oncertiorari of the Decision of the Court of Appeals FACTS - Intercapitol Marketing Corporation (IMC) is themaker of Wrangler Blue Jeans. Levi Strauss (Phils.)Inc. (LSPI) is the local distributor of products bearingtrademarks owned by Levi Strauss & Co..IMC andLSPI separately obtained from respondent fireinsurance policies with book debt endorsements.The insurance policies provide for coverage on bookdebts in connection with ready made clothingmaterials which have been sold or delivered tovarious customers and dealers of the Insuredanywhere in the Philippines. The policies definedbook debts as the unpaid account still appearing inthe Book of Account of the Insured 45 days after thetime of the loss covered under this Policy. Thepolicies also provide for the following conditions: 1.Warranted that the Company shall notbe liable for any unpaid account inrespect of the merchandise sold anddelivered by the Insured which areoutstanding at the date of loss for aperiod in excess of six (6) months fromthe date of the covering invoice oractual delivery of the merchandisewhichever shall first occur. 2. Warranted that the Insured shall submit to theCompany within twelve (12) days after the close ofevery calendar month all amount shown in theirbooks of accounts as unpaid and thus becomereceivable item from their customers and dealers.xx x - Petitioner is a customer and dealer of the productsof IMC and LSPI. On February 25, 1991, the GaisanoSuperstore Complex in Cagayan de Oro City, ownedby petitioner, was consumed by fire.Included in theitems lost or destroyed in the fire were stocks ofready-made clothing materials sold and delivered byIMC and LSPI.On February 4, 1992, respondentfiled a complaint for damages against petitioner. Italleges that IMC and LSPI filed with respondent theirclaims under their respective fire insurance policieswith book debt endorsements; that as of February25, 1991, the unpaid accounts of petitioner on thesale and delivery of ready-made clothing materialswith IMC was P2,119,205.00 while with LSPI it wasP535,613.00; that respondent paid the claims of IMCand LSPI and, by virtue thereof, respondent wassubrogated to their rights against petitioner; thatrespondent made several demands for payment uponpetitioner but these went unheeded. In its Answerwith Counter Claim dated July 4, 1995, petitionercontends that it could not be held liable because theproperty covered by the insurance policies weredestroyed due to fortuities event or force majeure;that respondents right of subrogation has no basisinasmuch as there was n o breach of contractcommitted by it since the loss was due to fire whichit could not prevent or foresee; that IMC and LSPInever communicated to it that they insured theirproperties; that it never consented to paying theclaim of the insured. - At the pre-trial conference the parties failed toarrive at an amicable settlement. Thus, trial on themerits ensued. On August 31, 1998, the RTCrendered its decision dismissing respondentscomplaint. It held that the fire was purely accidental;that the cause of the fire was not attributable to thenegligence of the petitioner; that it has not beenestablished that petitioner is the debtor of IMC andLSPI; that since the sales invoices state that it isfurther agreed that merely for purpose of securingthe payment of purchase price, the above-describedmerchandise remains the property of the vendoruntil the purchase price is fully paid, IMC and LSPIretained ownership of the delivered goods and mustbear the loss. Dissatisfied, petitioner appealed to theCA. On October 11, 2000, the CA rendered itsdecision setting aside the decision of the RTC. TheCA held that the sales invoices are proofs of sale,being detailed statements of the nature, quantity andcost of the thing sold; that loss of the goods in thefire must be borne by petitioner since theprovisocontained in the sales invoices is an exception under Article 1504 (1) of the Civil Code, to the general rulethat if the thing is lost by a fortuitous event, the riskis borne by the owner of the thing at the time theloss under the principle of res perit domino; thatpetitioners obligation to IMC and LSPI is not thedelivery of the lost goods but the payment of itsunpaid account and as such the obligation to pay isnot extinguished, even if the fire is considered afortuitous event; that by subrogation, the insurer hasthe right to go against petitioner; that, being a fireinsurance with book debt endorsements, what wasinsured was the vendors interest as a creditor.Petitioner filed a motion for reconsideration but itwas denied by the CA in its Resolution dated April11, 2001.

ISSUES 1. WON the CA erred in construing a fire insurancepolicy on book debts as one covering the unpaidaccounts of IMC and LSPI since such insuranceapplies to loss of the ready-made clothing materialssold and delivered to petitioner. 2. WON IMC bears the risk of loss because itexpressly reserved ownership of the goods bystipulating in the sales invoices that [i]t is furtheragreed that merely for purpose of securing thepayment of the purchase price the above describedmerchandise remains the property of the vendoruntil the purchase price thereof is fully paid. 3. WON the petitioner liable for the unpaid accounts HELD 1. NO - It is well-settled that when the words of a contractare plain and readily understood, there is no roomfor construction. In this case, the questionedinsurance policies provide coverage for book debtsin connection with ready-made clothing materialswhich have been sold or delivered to variouscustomers and dealers of the Insured anywhere inthe Philippines.; and defined book debts as theunpaid account still appearing in the Book ofAccount of the Insured 45 days after the time of theloss covered under this Policy. Nowhere is itprovided in the questioned insurance policies that thesubject of the insurance is the goods sold anddelivered to the customers and dealers of the insured. - Indeed, when the terms of the agreement are clearand explicit that they do not justify an attempt toread into it any alleged intention of the parties, theterms are to be understood literally just as they appear on the face of the contract. Thus, what wereinsured against were the accounts of IMC and LSPIwith petitioner which remained unpaid 45 days afterthe loss through fire, and not the loss or destructionof the goods delivered. 2. NO - The present case clearly falls under paragraph (1), Article 1504 of the Civil Code: ART. 1504. Unless otherwise agreed, the goodsremain at the sellers risk until the ownershiptherein is transferred to the buyer, but when theownership therein is transferred to the buyer thegoods are at the buyers risk whether actualdelivery has been made or not, except that: (1) Where delivery of the goods has been made tothe buyer or to a bailee for the buyer, in pursuanceof the contract and the ownership in the goods has been retained by the seller merely tosecure performance by the buyer of hisobligations under the contract, the goods areat the buyers risk from the time of suchdelivery; (Emphasis supplied) - Thus, when the seller retains ownership only toinsure that the buyer will pay its debt, the risk of lossis borne by the buyer. Accordingly, petitioner bearsthe risk of loss of the goods delivered. - IMC and LSPI did not lose complete interest overthe goods.They have an insurable interest until fullpayment of the value of the delivered goods.Unlikethe civil law concept of res perit domino, whereownership is the basis for consideration of who bearsthe risk of loss, in property insurance, ones interest is not determined by concept of title,but whether insured has substantial economicinterest in the property. - Section 13 of our Insurance Code defines insurableinterest as every interest in property, whether realor personal, or any relation thereto, or liability inrespect thereof, of such nature that a contemplatedperil might directly damnify the insured.Parenthetically, under Section 14 of the same Code,an insurable interest in property may consist in: (a)an existing interest; (b) an inchoate interest foundedon existing interest; or (c) an expectancy, coupledwith an existing interest in that out of which theexpectancy arises. - Therefore, an insurable interest in property doesnot necessarily imply a property interest in, or a lienupon, or possession of, the subject matter of theinsurance, and neither the title nor a beneficialinterest is requisite to the existence of such aninterest, it is sufficient that the insured is so situated with reference to the property that he would be liableto loss should it be injured or destroyed by the perilagainst which it is insured. Anyone has an insurableinterest in property who derives a benefit from itsexistence or would suffer loss from its destruction.Indeed, a vendor or seller retains an insurableinterest in the property sold so long as he has anyinterest therein, in other words, so long as he wouldsuffer by its destruction, as where he has a vendorslien. In this case, the insurable interest

of IMC andLSPI pertain to the unpaid accounts appearing intheir Books of Account 45 days after the time of theloss covered by the policies. 3. YES - Petitioners argument that it is not liable becausethe fire is a fortuitous event under Article 1174 of theCivil Code is misplaced.As held earlier, petitionerbears the loss under Article 1504 (1) of the Civil Code. - Moreover, it must be stressed that the insurance inthis case is not for loss of goods by fire but forpetitioners accounts with IMC and LSPI thatremained unpaid 45 days after the fire.Accordingly,petitioners obligation is for the payment of money.Where the obligation consists in the payment ofmoney, the failure of the debtor to make thepayment even by reason of a fortuitous event shallnot relieve him of his liability. The rationale for this isthat the rule that an obligor should be held exemptfrom liability when the loss occurs thru a fortuitousevent only holds true when the obligation consists inthe delivery of a determinate thing and there is nostipulation holding him liable even in case offortuitous event. It does not apply when the obligation is pecuniary in nature. - Under Article 1263 of the Civil Code, [i]n anobligation to deliver a generic thing, the loss ordestruction of anything of the same kind does notextinguish the obligation. If the obligation is genericin the sense that the object thereof is designatedmerely by its class or genus without any particulardesignation or physical segregation from all others ofthe same class, the loss or destruction of anything ofthe same kind even without the debtors fault andbefore he has incurred in delay will not have theeffect of extinguishing the obligation. This rule isbased on the principle that the genus of a thing cannever perish. Genus nunquan perit. An obligation to pay money is generic; therefore, it is notexcused by fortuitous loss of any specificproperty of the debtor. - Thus, whether fire is a fortuitous event or petitionerwas negligent are matters immaterial to this case.What is relevant here is whether it has beenestablished that petitioner has outstanding accountswith IMC and LSPI. - With respect to IMC, the respondent hasadequately established its claim. Petitioner has anoutstanding account with IMC in the amount ofP2,119,205.00, check voucher evidencing paymentto IMC, subrogation receipt executed by IMC in favorof respondent upon receipt of the insuranceproceeds. All these documents have been properlyidentified, presented and marked as exhibits in court.The subrogation receipt, by itself, is sufficient toestablish not only the relationship of respondent asinsurer and IMC as the insured, but also the amountpaid to settle the insurance claim.The right ofsubrogation accrues simply upon payment by theinsurance company of the insurance claim.Respondents action against petitioner is squarelysanctioned by Article 2207 of the Civil Code whichprovides: Art. 2207.If the plaintiffs property has beeninsured, and he has received indemnity from theinsurance company for the injury or loss arisingout of the wrong or breach of contract complainedof, the insurance company shall be subrogated tothe rights of the insured against the wrongdoer orthe person who has violated the contract.x x x - Petitioner failed to refute respondents evidence. - As to LSPI, respondent failed to present sufficientevidence to prove its cause of action.No evidentiaryweight can be given to Exhibit F Levi Strauss, ale tter dated April 23, 1991 from petitioners GeneralManager, Stephen S. Gaisano, Jr., since it is not anadmission of petitioners unpaid account with LSPI.It only confirms the loss of Levis products in theamount of P535,613.00 in the fire that razedpetitioners building on February 25, 1991. - Moreover, there is no proof of full settlement of theinsurance claim of LSPI; no subrogation receipt wasoffered in evidence. Thus, there is no evidence thatrespondent has been subrogated to any right whichLSPI may have against petitioner. Failure tosubstantiate the claim of subrogation is fatal topetitioners case for recovery of the amount ofP535,613.00. Disposition Petition is partlyGRANTED. Theassailed Decision dated October 11, 2000 andResolution dated April 11, 2001 of the Court ofAppeals in CA-G.R. CV No. 61848 areAFFIRMEDwith theMODIFICATIO N that the order to pay theamount of P535,613.00 to respondent is DELETED for lack of factual basis.

TAI TONG CHUACHE & CO v. INSURANCE COMMISSION and TRAVELLERS MULTIINDEMNITY CORPORATION 158 SCRA 366 GANCAYCO; February 29, 1988 NATURE Petition for review on certiorari of the decision of the Insurance Commission FACTS - Complainants Palomo acquired a parcel of land anda building located in Davao City. They assumed themortgage of the building in favor of SSS, whichbuilding was insured with respondent SSS AccreditedGroup of Insurers for P25K. - On April 19, 1975, Azucena Palomo obtained a P100K loan from Tai Tong Chuache Inc. (TTCC) and executed a mortgage over the land and the buildingin favor of Tai Tong Chuache & Co. as security ofpayment .On April 25, 1975, Arsenio Chua, representative of TTCC insured the latter's interest with Travellers Multi-Indemnity Corporation(Travellers) for P100K (P70K for bldg and P30K forthe contents thereof) - On June 11, 1975, Pedro Palomo secured a FireInsurance Policy, covering the building for P50K withrespondent Zenith Insurance Corporation (ZIC).Another Fire Insurance Policy was later procuredfrom respondent Philippine British AssuranceCompany (PBAC), covering the same building forP50K and contents thereof for P70K. On July 31,1975, the building and the contents were totallyrazed by fire. - Based on the computation of the loss, including theTravellers, respondents, ZIC, PBAC, and SSS paidtheir corresponding shares of the loss. Complainantswere paid the following: P41,546.79 by PBAC,P11,877.14 by ZIC, and P5,936.57 by SSS. Demandwas made from respondent Travellers for its share inthe loss but was refused. Hence, complainantsdemanded from the other 3 respondents the balanceof each share in the loss based on the computationexcluding Travellers Multi-Indemnity in the amount ofP30,894.31 (P5,732.79-ZIC: P22,294.62, PBAC: andP2,866.90, SSS) but was refused, hence, this action. ISSUE WON petitioner Tai Tong has insurable interest in the said policy HELD YES - First, respondent insurance commission based itsfindings on mere inference. Respondent InsuranceCommission absolved respondent insurance companyfrom liability on the basis of the certification issuedby the then CFI, that in a certain civil action againstthe Palomos, Arsenio Lopez Chua stands as thecomplainant andnot Tai Tong Chuache. From saidevidence respondent commission inferred that thecredit extended by herein petitioner to the Palomossecured by the insured property must have beenpaid. Such is a glaring error which this Court cannotsanction. - Second, it has been held in a long line of cases thatwhen the creditor is in possession of the document ofcredit, he need not prove non-payment for it ispresumed. The validity of the insurance policy takenb petitioner was not assailed by private respondent.Moreover, petitioner's claim that the loan extended tothe Palomos has not yet been paid was corroboratedby Azucena Palomo who testified that they are stillindebted to herein petitioner. So at the time of thefire, petitioner as mortgagee still had insurableinterest therein. - And third, petitioner's declaration that ArsenioLopez Chua acts as the managing partner of thepartnership was corroborated by respondentinsurance company. Thus Chua as the managingpartner of the partnership may execute all acts ofadministration including the right to sue debtors ofthe partnership in case of their failure to pay theirobligations when it became due and demandable. Orat the least, Chua being a partner of petitioner TaiTong Chuache & Company is an agent of thepartnership. Being an agent, it is understood that heacted for and in behalf of the firm. Disposition

Appealed decision SET ASIDE andANOTHER judgment is rendered order privaterespondent Travellers to pay petitioner the face valueof Fire Insurance Policy in the amount of P100K.Costs against said private respondent.

PEREZ v. CA (BF LIFEMAN INSURANCE CORP.) 323 SCRA 613 YNARES-SANTIAGO; January 28, 2000 NATURE Petition for review on certiorari FACTS - Primitivo Perez has been insured with the BFLifeman Insurance Corporation (BF hereafter) since1980 for Php20,000. Sometime in 1987, Rodolfo Lalog (agent of BF)convinced him to apply for additional insurancecoverage of Php50,000. Perez accomplished the application form and passedthe required medical examination. He also paidPhp2,075 premium) to Lalog. - On November 25, 1987, Perez died while riding abanca which capsized during a storm. During thistime his application papers for the additionalinsurance coverage was still with the Gumaca, Quezon office of BF. - Without knowing that Perez died on November 25,1987, BF approved Perez's application and issued thecorresponding policy for the Php50,000 on December2, 1987. - Virginia Perez (wife of the deceased) claimed thebenefits under the insurance policies of thedeceased, but she was only able to receivePhp40,000 under the first insurance policy. BF refused to pay the proceeds amounting toPhp150,000 under the additional policy coverage ofPhp50,000 because they maintain that such policyhad not been perfected. - On September 21, 1990, BF filed a complaintagainst Mrs. Perez seeking recission and declarationof nullity of the insurance contract in question. Mrs.Perez filed a conterclaim for the collection of Php150,000 plus damages. ISSUE WON there was a consummated contract of insurance between Perez and BF HELD NO - An essential requisite of a valid contract is consent.Consent must be manifested by the meeting of theoffer and the acceptance upon the thing and thecause which are to constitute the contract. - The offer must be certain and the acceptanceabsolute. When Perez filed the application, it wassubject to the acceptance of BF. The perfection wasalso further conditionedupon (1) the issuance of thepolicy, (2) the payment of the premium, and (3) thedelivery to and acceptance by the applicant in goodhealth. - The delivery and acceptance by the applicant was asuspensive condition which was not fulfilledinasmuch as the applicant was already dead at thetime the policy was issued. The nonfulfillment of thecondition resulted in the non-perfection of the contract. - An application for insurance is merely an offerwhich requires the overt act of the insurer for it toripen to a contract. Delay in acting on the applicationdoes not constitute acceptance even though theinsured has forwarded his first premium with his application. Delay, in thiscase, does not constitute gross negligence becausethe application was granted within the normalprocessing time. Disposition Decision of CA affirmed in so far as it declared the insurance policy for Php50,000 issuedby BF null and void (no recission because itpresupposes a valid contract)

VDA. DE SINDAYEN v. INSULAR 62 Phil 51 BUTTE; September 4, 1935 FACTS - Arturo Sindayen, up to the time of his death onJanuary 19, 1933, was employed as a linotypeoperator in the Bureau of Printing at Manila and hadbeen such for eleven years prior thereto. While therehe made a written application on December 26,1932, to the defendant Insular Life Assurance Co.,Ltd., through its agent, Cristobal Mendoza, for apolicy of insurance on his life in the sum of P1,000and he paid to the agent P15 cash as part of the firstpremium. It was agreed with the agent that thepolicy, when and if issued, should be delivered to hisaunt. Felicidad Estrada, with whom Sindayen left thesum of P26.06 to complete the payment of the firstannual premium of P40.06. - On January 1, 1933, Sindayen, who was thentwenty-nine years of age, was examined by thecompany's doctor who made a favorable report, tothe company. On January 11, 1933, The companyaccepted the risk and issued policy No. 47710 datedback to December 1, 1932, and mailed the same toits agent, Cristobal Mendoza, in Camiling, Tarlac, fordelivery to the insured. -On January 11, 1933,Sindayen was at work in the Bureau of Printing. OnJanuary 12, he complained of a severe headache and remained at home. On January 15, he called aphysician who found that he was suffering fromacute nephritis and uremia and on January 19, 1933,he died. - On January 18, 1933, the agent, in accordance withhis agreement with the insured, delivered the policyto Felicidad Estrada upon her payment of the balanceof the first year's annual premium. The agent askedFelicidad Estrada if her nephew was in good healthand she replied that she believed so because she hadno information that he was sick and he thereupondelivered to her the policy. - On January 20, 1933, the agent learned of thedeath of Arturo Sindayen and called on FelicidadEstrada and asked her to return the policy. But hedid not return or offer to return the premium paid.Felicidad Estrada on his aforesaid statement gavehim the policy. - On February 4, 1933 Insular Life obtained from thebeneficiary, Sind ayens wife, her signature to a legaldocument entitled "ACCORD, SATISFACTION ANDRELEASE" whereby in consideration of the sum ofP40.06 paid to her by a check of the company, she"assigns, releases and forever discharges said IsularLife Assurance Co., Ltd., its successors and assigns,of all claims, obligation in or indebtedness. The saidcheck for P40.06 was never cashed but returned tothe company and appears in the record of this caseas Exhibit D. Thereupon this action was brought toenforce payment of the policy. By the terms of the policy, an annual premium ofP40.06 is due on the first day of December of eachyear, the first premium already paid by the insuredcovering the period from December 1, 1932. It is toDecember 1, 1933. It is to be noted that the policywas not issued and the company assumed no actualrisk prior to January 11, 1933.The application whichthe insured signed in Camiling, Tarlac, on December26, 1932, contained among others the followingprovisions: 3 That the said policy shall not take effect until thefirst premium has been paid and the policy has beendelivered to and accepted by me, while I am in goodhealth. -Main defense of the company in this case, namely,that the said policy never took effect because ofparagraph 3 of the application above quoted, for atthe time of its delivery by the agent as aforesaid theinsured was not in good health ISSUE WON the insurance policy is valid HELD YES - There is one line of cases which holds that thestipulation contained in paragraph 3 is in the natureof a condition precedent, that is to say, that therecan be no valid delivery to the insuredunless he is ingood health at the time; that this conditionprecedent goes to the very essence of the contractand cannot be waived by the agent making deliveryof the policy - On the other hand, a number of American decisionshold that an agent to whom a life insurance policysimilar to the one here involved was sent withinstructions to deliver it to the insured has authorityto bind the company by making such delivery,although the insured was not in good health at thetime of delivery, on the theory that the delivery ofthe policy being the final act to the consummation ofthe contract, the condition as to the insurer's goodhealth was waived by the company. - we are inclined to the view that it is moreconsonant with the well known practice of lifeinsurance companies and the evidence in the presentcase to rest our decision on the proposition thatMendoza was authorized by the company to makethe delivery of the policy when he received thepayment of the first premium and he was satisfiedthat the insured was in good health. As was well saidin the

case of MeLaurinvs. Mutual Life Insurance Co.It is plain, therefore, that upon the facts it is notnecessarily a case of waiver or of estoppel, but acase where the local agents, in the exercise of thepowers lodged in them, accepted the premium anddelivered the policy. That act binds their principal,the defendant. - Mendoza was duly licensed by the InsuranceCommissioner to act as the agent of the defendantinsurance company. The well known custom of theinsurance business and the evidence in this caseprove that Mendoza was not regarded by thecompany as a mere conduit or automaton for theperformance of the physical act of placing the policyin the hands of the insured - Granted that Mendoza's decision that the conditionhad been met by the insured and that it was properto make a delivery of the policy to him is just asbinding on the company as if the decision had beenmade by its board of directors. Granted thatMendoza made a mistake of judgement because heacted on insufficient evidence as to the state ofhealth of the insured. But it is not charged that the mistake was induced by any misconduct or omission of duty of the insured. - It is the interest not only the applicant but of allinsurance companies as well that there should besome act which gives the applicant the definiteassurance that the contract has been consummated.This sense of security and of peace of mind thatone's defendants are provided for without risk eitherof loss or of litigation is the bedrock of life insurance.When the policy is issued and delivered, in theabsence of fraud or other grounds for rescission, it isplainly not within the intention of the parties thatthere should be any questions held in abeyance orreserved for future determination that leave the veryexistence of the contract in suspense and doubt. - It is therefore in the public interest, for the public isprofoundly and generally interested in life insurance,as well as in the interest of the insurance companiesthemselves by giving certainly and security to their policies, that we are constrained to hold, as we, do,that the delivery of the policy to the insured by anagent of the company who is authorized to makedelivery or without delivery is the final act whichbinds the company (and the insured as well) in theabsence of fraud or other legal ground for rescission- The company therefore having decided that all theconditions precedent to the taking effect of the policyhad been complied with and having accepted thepremium and delivered the policy thereafter to theinsured, the company is now estopped to assert thatit never intended that the policy should take effect. SEPARATE OPINION IMPERIAL [dissent] - "A local agent of an insurance company, whose onlypower is to solicit applications for insurance, andforward them to the company for approval, when, ifapproved to the insured, has no power to waive anyof the provision of the policy so delivered." - It is clear, therefore, that the delivery of the policyby Mendoza does not bind the defendant, nor is thedefendant estopped from alleging its defense, for thesimple reason that Mendoza was not an agent withauthority to issue policies or to accept risks in thename of his principle. -There is another ground upon which the majorityopinion is based, namely, that the defendant waivedthe defense it now invokes, by reason of the deliveryof the policy by its invokes, by reason of the deliveryof the policy by its agent. It is admitted that if thedelivery of the policy was due to fraud, legally there could have been no waiver. In view of the factsestablished and admitted, there is no doubt, as tothe existence of the fraud. -Estrada, as arepresentative of the insured was not only bound togive a truthful information on the state of health ofthe insured, but it was her duty to find out it his truestate of health in order to give true and correctinformation. When she gave Mendoza an incorrectinformation tending to create the impression that theinsured was well when in fact he was seriously ill,there is no doubt that she committed fraud andimparted a deceitful information to the defendant agent

ENRIQUEZ v. SUN LIFE OF CANADA 41 PHIL 269 MALCOLM; November 29, 1920 NATURE Appeal from judgment of trial court denyingplaintiffs (administrator of the estate of the lateJoaquin Ma. Herrer) action to recover from thedefendant life insurance company the sum of pesos6,000 paid by the deceased for a life annuity. FACTS - On September 24, 1917, Joaquin Herrer madeapplication to the Sun Life Assurance Company ofCanada through its office in Manila for a life annuity.Two days later he paid the sum of P6,000 to themanager of the company's Manila office and wasgiven a receipt. - The application was immediately forwarded to thehead office of the company at Montreal, Canada. OnNovember 26, 1917, the head office gave notice ofacceptance by cable to Manila. (Whether on thesame day the cable was received, notice was sent bythe Manila office of Herrera that the application hadbeen accepted, is a disputed point, which will bediscussed later.) On December 4, 1917, the policywas issued at Montreal. On December 18, 1917,attorney Aurelio A. Torres wrote to the Manila officeof the company stating that Herrer desired towithdraw his application. The following day the localoffice replied to Mr. Torres, stating that the policyhad been issued, and called attention to thenotification of November 26, 1917. This letter wasreceived by Mr. Torres on the morning of December21, 1917. Mr. Herrer died on December 20, 1917. - The chief clerk of the Manila office of Sun Lifetestified that he prepared the letter and handed it tothe local manager, Mr. E. E. White, for signature. The local manager, Mr. White, testified to having receivedthe cablegram accepting the application of Mr. Herrerfrom the home office on November 26, 1917. He saidthat on the same day he signed a letter notifying Mr.Herrer of this acceptance. The witness further saidthat letters, after being signed, were sent to thechief clerk and placed on the mailing desk fortransmission. Mr. Tuason, who was the chief clerk onNovember 26, 1917, was not called as a witness. - For the defense, attorney Manuel Torres testified tohaving prepared the will of Joaquin Ma. Herrer. Thaton this occasion, Mr. Herrer mentioned hisapplication for a life annuity, and that he said thatthe only document relating to the transaction in hispossession was the provisional receipt. RafaelEnriquez, the administrator of the estate, testifiedthat he had gone through the effects of the deceasedand had found no letter of notification from theinsurance company to Mr. Herrer. ISSUE WON there exists a contract for life annuity between Herrer and defendant HELD NO Ratio The law applicable to the case is found to be the second paragraph of article 1262 of the CivilCode providing that an acceptance made by lettershall not bind the person making the offer exceptfrom the time it came to his knowledge. Reasoning - Until quite recently, all of the provisions concerning life insurance in the Philippines were found in theCode of Commerce and the Civil Code. After July 1,1915, there was, however, in force the InsuranceAct. No. 2427. Chapter IV of this Act concerns lifeand health insurance. The Act expressly repealedTitle VIII of Book II and Section III of Title III ofBook III of the code of Commerce. The law ofinsurance is consequently now found in theInsurance Act and the Civil Code. - While, as just noticed, the Insurance Act deals withlife insurance, it is silent as to the methods to befollowed in order that there may be a contract ofinsurance. On the other hand, the Civil Code, inarticle 1802, not only describes a contact of lifeannuity markedly similar to the one we areconsidering, but in two other articles, gives strongclues as to the proper disposition of the case. Forinstance, article 16 of the Civil Code provides that"In matters which are governed by special laws, any deficiency of the latter shall be supplied by theprovisions of this Code." On the supposition,therefore, which is incontestable, that the special lawon the subject of insurance is deficient in enunciatingthe principles governing acceptance, the subject-matter of the Civil code, if there be any, would becontrolling. In the Civil Code is found article 1262providing that "Consent is shown by the concurrenceof offer and acceptance with respect to the thing andthe consideration which are to constitute thecontract. An acceptance made by letter shall not bindthe person making the offer except from the time itcame to his knowledge.

- According to the provisional receipt, three thingshad to be accomplished by the insurance companybefore there was a contract: (1) There had to be amedical examination of the applicant; (2) there hadto be approval of the application by the head office ofthe company; and (3) this approval had in some wayto be communicated by the company to theapplicant. The further admitted facts are that thehead office in Montreal did accept the application, didcable the Manila office to that effect, did actuallyissue the policy and did, through its agent in Manila,actually write the letter of notification and place it inthe usual channels for transmission to the addressee.- The contract for a life annuity in the case at barwas not perfected because it has not been provedsatisfactorily that the acceptance of the applicationever came to the knowledge of the applicant. DispositionJudgment is reversed, and the plaintiff shall have and recover from the defendant the sumof P6,000 with legal interest from November 20,1918, until paid, without special finding as to costs ineither instance.

VELASCO and ACOSTA v. APOSTOL and MAHARLIKA INSURANCE CO., INC. 173 SCRA 228 REGALADO.; May 9, 1989 NATURE Petition for review on certiorari FACTS - Petitioners were plaintiffs in a civil case of which public respondent Hon. Apostol was the judge. - The case was an offshoot of an incident: plaintiffs were riding in their car, when a taxicab crossed a center island in the road and collided with their car.Private respondent Maharlika was eventuallyimpleaded as a defendant in this case, with anallegation that the taxicab involved was insuredagainst third party liability for P20,000.00 withprivate respondent at the time of the accident - Maharlika claimed there was no cause of actionagainst it because at the time of the accident, thealleged insurance policy was not in force due to thenon-payment of the premium thereon. Also, even ifthe cab had been insured, the complaint would bepremature since the policy provides that the insurerwould be liable only when the insured becomeslegally liable. - Trial court ruled in favor of the plaintiff, holding thedefendants liable for repair of the car, medicalexpenses, etc. BUT Maharlike was exonerated on thegnd that the policy was not in force. - Petitioners elevated this case to this court, faultingthe respondent judge for considering the defense oflate payment of premium when the same waswaived at the pre -trial, hence the evidence of latepayment should be disregarded supposedly becausethe private respondent had admitted that such factwas not in issue. - (More pertinent to this class: ) petitioners assertthat the private respondent had agreed to grant thethen prospective insured a credit extension of thepremium due. - This controversy arose under the old insurance law, Act No. 2427. - The accident occurred in 1973. The complaint was filed on July 20, 1974. both before theeffectivity of Presidential Decree no. 612, thesubsequent insurance law which repealed itspredecessor - The former insurance law, which applies to the casehere, provided: An insurer is entitled to the paymentof premium as soon as the thing insured is exposedto the peril insured against, unless there is clearagreement to grant the insured credit extension ofthe premium due. No policy issued by an insurancecompany is valid and binding unless and until thepremium thereof has been paid. ISSUE WON the insurance policy would be valid and binding notwithstanding the non-payment of the premium HELD NO Ratio Act No. 2427: an insurance policy would be valid and binding notwithstanding the non-paymentof the premium if there was a clear agreement togrant to the insured credit extension. Suchagreement may be express or implied. Reasoning - Petitioners maintain that in spite of their latepayment, the policy is binding because there was animplied agreement to grant a credit extension so asto make the policy effective. To them, thesubsequent acceptance of the premium and deliveryof the policy estops the respondent company fromasserting that the policy is ineffective. The court however sees no proof of any such impliedagreement. The purported nexus between thedelivery of the policy and the grant of creditextension is too tenuous to support the conclusionfor which petitioners contend. Parenthetically mention: in the present law, Section 77 of the Insurance Code of 1978 hasdeleted the clause "unless there is clear agreementto grant the insured credit extension of the premiumdue" which was then involved in this controversy. Disposition Fnding no reversible error, the judgment appealed from is hereby AFFIRMED.

TIBAY v. CA (FORTUNE LIFE & GENERAL INSURANCE) 257 SCRA 126 BELLOSILLO; May 24, 1996 FACTS - On 22 January 1987 Fortune Life and GeneralInsurance Co., Inc. (FORTUNE) issued Fire InsurancePolicy No. 136171 in favor of Violeta R. Tibay and/orNicolas Roraldo on their two-storey residentialbuilding located at 5855 Zobel Street, Makati City,together with all their personal effects therein. Theinsurance was for P600,000 covering the period from23 January 1987 to 23 January 1988. On 23 January1987, of the total premium of P2,983.50, VioletaTibay only paid P600 thus leaving a considerablebalance unpaid. - On 8 March 1987 the insured building wascompletely destroyed by fire. Two days later, VioletaTibay paid the balance of the premium. On the sameday, she filed with FORTUNE a claim on the fireinsurance policy. Her claim was accordingly referred to its adjuster, Goodwill Adjustment Services, Inc.(GASI), which immediately wrote Violeta requestingher to furnish it with the necessary documents forthe investigation and processing of her claim.Petitioner forthwith complied. On 28 March 1987 shesigned a nonwaiver agreement with GASI to theeffect that any action taken by the companies shallnot be, or be claimed to be, an admission of liability.- FORTUNE denied the claim of Violeta for violation ofPolicy Condition No. 2 and of Sec. 77 of theInsurance Code. Efforts to settle the case before theInsurance Commission proved futile. On 3 March1988 Violeta and the other petitioners sued FORTUNEfor damages in the amount of P600,000 representingthe total coverage of the fire insurance policyplus12% interest per annum, P100,000 moral damages,and attorney's fees equivalent to 20% of the totalclaim. The trial court ruled for petitioners. CA reversed. ISSUE WON a fire insurance policy is valid, binding and enforceable upon mere partial payment of premium HELD NO Ratio Where the insurer and the insured expressly stipulated that the policy is not in force until thepremium has been fully paid the payment of partialpremium by the assured in this particular instanceshould not be considered the payment required bythe law and the stipulation of the parties. Rather, itmust be taken in the concept of a deposit to be heldin trust by the insurer until such time that the fullamount has been tendered and duly receipted for. Reasoning - As expressly agreed upon in the contract, full payment must be made before the risk occurs for thepolicy to be considered effective and in force. Thus,no vinculum juris whereby the insurer bound itself toindemnify the assured according to law ever resultedfrom the fractional payment of premium. Theinsurance contract itself expressly provided that thepolicy would be effective only when the premium waspaid in full. It would have been altogether differentwere it not so stipulated. Ergo, petitioners hadabsolute freedom of choice whether or not to beinsured by FORTUNE under the terms of its policyand they freely opted to adhere thereto. This policy including any renewal thereof and/or any endorsement thereon is not in force until the premium has been fully paid to and duly receipted by the Company in the manner provided herein. - Indeed, and far more importantly, the cardinalpolestar in the construction of an insurance contractis the intention of the parties as expressed in thepolicy. Courts have no other function but to enforcethe same. The rule that contracts of insurance will beconstrued in favor of the insured and most stronglyagainst the insurer should not be permitted to havethe effect of making a plain agreement ambiguousand then construe it in favor of the insured. Verily, itis elemental law that the payment of premium isrequisite to keep the policy of insurance in force. Ifthe premium is not paid in the manner prescribed inthe policy as intended by the parties the policy isineffective. Partial payment even when accepted as apartial payment will not keep the policy alive evenfor such fractional part of the year as the partpayment bears to the whole payment. Disposition Petition is DENIED.Decision of the CA is AFFIRMED. SEPARATE OPINION VITUG [dissent] - The law neither requires, nor measures thestrength of the vinculum juris by, any specificamount of premium payment. It should thus beenough that payment on the premium, partly or infull, is made by

the insured which the insureraccepts. In fine, it is either that a juridical tie exists(by such payment) or that it is not extant at all (byan absence thereof). Once the juridical relationcomes into being, the full efficacy, not merely protanto, of the insurance contract naturally follows.Verily, not only is there an insurance perfected butalso a partially performed contract. In case of loss,recovery on the basis of the full contract value, lessthe unpaid premium can accordingly be had;conversely, if no loss occurs, the insurer can demandthe payment of the unpaid balance of the premium.The insured, on the one hand, cannot avoid theobligation of paying the balance of the premiumwhile the insurer, upon the other hand, cannot treatthe contract as valid only for the purpose ofcollecting premiums and as invalid for the purpose ofindemnity.

MAKATI TUSCANY v. CA ( AMERICAN HOME ASSURANCE CO.) 215 SCRA 462 BELLOSILLO; November 6, 1992 NATURE Appeal from decision of the CA FACTS - American Home Assurance Co. (AHAC),represented by American International Underwriters(Phils.), Inc., issued in favor of petitioner MakatiTuscany Condominium Corporation an insurancepolicy on the latter's building and premises, for theperiod 1 March 1982 to1 March 1983. The premiumwas paid on installments all of which were acceptedby AHAC. - A second policy was issued to renew the first one,this time covering the period 1 March 1983 to 1March 1984. This was also pain in installment basis. - A third policy was again issued for the period 1 March 1984 to 1 March 1985. For this, petitionermade two installment payments, both accepted byAHAC. Thereafter, petitioner refused to pay thebalance of the premium. AHAC filed an action torecover the unpaid balance of P314,103.05. - Petitioner explained that it discontinued thepayment of premiums because the policy did notcontain a credit clause in its favor and the receiptsfor the installment payments covering the policy for198485, as well as the two (2) previous policies,stated the following reservations: 2. Acceptance of this payment shall not waiveany of the company rights to deny liability on anyclaim under the policy arising before suchpayments or after the expiration of the creditclause of the policy; and 3. Subject to no loss prior to premium payment. If there be any loss such is not covered. - Petitioner further claimed that the policy was neverbinding and valid, and no risk attached to the policy.It then pleaded a counterclaim for P152k for thepremiums already paid for 1984-85, and in itsanswer with amended counterclaim, sought therefund of P924,206.10 representing the premiumpayments for 1982-85. - Trial court dismissed the complaint and thecounterclaim upon the following findings: (1)payment of the premiums of the three policies weremade during the term of said policies, hence, it couldnot be said, inspite of the reservations, that no riskattached under the policies; (2) as regards theunpaid premiums, in view of the reservation in thereceipts ordinarily issued by AHAC on premiumpayments the only plausible conclusion is that AHAChas no right to demand their payment after the lapse of the term of said policy on March 1, 1985.Therefore, Tuscany was justified in refusing to paythe same. - CA modified the decision by ordering Tuscany topay the balance of the premiums due on the thirdpolicy plus legal interest until fully paid, andaffirming the denial of the counterclaim. Petitioners Claims Petitioner argues that where the premiums is notactually paid in full, the policy would only be effectiveif there is an acknowledgment in the policy of thereceipt of premium pursuant to Sec. 78 of theInsurance Code. The absence of an expressacknowledgment in the policies of such receipt of thecorresponding premium payments, and petitioner'sfailure to pay said premiums on or before theeffective dates of said policies rendered them invalid.Petitioner thus concludes that there cannot be aperfected contract of insurance upon mere partialpayment of the premiums because under Sec. 77 ofthe Insurance Code, no contract of insurance is validand binding unless the premium thereof has beenpaid, notwithstanding any agreement to the contrary. ISSUE WON payment by installment of the premiums dueon an insurance policy invalidates the contract ofinsurance HELD Ratio Where the risk is entire and the contract is indivisible, the insured is not entitled to a refund ofthe premiums paid if the insurer was exposed to therisk insured for any period, however brief ormomentary. Reasoning - The obligation to pay premiums when due isordinarily as indivisible obligation to pay the entirepremium. Here, the parties herein agreed to makethe premiums payable in installments, and there isno pretense that the parties never envisioned tomake the insurance contract binding between them.And the insured never informed the insurer that itwas terminating the policy because the terms wereunacceptable.

- There is nothing in Section 77 which suggests thatthe parties may not agree to allow payment of thepremiums in installment, or to consider the contractas valid and binding upon payment of the firstpremium. - The records clearly show that petitioner and private respondent intended subject insurance policies to be binding and effective notwithstanding the staggeredpayment of the premiums. Acceptance of paymentsspeaks loudly of the insurer's intention to honor thepolicies it issued to petitioner. - Section 78 of the Insurance Code in effect allows waiver by the insurer of the condition of prepaymentby making an acknowledgment in the insurancepolicy of receipt of premium as conclusive evidenceof payment so far as to make the policy bindingdespite the fact that premium is actually unpaid.Section 77 merely precludes the parties fromstipulating that the policy is valid even if premiumsare not paid, but does not expressly prohibit anagreement granting credit extension, and such anagreement is not contrary to morals, good customs,public order or public policy. - At the very least, both parties should be deemed inestoppel to question the arrangement they havevoluntarily accepted. Disposition Judgment affirmed. Costs against petitioner.

SOUTH SEA SURETY AND INSURANCE v. CA (VALENZUELA HARDWOOD) 244 SCRA 744 VITUG; June 2, 1995 NATURE Petition for review on certiorari FACTS - Hardwood entered into agreement with Seven BrosShipping, where latter undertook to load the formerslogs on vessel.Hardwood insured the logs withSouth Sea Surety which issued Marine CargoInsurance Policy.The vessel sank Jan 25, 1984. - Hardwood filed claim with South Sea and SevenBros.Trial Court favored Hardwood.CA decidedagainst South Sea, but absolved Seven Bros.SouthSea filed this instant petition. ISSUES WON the insurance contract was already in effect when the vessel sank HELD YES - It is already in effect because Hardwood has already paid the insurance premium. It delivered the check to Victorio Chua before thevessel sank, but Victorio Chua was only to deliver thecheck to South Sea five days after the vessel sank. Appellant argues that Chua was not its broker, but itwas found that Chua was authorized by South Sea toreceive the premium on its behalf.

AREOLA v. CA (PRUDENTIAL GUARANTEE AND ASSURANCE, INC.) 236 SCRA 643 ROMERO; September 22, 1994 NATURECERTIORARI FACTS - June 29, 1985- 7 months after the issuance ofSantos Areola's Personal Accident Insurance PolicyNo. PA-20015 (covering a period of one year),Prudential unilaterally cancelled the same sincecompany records revealed that Areola failed to payhis premiums. o Under the terms of the statement of accountissued by Prudential, Areola was supposed topay the total amount of P1,609.65 whichincluded the premium of P1,470.00,documentary stamp of P110.25 and 2%premium tax of P29.40. o The statement of account stated that it mustnot be considered a receipt as an officialreceipt will be issued upon payment of theaccount. And if payment was made to arepresentative, the client must demand for aProvisional Receipt and if Official Receiptsarent received within 7 days, Prudentialshould be notified. If payment is made to theiroffice, clients should demand for an OR. - August 3, 1985- Prudential offered to reinstatesame policy it had previously cancelled and evenproposed to extend its lifetime to December 17,1985, upon a finding that the cancellation waserroneous and that the premiums were paid in full byAreola but were not remitted by Teofilo M. Malapit,Prudential's branch manager. Petitioners Claims - The fraudulent act of in misappropriating Areolaspremium payments is the proximate cause of thecancellation of the insurance policy. - Areola theorized that Malapit's act of signing andeven sending the notice of cancellation himself,notwithstanding his personal knowledge ofpetitioner-insured's full payment of premiums,further reinforces the allegation of bad faith. - Such fraudulent act committed by Malapit is attributable to Prudential. - Malapit's actuations are therefore not separate anddistinct from that of Prudentials. It must, therefore,bear the consequences of the erroneous cancellationof subject insurance policy caused by the non-remittance by its own employee of the premiums paid. - Subsequent reinstatement could not possiblyabsolve respondent insurance company from liability,there being an obvious breach of contract. After alldamage had already been inflicted on him and noamount of rectification could remedy the same. Respondents Argument - Prudential argues that where reinstatement, theequitable relief sought by Areola was granted at anopportune moment, i.e. prior to the filing of thecomplaint, Areola is left without a cause of action onwhich to predicate his claim for damages. - Reinstatement effectively restored Areola to all his rights under the policy. ISSUES 1.WON the erroneous act of canceling subjectinsurance policy entitle petitioner-insured to paymentof damages 2. WON the subsequent act of reinstating thewrongfully cancelled insurance policy obliteratewhatever liability for damages Prudential has HELD 1. YES 2. NO Reasoning - Malapit's fraudulent act of misappropriating thepremiums paid by petitioner-insured is beyond doubtdirectly imputable to Prudential.

- A corporation, such as respondent insurancecompany, acts solely thru its employees. The lattersacts are considered as its own for which it can beheld to account. - The facts are clear as to the relationship betweenprivate respondent insurance company and Malapit.His act of receiving the premiums collected is wellwithin the province of his authority as manager.Thus, his receipt of said premiums is receipt byprivate respondent insurance company who, byprovision of law, particularly under Article 1910 ofthe Civil Code, is bound by the acts of its agent. - Article 1910 thus reads: Art. 1910. The principal must comply with all theobligations which the agent may have contractedwithin the scope of his authority. As for any obligation wherein the agent hasexceeded his power, the principal is not boundexcept when he ratifies it expressly or tacitly. - Malapit's failure to remit the premiums he receivedcannot constitute a defense for private respondentinsurance company; no exoneration from liabilitycould result therefrom. - Prudentials earlier act of reinstating the insurancepolicy can not obliterate the injury inflicted onpetitioner-insured. - Respondent company should be reminded that acontract of insurance creates reciprocal obligationsfor both insurer and insured. - Reciprocal obligations are those which arise fromthe same cause and in which each party is both adebtor and a creditor of the other, such that theobligation of one is dependent upon the obligation ofthe other. - Under the circumstances of instant case, therelationship as creditor and debtor between theparties arose from a common cause: i.e., by reasonof their agreement to enter into a contract ofinsurance under whose terms, Prudential promised toextend protection to Areola against the risk insuredfor a consideration in the form of premiums to bepaid by the latter. - Under the law governing reciprocal obligations,particularly the second paragraph of Article 1191, theinjured party, Areola in this case, is given a choicebetween fulfillment or rescission of the obligation incase one of the obligors, such as respondentinsurance company, fails to comply with what isincumbent upon him. - However, said article entitles the injured party topayment of damages, regardless of whether hedemands fulfillment or rescission of the obligation. - Untenable then is reinstatement insurancecompany's argument, namely, that reinstatementbeing equivalent to fulfillment of its obligation,divests petitioner-insured of a rightful claim forpayment of damages. Such a claim finds no supportin our laws on obligations and contracts. DAMAGES: - The nature of damages to be awarded, however, would be in the form of nominal damages - Although the erroneous cancellation of theinsurance policy constituted a breach of contract,Prudential within a reasonable time took steps torectify the wrong committed by reinstating theinsurance policy of petitioner. - Moreover, no actual or substantial damage or injurywas inflicted on petitioner Areola at the time theinsurance policy was cancelled. - Nominal damages are "recoverable where a legalright is technically violated and must be vindicatedagainst an invasion that has produced no actualpresent loss of any kind, or where there has been abreach of contract and no substantial injury or actualdamages whatsoever have been or can be shown. Disposition Petition for review oncertiorari is hereby GRANTED. RTC s DECISION is REINSTATED.

UCPB GENERAL INSURANCE CO., INC. v. MASAGANA TELAMART, INC. 308 SCRA 259 PARDO; June 15, 1999 NATURE Petition for review on certiorari of a decision of the Court of Appeals. FACTS - On April 15, 1991, petitioner issued five (5)insurance policies covering respondent's variousproperty described therein against fire, for the periodfrom May 22, 1991 to May 22, 1992. - In March 1992, petitioner evaluated the policiesand decided not to renew them upon expiration oftheir terms on May 22, 1992. Petitioner advisedrespondent's broker, Zuellig Insurance Brokers, Inc.of its intention not to renew the policies.- On April 6, 1992, petitioner gave written notice torespondent of the non-renewal of the policies at theaddress stated in the policies. - On June 13, 1992, fire razed respondent's propertycovered by three of the insurance policies petitionerissued. - On July 13, 1992, respondent presented topetitioner's cashier at its head office five (5)manager's checks in the total amount ofP225,753.95, representing premium for the renewalof the policies from May 22, 1992 to May 22, 1993.No notice of loss was filed by respondent under thepolicies prior to July 14, 1992. - On July 14, 1992, respondent filed with petitionerits formal claim for indemnification of the insuredproperty razed by fire. On the same day, petitionerreturned to respondent the five manager's checksthat it tendered, and at the same time rejectedrespondent's claim for the reasons (a) that thepolicies had expired and were not renewed, and (b) that the fire occurred on June 13, 1992, before respondent's tender of premium payment. - On July, 21, 1992, respondent filed with theRegional Trial Court, Branch 58, Makati City, a civilcomplaint against petitioner for recovery, ofP18.645,000.00, representing the face value of thepolicies covering respondent's insured property razedby fire, and for attorney's fees. - On October 23, 1992, after its motion to dismisshad been denied, petitioner filed an answer to thecomplaint. It alleged that the complaint "fails to statea cause of action"; that petitioner was not liable to-respondent for insurance proceeds under thepolicies because at the time of the loss ofrespondent's property due to fire, the policies hadlong expired and were not renewed. After due trial, on March 10, 1993, the Regional TrialCourt, Branch 58, Makati, rendered decision, thedispositive portion of which reads: "WHEREFORE, premises considered, judgment ishereby rendered in favor of the plaintiff and againstthe defendant, as follows."(1) Authorizing and allowing the plaintiff toconsign/deposit with this Court the sum ofP225,753.95 (refused by the defendant) as fullpayment of the corresponding premiums for thereplacement-renewal policies for Exhibits A, B, C, Dand E; "(2) Declaring plaintiff to have fully compliedwith its obligation to pay the premium therebyrendering the replacement-renewal policy of ExhibitsA, B, C, D and E effective and binding for theduration May 22, 1992 until May 22, 1993; and,ordering defendant to deliver forthwith to plaintiff thesaid replacementrenewal policies; "(3) DeclaringExhibits A & B, in force from August 22, 1991 up toAugust 23, 1992 and August 9, 1991 to August 9,1992, respectively; and "(4) Ordering the defendantto pay plaintiff the sums of. (a) P18,645,000.00representing the latter's claim for indemnity underExhibits A, B & C and/or its replacement-renewalpolicies; (b) 25% of the total amount due as and forattorney's fees; (c) P25,000.00 as necessarylitigation expenses; and, (d) the costs of suit. xxx - In due time, petitioner appealed to the Court ofAppeals (CA). The CA promulgated its decisionaffirming that of the Regional Trial Court with themodification that item No. 3 of the dispositive portionwas deleted, and the award of attorney's fees wasreduced to 10% of the total amount due. It held that following previous practice, respondent was allowed a 60- to 90-day credit term for the renewal of its policies, and that the acceptance of thelate premium payment suggested an understandingthat payment could be made later. Hence, this appeal. ISSUE

WON the fire insurance policies issued by petitionerto the respondent covering the period May 22, 1991to May 22, 1992, had expired on the latter date orhad been extended or renewed by an implied creditarrangement though actual payment of premium wastendered on a later date after the occurrence of therisk (fire) insured against HELD NO - An insurance policy, other than life, issuedoriginally or on renewal, is not valid and binding untilactual payment of the premium. Any agreement tothe contrary is void. The parties may not agreeexpressly or impliedly on the extension of credit ortime to pay the premium and consider the policybinding before actual payment. Disposition Judgment reversed and set aside

UCPB GENERAL INSURANCE CO., INC. v. MASAGANA TELAMART, INC. (EN BANC) 356 SCRA 307 DAVIDE; April 4, 2001 NATURE Motion for reconsideration of the decision of the Supreme Court. FACTS - In its decision of 15 June 1999, the SC defined themain issue to be whether the fire insurance policiesissued by petitioner to the respondent covering theperiod from May 22, 1991 to May 22, 1992 had beenextended or renewed by an implied creditarrangement though actual payment of premium wastendered on a later date and after the occurrence ofthe (fire) risk insured against.The Court resolved this issue in the negative in view of Section 77 of the Insurance Code and its decisions in Valenzuela v. Court of Appeals; South Sea Surety and InsuranceCo., Inc. v. Court of Appeals; and Tibay v. Court ofAppeals. Accordingly, it reversed and set aside the decision of the Court of Appeals. - Respondent seasonably filed a motion for thereconsideration of the adverse verdict.It alleges inthe motion that the SC had made in the decision itsown findings of facts, which are not in accord withthose of the trial court and the Court of Appeals.The courts below correctly found that no notice ofnon-renewal was made within 45 days before 22 May1992, or before the expiration date of the fireinsurance policies.Thus, the policies in questionwere renewed by operation of law and were effectiveand valid on 30 June 1992 when the fire occurred,since the premiums were paid within the 60- to 90-day credit term. - Respondent likewise disagrees with its ruling thatparties may neither agree expressly or impliedly onthe extension of credit or time to pay the premiumnor consider a policy binding before actual payment.It urges the Court to take judicial notice of the factthat despite the express provision of Section 77 ofthe Insurance Code, extension of credit terms inpremium payment has been the prevalent practice inthe insurance industry.Most insurance companies,including Petitioner, extend credit terms becauseSection 77 of the Insurance Code is not a prohibitiveinjunction but is merely designed for the protectionof the parties to an insurance contract.The Codeitself, in Section 78, authorizes the validity of a policynotwithstanding non-payment of premiums. - Respondent also asserts that the principle ofestoppel applies to Petitioner. Despite its awarenessof Section 77 Petitioner persuaded and inducedRespondent to believe that payment of premium onthe 60- to 90-day credit term was perfectly alright;in fact it accepted payments within 60 to 90 daysafter the due dates. By extending credit andhabitually accepting payments 60 to 90 days fromthe effective dates of the policies, it has implicitlyagreed to modify the tenor of the insurance policyand in effect waived the provision therein that itwould pay only for the loss or damage in case thesame occurred after payment of the premium. - Petitioner filed an opposition to the Respondentsmotion for reconsideration.It argu es that both thetrial court and the Court of Appeals overlooked thefact that on 6 April 1992 Petitioner sent by ordinarymail to Respondent a notice of non-renewal and sent by personal delivery a copy thereof to Respondentsbroker, Zuellig.Both courts likew ise ignored the factthat Respondent was fully aware of the notice ofnon-renewal. A reading of Section 66 of theInsurance Code readily shows that in order for aninsured to be entitled to a renewal of a nonlifepolicy, payment of the premium due on the effectivedate of renewal should first be made.Respondentsargument that Section 77 is not a prohibitiveprovision finds no authoritative support. - The following facts, as found by the trial court and the Court of Appeals, are indeed duly established: 1.For years, Petitioner had been issuing firepolicies to the Respondent, and these policieswere annually renewed. 2.Petitioner had been granting Respondent a60- to 90-day credit term within which to paythe premiums on the renewed policies. 3.There was no valid notice of non-renewal ofthe policies in question, as there is no proof atall that the notice sent by ordinary mail wasreceived by Respondent, and the copy thereofallegedly sent to Zuellig was ever transmittedto Respondent.

4.The premiums for the policies in question inthe aggregate amount of P225,753.95 werepaid by Respondent within the 60- to 90-daycredit term and were duly accepted andreceived by Petitioners cashier. ISSUE WON Sec. 77 of the Insurance Code of 1978 must bestrictly applied to Petitioners advan tage despite itspractice of granting a 60- to 90-day credit term forthe payment of premiums HELD NO - Section 77 of the Insurance Code of 1978 provides:SEC. 77.An insurer is entitled to payment of thepremium as soon as the thing insured is exposedto the peril insured against.Notwithstanding anyagreement to the contrary, no policy or contract ofinsurance issued by an insurance company is validand binding unless and until the premium thereofhas been paid, except in the case of a life or anindustrial life policy whenever the grace periodprovision applies. - This Section is a reproduction of Section 77 of P.D.No. 612 (The Insurance Code) promulgated on 18December 1974.In turn, this Section has its sourcein Section 72 of Act No. 2427 otherwise known asthe Insurance Act as amended by R.A. No. 3540, approved on 21 June 1963, which read: SEC. 72.An insurer is entitled to payment ofpremium as soon as the thing insured is exposedto the peril insured against, unless there is clearagreement to grant the insured credit extension ofthe premium due. No policy issued by aninsurance company is valid and binding unless anduntil the premium thereof has been paid.(Underscoring supplied) - It can be seen at once that Section 77 does notrestate the portion of Section 72 expresslypermitting an agreement to extend the period to paythe premium.But there are exceptions to Section 77. The first exception is provided by Section 77 itself,and that is, in case of a life or industrial life policywhenever the grace period provision applies. The second is that covered by Section 78 of the Insurance Code, which provides: SEC. 78.Any acknowledgment in a policy orcontract of insurance of the receipt of premium isconclusive evidence of its payment, so far as tomake the policy binding, notwithstanding anystipulation therein that it shall not be binding untilpremium is actually paid. - A third exception was laid down in Makati Tuscany Condominium Corporation vs. Court of Appeals, wherein we ruled that Section 77 may not apply ifthe parties have agreed to the payment ininstallments of the premium and partial payment hasbeen made at the time of loss. Tuscanyhas provided a fourth exception to Section 77, namely,that the insurer may grant credit extension for thepayment of the premium.This simply means that ifthe insurer has granted the insured a credit term forthe payment of the premium and loss occurs beforethe expiration of the term, recovery on the policyshould be allowed even though the premium is paidafter the loss but within the credit term. Moreover, there is nothing in Section 77 whichprohibits the parties in an insurance contract toprovide a credit term within which to pay thepremiums.That agreement is not against the law,morals, good customs, public order or public policy.The agreement binds the parties.Article 1306 of theCivil Code provides: ART. 1306.The contracting parties may establishsuch stipulations clauses, terms and conditions asthey may deem convenient, provided they are notcontrary to law, morals, good customs, publicorder, or public policy. - Finally, it would be unjust and inequitable if recovery on the policy would not be permittedagainst Petitioner, which had consistently granted a60to 90-day credit term for the payment ofpremiums despite its full awareness of Section 77.Estoppel bars it from taking refuge under saidSection since Respondent relied in good faith on suchpractice.Estoppel then is the fifth exception toSection 77. DispositionJudgment reconsidered and set aside, that of the Court of Appeals affirmed in toto. SEPARATE OPINION VITUG - An essential characteristic of an insurance is itsbeing synallagmatic, a highly reciprocal contractwhere the rights and obligations of the partiescorrelate and mutually correspond.

- By weight of authority, estoppel cannot create acontract of insurance, neither can it be successfullyinvoked to create a primary liability, nor can it givevalidity to what the law so procribes as a matter ofpublic policy. PARDO [dissent] - An assureds failure to give notice of the fireimmediately upon its occur rence blatantly showedthe fraudulent character of its claims. Respondent isrequired by law and by express terms of the policy togive immediate written notice of loss. This must becomplied with in the utmost good faith. - Assuming arguendo that the 60- to 90-day credithas been agreed between the parties, respondentcould not still invoke estoppel to back up its claim.Estoppel cannot give validity to an act that isprohibited by law or against public policy. The actualpayment of premiums is a condition precedent to thevalidity of an insurance contract other than lifeinsurance policy. Any agreement to the contrary isvoid as against law and public policy.

ACME SHOE RUBBER & PLASTIC CORP. v. CA(DOMESTIC INSURANCE COMPANY OF THEPHILS.) 134 SCRA 155 MELENCIO-HERRERA; January 17, 1985. NATURE Petition for Review on Certiorari of the Decision ofthe then Court of Appeals (CA-G. R. No. 58917R),denying recovery on an insurance policy, therebyreversing the judgment of the Court of First Instance of Rizal, Branch XII, at Caloocan City, which had allowed such recovery. FACTS - ACME Shoe Rubber and Plastic Corporation (ACME)had been insuring yearly against fire its building,machines and general merchandise with DomesticInsurance Company (INSURER) since 1946. On May14, 1962, ACME continued to insure its propertieswith INSURER in the amount of P200,000 for theperiod May 15, 1962 up to May 15, 1963. - On May 14, 1963, INSURER issued Renewal Receipt to cover the period May 15, 1963 to May 15, 1964. - On January 8, 1964, ACME paid P3,331.26 aspremium. The INSURER applied the payment asrenewal premium for the period of May 15, 1963 toMay 15, 1964. - On May 15, 1964, INSURER issued a RenewalReceipt for the period of May 15, 1964 to May 15,1965 (for renewal premium of P3,331.26 yet to bepaid) with a stamped note that says that theinsurance will be deemed valid and binding onlywhen the premium and documentary stamps haveactually been paid in full and duly acknowledged inan official receipt. ACME was given 90 days to payotherwise the policy would automatically becomevoid and ineffective. (ACME should pay short periodpremium for 90 days before the period expires. Ifthey are able to pay the whole amount before the90-day period, the automatic termination wont applyanymore). - On May 26, 1964, ACME, through its President,signed a promissory note saying that they promise topay the premium and documentary stamps andagreed to the automatic cancellation penalty for notcomplying. - On October 13, 1964, ACMEs properties werecompletely destroyed by fire. ACME filed insuranceclaim but the INSURER disclaimed liability on theground that as of the date of loss, the propertiesburned were not covered by insurance. - ACME claims that the January 8, 1964 paymentwas for the period 1964-1965 and that INSURER hadno right to apply it to the period 1963-1964 becauseunder RA 3540, the policy was void and INSURERcould have validly disclaimed liability for loss had oneoccurred then. - TC found INSURER liable for P200k and opined thatthere was a clear intention on the INSURER's part togrant ACME a credit extension for the payment of thepremium due; and that to allow the INSURER toapply the premium ACME paid on January 8, 1964. CA reversed TC and dismissed the suit on the groundthat, as of the moment of loss, ACME's propertieswere not insured and the INSURER could not be heldliable for any indemnity as a result of the loss. ISSUE WON the premium payment for 1964-1965 was paid HELD NO - Not having paid the 1964-1965 premium within theextension granted, and pursuant to R.A. No. 3540,the policy was automatically cancelled and there wasno insurance coverage to speak of as of the date ofthe fire on October 13, 1964. - The pertinent provision of Republic Act No. 3540 reads: "Sec. 72.An insurer is entitled to payment of thepremium as soon as the thing insured is exposedto the peril insured against, unless there is clearagreement to grant the insured credit extension ofthe premium due. No policy issued by aninsurance company is valid and binding unless anduntil the premium thereof has been paid." - RA 3540 was approved on June 20, 2963 and wasput into effect on Oct 1, 1963. It could not be appliedretroactively to the renewal of the policy for the1963-1964 period because said policy was renewedon May 14, 1963. (Laws have no retroactive effectunless the contrary is provided.) Therefore, the Jan8, 1964 payment was properly applied to the 1963-1964 premium. The Trial Court's opinion that therewas a clear agreement to grant ACME creditextension for 1964-1965 is negated by

ACME'sPromissory Note binding itself to pay within ninetydays from the effective date of this policy, 15th May,1964. The credit extension was granted for 90 daysonly. (So wala na by August 16, 1964.) - If ACME was granted credit extensions in the past,the promissory note it signed did away with suchcredit arrangement. Also, before RA 3540, theRenewal Receipts issued by INSURER did not containthe auto-cancellation after 90 days note. By 1964,however, the situation had changed by the passageof the RA: no policy could be valid and binding unless and until the premium thereof had been paid. - What became automatically cancelled by R.A. No.3540 was the 1964-1965 policy for ACME's failure topay the premium within the 90-day extensiongranted, and in accordance with the express terms ofthe Promissory Note that it had signed. Disposition The judgment under review is hereby affirmed. Without pronouncement as to costs.

PEDRO ARCE v. THE CAPITAL INSURANCE & SURETY CO., INC. 11 SCRA 63 ABAD SANTOS; September 30, 1982. NATURE Appeal from CFI decision on question of law. FACTS - Arce (INSURED) owned a residential house whichwas insured with the appellant COMPANY since 1961.In November 1965, the COMPANY sent to theINSURED a Renewal Certificate to cover the periodfrom December 5, 1965 to December 5,1966, andrequested payment of the corresponding premium.Anticipating that the premium could not be paid ontime, the INSURED asked for an extension which wasgranted by the COMPANY. After the lapse of therequested extension, INSURED still failed to pay thepremium. Thereafter, the house of the INSURED wastotally destroyed by fire. Upon INSURED'spresentation of claim for indemnity, he was told thatno indemnity was due because the premium was notpaid. The INSURED sued the COMPANY for indemnity. - The trial court held the COMPANY liable toindemnify the INSURED on the ground that since theCOMPANY could have demanded payment of thepremium, mutuality of obligation required that itshould be liable on the policy. ISSUE WON the COMPANY can be held liable on its policy HELD NO. - The Court commiserates with the INSURED. Theyare well aware that many insurance companies havefallen into the condemnable practice of collectingpremiums promptly but resort to all kinds of excusesto deny or delay payment of just claims. Unhappilythe instant case is one where the insurer has the lawon its side. - Sec. 72 of the Insurance Act, as amended by R.A. No. 3540 reads: "SEC. 72.An insurer is entitled to payment ofpremium as soon as the thing insured is exposedto the perils insured against, unless there is clearagreement to grant credit extension for the premium due. No policy issued by an insurance company is valid and binding unless and until the premium thereof has been paid." - It is obvious from both the Insurance Act, asamended, and the stipulation of the parties that timeis of the essence in respect of the payment of theinsurance premium so that if it is not paid thecontract does not take effect unless there is stillanother stipulation to the contrary. In the instantcase, the INSURED was given a grace period to paythe premium but the period having expired with nopayment made, he cannot insist that the COMPANY isnonetheless obligated to him. - Prior to the amendment (italicized portion above),an insurance contract was effective even if thepremium had not been paid so that an insurer wasobligated to pay indemnity in case of loss andcorrelatively he had also the right to sue for paymentof the premium. But the amendment to Sec. 72 hasradically changed the legal regime in that unless thepremium is paid there is no insurance. DispositionThe decision of the court a quo is reversed; the appellee's complaint is dismissed. No special pronouncement as to costs. - Irrelevant facts: The premium costs P38.10. Afterthe fire, the COMPANY issued a check for P300 toArce as donation. Arce accepted the check, but stillsued the company.

CAPITAL INC. v. PLASTIC ERA CO. 65 SCRA 134 MARTIN; July 18, 1975 NATURE Petition for review of a decision of the CA affirming the decision of the CFI of Manila FACTS - On December 17, 1960, petitioner CapitalInsurance & Surety Co., Inc. delivered to therespondent Plastic Era Manufacturing Co., Inc., itsopen Fire Policy No. 22760 wherein the formerundertook to insure the latter's building, equipments,raw materials, products and accessories located atSheridan Street, Mandaluyong, Rizal. The policyexpressly provides that if the property insured wouldbe destroyed or damaged by fire after the paymentof the premiums, at anytime between the 15th dayof December 1960 and one o'clock in the afternoonof the 15th day of December 1961, the insurancecompany shall make good all such loss or damage inan amount not exceeding P100,000.00. When thepolicy was delivered, Plastic Era failed to pay the corresponding insurance premium. On January 8,1961, in partial payment of the insurance premium,Plastic Era delivered to Capital Insurance, a check forthe amount of P1,000.00 postdated January 16,1961. However, Capital Insurance tried to deposit thecheck only on February 20, 1961 and the same wasdishonored by the bank for lack of funds. - Two days after the insurance premium became due,at about 4:00 to 5:00 o'clock in the morning, theproperty insured by Plastic Era was destroyed by fire.In less than a month Plastic Era demanded fromCapital Insurance the payment of the sum ofP100,000.00 as indemnity for the loss of the insuredproperty under Policy No. 22760 but the latterrefused for the reason that, among others, PlasticEra failed to pay the insurance premium. ISSUES 1. WON a contract of insurance has been duly perfected between petitioner and respondent 2. WON the dishonored check constituted payment HELD 1. YES - Tender of draft or check in order to effect paymentthat would extinguish the debtor's liability should beactually cashed. If the delivery of the check of PlasticEra to Capital Insurance were to be viewed in thelight of the foregoing, no payment of the premiumhad been effected. Significantly, Capital Insuranceaccepted the promise of Plastic Era to pay theinsurance premium within 30 days from the effectivedate of policy. By so doing, it has implicitly agreed tomodify the tenor of the insurance policy and ineffect, waived the provision therein that it would onlypay for the loss or damage in case the same occursafter the payment of the premium. Considering thatthe insurance policy is silent as to the mode ofpayment, Capital Insurance is deemed to haveaccepted the promissory note in payment of thepremium. This rendered the policy immediatelyoperative on the date it was delivered. 2. YES - Although the check was due for payment onJanuary 16, 1961 and Plastic Era had sufficient fundsto cover it as of January 19, 1961, Capital Insurancedecided to hold the same for thirty-five (35) daysbefore presenting it for payment. Having held thecheck for such an unreasonable period of time,Capital Insurance was estopped from claiming aforfeiture of its policy for non-payment even if thecheck had been dishonored later. Where the check is held for an unreasonable time before presenting itfor payment, the insurer may be held estopped fromclaiming a forfeiture if the check is dishonored. Disposition The decision of the CA is AFFIRMEDin toto.

MALAYAN INSURANCE CO., INC. v. ARNALDO and PINCA 154 SCRA 672 CRUZ; October 12, 1987 FACTS - On June 7, 1981, Malayan Insurance Co. (MICO),issued fire insurance for the amount of P14,000 onthe property of private respondent, Pinca, effectiveJuly 1981-1982. MICO later allegedly cancelled thepolicy for non-payment of the premium and sent anotice to Pinca. On Dec. 24 Adora, an agent of MICO,received Pincas payment, which was remitted toMICO. On Jan. 18, 1982, Pincas property wascompletely burned. On Feb. 5, MICO returned Pincaspayment to Adora on the ground that her policy hadbeen cancelled; the latter refused to accept it. Herdemand for payment having been rejected by MICO,Pinca went to the Insurance Commission. Publicrespondent Arnaldo, the Insurance Commissioner,sustained Pinca, hence this petition from MICO.Records show MICO received Arnaldos decision onApril 10; MICO filed a MFR on April 25 which wasdenied on June 4; MICO received notice of this denialon June 14; instant petition was filed on July 2. ISSUES Procedural 1.WON the petition should be dismissed for late filing Substantive 2.WON there was a valid insurance contract at the time of the loss 3. WON Adora was authorized to receive such payment 4. WON an adjuster is indispensable in the valuation of the loss HELD Procedural 1. YES - Petitioner invokes Sec 416 of the Insurance Codewhich grants it 30 days from notice of the InsuranceCommission within which to appeal by certiorari withthe Court. MICO filed its MFR on April 25, 15 daysafter the notice; the reglementary period began to run again after June 13. Since the petition was filedonly on July 2, it was tardy by 4 days. Alternatively itinvokes Rule 45 of the Rules of Court for certioraribut the petition still exceeds the 15 day limit fromthe June 13 notice. -Respondents, on the other hand, invoke Sec. 39 ofB.P. 129 which pegs the period for appeal fromdecisions of any court in all cases at 15 days fromthe notice of the decision appealed from. Since theMFR was filed only 15 days after receiving notice ofthe decision, it was already 18 days late by July 2.So whichever is applied, the petition is still late. Substantive 2. YES - A valid cancellation requires the followingconditions based on Sections 64-65 of the Code:prior notice which must be based on the occurrenceof one or more of the grounds mentioned in Sec 64(in this case, non-payment of premium), after theeffective date of the policy; the notice must bewritten and mailed to the address on the policy; itmust state the ground(s) for cancellation and theinsurer must furnish details upon the request of theinsured. - It is undisputed that payment of premium wasmade. Petitioner relies heavily on Sec 77 of theInsurance Code to contest this, the said provisionrequiring payment of premium as soon as the thingis exposed to the peril insured against and that thepolicy is invalid without it. However, this is notapplicable in the instant case as payment waseventually made. It is to be noted that the premiuminvoice was stamped Payment Received, indicatingan understanding between the parties that paymentcould be made later. This is furthered by the fact thatAdora had earlier told her to call him anytime shewas ready with her payment. The Court also finds itstrange that MICO only sought to return Pincas Jan.15 payment only on Feb. 5, long after her house hadburned downthis makes petitioners motives highlysuspect. - MICO claims to have sent a notice to Pinca, whoflatly denied receiving one. Pinca did not have toprove this since the strict language of Sec 64requires that MICO ensure the cancellation wasactually sent to and received by the insured.

- MICO also suggests that Pinca knew the policy hadbeen cancelled and was paying the premium in orderto renew the policy. A close study of the transcriptsshow, however, that Pinca only meant to renew thepolicy had it been cancelled but not if it was still ineffect it was conditional. Payment was thus legally made on the original transaction and validly receivedby Adora, who was not informed of the allegedcancellation and thus saw no reason to reject thepayment. 3. YES - Sec. 306 of the Insurance Code provides that anyinsurance company that delivers a policy to its agentis deemed to have authorized such agent to receivepayment of premium on its behalf. It is a well-knownprinciple under the law of agency that payment to anauthorized agent is equivalent to payment to theprincipal himself. MICOs acknowledgement of Adoraas its agent thus defeats its contention that he wasnot authorized to receive payments on its behalf. 4. NO - In absence of fraud, the amount of the loss may bedetermined on the basis of such proof offered by theinsured. Here. The certification of the IntegratedNational Police as the extent of the loss should suffice. Disposition petition is DENIED

MANUFACTURERS LIFE INSURANCE CO. v . MEER 89 PHIL 351 BENGZON, June 29, 1951 NATURE APPEAL from a judgment of the Court of First Instance of Manila FACTS (this is a tax case. Whats really important here is the definition of CASH SURRENDER VALUE). - Manufacturers Life Insurance Company is a dulyorganized corporation which has its head office atToronto. It is duly registered and licensed to engagein life insurance business in the Philippines, and,maintains a branch office in Manila. It was engagedin such business in the Philippines for more than fiveyears before and including the year 1941. But due tothe exigencies of the war It closed the branch officeat Manila during 1942 up to September 1945. - Plaintiff issued a number of life insurance policies inthe Philippines containing stipulations referred to asNONFORFEITURE CLAUSES5 - From January 1, 1942 to December 31, 1946,Plaintiff head office at Toronto applied the provisionsof the automatic premium loan clauses upon thenonpayment of the corresponding premiums by thepeople who subscribed to the insurance. The netamount of premiums advanced (by the company) orloaned (to the insured) as payment for the premiumdue totaledP1,069,254.98. - Meer, the Collector of the National Internal Revenueassessed the net amount of premium at P17,917.12pursuant to SEC.255, National Internal Revenue Code6 - Company protested the assessment, but paid thetaxes anyway. Then they filed a complaint to recovermoney paid under protest for taxes - CFI: Dissmiss complaint - PLAINTIFFs MAIN CONTENTION: when it madepremium loans or premium advances by virtue of thenon-forfeiture clauses, it did not collect premiumswithin the meaning of the above sections of the law,and therefore it is not amenable to the tax thereinprovided. ISSUES 1. WON premium advances made by plaintiff-appellant under the automatic premium loan clauseof its policies are premiums collected' by theCompany subject to tax 2. WON, in the application of the automatic premiumloan clause of plaintiff-appellant's policies, there is'payment in money, notes, credits, or any substitutesfor money 3. WON the collection of the alleged deficiency premium taxes constitutes double taxation 4. WON the making of premium advances, grantingfor the sake of argument that it amounted tocollection of premiums, were done in Toronto, Canada 5. WON the fact that plaintiff-appellant was not doingbusiness in the Philippines during the period fromJanuary 1, 1942 to September 30, 1945, inclusive,exempts it from payment of premium taxescorresponding to said period HELD NOTE (example given by the plaintiff): "Suppose that 'A', 30 years of age, secures a 20-yearendowment policy for P5,000 from plaintiffappellantCompany and pays an annual premium of P250. 'A'pays the first ten yearly premiums amounting toP2,500 and on this amount plaintiff-appellant paysthe corresponding taxes under section 255 of theNational Internal Revenue Code. Suppose also thatthe cash value of said policy after the payment of the10th annual premium amounts to P1,000." When onthe eleventh year the annual premium fell due andthe insured remitted no money within the mouthgrace, the insurer treated the premium then overdue as paid from the cash value, the amount being a loan to the policyholder1 who could discharge it at any time with interest at 6 per cent. The insurancecontract, therefore, continued in force for theeleventh year. 1. YES - Based on the example given by the plaintiff, theinsurer collected the amount of P250 as the annualpremium for the eleventh year on the said policywhen it loaned to A the sum of P250. The insurerbecame a creditor of theloan, but not of the premium that had already been paid (advanced by the insurer). The insurer is entitled to collect intereston the loan, not on the premium. "A" paid thepremium for the eleventh year; but in turn hebecame a debtor of the company for the sum ofP250. This debt he could repay either by laterremitting the money to the insurer or by letting thecash value

compensate for it. The debt may also bededucted from the amount of the policy should "A"die thereafter during the continuance of the policy. - ON ARGUMENT THAT THE ASSETS OF THE INSURER REMAINED THE SAME AFTER THE APPLICATION OF THE AUTOMATIC PREMIUM LOAN CLAUSE: there was an increase in assets in the form ofCREDIT for the advances made (in the example, theP250 for the 11th year). - ON ARGUMENT THAT IF THE CREDIT IS PAID OUT OF THE CASH SURRENDER VALUE, THERE WERE NO NEW FUNDS ADDED TO THE COMPANY'S ASSETS: Cash surrender value "as applied to a life insurance policy, is the amount of money the company agrees to pay to the holder of the policy if he surrenders it and releases his claims upon it. Themore premiums the insured has paid the greater willbe the surrender value; but the surrender value isalways a lesser sum than the total amount ofpremiums paid." (Cyclopedia Law Dictionary 3d. ed.1077.) The cash value or cash surrender value istherefore an amount which the insurance companyholds In trust for the insured to be delivered to himupon demand. It is therefore a liability of thecompany to the insured. Now then, when thecompany's credit for advances is paid out of the cashvalue or cash surrender value, that value and thecompany's liability is thereby diminished pro tanto. 2. YES - the insurer agreed to consider the premium paid onthe strength of the automatic loan. The premium wastherefore paid by means of a "note" or "credit" or"other substitute for money" and the tax is duebecause section 255 above quoted levies taxesaccording to the total premiums collected by theinsurer "whether such premiums are paid in money,notes, credits or any substitute for money. 3. NO No constitutional prohibition against double taxation. 4. NO - The loans are made to policyholders in thePhilippines, who in turn pay therewith the premiumto the insurer thru the Manila branch. Approval ofappellant's position will enable foreign insurers toevade the tax by contriving to require that premiumpayments shall be made at their head offices. Whatis important, the law does not contemplatepremiums collected in the Philippines. It is enoughthat the insurer is doing insurance business in thePhilippines, irrespective of the place of itsorganization or establishment. 5. NO - Although during those years the appellant was notopen for new business because its branch office wasclosed, still it was practically and legally, operating inthis country by collecting premiums on itsoutstanding policies, incurring the risks and/orenjoying the benefits consequent thereto, withouthaving previously taken any steps indicatingwithdrawal in good faith from this field of economicactivity. Disposition finding no prejudicial error in the appealed decision, we hereby affirm it with costs.

ANDRES v. CROWN LIFE INSURANCE 102 Phil. 919 REYES, J.B.L., Jan.28, 1958 NATURE Appeal from judgment of CFI FACTS - Feb. 13, 1950: For the sum of P5,000, defendant-appellee Crown Life issued an insurance policy in thename of plaintiff-appellant Rufino and his wife, withthe stipulation that the premiums are to be paidsemi-annually. - The premiums for the 1st and 2nd semester of the 1styear, in the amount of P165.15 were paid by Rufinobut the premium for the third semester, in the sameamount,was not paid. - Jan. 6, 1951, Crown Life, through its branch secretary, wrote to Mr. and Mrs. Andres advisingthem that their insurance policy lapsed on Dec. 26,1950 and the amount of P165.15 was overdue,giving them 60 days from the date of lapse to file anapplication for reinstatement. Crown Life later sentanother letter telling the spouses Andres that theirinsurance policy was no longer in force. - Feb. 1951: Plaintiff and his wife executed a Statement of Health and application for reinstatement of the aforesaid policy. - Feb. 20, 1951: Plaintiff wrote a letter to thedefendant, enclosed with a money order for P100.Upon acceptance, defendant advised Rufino that itsmain office had approved the application and thatthe reinstatement of the lapsed policy was subject tothe payment of the remaining premium balance ofP65.15. - May 3, 1951: Severa Andres died of dystocia, contracted pelvis. - May 5, 1951: Plaintiff sent a letter enclosed with amoney order in the amount of P65, for the remainingbalance due. - May 15, 1951: Defendant sent a letter with officialreceipt of the P165.15 paid by Rufino as well as aCertificate of Reinstatement. - June 7, 1951: Rufino presented a death claim as survivor-beneficiary of his deceased wife. Payment was denied by the defendant. - April 1952: Rufino filed a complaint in CFI against Crown Life for the recovery of the amount of P5,000as the face value of a joint 20-year endowmentinsurance policy issued by defendant in favor ofplaintiff and his wife, on Feb. 13, 1950. In its answer,Crown Life disclaimed liability and set forth thespecial defense that the aforementioned policy hadalready lapsed. - Aug. 5, 1954: CFI rendered a decision absolving the defendant company from any liability on theground that the policy had lapsed and it was notreinstated at the time of the plaintiffs wifes death.Plaintiff later appealed to the CA but the same wascertified by the CA to the SC for having no questionof fact. ISSUE WON the insurance policy, which has been in a stateof lapse before May 3, 1951, has been validly andcompletely reinstated after said date(Was there aperfected contract of reinstatement after the policylapsed due to non-payment of premiums?) HELD NO Ratio The stipulation in a life insurance policy giving the insured the privilege to reinstate it upon writtenapplication does not give the insured absolute rightto such reinstatement by the mere filing of anapplication. The Company has the right to deny thereinstatement if it is not satisfied as to theinsurability of the insured and if the latter does notpay all overdue premiums and all other indebtednessto the Company. After the death of the insured theinsurance Company cannot be compelled to entertainan application for reinstatement of the policybecause the conditions precedent to reinstatementcan no longer be determined and satisfied. Reasoning - The stipulations of facts render it undisputable that

the original policy lapsed for non-payment ofpremiums on Dec. 26, 1950, upon expiration of the31day grace period. - As found by the lower court, the conditions setforth in the policy for reinstatement as provided inthe contract itself are the following: (A) applicationshall be made within 3 years from the date of lapse;(B) there should be a production of evidence of thegood health of the insured; (C) if the rate ofpremium depends upon the age of the Beneficiary,there should likewise be a production of evidence ofhis or her good health; (D) there should be presented such other evidence of insurability at thedate of application for reinstatement; (E) thereshould be no change which has taken place in suchgood health and insurability subsequent to the dateof such application and before the policy isreinstated; and (F) all overdue premiums and otherindebtedness in respect of the policy, together withinterest at 6%, compounded annually, should first bepaid. - The plaintiff did not comply with the last condition;for he only paid P100 before his wifes death; anddespite the Companys reminders, he only remittedthe balance of P65.15 two days a fter his wife died.On the face of such facts, the Company had the rightto treat the contract as lapsed and refuse payment ofthe policy. - Rufino contends that the condition regardingpayment of the premium was waived by theinsurance Company through its letters, wherein itmade statements such as: If you are unable to paythe full amount immediately, send as large amountas possible and advise us how soon you expect to beable to pay the balance; we will work out anadjustment most beneficial to you. The Court f oundthe statements to be too vague and indefinite toindicate an intention on the insurers part to waivethe full payment as prerequisite to the reinstatementof the lapsed policy. The Court reiterated the rulethat a waiver must be clear and positive, the intentto waive shown clearly and convincingly.On theother hand, It found subsequent letters sent bydefendant indicating that they insisted on fullpayment of the premium before the policy wasreinstated and that defendant did not consider partialpayment as sufficient consideration for thereinstatement. Plaintiff-Appellants failure to remitthe balance before the death of his wife operated todeprive him of any right to waive the policy andrecover the face value thereof. Disposition Judgment appealed from is affirmed.

VALENZUELA v. CA (PHILIPPINE AMERICAN GENERAL INSURANCE COMPANY, INC.) 191 SCRA 1 GUTIERREZ; October 19, 1990 NATURE Petition for review of the decision of theca. FACTS - Petitioner Arturo P. Valenzuela is a General Agent of private respondent Philippine American General Insurance Company, Inc. (Philamgen for short) since1965. As such, he was authorized to solicit and sellin behalf of Philamgen all kinds of non-life insurance,and in consideration of services rendered wasentitled to receive the full agent's commission of32.5% from Philamgen under the scheduledcommission rates. - From 1973 to 1975, Valenzuela solicited marineinsurance from one of his clients, the Delta Motors,Inc. (Division of Electronics Airconditioning andRefrigeration) in the amount of P4.4 Million fromwhich he was entitled to a commission of 32%.However, Valenzuela did not receive his fullcommission which amounted to P1.6 Million from theP4.4 Million insurance coverage of the Delta Motors.During the period 1976 to 1978, premium paymentsamounting to P1,946,886.00 were paid directly toPhilamgen and Valenzuela's commission to which heis entitled amounted to P632,737.00. - In 1977, Philamgen started to become interested inand expressed its intent to share in the commissiondue Valenzuela on a fifty-fifty basis. Valenzuelarefused. - Because of the refusal of Valenzuela, Philamgenand its officers took drastic action againstValenzuela. They: (a) reversed the commission duehim by not crediting in his account the commissionearned from the Delta Motors, Inc. insurance ; (b)placed agency transactions on a cashand-carrybasis; (c) threatened the cancellation of policiesissued by his agency; and (d) started to leak outnews that Valenzuela has a substantial account withPhilamgen. All of these acts resulted in the decline ofhis business as insurance agent. - Then on December 27, 1978, Philamgen terminated the General Agency Agreement of Valenzuela. - Lower court: the termination of Valenzuela asGeneral Agent was improper because the record willshow the principal cause of the termination of theplaintiff as General Agent of defendant Philamgenwas his refusal to share his Delta commission. -CA: In any event the principal's power to revoke an agency at will is so pervasive, that the SupremeCourt has consistently held that termination may beeffected even if the principal acts in bad faith,subject only to the principal's liability for damages.(CA ordered Valenzuela to pay Philamgen the amountof One Million Nine Hundred Thirty-Two ThousandFive Hundred Thirty-Two and 17/100 Pesos(P1,932,532.17) with legal interest) ISSUES 1. WON whether or not Philamgen and/or its officerscan be held liable for damages due to thetermination of the General Agency Agreement itentered into with the petitioners 2. WON petitioners are liable to Philamgen for the unpaid and uncollected premiums HELD 1. YES - If a principal acts in bad faith and with abuse ofright in terminating the agency, then he is liable indamages. - There is an exception to the principle that anagency is revocable at will and that is when theagency has been given not only for the interest ofthe principal but for the interest of third persons orfor the mutual interest of the principal and the agent.In these cases, it is evident that the agency ceasesto be freely revocable by the sole will of the principal(PROCEDURAL: Where the findings of the Court ofAppeals and the trial court are contrary to eachother, this Court may scrutinize the evidence on record - After a painstaking review of the entire records ofthe case and the findings of facts of both the court aquo and respondent appellate court, the Courtaffirmed the trial courts findings.) - The principal cause of the termination of Valenzuelaas General Agent of Philamgen arose from his refusalto share his Delta commission. The records sustainthe conclusions of the trial court on the apparent badfaith of the private respondents in terminating theGeneral Agency Agreement of petitioners. - It is also evident from the records that the agencyinvolving petitioner and private respondent is one"coupled with an interest," and, therefore, should notbe freely revocable at the unilateral will of the latter.- The private respondents by the simple expedient ofterminating the

General Agency Agreementappropriated the entire insurance business ofValenzuela. With the termination of the GeneralAgency Agreement, Valenzuela would no longer beentitled to commission on the renewal of insurancepolicies of clients sourced from his agency. Worse,despite the termination of the agency, Philamgencontinued to hold Valenzuela jointly and severallyliable with the insured for unpaid premiums. Underthese circumstances, it is clear that Valenzuela hadan interest in the continuation of the agency when itwas unceremoniously terminated not only because ofthe commissions he should continue to receive fromthe insurance business he has solicited and procured but also for the fact that by the very acts of therespondents, he was made liable to Philamgen in theevent the insured fail to pay the premiums due. Theyare estopped by their own positive averments andclaims for damages. - "The principal may not defeat the agent's right toindemnification by a termination of the contract ofagency (Erskine v. Chevrolet Motors Co. 185 NC 479,117 SE 706, 32 ALR 196). - For the pivotal factor rendering Philamgen and the other private respondents liable in damages is that the termination by them of the General Agency Agreement was tainted with bad faith. This is in accordance with the precepts in Human Relations enshrined in our Civil Code. 2. NO. The respondent court erred in holdingValenzuela liable. There was no factual and legalbasis for the award. Under Section 77 of the Insurance Code, the remedy for the non-payment of premiums is to put an end to and render theinsurance policy not binding - "Sec. 77 . . .[N]otwithstanding any agreement to the contrary, nopolicy or contract of insurance is valid and bindingunless and until the premiums thereof have beenpaid except in the case of a life or industrial lifepolicy whenever the grace period provision applies(P.D. 612, as amended otherwise known as theInsurance Code of 1974) - This is buttressed by Section 776 of the Insurance Code (Presidential Decree No. 612, promulgated on December 18, 1974), which nowprovides that no contract of Insurance by aninsurance company is valid and binding unless anduntil the premium thereof has been paid,notwithstanding any agreement to the contrary." Disposition Petition is GRANTED. CA decision SET ASIDE. The decision of the TC REINSTATED with the MODIFICATIONS. And that the contractualrelationship between Arturo P. Valenzuela andPhilippine American General Insurance Companyshall be deemed terminated upon the satisfaction ofthe judgment as modified.

CHAPTER V THE POLICY, PARTIES THERETO, & RIGHTS THEREON DE LIM v. SUN LIFE ASSURANCE COMPANY OF CANADA 41 PHIL 263 MALCOLM; November 29, 1920 NATURE Appeal from an order of the CFI of Zamboangasustaining a demurrer to plaintiff's complaint uponthe ground that it fails to state a cause of action. FACTS - On July 6, 1917, Luis Lim of Zamboanga madeapplication to the Sun Life Assurance Company ofCanada for a policy of insurance on his life in thesum of P5,000. In his application Lim designated hiswife, Pilar de Lim, the plaintiff herein, as thebeneficiary. The first premium of P433 was paid byLim, and upon such payment the company issuedwhat was called a ''provisional policy." Luis Lim diedon August 23, 1917, after the issuance of theprovisional policy but before approval of theapplication by the home office of the insurancecompany. Pilarde Lim brought an action to recoverfrom the Sun Life sum of P5,000, the amount namedin the provisional policy. - The "provisional policy" reads: "Received (subjectto the following stipulations and agreements) thesum of P433, being the amount of the first year'spremium for a Life Assurance Policy on the life of Mr.Luis D. Lim of Zamboanga for P5,000, for which anapplication dated the 6th day of July, 1917, has beenmade to the Sun Life Assurance Company of Canada.- The above-mentioned life is to be assured inaccordance with the terms and conditions containedor inserted by the Company in the policy which maybe granted by it in this particular case for fourmonths only from the date of the application, provided that the Company shall confirm this agreement by issuing a policy on said applicationwhen the same shall be submitted to the Head Officein Montreal. Should the Company not issue such apolicy, then this agreement shall be null and void ab initio, and the Company shall be held not to have been on the risk at all, but in such case the amount herein acknowledged shall be returned. ISSUE WON the contract of insurance between Luis Lim andSun Life Assurance Company of Canada was perfected HELD NO. - The document it is to be a provisional policy "for four months only from the date of this application."Immediately following the words fixing the fourmonths period comes the word "provided" which hasthe meaning of "if." Otherwise stated, the policy for four months is expressly made subject to theaffirmative condition that the company shall confirmthis agreement by issuing a policy on said applicationwhen the same shall be submitted to the head officein Montreal. To re-enforce the same there follows thenegative condition -"Should the company not issuesuch a policy, then this agreement shall be null andvoid ab initio, and the company shall be held not tohave been on the risks." Certainly language couldhardly be used which would more clearly stipulatethat the agreement should not go into effect until thehome office of the company should confirm it byissuing a policy. As we read and understand the so-called provisional policy, it amounts to nothing but an acknowledgment on behalf of the company, thatit has received from the person named therein thesum of money agreed upon as the first year'spremium upon a policy to be issued upon theapplication, if the application is accepted by thecompany. - It is of course a primary rule that a contract ofinsurance, like other contracts, must be assented toby both parties either in person or by their agents. So long as an application for insurance has not beeneither accepted or rejected, it is merely an offer orproposal to make a contract. The contract, to bebinding from the date of the application must havebeen a completed contract, one that leaves nothingto be done, nothing to be completed, nothing to bepassed upon, or determined, before it shall takeeffect. There can be no contract of insurance unlessthe minds of the parties have met in agreement. Our view is, that a contract of insurance was not here consummated by the parties.

- The trial court committed no error in sustaining thedemurrer and dismissing the case. It is to be noted,however that counsel for appellee admits the liabilityof the company for the return of the first premium to the estate of the deceased.

GREAT PACIFIC LIFE v. CA (LEUTERIO) 316 SCRA 677 QUISUMBING; October 13, 1999 NATURE Petition for Review of CA decision FACTS - A contract of group life insurance was executed between petitioner Great Pacific Life Assurance Corporation (hereinafter Grepalife) and Development Bank of the Philippines (hereinafter DBP). Grepalife agreed to insure the lives of eligible housing loan mortgagors of DBP. - In Nov. 1983, Dr. Wilfredo Leuterio, a physician and a housing debtor of DBP applied for membership in the group life insurance plan. In an application form, Dr. Leuterio answered Qs concerning his health condition as follows: Q:Have you ever had, or consulted, a physician fora heart condition, high blood pressure, cancer,diabetes, lung, kidney or stomach disorder or anyother physical impairment? No. Q: Are you now, to the best of your knowledge, ingood health? Yes. - Grepalife issued an insurance coverage of Dr. Leuterio, to the extent of his DBP mortgage indebtedness of P86,200.00. In Aug. 1984, Dr. Leuterio died due to "massive cerebral hemorrhage."DBP submitted a death claim to Grepalife. Grepalife denied the claim because Dr. Leuterio was not physically healthy when he applied for an insurance. Grepalife insisted that Dr. Leuterio did not disclose he had been suffering from hypertension, which caused his death. Allegedly, such non-disclosure constituted concealment that justified the denial of the claim. - Herein respondent Medarda Leuterio, widow, filed acomplaint with RTC against Grepalife for "Specific Performance with Damages." Dr. Mejia, who issued the death certificate, testified that Dr. Leuterio complained of headaches presumably due to highblood pressure. The inference was not conclusive because Dr. Leuterio was not autopsied, hence, other causes were not ruled out. - RTC ruled in favor of respondent widow and against Grepalife. CA sustained the RTC decision. Hence, the present petition. ISSUES 1. WON CA erred in holding petitioner liable to DBP as beneficiary in a group life insurance contract from a complaint filed by the widow of the decedent/mortgagor 2. WON CA erred in not finding that Dr. Leuterio concealed that he had hypertension, which would vitiate the insurance contract 3. WON CA erred in holding Grepalife liable for P86,200.00 without proof of the actual outstanding mortgage payable by the mortgagor to DBP HELD 1. NO Ratio: Insured, being the person with whom the contract was made, is primarily the proper person to bring suit. Subject to some exceptions, insured may thus sue, although the policy is taken wholly or in part for the benefit of another person named or unnamed, and although it is expressly made payable to another as his interest may appear or otherwise. Although a policy issued to a mortgagor is taken out for the benefit of the mortgagee and is made payable to him, yet the mortgagor may sue thereon in his own name, especially where the mortgagee's interestis less than the full amount recoverable under the policy. (See Sec. 8, Insurance Code) Reasoning: [a] The insured private respondent did not cede to the mortgagee all his rights or interests in the insurance, the policy stating that: In the event ofthe debtor's death before his indebtedness with the Creditor (DBP) shall have been fully paid, an amount to pay the outstanding indebtedness shall first be paid to the creditor and the balance of sum assured, if there is any, shall then be paid to the beneficiary/ies designated by the debtor. When DBP submitted the insurance claim against

Grepalife, the latter denied payment thereof, interposing the defense of concealment committed by the insured. Thereafter, DBP collected the debt from the mortgagor and took the necessary action of foreclosure on the residential lot of private respondent. [b] Since a policy of insurance upon life or health may pass by transfer, will or succession to any person, whether he has an insurable interest or not, and such person may recover it whatever the insured might have recovered, the widow of the decedent Dr. Leuterio may file the suit against the insurer, Grepalife. 2. NO Ratio: The fraudulent intent on the part of the insured must be established to entitle the insurer to rescind the contract. Misrepresentation as a defense of the insurer to avoid liability is an affirmative defense and the duty to establish such defense by satisfactory and convincing evidence rests upon the insurer. In the case at bar, the petitioner failed to clearly and satisfactorily establish its defense, and is therefore liable to pay the proceeds of the insurance. Reasoning: [a] The insured, Dr. Leuterio, had answered in his insurance application that he was in good health and that he had not consulted a doctor or any of the enumerated ailments, including hypertension; when he died the attending physician had certified in the death certificate that the former died of cerebral hemorrhage, probably secondary to hypertension. From this report, petitioner Grepalife refused to pay the insurance claim. It alleged that the insured had concealed the fact that he had hypertension. [b] Contrary to Grepalifes allegations, there was no sufficient proof that the insured had suffered from hypertension. Aside from the statement of the insured's widow who was not even sure if the medicines taken by Dr. Leuterio were for hypertension, the appellant had not proven nor produced any witness who could attest to Dr.Leuterio's medical history. [c] Grepalife had failed to establish that there was concealment made by the insured, hence, it cannot refuse payment of the claim. 3. NO - Considering the supervening event that DBP foreclosed in 1995 their residential lot, in satisfaction of mortgagor's outstanding loan, the insurance proceeds shall inure to the benefit of the heirs of the deceased person or his beneficiaries. Equity dictates that DBP should not unjustly enrich itself at the expense of another. Hence, it cannot collect the insurance proceeds, after it already foreclosed on the mortgage. The proceeds now rightly belong to Dr. Leuterio's heirs represented by his widow, herein private respondent. - The Court ruled this issue based on the clear provisions of the policy. The mortgagor paid the premium according to the coverage of his insurance, which states that: "The policy states that upon receipt of due proof of the Debtor's death during the terms of this insurance, a death benefit in the amount of P86,200.00 shall be paid In the event of the debtor's death before his indebtedness with the creditor shall have been fully paid, an amount to pay the outstanding indebtedness shall first be paid to the Creditor and the balance of the Sum Assured, if there is any shall then be paid to the beneficiary/ies designated by the debtor." From this, it is clear that Grepalife is liable and that Dr. Leuterios heirs must get the proceeds. Disposition: Petition DENIED. CA Decision AFFIRMED with modification.

PACIFIC TIMBER EXPORT CORPORATION v. CA (WORKMENS INSURANCE CO) 112 SCRA 199 DE CASTRO; February 25, 1982 FACTS -March 19, 1963 - the plaintiff secured temporaryinsurance from the defendant for its exportation of1,250,000 board feet of Philippine Lauan and Apitonglogs to be shipped from the Diapitan Bay, Quezon toOkinawa and Tokyo, Japan. The defendant issued onsaid date Cover Note No. 1010, insuring the saidcargo of the plaintiff "Subject to the Terms andConditions of the WORKMEN'S INSURANCECOMPANY, INC. printed Marine Policy form as filedwith and approved by the Office of the InsuranceCommissioner. -April 2, 1963 - The two (2) regular marine cargopolicies were issued by the defendant in favor of theplaintiff. The total cargo insured under the twomarine policies accordingly consisted of 1,395 logs,or the equivalent of 1,195,498 bd. ft. -After the issuance of cover note but before theissuance of the two marine policies some of the logsintended to be exported were lost during loadingoperations in the Diapitan Bay due to bad weather. - April 4, 1963 - The plaintiff informed the defendantabout the loss of 'approximately 32 pieces of logs'during loading through a letter. -The plaintiff subsequently submitted a 'ClaimStatement' demanding payment of the loss under thesecond marine cargo policy. -July 17, 1963 -the defendant requested the FirstPhilippine Adjustment Corporation to inspect the lossand assess the damage. -August 23, 1963 -the adjuster reported that 'theloss of 30 pieces of logs is not covered by the twopolicies inasmuch as said policies covered the actualnumber of logs loaded on board.But it is covered byCover Note. -On January 13, 1964 - the defendant wrote theplaintiff denying the latter's claim, on the groundthat defendant's investigation revealed that theentire shipment of logs covered by the two marinespolicies were received in good order at their point ofdestination. It was further stated that the said lossmay not be considered as covered under cover notebecause the said note had become 'null and void byvirtue of the issuance of two marine policies. -The CFI of Manila ruled in favour of the petitioner. -The Court of Appeals reversed the decision of the CFI. ISSUES 1. WON the cover note is null and void for lack of valuable consideration because no separate premiums are collected by private respondent on all its cover notes. 2. WON the court of appeals erred in holding thatprivate respondent was released from liability underthe cover note due to unreasonable delay in givingnotice of loss because the court disregarded theproven fact that private respondent did not promptlyand specifically object to the claim on the ground ofdelay in giving notice of loss and, consequently,objections on that ground are waived under section84 of the insurance act. HELD 1. NO Ratio: Cover note is issued with a consideration when, by express stipulation, the cover note is madesubject to the terms and conditions of the marinepolicies, and the payment of premiums is one of theterms of the policies.

Reasoning: a.) The cover note in question is subject to the terms and conditions of the marine policies b.) Nature of the Cover Note:The fact that noseparate premium was paid on the Cover Note beforethe loss insured against occurred, does not militateagainst the validity of petitioner's contention, for nosuch premium could have been paid, since by thenature of the Cover Note, it did not contain, as allCover Notes do not contain particulars of theshipment that would serve as basis for thecomputation of the premiums. As a logicalconsequence, no separate premiums are intended orrequired to be paid on a Cover Note. c.) The petitioner paid in full all the premiums ascalled for by the statement issued by privaterespondent after the issuance of the two regularmarine insurance policies, thereby leaving noaccount unpaid by petitioner due on the insurancecoverage, which must be deemed to include theCover Note. If the Note is to be treated as a separatepolicy instead of integrating it to the regular policiessubsequently issued, the purpose and function of theCover Note would be set at naught or renderedmeaningless, for it is in a real sense a contract, not amere application for insurance which is a mere offer.Had all the logs been lost during the loadingoperations, but after the issuance of the Cover Note,liability on the note would have already arisen evenbefore payment of premium. This is how the covernote as a "binder" should legally operate; otherwise,it would serve no practical purpose in the realm ofcommerce, and is supported by the doctrine that where a policy is delivered without requiringpayment of the premium, the presumption is that acredit was intended and policy is valid. 2. NO - The private respondent company never raised thisground in the proceedings.It must be because it didnot find any delay, as this Court fails to find a realand substantial sign thereof. But even on theassumption that there was delay, this Court issatisfied and convinced that as expressly provided bylaw, waiver can successfully be raised against privaterespondent. Thus Section 84 of the Insurance Actprovides: "Section 84. - Delay in the presentation to aninsurer of notice or proof of loss is waived ifcaused by any act of his or if he omits to takeobjection promptly and specifically upon thatground." - From what has been said, We find duly substantiated petitioner's assignments of error. Disposition: The appealed decision is set aside and the decision of the Court of First Instance is reinstated in toto with the affirmance of this Court.

DEVELOPMENT INSURANCE v. IAC (PHIL UNION REALTY DEVELOPMENT CORP) 143 SCRA 62 CRUZ; July 16, 1986 FACTS - A fire occurred in the building of the privaterespondent and it sued for recovery of damages fromthe petitioner on the basis of an insurance contractbetween them. The petitioner allegedly failed toanswer on time and was declared in default by TC. Ajudgment of default was rendered on the strength ofthe evidence submitted ex parte by the privaterespondent, which was allowed full recovery of itsclaimed damages. - On learning of this decision, the petitioner movedto lift the order of default, invoking excusableneglect, and to vacate the judgment by default. Itsmotion was denied. - On appeal, IAC affirmed the TC decision in toto. ISSUE 1. WON default of petitioner is based on excusable neglect 2. What is the amount of indemnity due to theprivate respondent under its insurance contract?WON CFI was correct in interpreting the contract HELD 1. NO - Summons was served through its vice-president.There were even several extensions to the originalperiod to answer. As a consequence, the TC, onmotion of the private respondent filed declared thepetitioner in default. This was done almost onemonth later. Even so, the petitioner made no moveat all for two months thereafter. It was only morethan one month after the judgment of default wasrendered by the TC that it filed a motion to lift theorder of default and vacate the judgment by default.- There is a pattern of inexcusable neglect. 2. The policy is an open policy which means that theactual loss, as determined, will represent the totalindemnity due the insured from the insurer exceptonly that the total indemnity shall not exceed theface value of the policy. - The petitioner argues that since at the time of thefire the building insured was worth P5,800,000.00,the private respondent should be considered its owninsurer for the difference between that amount andthe face value of the policy and should share pro ratain the loss sustained. Accordingly, the privaterespondent is entitled to an indemnity of onlyP67,629.31, the rest of the loss to be shouldered byit alone. The petitioner cites Condition 17 of thepolicy, which provides: "If the property hereby insured shall, at thebreaking out of any fire, be collectively of greatervalue than the sum insured thereon then theinsured shall be considered as being his owninsurer for the difference, and shall bear a ratableproportion of the loss accordingly. Every item, ifmore than one, of the policy shall be separatelysubject to this condition." - However, there is no evidence on record that thebuilding was worth P5,800,000.00 at the time of theloss. On the contrary, the building was insured atP2,500,000.00, and this must be considered, byagreement, the actual value of the property insuredon the day the fire occurred. This valuation becomeseven more believable if it is remembered that at thetime the building was burned it was still underconstruction and not yet completed. - The Court notes that the policy in this case is anopen policy and is subject to the express conditionthat: "Open Policy. This is an open policy as defined in Sec57 of the Insurance Act. In the event of loss, whether total or partial, it is understood that the amount of theloss shall be subject to appraisal and the liability ofthe company, if established, shall be limited to theactual loss, subject to the applicable terms,conditions, warranties and clauses of this Policy,and in no case shall exceed the amount of the policy." - As defined in the aforestated provision, which isnow Sec60 of the Insurance Code, "an open policy isone in which the value of the thing insured is notagreed upon but is left to be ascertained in case ofloss.". - The actual loss has been ascertained in this caseand the Court will respect such factual determinationin the absence of proof that it was arrived atarbitrarily. There is no such showing. Hence, applyingthe open policy clause as expressly agreed upon bythe parties in their contract, we hold that the privaterespondent is entitled to the payment of indemnityunder the said contract in the total amount ofP508,867.00.

- The refusal of its vice-president to receive theprivate respondent's complaint, as reported in thesheriff's return, was the first indication of thepetitioner's intention to prolong this case andpostpone the discharge of its obligation to the privaterespondent under this agreement. That intention wasrevealed further in its subsequent acts---- orinaction- --- which indeed enabled it to avoidpayment for more than five years from the filing ofthe claim against it in 1980. DispositionThe appealed decision is affirmed in full, with costs against the petitioner.

HARDING v. COMMERCIAL UNION ASSURANCE 38 PHIL 464 FISHER; August 10, 1918 FACTS - Mrs. Harding was the owner of a Studebakerautomobile; in consideration of the payment to thedefendant of the premium of P150, by said plaintiff,Mrs. Henry E. Harding, with the consent of herhusband, the defendant by its duly authorized agent,Smith, Bell & Company (limited), made its policy ofinsurance in writing upon said automobile was setforth in said policy to be P3,000 that the value ofsaid automobile was set forth in said policy to beP3,000; that on March 24, 1916, said automobilewas totally destroyed by fire; that the loss thereby toplaintiffs was the sum of P3,000. - The defendants version is that by request of Mrs.Harding, it issued the policy of insurance on anautomobile alleged by the said plaintiff to be herproperty. It was made by means of a proposal inwriting signed and delivered by said plaintiff to thedefendant, guaranteeing the truth of the statementscontained therein which said proposal is referred toin the said policy of insurance made a part thereof;that certain of the statements and representationscontained in said proposal and warranted by saidplaintiff to be true, to wit: (a) the price paid by theproposer for the said automobile; (b) the value ofsaid automobile at the time of the execution anddelivery of the said proposal and (c) the ownership ofsaid automobile, were false and known to be false bythe said plaintiff at the time of signing and deliveringthe said proposal and were made for the purpose ofmisleading and deceiving the defendant, andinducing the defendant, relying upon the warranties,statements, and representations contained in thesaid proposal and believing the same to be true,issued the said policy of insurance. - The evidence shows that Hermanos, the Manilaagents for the Studebaker automobile, sold theautomobile to Canson for P3,200 (testimony of Mr.Diehl); who sold the said automobile to HenryHarding for the sum of P1,500. Harding sold the saidautomobile to J. Brannigan for the sum of P2,000who sold the said automobile Henry Harding for thesum of P2,800; Henry Harding gave the saidautomobile to his wife as a present; that saidautomobile was repaired and repainted at the LunetaGarage at a cost of some P900; that while the saidautomobile was at the Luneta Garage; the lattersolicited of Mrs. Harding the insurance of saidautomobile by the Company; that a proposal wasfilled out by the said agent and signed by the plaintiffMrs. Henry E. Harding, and in said proposal underthe heading "Price paid by proposer," is the amountof "3,500" and under another heading "Presentvalue" is the amount of "3,000". - After the said proposal was made a representativeof the Manila agent of defendant went to the LunetaGarage and examined said automobile and Mr.Server, the General Manager of the Luneta Garage,an experienced automobile mechanic, testified thatat the time this automobile was insured it was worthabout P3,000, and the defendant, by and through itssaid agent Smith, Bell & Company (limited),thereafter issued a policy of insurance upon proposalin which policy the said automobile was described asof the "present value" of P3,000 and the said defendant charged the said plaintiff Mrs. Henry E.Harding as premium on said policy the sum of P150,or 5 per cent of the then estimated value of P3,000.- The "Schedule" in said policy of insurance describesthe automobile here in question, and provides in partof follows: "That during the period above set forth and duringany period for which the company may agree torenew this policy the company will subject to theexception and conditions contained herein orendorsed hereon indemnify the insured against lossof or damage to any motor car described in theschedule hereto (including accessories) by whatevercause such loss or damage may be occasioned andwill further indemnify the insured up to the value ofthe car or P3,000 whichever is the greater againstany claim at common law made by any person (notbeing a person in the said motor car nor in theinsured's service) for loss of life or for accidentalbodily injury or damage to property caused by thesaid motor car including law costs payable inconnection with such claim when incurred with theconsent of the company." - On March 24, 1916, the said automobile was totallydestroyed by fire, and that the iron and steelportions of said automobile which did not burn weretaken into the possession of the defendant by andthrough its agent Smith, Bell & Company (limited),and sold by it for a small sum, which had never beentendered to the plaintiff prior to the trial of this case,but in open court during the trial the sum of P10 asthe proceeds of such sale was tendered to plaintiffand refused. - Trial judge decided that there was no proof of fraudon the part of plaintiff in her statement of the valueof the automobile, or with respect to its ownership;that she had an insurable interest therein; and thatdefendant, having agreed to the estimated value,P3,000, and having insured the automobile for thatamount, upon the basis of which the premium waspaid, is bound by it and must pay the loss inaccordance with the stipulated insured value.

ISSUE 1. WON Mrs. Harding was not the owner of theautomobile at the time of the issuance of the policy,and, therefore, had no insurable interest in it 2. WON the statement regarding the cost of theautomobile was a warranty, that the statement wasfalse, and that, therefore, the policy never attachedto the risk HELD 1. NO - Article 1334 of the Civil Code which provides that"All gifts between spouses during the marriage shallbe void. Moderate gifts which the spouses bestow oneach other on festive days of the family are notincluded in this rule." - Even assuming that defendant might have invokedarticle 1334 as a defense, the burden would be uponit to show that the gift in question does not fallwithin the exception therein established. We cannotsay, as a matter of law, that the gift of an automobileby a husband to his wife is not a moderate one.Whether it is or is not would depend upon thecircumstances of the parties, as to which nothing isdisclosed by the record. - We are of the opinion that it would be unfair to holdthe policy void simply because the outlayrepresented by the automobile was made by theplaintiff's husband and not by his wife, to whom hehad given the automobile. It cannot be assumed thatdefendant should not have issued the policy unless itwere strictly true that the price representing the costof the machine had been paid by the insured and byno other person? that it would no event insure anautomobile acquired by gift, inheritance, exchange,or any other title not requiring the owner to make aspecific cash outlay for its acquisition. 2. NO - It has not been shown by the evidence that thestatement was false; on the contrary we believe thatit shows that the automobile had in fact cost morethan the amount mentioned. The court below found,and the evidence shows, that the automobile wasbought by plaintiff's husband a few weeks before theissuance of the policy in question for the sum ofP2,800, and that between that time and the issuanceof the policy some P900 was spent upon it in repairsand repainting. - The witness Server, an expert automobilemechanic, testified that the automobile waspractically as good as new at the time the insurancewas effected. The form of proposal upon which thepolicy was issued does not call for a statementregarding the value of the automobile at the time ofits acquisition by the applicant for the insurance, butmerely a statement of its cost. The amount statedwas less than the actual outlay which the automobilerepresented to Mr. Harding, including repairs, whenthe insurance policy was issued. - The court below found and the evidence shows, without dispute, that the proposal upon which the policy in question was issued was made out bydefendant's agent by whom the insurance wassolicited, and that appellee simply signed the same.It also appears that an examiner employed by thedefendant made an inspection of the automobilebefore the acceptance of the risk, and that the sumafter this examination. The trial court found that Mrs.Harding, in fixing the value of the automobile atP3,000, acted upon information given her by herhusband and by Mr. Server, the manager of theLuneta Garage. She merely repeated the informationwhich had been given her by her husband, and at thesame time disclosed to defendant's agent the sourceof her information. There is no evidence to sustainthe contention that this communication was made inbad faith.We do not think that the facts stated inthe proposal can be held as a warranty of theinsured, even if it should have been shown that theywere incorrect in the absence of proof of willfulmisstatement. Under such circumstance, theproposal is to be regarded as the act of the insurerand not of the insured. Disposition Plaintiff was the owner of the automobile in question and had an insurable interesttherein; that there was no fraud on her part inprocuring the insurance; that the valuation of theautomobile, for the purposes of the insurance, isbinding upon the defendant corporation, and that thejudgment of the court below is, therefore, correctand must be affirmed, with interest, the costs of thisappeal to be paid by the appellant.

WHITE GOLD MARINE SERVICES v. PIONEER INSURANCE 464 SCRA 448 QUISUMBING; July 28, 2005 NATURE This petition for review assails the Decision of theCourt of Appeals, affirming the Decision of theInsurance Commission.Both decisions held thatthere was no violation of the Insurance Code and therespondents do not need license as insurer andinsurance agent/broker. FACTS - White Gold procured a protection and indemnitycoverage for its vessels from Steamship Mutualthrough Pioneer Insurance.Subsequently, WhiteGold was issued a Certificate of Entry andAcceptance. Pioneer also issued receipts evidencingpayments for the coverage.When White Gold failed to fully pay its accounts, Steamship Mutual refused to renew the coverage. - Steamship Mutual thereafter filed a case againstWhite Gold for collection of sum of money to recoverthe latters unpaid balance.White Gold on the otherhand, filed a complaint before the InsuranceCommission claiming that Steamship Mutual violatedSections 186 and 187 of the Insurance Code, whilePioneerviolatedSections299, 300 and 301 inrelation to Sections 302 and 303, thereof. - The Insurance Commission dismissed thecomplaint.It said that there was no need forSteamship Mutual to secure a license because it wasnot engaged in the insurance business.It explainedthat Steamship Mutual was a Protection andIndemnity Club (P & I Club).Likewise, Pioneer neednot obtain another license as insurance agent and/ora broker for Steamship Mutual because SteamshipMutual was not engaged in the insurance business.Moreover, Pioneer was already licensed, hence, aseparate license solely as agent/broker of SteamshipMutual was already superfluous. - The Court of Appeals affirmed the decision of theInsurance Commissioner.In its decision, theappellate court distinguished between P & I Clubs vis--vis conventional insurance.The appellate court also held that Pioneer merely acted as a collection agent of Steamship Mutual. ISSUES 1.WON Steamship Mutual, a P & I Club, is engaged in the insurance business in the Philippines 2.WON Pioneer needs a license as an insurance agent/broker for Steamship Mutual HELD 1. YES - The test to determine if a contract is an insurancecontract or not, depends on the nature of thepromise, the act required to be performed, and theexact nature of the agreement in the light of theoccurrence, contingency, or circumstances underwhich the performance becomes requisite.It is notby what it is called.Basically, an insurance contractis a contract of indemnity.In it, one undertakes fora consideration to indemnify another against loss,damage or liability arising from an unknown orcontingent event. - In particular, a marine insurance undertakes toindemnify the assured against marine losses, such asthe losses incident to a marine adventure. Section 99 of the Insurance Code enumerates the coverage of marine insurance. - Relatedly, a mutual insurance company is acooperative enterprise where the members are boththe insurer and insured.In it, the members allcontribute, by a system of premiums orassessments, to the creation of a fund from which alllosses and liabilities are paid, and where the profitsare divided among themselves, in proportion to theirinterest. Additionally, mutual insurance associations,or clubs, provide three types of coverage, namely,protection and indemnity, war risks, and defense costs. - A P & I Club is a form of insurance against thirdparty liability, where the third party is anyone otherthan the P & I Club and the members. By definitionthen, Steamship Mutual as a P & I Club is a mutualinsurance association engaged in the marineinsurance business. - The records reveal Steamship Mutual is doingbusiness in the country albeit without the requisitecertificate of authority mandated by Section 187 ofthe Insurance Code.It maintains a resident agent inthe Philippines to solicit insurance and to collectpayments in its behalf.We note that SteamshipMutual even renewed its P & I Club cover until it wascancelled due to non-payment of the calls.Thus, tocontinue doing business here, Steamship Mutual orthrough its agent Pioneer, must secure a license fromthe Insurance Commission.

- Since a contract of insurance involves publicinterest, regulation by the State is necessary.Thus,no insurer or insurance company is allowed toengage in the insurance business without a license ora certificate of authority from the InsuranceCommission. 2. YES - SEC. 299 . . . - No person shall act as an insurance agent or as aninsurance broker in the solicitation or procurement ofapplications for insurance, or receive for services inobtaining insurance, any commission or othercompensation from any insurance company doingbusiness in the Philippines or any agent thereof,without first procuring a license so to act from theCommissioner, which must be renewed annually onthe first day of January, or within six months thereafter. DispositionThe petition is PARTIALLY GRANTED. The Decision dated July 30, 2002 of the Court ofAppeals affirming the Decision dated May 3, 2000 ofthe Insurance Commission is hereby REVERSED AND SET ASIDE. The Steamship Mutual UnderwritingAssociation (Bermuda) Ltd., and Pioneer Insuranceand Surety Corporation are ORDERED to obtainlicenses and to secure proper authorizations to dobusiness as insurer and insurance agent, respectively. The petitioners prayer for therevocation of Pioneers Certificate of Authority andremoval of its directors and officers, is DENIED.

PANDIMAN v. MARINE MANNING MNGT CORP. 460 SCRA 418 GARCIA; June 21, 2005 NATURE Petition for certiorari to review CA decision FACTS - Benito Singhid was hired as chief cook on board thevessel MV Sun Richie Five for a term of one year byFullwin Maritime Limited through its Philippine agent,Marine Manning and Management Corporation. Whilethe said vessel was on its way to Shanghai from HoChih Minh City, Benito suffered a heart attack andsubsequently died on June 24, 1997. - Apparently, the vessel and the crew were insuredwith Ocean Marine Mutual Insurance AssociationLimited (OMMIAL), a Protective and Indemnity Clubof which Sun Richie Five Bulkers S.A. is a member.Pandiman Philippines, the petitioner, is the localcorrespondent of OMMIAL. - Benitos widow, Rosita, filed a claim for deathbenefits with Marine which referred her to Pandiman.After her submission of the required documentation,Pandiman recommended payment of the deathbenefits amounting to $79,000. However, paymenthas not been made. - Rosita filed a complaint with the Labor Arbiternaming Marine, Pandiman, OMMIAL, and Fullwin asrespondents. The Arbiter ordered all therespondents, except Pandiman, to jointly andseverally pay the widow the death benefits plus legalfees. The NLRC, on appeal by Marine, limited theliable parties to Pandiman and OMMIAL butmaintained the money award. The CA sustained thedecision of the NLRC. Hence this appeal. ISSUE 1.WON Pandimanmay be held liable for the death benefits 2. WON Marine and its foreign principal, Fullwin, should be absolved from the death claim liabilities HELD 1. NO - Pandiman is not an insurance agent as defined bySection 3007 of the Insurance Code. In this case,there was no showing that Pndiman in factnegotiated the insurance contract between SunRichie Five and the insurer OMMIAL. Even, ifPandiman were an agent, payment for claims arisingfrom peril insured against, to which the insurer isliable, is definitely not one of the liabilities of aninsurance agent. Thus, there is no legal basiswhatsoever for holding petitioner solidarily liable withinsurer OMMIAL for the widows claim for deathbenefits. Also, Pandiman is not a party to theinsurance contract and hence under Article 1311 ofthe Civil Code, it is not liable for the obligationarising out of the insurance contract. 2. NO - Fullwin, as Benitos principal employer is liableunder theemployment contract. Marine is alsobound by its undertaking pursuant to the Rules andRegulations Governing Overseas Employment that itshall assume joint and solidary liability with theemployer for all the claims and liabilities which mayarise in connection with the implementation of thecontract, including but not limited to the payment ofwages, heath and disability compensation andrepatriation. In ot her words, both Fullwin andMarine should be held liable for whatever deathbenefits the widow of Benito may be entitled to. Disposition The petition is granted and the CA decision is reversed and set aside.

FILIPINAS COMPANIA DE SEGUROS V CHRISTERN, HUENEFELD AND CO INC 89 PHIL 54 PARAS; May 25, 1951 FACTS - October 1, 1941 - Christern Huenefeld, & Co., Inc.,after payment of corresponding premium, obtainedfrom the Filipinas Cia. de Seguros a fire policy in thesum of P1000,000, covering merchandise containedin No. 711 Roman Street, Binondo Manila. - February 27, 1942 or during the Japanese militaryoccupation - building and insured merchandise wereburned. In due time the Huenefeld Co submitted tothe Filipinas Cia its claim under the policy. Thesalvage goods were sold at public auction and, afterdeducting their value, the total loss suffered by therespondent was fixed at P92,650. - Filipinas Cia refused to pay the claim on the groundthat the policy in favor of the respondent had ceasedto be in force on the date the United States declaredwar against Germany, the respondent Corporation(though organized under and by virtue of the laws ofthe Philippines) being controlled by the Germansubjects and the Filipinas Cia being a company underAmerican jurisdiction when said policy was issued onOctober 1, 1941. Filipinas Cia, however, in pursuanceof the order of the Director of Bureau of Financing,Philippine Executive Commission, dated April 9,1943, paid to the Huenefeld Co the sum of P92,650on April 19, 1943. - August 6, 1946 action filed in CFI Manila torecover from the Huenefeld Co the sum of P92,650above mentioned. The theory of the Filipinas Cia isthat the insured merchandise were burned up afterthe policy issued in 1941 in favor of Huenefeld Cohas ceased to be effective because of the outbreak ofthe war between the United States and Germany onDecember 10, 1941, and that the payment made bythe Filipinas Cia to Huenefeld Co during the Japanesemilitary occupation was under pressure. - CFI:dismissed the action without pronouncement as to costs. - CA:CFI judgment affirmed, with costs. The case isnow before us on appeal bycertiorari from thedecision of the Court of Appeals. ISSUE WON the policy in question became null and voidupon the declaration of war between United Statesand Germany HELD YES RatioThe Philippine Insurance Law (Act No. 2427, as amended,) in section 8, provides that "anyoneexcept a public enemy may be insured." It stands toreason that an insurance policy ceases to beallowable as soon as an insured becomes a public enemy. the law of nations. Such prohibition includes allnegotiations, commerce, or trading with theenemy; all acts which will increase, or tend toincrease, its income or resources; all acts ofvoluntary submission to it; or receiving itsprotection; also all acts concerning thetransmission of money or goods; and all contractsrelating thereto are thereby nullified. It furtherprohibits insurance upon trade with or by theenemy, upon the life or lives of aliens engaged inservice with the enemy; this for the reason thatthe subjects of one country cannot be permitted tolend their assistance to protect by insurance thecommerce or property of belligerent, aliensubjects, or to do anything detrimental too theircountry's interest. The purpose of war is to cripplethe power and exhaust the resources of theenemy, and it is inconsistent that one countryshould destroy its enemy's property and repay ininsurance the value of what has been sodestroyed, or that it should in such mannerincrease the resources of the enemy, or render itaid, and the commencement of war determines,for like reasons, all trading intercourse with theenemy, which prior thereto may have been lawful.All individuals therefore, who compose thebelligerent powers, exist, as to each other, in astate of utter exclusion, and are public enemies. (6Couch, Cyc. of Ins. Law, pp. 5352-5353.) > In the case of an ordinary fire policy, whichgrants insurance only from year, or for some otherspecified term it is plain that when the partiesbecome alien enemies, the contractual tie isbroken and the contractual rights of the parties,so far as not vested . lost. (Vance, the Law on Insurance, Sec. 44, p. 112.) Reasoning

- The Court of Appeals overruled the contention ofthe petitioner that the respondent corporationbecame an enemy when the United States declaredwar against Germany, relying on English andAmerican cases which held that a corporation is acitizen of the country or state by and under the lawsof which it was created or organized. It rejected thetheory that nationality of private corporation isdetermined by the character or citizenship of itscontrolling stockholders. - There is no question that majority of thestockholders of the respondent corporation wereGerman subjects. Therefore, Huenefeld Co becamean enemy corporation upon the outbreak of the warbetween the United States and Germany. The English and American cases relied upon by the Court ofAppeals have lost their force in view of the latestdecision of the Supreme Court of the United States inClarkvs. Uebersee Finanz Korporation, decided onDecember 8, 1947, in which the controls test hasbeen adopted. In "Enemy Corporation" by MartinDomke, a paper presented to the SecondInternational Conference of the Legal Profession heldat the Hague (Netherlands) in August. 1948 alsodiscussed this dilemma > In Clarkvs. Uebersee Finanz Korporation, A. G.,dealing with a Swiss corporation allegedlycontrolled by German interest, the Court: "Theproperty of all foreign interest was placed withinthe reach of the vesting power (of the AlienProperty Custodian) not to appropriate friendly orneutral assets but to reach enemy interest whichmasqueraded under those innocent fronts. . . . Thepower of seizure and vesting was extended to allproperty of any foreign country or national so thatno innocent appearing device could become aTrojan horse." - The respondent having become an enemycorporation on December 10, 1941, the insurancepolicy issued in its favor on October 1, 1941, by thepetitioner (a Philippine corporation) had ceased to bevalid and enforcible, and since the insured goodswere burned after December 10, 1941, and duringthe war, the respondent was not entitled to anyindemnity under said policy from the petitioner.However, elementary rules of justice (in the absenceof specific provision in the Insurance Law) requirethat the premium paid by the respondent for theperiod covered by its policy from December 11,1941, should be returned by the petitioner. Disposition the appealed decision is herebyreversed and the respondent corporation is orderedto pay to the petitioner the sum of P77,208.33,Philippine currency, less the amount of the premium,in Philippine currency, that should be returned by thepetitioner for the unexpired term of the policy inquestion, beginning December 11, 1941.

INSULAR LIFE ASSURANCE CO. v. EBRADO 80 SCRA 181 MARTIN; October 28, 1977 NATURE Appeal from judgment of RTC. FACTS - Buenaventura Ebrado obtained a whole-lifeinsurance policy from Insular, for P5,882.00 with arider for accidental death benefits for the sameamount. He designated Carponia Ebrado as therevocable beneficiary, referring to her as the wife. - Afterwards, he died as a result of an accident whenhe was hit by a falling branch of a tree. Carponiafiled a claim for the proceeds as the designatedbeneficiary in the policy, although she admits thatshe and Buenaventura were merely living as husbandand wife without the benefit of marriage. The legalwife, Pascuala Vda De Ebrado, also filed her claim asthe widow of the deceased. - Insular then filed an interpleader in court (CFIRizal) to determine to whom the proceeds should bepaid. CFI declared that Carponia was disqualifiedfrom becoming beneficiary of the insured anddirecting the Insular to pay the proceeds to theestate of Buenaventura. ISSUE 1. WON a common-law wife named as beneficiary inthe insurance policy of a legally married man claimthe proceeds of the same HELD 1. NO RatioThe prohibition that husband and wife cannot donate to each other applies to common-lawrelationships. As the appointment of a beneficiary ininsurance may be considered a donation, one cannotname as beneficiary his common-law wife. Reasoning - It is quite unfortunate that the Insurance Codedoes not contain any specific provision grosslyresolutory of the prime question at hand. - Rather, general rules of civil law should be appliedto resolve the issue. Art.2011, CC states: Thecontract of insurance is governed by special laws. Matters not expressly provided for in such special laws shall be regulated by this Code. Thus, when not otherwise specifically provided for by the InsuranceLaw, the contract of life insurance is governed by thegeneral rules of the civil law regulating contracts. - Also, Art.2012 any person who is forbidden fromreceiving any donation under Article 739 cannot benamed beneficiary of a life insurance policy by the person who cannot make a donation to him.Common -law spouses are, definitely, barred fromreceiving donations from each other. - Art.739, CC: The following donations shall be void: 1. Those made between persons who were guilty of adultery or concubinage at the time of donation; - In essence, a life insurance policy is no differentfrom a civil donation insofar as the beneficiary isconcerned. Both are founded upon the sameconsideration: liberality. A beneficiary is like a donee,because from the premiums of the policy which theinsured pays out of liberality, the beneficiary willreceive the proceeds or profits of said insurance. Asa consequence, the proscription in Art.739 CC shouldequally operate in life insurance contracts. Themandate of Art.2012 cannot be laid aside: anyperson who cannot receive a donation cannot benamed as beneficiary in the life insurance policy ofthe person who cannot make the donation. - Policy considerations and dictates of morality rightlyjustify the institution of a barrier between common-law spouses in regard to property relations sincesuch relationship ultimately encroaches upon thenuptial and filial rights of the legitimate family. Thereis every reason to hold that the bar in donationsbetween legitimate spouses and those betweenillegitimate ones should be enforced in life insurancepolicies since the same are based on similarconsideration. - So long as marriage remains the threshold offamily laws, reason and morality dictate that theimpediments imposed upon married couple shouldlikewise be imposed upon extra-marital relationship.If legitimate relationship is circumscribed by theselegal disabilities, with more reason should an illicitrelationship be restricted by these disabilities. Disposition Decision AFFIRMED.

CONSUEGRA v. GSIS 37 SCRA 315 ZALDIVAR; January 30, 1971 NATURE Appeal from the decision of the Court of FirstInstance of Surigao del Norte awarding the 8/16 partof the proceeds of the deceased Consuegrasretirement benefits to Rosario Diaz. FACTS - The late Jose Consuegra, at the time of his death,was employed as a shop foreman of the office of theDistrict Engineer in the province of Surigao del Norte.In his lifetime, Consuegra contracted two marriages, the first with herein respondent Rosario Diaz,solemnized in the parish church of San Nicolas deTolentino, Surigao, Surigao, on July 15, 1937, out ofwhich marriage were born two children, namely, JoseConsuegra, Jr. and Pedro Consuegra, but bothpredeceased their father; and the second, which wascontracted in good faith while the first marriage wassubsisting, with herein petitioner Basilia Berdin, onMay 1, 1957 in the same parish and municipality, outof which marriage were born seven children, namely,Juliana, Pacita, Maria Lourdes, Jose, Rodrigo, Lenidaand Luz, all surnamed Consuegra. - Being a member of the Government ServiceInsurance System (GSIS, for short) when Consuegradied on September 26, 1965, the proceeds of his lifeinsurance under policy No. 601801 were paid by theGSIS to petitioner Basilia Berdin and her childrenwho were the beneficiaries named in the policy. - However, Consuegra did not designate anybeneficiary who would receive the retirementinsurance benefits due to him. Respondent RosarioDiaz, the widow by the first marriage, filed a claimwith the GSIS asking that the retirement insurancebenefits be paid to her as the only legal heir ofConsuegra, considering that the deceased did notdesignate any beneficiary with respect to hisretirement insurance benefits. Petitioner BasiliaBerdin and her children, likewise, filed a similar claimwith the GSIS, asserting that being the beneficiariesnamed in the life insurance policy of Consuegra, theyare the only ones entitled to receive the retirementinsurance benefits due the deceased Consuegra.Resolving the conflicting claims, the GSIS ruled thatthe legal heirs of the late Jose Consuegra wereRosario Diaz, his widow by his first marriage who isentitled to one-half, or 8/16, of the retirementinsurance benefits, on the one hand; and BasiliaBerdin, his widow by the second marriage and theirseven children, on the other hand, who are entitledto the remaining one-half, or 8/16, each of them toreceive an equal share of 1/16. - Dissatisfied with the foregoing ruling andapportionment made by the GSIS, Basilia Berdin andher children filed on October 10, 1966 a petition formandamus with preliminary injunction in the Court ofFirst Instance of Surigao. - The CFI of Surigao ruled in favor of respondentRosario Diaz and upheld the ruling of GSIS in allaspect. Thus, Basilia Berdin and her childrenappealed said decision to the Supreme Court. ISSUE WON GSIS was correct in awarding half of theretirement benefit of the deceased to Rosario Diaz,the first wife, notwithstanding the fact that thepetitioners were named as beneficiaries of the lifeinsurance HELD YES - The GSIS offers two separate and distinct systemsof benefits to its members, one is the life insuranceand the other is the retirement insurance. These twodistinct systems of benefits are paid out from twodistinct and separate funds that are maintained bythe GSIS. Thus, it doesnt necessarily mean that thebeneficiaries in the life insurance are also thebeneficiaries in the retirement insurance. - Consuegra started in the government service sometime during the early part of 1943, or before1943. In 1943 Com. Act 186 was not yet amended,and the only benefits then provided for in said Com.Act 186 were those that proceed from a lifeinsurance. Upon entering the government serviceConsuegra became a compulsory member of theGSIS, being automatically insured on his life,pursuant to the provisions of Com. Act 186 whichwas in force at the time. During 1943 the operationof the Government Service Insurance System wassuspended because of the war, and the operationwas resumed sometime in 1946. When Consuegradesignated his beneficiaries in his life insurancehe could not have intended those beneficiaries of his lifeinsurance as also the beneficiaries of his retirementinsurance because the provisions on retirementinsurance under the GSIS came about only whenCom. Act 186 was amended by Rep. Act 660 on June16, 1951. Hence, it cannot be said that becauseherein appellants were designated beneficiaries inConsuegra's life insurance they automaticallybecame the beneficiaries also of his retirementinsurance.

- The provisions of subsection (b) of Section 11 ofCommonwealth Act 186, as amended by Rep. Act660, clearly indicate that there is need for theemployee to file an application for retirementinsurance benefits when he becomes a member ofthe GSIS, and he should state in his application thebeneficiary of his retirement insurance. Hence, thebeneficiary named in the life insurance does notautomatically become the beneficiary in theretirement insurance unless the same beneficiary inthe life insurance is so designated in the applicationfor retirement insurance. - In the case of the proceeds of a life insurance, thesame are paid to whoever is named the beneficiaryin the life insurance policy. As in the case of a lifeinsurance provided for in the Insurance Act, thebeneficiary in a life insurance under the GSIS maynot necessarily be an heir of the insured.The insured in a life insurance may designate anyperson as beneficiary unless disqualified to beso under the provisions of the Civil Code. And in the absence of any beneficiary named in the lifeinsurance policy, the proceeds of the insurance willgo to the estate of the insured. - On the other hand, the beneficiary of theretirement insurance can only claim the proceeds ofthe retirement insurance if the employee dies beforeretirement. If the employee failed or overlooked tostate the beneficiary of his retirement insurance, theretirement benefits will accrue to his estate and willbe given to his legal heirs in accordance with law, asin the case of a life insurance if no beneficiary isnamed in the insurance policy. Disposition Petition Denied. It is Our view,therefore, that the respondent GSIS had correctlyacted when it ruled that the proceeds of theretirement insurance of the late Jose Consuegrashould be divided equally between his first living wifeRosario Diaz, on the one hand, and his second wifeBasilia Berdin and his children by her.

SSS v. DAVAC 17 SCRA 863 BARRERA: July 30, 1966 NATURE APPEAL from a resolution Of the Social Security Commission. FACTS - Petronilo Davac,became a member of the SocialSecurity System (SSS for short) on September 1,1957. In the Member's Record he designatedrespondent, Candelaria Davac as his beneficiary andindicated his relationship to her as that of "wife". - He died on April 5, 1959. It appears that thedeceased contracted two marriages, the first, withLourdes Tuplano on August 29, 1946, who bore hima child, Romeo Davac, and the second, withCandelaria Davac on January 18, 1949, with whomhe had a minor daughter, Elizabeth Davac. Both filedtheir claims for death benefit with the SSS. - Social Security Commission issued the resolution declaring respondent Candelaria Davac as the person entitled to receive the death benefits payable for the death of Petronilo Davac. ISSUES 1. WON the Social Security Commission Candelaria Davac is entitled to receive the death benefits 2. WON a beneficiary under the Social SecuritySystem partakes of the nature of a beneficiary in alife insurance policy and, therefore the designationmade in the person DAVACas bigamous wife is nulland void, because it contravenes the provisions ofthe Civil Code 3. WON the benefits accruing from membership withSSS forms part of the conjugal property thus theresolution deprives the lawful wife of her share in theconjugal property as well as of her own and herchild's legitime in the inheritance HELD 1. YES - Section 13, RA1161 provides that the beneficiary"as recorded" by the employee's employer is the oneentitled to the death benefits. - Section 13, Republic Act No. 1161, as amended byRepublic Act No. 1792, in force at the time ofPetronilo Davac's deathprovides: Upon the covered employee's death or total and permanent disabilityunder such conditions as the Commission maydefine, before becoming eligible for retirement and ifeither such death or disability is not compensableunder the Workmen's Compensation Act, he or. incase of his death, his beneficiaries, as recorded byhis employer shall be entitled to the following benefit: - In Tecson vs. Social Security System. Section 13was construed:"it may be true that the purpose ofthe coverage under the Social Security System isprotection of the employee as well as of his family,but this purpose or intention of the law cannot beenforced to the extent of contradicting the veryprovisions of said law contained in Section 13,thereof - When the provisions of a law are clear and explicit,the courts can do nothing but apply its clear andexplicit provisions (Velasco vs. Lopez) 2. NO - The disqualification mentioned in Article 739 is notapplicable to herein appellee Candelaria Davacbecause she was not guilty of concubinage, therebeing no proof that she had knowledge of theprevious marriage of her husband Petronilo. ART. 2012. Any person who is forbidden fromreceiving any donation under Article 739 cannot benamed beneficiary of a life insurance policy by theperson who cannot make any donation to himaccording to said article. ART. 739. The following donations shall be void: (1) Those made between persons who were guiltyof adultery or concubinage at the time of thedonation; (the court did not decide whether thispartakes the nature of a life insurance policy) 3. NO - The benefit receivable under the Act is in thenature of a special privilege or an arrangementsecured by the law pursuant to the policy of theState to provide social security to the workingmen.The amounts that may thus be received cannot beconsidered as property earned by the member duringhis lifetime. His contribution to the fund constitutesonly an insignificant portion thereof. Then, thebenefits are specifically declared not transferable,and exempted from tax, legal processes, and lien.Furthermore, in the settlement of claims thereunder,the procedure to be observed is governed not by thegeneral provisions of law, but by rules andregulations promulgated by the Commission.

Thus, ifthe money is payable to the estate of a deceasedmember, it is the Commission, not the probate orregular court that determines the person or personsto whom it is payable. - They are disbursed from a public special fundcreated by Congress.The sources of this special fundare the covered employee's contribution (equal to 2-1/2 per cent of the employee's monthlycompensation) ; the employer's 'Contribution(equivalent to 3-1/2 per cent of the monthlycompensation of the covered employee) ;and theGovernment contribution which consists in yearlyappropriation of public funds to assure themaintenance of an adequate working balance of thefunds of the System. Additionally, Section 21 of theSocial Security Actprovides that the benefitsprescribed in this Act shall not be diminished and theGovernment of the Republic of the Philippinesaccepts general responsibility for the solvency of theSystem. - The benefits under the Social Security Act are notintended by the lawmaking body to form part of theestate of the covered members. - Social Security Act is not a law of succession. Disposition Resolution of the Social Security Commission appealed is affirmed

FRANCISCO DEL VAL v. ANDRES DEL VAL 29 PHIL 534 MORELAND; February 16, 1915 NATURE Appeal from a judgment of the Court of FirstInstance of the city of Manila dismissing thecomplaint with costs. FACTS - Plaintiffs and defendant are brothers and sisters; that they are the only heirs at law and next of kin ofGregorio Nacianceno del Val, who died in Manila onAugust 4, 1910, intestate - During the lifetime of the deceased he took outinsurance on his life for the sum of P40,000 andmade it payable to the defendant ANDRES DEL VALas sole beneficiary.After his death the defendantcollected the face of the policy.From said policy hepaid the sum of P18,365.20 to redeem certain realestate which the decedent had sold to third personswith a right to repurchase. - The redemption of said premises was made by theattorney of defendant ANDRES in the name of theplaintiffs and the defendant as heirs of the deceasedvendor.It further appears from the pleadings thatthe defendant, on the death of the deceased, tookpossession of most of his personal property, which hestill has in his possession, and that he has also thebalance on said insurance policy amounting toP21,634.80. - Plaintiffs contend that the amount of the insurancepolicy belonged to the estate of the deceased andnot to the defendant personally; that, therefore, theyare entitled to a partition not only of the real andpersonal property, but also of the P40,000 lifeinsurance. The complaint prays a partition of all theproperty, both real and personal, left by thedeceased; that the defendant account forP21,634.80, and that the sum be divided equallyamong the plaintiffs and defendant along with theother property of deceased. - The defendant denies the material allegations ofthe complaint and sets up as special defense andcounterclaim that the redemption of the real estatesold by his father was made in the name of theplaintiffs and himself instead of in his name alonewithout his knowledge or consent. Andres contendsthat it was not his intention to use the proceeds ofthe insurance policy for the benefit of any person buthimself, he alleging that he was and is the soleowner thereof and that it is his individual property. He, therefore, asks that he be declared the owner ofthe real estate redeemed by the payment of theP18,365.20, the owner of the remaining P21,634.80,the balance of the insurance policy, and that theplaintiffs account for the use and occupation of thepremises so redeemed since the date of theredemption. - The trial court refused to give relief to either partyand dismissed the action. In this appeal, it is claimedby the attorney for the plaintiffs that insuranceprovisions in the Code of Commerce aresubordinated to the provisions of the Civil Code asfound in article 1035. This article reads: "An heir by force of law surviving with others of thesame character to a succession must bring into thehereditary estate the property or securities he mayhave received from the deceased during the life ofthe same, by way of dowry, gift, or for any goodconsideration, in order to compute it in fixing thelegal portions and in the account of the division." - Counsel also claims that the proceeds of theinsurance policy were a donation or gift made by thefather during his lifetime to the defendant and that,as such, its ultimate destination is determined bythose provisions of the Civil Code which relate todonations, especially article 819. This article providesthat "gifts made to children which are notbetterments shall be considered as part of their legalportion." ISSUES 1. WON the insurance belongs to the defendant and not to the decedents estate 2. WON the Civil code provisions on successionprevail over any other law with respect to theinsurance HELD 1. YES - The SC agreed with the finding of the trial court that the proceeds of the life-insurance policy belongexclusively to the defendant as his individual andseparate property, we agree. That the proceeds of aninsurance policy belong exclusively to the beneficiaryand not to the estate of the person whose life wasinsured, and that such proceeds are the separate andindividual property of the beneficiary, and not of theheirs of the person whose life was

insured, is thedoctrine in America. We believe that the samedoctrine obtains in these Islands by virtue of section428 of the Code of Commerce, which reads: "The amounts which the underwriter must deliverto the person insured, in fulfillment of thecontract, shall be the property of the latter, evenagainst the claims of the legitimate heirs orcreditors of any kind whatsoever of the personwho effected the insurance in favor of the former." 2. NO - The contract of life insurance is a special contract and the destination of the proceeds thereof isdetermined by special laws which deal exclusivelywith that subject. The Civil Code has no provisionswhich relate directly and specifically to lifeinsurancecontracts or to the destination of life insuranceproceeds. That subject is regulated exclusively bythe Code of Commerce which provides for the termsof the contract, the relations of the parties and thedestination of the proceeds of the policy. - Assuming that the proceeds of the life-insurancepolicy being the exclusive property of the defendantand he having used a portion thereof in therepurchase of the real estate sold by the decedentprior to his death with right to repurchase, and suchrepurchase having been made and the conveyancetaken in the names of all of the heirs instead of thedefendant alone, plaintiffs claim that the property belongs to the heirs in common and not to the defendant alone. - The Court rejected this contention unless the factappear or be shown that the defendant acted as hedid with the intention that the other heirs shouldenjoy with him the ownership of the estate---inother words, that he proposed, in effect, to make agift of the real estate to the other heirs. If it isestablished by the evidence that was his intentionand that the real estate was delivered to theplaintiffs with that understanding, then it is probablethat their contention is correct and that they areentitled to share equally with the defendant therein.If, however, it appears from the evidence in the casethat the conveyances were taken in the name of theplaintiffs without his knowledge or consent, or that itwas not his intention to make a gift to them of thereal estate, then it belongs to him. If the facts are asstated, he has two remedies. The one is to compelthe plaintiffs to reconvey to him and the other is tolet the title stand with them and to recover fromthem the sum he paid on their behalf. - For the complete and proper determination of the questions at issue in this case, the Court was of the opinion that the cause should be returned to the trialcourt with instructions to permit the parties to framesuch issues as will permit the settlement of all the questions involved and to introduce such evidence asmay be necessary for the full determination of theissues framed. Upon such issues and evidence takenthereunder the court will decide the questionsinvolved according to the evidence, subordinating hisconclusions of law to the rules laid down in thisopinion.REMANDED.

GERCIO v. SUN LIFE ASSURANCE OF CANADA 48 PHIL 53 MALCOLM; September 28, 1925 NATURE Mandamus to compel Sun Life Assurance Co. ofCanada to change the beneficiary in the policy issuedby the defendant company on the life of the plaintiffHilario Gercio FACTS - On January 29, 1910, the Sun Life Assurance Co. ofCanada issued an insurance policy on the life ofHilario Gercio. The policy was what is known as a 20-year endowment policy. By its terms, the insurancecompany agreed to insure the life of Hilario Gerciofor the sum of P2,000, to be paid him on February 1,1930, or if the insured should die before said date,then to his wife, Mrs. Andrea Zialcita, should shesurvive him; otherwise to the executors,administrators, or assigns of the insured. The policydid not include any provision reserving to the insuredthe right to change the beneficiary. - On the date the policy was issued, Andrea Zialcitawas the lawful wife of Hilario Gercio. Towards the endof the year 1919, she was convicted of the crime ofadultery. On September 4, 1920, a decree of divorcewas issued in civil case no. 17955, which had theeffect of completely dissolving their bonds ofmatrimony - On March 4, 1922, Hilario Gercio formally notifiedthe Sun Life that he had revoked his donation infavor of Andrea Zialcita, and that he had designatedin her stead his present wife, Adela Garcia de Gercio,as the beneficiary of the policy. Gercio requested theinsurance company to eliminate Andrea Zialcita asbeneficiary. This, the insurance company has refusedand still refuses to do. ISSUES 1.(Preliminary) WON the provisions of the Code ofCommerce and the Civil Code shall be in force in1910, or the provisions of the Insurance Act now in force, or the general principles of law, guide the court in its decision 2. WON the insured, the husband, has the power tochange the beneficiary, the former wife, and to nameinstead his actual wife, where the insured and thebeneficiary have been divorced and where the policyof insurance does not expressly reserve to theinsured the right to change the beneficiary HELD 1. Whether the case be considered in the light of theCode of Commerce, the Civil Code, or the InsuranceAct, the deficiencies in the law will have to besupplemented by the general principles prevailing onthe subject. To that end, we have gathered the ruleswhich follow from the best considered Americanauthorities. In adopting these rules, we do so withthe purpose of having the Philippine Law ofInsurance conform as nearly as possible to themodern Law of Insurance as found in the UnitedStates proper. - Courts first duty is to determine what law shouldbe applied to the facts. The insurance policy wastaken out in 1910, that the Insurance Act. No. 2427,became effective in 1914, and that the effort tochange the beneficiary was made in 1922. - Code of Commerce- there can be found in it noprovision either permitting or prohibiting the insuredto change the beneficiary. - Civil Code- it would be most difficult, if indeed it ispracticable, to test a life insurance policy by itsprovisions. In the case of Del Val vs. Del Val, itdeclined to consider the proceeds of the insurancepolicy as a donation or gift, saying "the contract oflife insurance is a special contract and the destinationof the proceeds thereof is determined by special lawswhich deal exclusively with that subject. The CivilCode has no provisions which relate directly andspecifically to life-insurance contracts or to thedestination of life-insurance proceeds. . . . " - Insurance Act- there is likewise no provisioneither permitting or prohibiting the insured to changethe beneficiary. 2. NO Ratio The wife has an insurable interest in the life of her husband. The beneficiary has an absolute vestedinterest in the policy from the date of its issuanceand delivery. So when a policy of life insurance istaken out by the husband in which the wife is namedas beneficiary, she has a subsisting interest in thepolicy. And this applies to a policy to which there areattached the incidents of a loan value, cash

surrender value, an automatic extension bypremiums paid, and to an endowment policy, as wellas to an ordinary life insurance policy. If the husbandwishes to retain to himself the control and ownershipof the policy he may so provide in the policy. But ifthe policy contains no provision authorizing a changeof beneficiary without the beneficiary's consent, theinsured cannot make such change. Accordingly, it isheld that a life insurance policy of a husband madepayable to the wife as beneficiary, is the separateproperty of the beneficiary and beyond the control ofthe husband. - Unlike the statutes of a few jurisdictions, there isno provision in the Philippine Law permitting thebeneficiary in a policy for the benefit of the wife ofthe husband to be changed after a divorce. It mustfollow, therefore, in the absence of a statute to thecontrary, that if a policy is taken out upon ahusband's life the wife is named as beneficiarytherein, a subsequent divorce does not destroy herrights under the policy. Reasoning - Yore vs. Booth . . . It seems to be the settled doctrine, with butslight dissent in the courts of this country, that aperson who procures a policy upon his own life,payable to a designated beneficiary, although hepays the premiums himself, and keeps the policyin his exclusive possession, has no power tochange the beneficiary, unless the policy itself, orthe charter of the insurance company, so provides.In policy, although he has parted with nothing,and is simply the object of another's bounty, hasacquired a vested and irrevocable interest in thepolicy, which he may keep alive for his own benefitby paying the premiums or assessments if theperson who effected the insurance fails or refusesto do so. - Connecticut Mutual Life Insurance Company vs Schaefer We do not hesitate to say, however, that a policytaken out in good faith and valid at its inception, isnot avoided by the cessation of the insurableinterest, unless such be the necessary effect of theprovisions of the policy itself.. . . .In our judgmentof life policy, originally valid, does not cease to beso by the cessation of the assured party's interestin the life insured. - Central National Bank of Washington City vs. Hume It is indeed the general rule that a policy, and the money to become due under it, belong, the moment it is issued, to the person or personsnamed in it as the beneficiary or beneficiaries, andthat there is no power in the person procuring theinsurance, by any act of his, by deed or by will, totransfer to any other person the interest of theperson named. - In re Dreuil & Co. In so far as the law of Louisiana is concerned, itmay also be considered settled that where a policyis of the semitontine variety, as in this case, thebeneficiary has a vested right in the policy, ofwhich she cannot be deprived without her consent - Wallace vs Mutual Benefit Life Insurance Co. As soon as the policy was issued Mrs. Wallace acquired a vested interest therein, of which shecould not be deprived without her consent, exceptunder the terms of the contract with the insurancecompany. No right to change the beneficiary wasreserved. Her interest in the policy was herindividual property, subject to be divested only byher death, the lapse of time, or by the failure ofthe insured to pay the premiums. She could keepthe policy alive by paying the premiums, if theinsured did not do so. It was contingent uponthese events, but it was free from the control ofher husband. He had no interest in her property inthis policy, contingent or otherwise. Her interestwas free from any claim on the part of the insuredor his creditors. He could deprive her of herinterest absolutely in but one way, by living morethan twenty years. - Filley vs. Illinois Life Insurance Company The benefit accruing from a policy of lifeinsurance upon the life of a married man, payableupon his death to his wife, naming her, is payableto the surviving beneficiary named, although shemay have years thereafter secured a divorce fromher husband, and he was thereafter again marriedto one who sustained the relation of wife to him atthe time of his death. The rights of a beneficiary in an ordinary lifeinsurance policy become vested upon the issuanceof the policy, and can thereafter, during the life ofthe beneficiary, be defeated only as provided bythe terms of the policy. - On the admitted facts and the authoritiessupporting the nearly universally accepted principlesof insurance, we are irresistibly led to the conclusionthat the question at issue must be answered in thenegative Disposition The judgment appealed from will be

reversed and the complaint ordered dismissed as to the appellant. SEPARATE OPINION JOHNSON [concur] - I agree with the majority of the court, that thejudgment of the lower court should be revoked, butfor a different reason. The purpose of the petition isto have declared the rights of certain persons in aninsurance policy which is not yet due and payable. Itmay never become due and payable. The premiumsmay not be paid, thereby rendering the contract ofinsurance of non effect, and many other things mayoccur, before the policy becomes due, which wouldrender it non effective. The plaintiff and the otherparties who are claiming an interest in said policyshould wait until there is something due them underthe same. For the courts to declare now who are thepersons entitled to receive the amounts due, if theyever become due and payable, is impossible, for thereason that nothing may ever become payable underthe contract of insurance, and for many reasons suchpersons may never have a right to receive anythingwhen the policy does become due and payable. Inmy judgment, the action is premature and shouldhave been dismissed. PHIL. AMERICAN LIFE INSURANCE v. PINEDA 175 SCRA 416 PARAS; July 19, 1989 NATURE Petition for review on certiorari the orders of CFI Judge Pineda FACTS - In 1968, Private Respondent Rodolfo Dimayugaprocured an ordinary life insurance policy from thepetitioner company and designated his wife andchildren as irrevocable beneficiaries. On Feb. 22,1980, Dimayuga filed with the CFI a petition toamend the designation of the beneficiaries in his lifepolicy from irrevocable to revocable. Petitioner filedan Urgent Motion to reset hearing as well as itscomment and/or Opposition to the respondentspetition. - Respondent Judge denied petitioners UrgentMotion, thus allowing private respondent to adduceevidence, the consequence of which was theissuance of the questioned Order granting the petition. Petitioner then filed a MFR which was also denied hence this petition. ISSUE 1. WON the designation of the irrevocablebeneficiaries could be changed or amended withoutthe consent of all the irrevocable beneficiaries 2. WON the irrevocable beneficiaries herein, one ofwhom is already deceased while the others are allminors could validly give consent to the change oramendment in the designation of the irrevocablebeneficiaries HELD 1. NO - Based on the provision of their contract and the lawapplicable, it is only with the consent of all thebeneficiaries that any change or amendment in thepolicy concerning the irrevocable beneficiaries maybe legally and validly effected. Both the law and thePolicy do not provide for any other exception. Reasoning - Since the policy was procured in 1968, the applicable law in this case is the Insurance Act andunder that law, the beneficiary designated in a lifeinsurance contract cannot be changed without theconsent of the beneficiary because he has a vestedinterest in the policy. - The Beneficiary Designation Indorsement in thepolicy in the name of Dimayuga states that thedesignation of the beneficiaries is irrevocable: no right or privilege under the Policy may be exercised,or agreement made with the Company to anychange in or amendment to the Policy, without theconsent of the said beneficiary/beneficiaries. - Contracts which are the private laws of thecontracting parties should be fulfilled according tothe literal sense of their stipulations, if their termsare clear and leave no room for doubt as to theintention of the contracting parties, for contracts areobligatory, no matter in what form they may be,whenever the essential requisites for their validityare present. - Finally, the fact that the contract of insurance doesnot contain a contingency when the change in thedesignation of beneficiaries could be validly effectedmeans that it was never within the contemplation ofthe parties. 2. NO

- The parent-insured cannot exercise rights and/orprivileges pertaining to the insurance contract, forotherwise, the vested rights of the irrevocable beneficiaries would be rendered inconsequential. Thealleged acquiescence of the 6 children beneficiariescannot be considered an effective ratification to thechange of the beneficiaries from irrevocable torevocable. They were minors at the time, and couldnot validly give consent. Neither could they actthrough their father-insured since their interests arequite divergent from one another. Disposition questioned Orders of respondent judge are nullified and set aside.

SUN LIFE ASSURANCE v. INGERSOLL 41 PHIL 331 STREET; November 8, 1921 NATURE Action of interpleader FACTS - April 16, 1918, Sun Life Assurance Company ofCanada (Sun Life), in consideration of the payment ofa stipulated annual premium during the period of thepolicy, or until the premiums had been completelypaid for twenty years, issued a policy of insurance onthe life of Dy Poco for US$12,500, payable to the saidassured or his assigns on the 21st day of February,1938, and if he should die before that date then tohis legal representatives. - June 23, 1919, the assured, Dy Poco, was adjudgedan involuntary insolvent by the CFI Manila, and FrankB. Ingersoll was appointed assignee of his estate. - July 10, 1919, Dy Poco died, and on August 21,1919, Tan Sit, was duly appointed as theadministratrix of his intestate estate. - By the terms of the policy it was provided that afterthe payment of three full premiums, the assuredcould surrender the policy to the company for a"cash surrender value," indicated in an annexedtable; but inasmuch as no more than two premiumshad been paid upon the policy now in question up tothe time of the death of the assured, this provisionhad not become effective; and it does not appearthat the company would in accordance with its ownusage or otherwise have made any concession to theassured in the event he had desired, before hisdeath, to surrender the policy. It must therefore beaccepted that this policy had no cash surrendervalue, at the time of the assured's death, either bycontract or by convention practice of the company insuch cases. - Both Ingersoll, as assignee, and Tan Sit, as administratix of Dy Poco's estate, asserted claims to the proceeds of the policy. The lower court foundthat Ingersoll had a better right and ordered Sun Lifeto pay the insurance proceeds to him. ISSUE WON Ingersoll, as assignee, has a right to the proceeds of the insurance HELD NO On the Philippine Insolvency Law (Act No. 1956) - The property and interests of the insolvent whichbecome vested in the assignee of the insolvent arespecified in section 32 of the Insolvency Law whichreads as follows: "SEC. 32. As soon as an assignee is electedor appointed and qualified, the clerk of the courtshall, by an instrument under his hand and seal ofthe court, assign and convey to the assignee all thereal and personal property, estate, and effects of thedebtor with all his deeds, books, and papers relatingthereto, and such assignment shall relate back to thecommencement of the proceedings in insolvency, andshall relate back to the acts upon which theadjudication was founded, and by operation of lawshall vest the title to all such property, estate, andeffects in the assignee, although the same is thenattached on mesne process, as the property of thedebtor. Such assignment shall operate to vest in theassignee all of the estate of the insolvent debtor notexempt by law from execution." - the Insolvency Law is in great part a copy of theInsolvency Act of California, enacted in 1895, thoughit contains a few provisions from the AmericanBankruptcy Law of 1898 - Under each of said laws the assignee acquires allthe real and personal property, estate, and effects ofthe debtor, not exempt by law from execution, withall deeds, books and papers relating thereto; andwhile this language is broad, it nevertheless lacks thecomprehensiveness of section 70 (a) of the AmericanBankruptcy Law of 1898 in at least two particulars;for under subsection 3 of section 70 (a) of the lastmentioned law, the trustee in bankruptcy acquiresthe right to exercise any powers which the insolventmight have exercised for his own benefit, and undersubsection 5 the trustee acquires any property of theinsolvent which the latter could by any means haveassigned to another. The Insolvency Law here inforce, in common with the predecessor laws above-mentioned, contains nothing similar to theseprovisions. On the applicability of the Insolvency Law - Sec 32 of the Insolvency Law among other things,declares that the assignment to be made by the clerkof the court "shall operate to vest in the assignee allof the estate of the insolvent debtor not exempt bylaw from execution." Moreover, by section 24, thecourt is required, upon making an order

adjudicatingany person insolvent, to stay any civil proceedingspending against him; and it is declared in section 60that no creditor whose debt is provable under the Actshall be allowed, after the commencement ofproceedings in insolvency, to prosecute to finaljudgment any action therefor against the debtor. Inconnection with the foregoing may be mentionedsubsections 1 and 2 of section 36, as well as theopening words of section 33, to the effect that theassignee shall have the right and power to recoverand to take into his possession, all of the estate,assets, and claims belonging to the insolvent, exceptsuch as are exempt by law from execution. - These provisions clearly evince an intention to vestin the assignee, for the benefit of all the creditors ofthe insolvent, such elements of property andproperty right as could be reached and subjected byprocess of law by any single creditor suing alone.And this is exactly as it should be: for it cannot besupposed that the Legislature would suppress theright of action of every individual creditor upon theadjudication of insolvency, and at the same timeallow the insolvent debtor to retain anything subjectto the payment of his debts in a normal state ofsolvency. - "leviable assets" and "assets in insolvency" arepractically coextensive terms. Hence, in determiningwhat elements of value constitute assets ininsolvency, SC is at liberty to consider what elementsof value are subject to be taken upon execution, andvice versa. On whether a policy of insurance having no cashsurrender value, but payable to insured or his legalrepresentative, is property that may be taken uponexecution against him. - Philippine laws declare no exemption with respectto insurance policies; and this species of property isnot enumerated, in section 48 of the Insolvency Law,among items from the ownership of which theassignee is excluded. Moreover, all life insurancepolicies are declared by law to be assignable,regardless of whether the assignee has an insurableinterest in the life of the insured or not (InsuranceAct No. 2427, sec. 166). - SC has held that insurance policies having apresent cash surrender value are subject to be takenupon execution. (Misut Garcia vs. West Coast SanFrancisco Life Ins. Co.) - a policy devoid of a cash surrender value cannot be either "leviable assets" or "assets in insolvency." - the assignee in insolvency acquired no beneficialinterest in the policy of insurance in question; thatits proceeds are not liable for any of the debtsprovable against the insolvent in the pendingproceedings, and that said proceeds should thereforebe delivered to his administratrix. On applicable US case - In re McKinney: no beneficial interest in the policyhad ever passed to the assignee over and beyondwhat constituted the surrender value, and that thelegal title to the policy was vested in the assigneemerely in order to make the surrender value-available to him. The assignee should surrender thepolicy upon the payment to him of said value, as hewas in fact directed to do. The assignee inbankruptcy had no right to keep the estate unsettledfor an indefinite period, for the mere purpose ofspeculating upon the chances of the bankrupt'sdeath. As regards everything beyond the surrendervalue, the assignee in bankruptcy would, after thedischarge of the bankrupt, have no insurable interestin the life of the bankrupt. - surrender value of a policy "arises from the factthat the fixed annual premiums is much in excess ofthe annual risk during the earlier years of the policy,an excess made necessary in order to balance thedeficiency of the same premium to meet the annualrisk during the latter years of the policy. This excessin the premium paid over the annual cost ofinsurance, with accumulations of interest, constitutesthe surrender value. Though this excess of premiumspaid is legally the sole property of the company, stillin practical effect, though not in law, it is moneys ofthe assured deposited with the company in advanceto make up the deficiency in later premiums to coverthe annual cost of insurance, instead of beingretained by the assured and paid by him to thecompany in the shape of greatly-increasedpremiums, when the risk is greatest. It is the 'netreserve' required by law to be kept by the companyfor the benefit of the assured, and to be maintainedto the credit of the policy. So long as the policyremains in force the company has not practically anybeneficial interest in it, except as its custodian, withthe obligation to maintain it unimpaired and suitablyinvested for the benefit of the insured. This is the practical, though not the legal, relation of thecompany to this fund. "Upon the surrender of thepolicy before the death of the assured, the company,to be relieved from all responsibility for the increasedrisk, which is represented by this accumulatingreserve, could well afford to surrender a considerablepart of it to the assured, or his representative. Areturn of a part in some form or other is now Usuallymade." (In re McKinney) - the stipulation providing for a cash surrender valueis a comparatively recent innovation in life insurance.Formerly the contracts provided as they stillcommonly do in the policies issued by fraternalorganizations and benefit societies for thepayment of a premium sufficient to keep

theestimated risk covered; and in case of a lapse thepolicy-holder received nothing. Furthermore, thepractice is common among insurance companieseven now to concede nothing in the character of cashsurrender value, until three full premiums have beenpaid, as in this case. - CONLUSION (from this case and other English and American cases cited following the same opinion):the assignee acquires no beneficial interest ininsurance effected on the life of the insolvent, exceptto the extent that such insurance contains assetswhich can be realized upon as of the date when thepetition of insolvency is filed. The explanation is tobe found in the consideration that the destruction ofa contract of life insurance is not only highlyprejudicial to the insured and those dependent uponhim, but is inimical to the interests of society.Insurance is a species of property that should beconserved and not dissipated. As is well known, lifeinsurance is increasingly difficult to obtain withadvancing years, and even when procurable after theage of fifty, the cost is then so great as to bepractically prohibitive to many. Insolvency is adisaster likely to overtake men in mature life; andone who has gone through the process of bankruptcyusually finds himself in his declining years with theaccumulated savings of years swept away andearning power diminished. The courts are thereforepractically unanimous in refusing to permit theassignee in insolvency to wrest from the insolvent apolicy of insurance which contains in it no presentrealizable assets. On the applicability of the Insolvency Law - Sec 32 of the Insolvency Law among other things,declares that the assignment to be made by the clerkof the court "shall operate to vest in the assignee allof the estate of the insolvent debtor not exempt by law from execution." Moreover, by section 24, thecourt is required, upon making an order adjudicatingany person insolvent, to stay any civil proceedingspending against him; and it is declared in section 60that no creditor whose debt is provable under the Actshall be allowed, after the commencement ofproceedings in insolvency, to prosecute to finaljudgment any action therefor against the debtor. Inconnection with the foregoing may be mentionedsubsections 1 and 2 of section 36, as well as theopening words of section 33, to the effect that theassignee shall have the right and power to recoverand to take into his possession, all of the estate,assets, and claims belonging to the insolvent, exceptsuch as are exempt by law from execution. - These provisions clearly evince an intention to vestin the assignee, for the benefit of all the creditors ofthe insolvent, such elements of property andproperty right as could be reached and subjected byprocess of law by any single creditor suing alone.And this is exactly as it should be: for it cannot besupposed that the Legislature would suppress theright of action of every individual creditor upon theadjudication of insolvency, and at the same timeallow the insolvent debtor to retain anything subjectto the payment of his debts in a normal state ofsolvency. - "leviable assets" and "assets in insolvency" arepractically coextensive terms. Hence, in determiningwhat elements of value constitute assets ininsolvency, SC is at liberty to consider what elementsof value are subject to be taken upon execution, andvice versa. On whether a policy of insurance having no cashsurrender value, but payable to the insured or hislegal representative, is property that may be takenupon execution against him. - Philippine laws declare no exemption with respectto insurance policies; and this species of property isnot enumerated, in section 48 of the Insolvency Law,among items from the ownership of which theassignee is excluded. Moreover, all life insurancepolicies are declared by law to be assignable,regardless of whether the assignee has an insurableinterest in the life of the insured or not (InsuranceAct No. 2427, sec. 166). - SC has held that insurance policies having apresent cash surrender value are subject to be takenupon execution. (Misut Garcia vs. West Coast SanFrancisco Life Ins. Co., 41 Phil., 258.) - a policy devoid of a cash surrender value cannot be either "leviable assets" or "assets in insolvency." the assignee in insolvency acquired no beneficialinterest in the policy of insurance in question; thatits proceeds are not liable for any of the debtsprovable against the insolvent in the pendingproceedings, and that said proceeds should thereforebe delivered to his administratrix. Disposition Judgment reversed. Sun Life is directed to pay the proceeds of the policy to Tan Sit.

CHAPTER VI RESCISSION OF INSURANCE CONTRACTS: CONCEALMENT, MISREPRESENTATION, & BREACH OF WARRANTIES NG v. ASIAN CRUSADER LIFE ASSURANCE CORP 122 SCRA 461 ESCOLIN; May 30, 1983 FACTS - On May 12, 1962, Kwong Nam applied for a 20-year endowment insurance on his life for the sum ofP20,000, with his wife, Ng Gan Zee, as beneficiary. - He died on Dec 1963 of cancer of the liver withmetastasis. All premiums had been paid at the timeof his death. - Ng presented a claim for payment of the face valueof the policy. Appellant (Asian Crusader) denied theclaim on the ground that the answers given by theinsured to the questions appearing in his applicationfor life insurance were untrue. -Appellant: the insured was guilty of misrepresentation when 1) he answered "No" to the question (in the application) of "Has any life insurance company ever refused your application for insurance or forreinstatement of a lapsed policy or offered you apolicy different from that applied for?" when in fact, Insular Life denied his application for reinstatement of his lapsed life insurance policy 2) he gave the appellant's medical examiner falseand misleading information as to his ailment andprevious operation when he said he was operated on for a Tumor [mayoma] of thestomach associated with ulcer of stomach. Tumortaken out was hard and of a hen's egg size.Operation was two years ago in Chinese GeneralHospital by Dr. Yap. Claims he is completelyrecovered. Medical report show that insured was operated on for "peptic ulcer", involving the excision of a portion of the stomach, not tumor. ISSUE WON there was concealment (Was appellant,because of insured's aforesaid representation, misledor deceived into entering the contract or in acceptingthe risk at the rate of premium agreed upon?) HELD NO -"concealment exists where the assured hadknowledge of a fact material to the risk, and honesty,good faith, and fair dealing requires that he shouldcommunicate it to the assurer, but he designedly andintentionally withholds the same." - It has also been held "that the concealment must,in the absence of inquiries, be not only material, butfraudulent, or the fact must have been intentionallywithheld." Reasoning 1) The evidence shows that the Insular LifeAssurance Co., Ltd. approved Kwong Nam's requestfor reinstatement and amendment of his lapsedinsurance policy on April 24, 1962. It results,therefore, that when on May 12, 1962 Kwong Namanswered `No' to the question whether any lifeinsurance company ever refused his application forreinstatement of a lapsed policy he did notmisrepresent any fact. 2) Assuming that the aforesaid answer given by the insured is false, Sec. 278 of the Insurance Lawnevertheless requires that fraudulent intent on thepart of the insured be established to entitle theinsurer to rescind the contract. And as correctlyobserved by the lower court, "misrepresentation as adefense of the insurer to avoid liability is

an`affirmative defense. The duty to establish such adefense by satisfactory and convincing evidencerests upon the defendant. The evidence before theCourt does not clearly and satisfactorily establishthat defense." -Kwong Nam had informed the appellant's medicalexaminer that the tumor for which he was operatedon was ''associated with ulcer of the stomach." In theabsence of evidence that the insured had sufficientmedical knowledge as to enable him to distinguishbetween "peptic ulcer" and "a tumor", his statementthat said tumor was "associated with ulcer of thestomach" should be construed as an expressionmade in good faith of his belief as to the nature of his ailment and operation. Indeed, such statementmust be presumed to have been made by himwithout knowledge of its incorrectness and withoutany deliberate intent on his part to mislead theappellant. 3) Waiver: While it may be conceded that, from the viewpoint ofa medical expert, the information communicated wasimperfect, the same was nevertheless sufficient tohave induced appellant to make further inquiriesabout the ailment and operation of the insured. Section 32 of Insurance Law [Act No. 2427] provides: The right to information of material facts may bewaived either by the terms of insurance or by neglectto make inquiries as to such facts where they aredistinctly implied in other facts of which informationis communicated. It has been held that where, "upon the face of theapplication, a question appears to be not answeredat all or to be imperfectly answered, and the insurersissue a policy without any further inquiry, they waivethe imperfection of the answer and render theomission to answer more fully immaterial. Disposition the judgment appealed from is hereby affirmed, with costs against appellant

CANILANG v. CA (GREAT PACIFIC LIFE ASSURANCE CORP.) 223 SCRA 443 FELICIANO; June 17, 1993 NATURE Petition for review on certiorari of the decision of the Court of Appeals FACTS - June 18, 1982 Jaime Canilang was diagnosed byDr. Claudio to have sinus tachycardia.He wasdirected by the doctor to take a tranquilizer(Trazepam) and a beta-blocker drug (Aptin). - August 3, 1982 Jaime consulted Dr. Claudio again and was diagnosed to have acute bronchitis. - August 4, 1982 Jaime applied for a nonmedicalinsurance policy with Great Pacific Life AssuranceCompany. He named his wife Thelma as hisbeneficiary.He was issue the policy with a facevalue of P19,700 effective August 9, 1982. - August 5, 1983 Jaime died of congestive heartfailure, anemia and chronic anemia.Thelma filed herclaim but the insurance company refused to grant iton the ground that Jaime had concealed information. - Thelma filed a complaint against Great Pacific torecover the insurance proceeds.She testified thatshe was not aware of her husbands ailments andthat she thought he had died from a kidney disorder.- Great Pacific presented as witness Dr. Quismoriowho testified that Jaimes insurance application wasthe basis of his medical declaration and sheexplained that an applicant was required to undergomedical examination only if the applicant haddisclosed that he had previously been consulted witha doctor and had been hospitalized. - The Insurance Commissioner ordered Great Pacificto pay Thelma the insurance proceeds, includingattorneys fees, holding that Jaimes illness was notthat serious as to Great Pacifics decision to insurehim and that there was no concealment on the partof Jaime with regard to his illness. Petitioners Claim: > Thelma argues that the non-disclosure of Jaime did not amount to fraud. > She also argues that the CA erred in not holdingthat the issue in the case agreed upon between theparties before the Insurance Commission is whetheror not Jaime 'intentionally' made materialconcealment in stating his state of health; Respondents Comments: > The CA reversed the Insurance Commissionersdecision, holding that the use of the word'intentionally" by the Insurance Commissioner indefining and resolving the issue agreed upon by theparties at pre-trial before the InsuranceCommissioner was not supported by the evidenceand that the issue agreed upon by the parties hadbeen whether Jaime made a material concealment asto the state of his health at the time of the filing ofinsurance application, justifying the denial of the claim. > It also found that the failure of Jaime to discloseprevious medical consultation and treatmentconstituted material information which should havebeen communicated to Great Pacific to enable thelatter to make proper inquiries. ISSUES 1. WON Jaime intentionally withheld information from Great Pacific 2. WON the information withheld would have beenmaterial to Great Pacifics decision to grant Jaime theinsurance policy HELD 1. YES Ratio Section 27 of the Insurance Code of 1978 is properly read as referring to "any concealmentwithout regard to whether such concealment isintentional or unintentional.The restoration in 1985by B.P. Blg. 874 of the phrase "whether intentional orunintentional" merely underscored the fact that allthroughout (from 1914 to 1985), the

statute did notrequire proof that concealment must be "intentional"in order to authorize rescission by the injured party. Reasoning - Art. 27 of the 1978 Insurance Code reads that aconcealment en titles the injured party to rescind acontract of insurance, which does not include thewords whether intentional or unintentional from theprevious statutes. The Insurance Commissionerrelied on this deletion in arguing that the statuteintended to limit the kinds of concealment whichgenerate a right to rescind on the part of the injuredparty to "intentional concealments." - In the case at bar, the nature of the facts notconveyed to the insurer was such that the failure tocommunicate must have been intentional rather thanmerely inadvertent. > Jaime could not have been unaware that hisheart beat would at times rise to high andalarming levels and that he had consulted a doctortwice two months before applying for non-medicalinsurance. > The last medical consultation took place just the day before the insurance application was filed. 2. YES Ratio Materiality relates rather to the "probable and reasonable influence of the facts" upon the party towhom the communication should have been made, inassessing the risk involved in making or omitting tomake further inquiries and in accepting theapplication for insurance; that "probable andreasonable influence of the farts" concealed must, ofcourse, be determined objectively, by the judgeultimately. Reasoning - The information which Jaime failed to disclose wasmaterial to the ability of Great Pacific to estimate theprobable risk he presented as a subject of lifeinsurance. - Had Canilang disclosed his visits to his doctor, thediagnosis made and the medicines prescribed bysuch doctor, in the insurance application, it may bereasonably assumed that Great Pacific would havemade further inquiries and would have probablyrefused to issue a non-medical insurance policy or, at the very least, required a higher premium for the same coverage. - As held in the case of Saturnino vs. PhilippineAmerican Life Insurance, the waiver of medical examination in a non-medical insurance contractrenders even more material the information inquiredof the applicant concerning previous condition ofhealth and diseases suffered, for such informationnecessarily constitutes an important factor which theinsurer takes into consideration in deciding whetherto issue the policy or not. Disposition the Petition for Review is DENIED for lack of merit and the Decision of the Court of Appealsdated 16 October 1989 in C.A.-G.R. SP No. 08696 ishereby AFFIRMED. YU PANG CHENG v. CA 105 PHIL 930 BAUTISTA ANGELO; May 29, 1959 FACTS - September 5, 1950: Yu Pang Eng submitted partsII and III of his application for insurance consistingof the medical declaration made by him to themedical examiner of defendant and the medicalexaminer's report - September 7: he submitted part I of his applicationwhich is the declaration made by him to an agent ofdefendant - September 8: defendant issued to the insured Policy No. 812858 - December 27, 1950:the insured entered St.Luke's Hospital for medical treatment but he died onFebruary 27, 1951. - According to the death certificate, he died of"infiltrating medullary carcinoma, Grade 4, advancedcardiac and of lesser curvature, stomach metastasesspleen." - Plaintiff, brother and beneficiary of the insured,demanded from defendant the payment of theproceeds of the insurance policy and when thedemand was refused, he brought the present action.- The insured, in his application for insurance,particularly in his declarations to the examiningphysician, stated the following in answering thequestions propounded to him: 14. Have you ever had any of the following diseasesor symtoms? Each question must be read andanswered "Yes" or "No.". "Gastritis, Ulcer of the Stomach or any disease of

that organ? No.

INSURANCE Page 59 "Vertigo, Dizziness, Fainting-spells or Unconsciouness? No. "Cancer, Tumors or Ulcers of any kind? No. - 15. Have you ever consulted any physician riotincluded in any of the above answers? Give namesand address or physicians list ailments or accidentsand date. No." - It appears that the insured entered the ChineseGeneral Hospital for medical treatment on January29, 1950 having stayed there up to February 11,1950. - An X-ray picture of his stomach was taken and thediagnosis made of him by his doctors showed thathis illness was "peptic ulcer, bleeding." ISSUE WON the insured is guilty of concealment of somefacts material to the risk insured against which hasthe effect of avoiding the policy as found byrespondent court. HELD - It should be noted that the insured's confinementin the Chinese General Hospital took place fromJanuary 29, 1950 to February 11, 1950, whereas hisapplication for insurance wherein he stated hisanswers to the questions propounded to him by theexamining physician of defendant was submitted todefendant on September 5, 1950. - It is apparent that when the insured gave hisanswers regarding his previous ailment, particularlywith regard to "Gastritis, Ulcer of the Stomach or anydisease of that organ" and "Vertigo, Dizziness,Fainting-spells or Unconsciousness", he concealedthe ailment of which he was treated in the ChineseGeneral Hospital which precisely has directconnection with the subject of the questionspropounded. - The negative answers given by the insuredregarding his previous ailment, or his concealment ofthe fact that he was hospitalized and treated forsometime of peptic ulcer and had suffered from"dizziness, anemia, abdominal pains and tarrystools", deprived defendant of the opportunity tomake the necessary inquiry as to the nature of hispast illness so that it may form its estimate relativeto the approval of his application. - Had defendant been given such opportunity,considering the previous illness of the insured asdisclosed by the records of the Chinese GeneralHospital, defendant would probably had neverconsented to the issuance of the policy in question. In fact, according to the death certificate, the insureddied of "infiltrating medullary carcinoma, Grade, 4,advanced cardiac and of lesser curvature, stomachmetastases spleen", which may have a directconnection with his previous illness. - Our Insurance Law provides that "A neglect tocommunicate that which a party knows and ought tocommunicate, is called concealment" (Section 25,Act No. 2427). Whether intentional or unintentional,the concealment entitles the insurer to rescind thecontract of insurance (Section 26). - Our law even requires the insured to communicateto the insurer all facts within his knowledge whichare material to the contract and which the otherparty has not the means of ascertaining (Section27), and the materiality is to be determined not bythe event but solely by the probable and reasonableinfluence of the facts upon the party to whom thecommunication is due (Section 30). - Argente vs. West Coast Life Insurance Co.: "Oneground for the rescission of a contract of insuranceunder the Insurance Act is 'a concealment', which insection 25 is defined 'A neglect to communicate thatwhich a party knows and ought to communicate.'Appellant argues that the concealment wasimmaterial and insufficient to avoid the policy. Wecannot agree. In an action on a life

insurance policywhere the evidence conclusively shows that theanswers to questions concerning diseases wereuntrue, the truth or falsity of the answers becomethe determining factor. If the policy was procured byfraudulent representations, the contract of insuranceapparently set forth therein was never legallyexistent. It can fairly be assumed that had the truefacts been disclosed by the assured, the insurancewould never have been granted." Disposition Decision affirmed.

GREAT PACIFIC LIFE v. CA (supra p.34) PACIFIC BANKING CORP v. CA (ORIENTAL ASSURANCE CORPORATION) 168 SCRA 1 PARAS; November 28, 1988 NATURE Petition for review on certiorari of the CA decision,which set aside the decision of CFI Manila, which hadin turn granted the complaint for a sum of money incivil case filed by Pacific Banking against OrientalAssurance. FACTS - October 21,1963: an open Fire Policy was issued tothe Paramount Shirt Manufacturing Co. (insured), bywhich Oriental Assurance Corporation bound itself toindemnify the insured for any loss or damage, notexceeding P61,000.00, caused by fire to its propertyconsisting of stocks, materials and supplies usual toa shirt factory, including furniture, fixtures,machinery and equipment while contained in theground, second and third floors of the buildingsituated at number 256 Jaboneros St., San Nicolas,Manila, for a period of one year commencing fromthat date to October 21, 1964. - Insured was at the time of the issuance of thepolicy and is up to this time, a debtor of PacificBanking in the amount of not less P800,000.00 andthe goods described in the policy were held in trustby the insured for the Pacific Banking under thrustreceipts. - Said policy was duly endorsed to Pacific Banking asmortgagee/trustor of the properties insured, with theknowledge and consent of Oriental Assurance to theeffect that "loss if any under this policy is payable tothe Pacific Banking Corporation". - While the aforesaid policy was in full force andeffect, a fire broke out on the subject premisesdestroying the goods contained in its ground andsecond floors. Counsel for the Pacific Banking sent aletter of demand to Oriental Assurance for indemnitydue to the loss of property by fire. OrientalAssurance informed counsel that it was not yet readyto accede to the latter's demand as the former isawaiting the final report of the insurance adjuster,H.H. Bayne Adjustment Company. - Said insurance adjuster notified counsel for thePacific Banking that the insured under the policy hadnot filed any claim with it, nor submitted proof ofloss which is a clear violation of Policy ConditionNo.11, and for which reason, determination of theliability of Oriental Assurance could not be had.Pacific Banking's counsel replied asking the insuranceadjuster to verify from the records of the Bureau ofCustoms the entries of merchandise taken into thecustoms bonded warehouse razed by fire as areliable proof of loss. - For failure of the insurance company to pay the lossas demanded, Pacific Banking field before CFI anaction for a sum of money against the OrientalAssurance, in the principal sum of P61,000.00 issuedin favor of Paramount Shirt Manufacturing Co. Oriental Assurance defenses (a) lack of formal claim by insured over the loss and (b) premature filing of the suit as neither plaintiff norinsured had submitted any proof of loss on the basisof which defendant would determine its liability andthe amount thereof, either to the Oriental Assuranceor its adjuster H.H. Bayne Adjustment Co. Pacific Banking > presented evidence that insured has undeclared co-insurances with the following: P30,000.00 with Wellington Insurance; P25,000. 00 with Empire Surety and P250,000.00 with Asian Surety; undertaken by insured Paramount on the sameproperty covered by its policy with OrientalAssurance whereas the only co-insurances declaredin the subject policy are those of P30,000.00 with Malayan, P50,000.00 with South Sea,and P25.000.00 with Victory - NOTE: the defense of fraud and/or violation of non-declaration of co-insurances was not pleaded in theanswer, also not pleaded in the Motion to Dismiss. - CFI denied Oriental Assurance's motion on the ground that since the defense was raised for the firsttime, it must be deemed to have waived therequirement of proof of loss. Case was submitted fordecision. But upon MR, Oriental Asurance wasallowed to present additional evidence, "in order toprove that 'insured has committed a violation ofcondition No. 3 of the policy in relation to the otherInsurance Clause.' " CFI eventually adjudged OrientalAssurance liable to the Pacific Banking under the saidcontract of insurance. - Court of Appeals reversed. Pacific Banking's MR denied.

ISSUES 1. WON insured is guilty of fraud 2. WON mortgagee/assignee can still claim from the insurance HELD 1. YES - The crux of the controversy centers on two points:(a) unrevealed co-insurances which violated policyconditions No. 3; and (b) failure of the insured to filethe required proof of loss prior to court action. - Policy Condition No. 3 explicitly provides: TheInsured shall give notice to the Company of anyinsurance already effected, or which maysubsequently be effected, covering any of theproperty hereby insured, and unless such notice begiven and the particulars of such insurance orinsurances be stated in or endorsed on this Policy byor on behalf of the Company before the occurrenceof any loss or damage, all benefit under this policyshall be forfeited. - It is not disputed that the insured failed to revealbefore the loss three other insurances. By reason ofsaid unrevealed insurances, the insured had beenguilty of a false declaration; a clearmisrepresentation and a vital one because where theinsured had been asked to reveal but did not, thatwas deception. Otherwise stated, had the insurerknown that there were many co-insurances, it couldhave hesitated or plainly desisted from entering intosuch contract. Hence, the insured was guilty of clearfraud. - Pacific Banking's contention that the allegation offraud is but a mere inference or suspicion isuntenable. Concrete evidence of fraud or falsedeclaration by the insured was furnished by thePacific Banking itself when the facts alleged in thepolicy under clauses "Co-Insurances Declared" and"Other Insurance Clause" are materially differentfrom the actual number of co-insurances taken overthe subject property. Consequently, the wholefoundation of the contract fails, the risk does notattach and the policy never becomes a contractbetween the parties. Representations of facts are thefoundation of the contract and if the foundation doesnot exist, the superstructure does not arise.Falsehood in such representations is not shown tovary or add to the contract, or to terminate acontract which has once been made, but to showthat no contract has ever existed (Tolentino). A voidor inexistent contract is one which has no force andeffect from the very beginning, as if it had neverbeen entered into, and which cannot be validatedeither by time or by ratification. - As the insurance policy against fire expresslyrequired that notice should be given by the insuredof other insurance upon the same property, the totalabsence of such notice nullifies the policy. - Argument that notice of co-insurances may bemade orally is preposterous and negates policycondition No. 20 which requires every notice andother communications to the insurer to be written orprinted. 2. NO - Subject mortgage clause pecifically provides: Loss,if any, under this policy, shall be payable to the PACIFIC BANKING CORPORATION Manilamortgagee/trustor as its interest may appear, itbeing hereby understood and agreed that thisinsurance as to the interest of the mortgagee/trustoronly herein, shall not be invalidated by any act orneglect except fraud or misrepresentation, or arsonof the mortgagor or owner/trustee of the propertyinsured; provided, that in case the mortgagor orowner/ trustee neglects or refuses to pay anypremium, the mortgagee/ trustor shall, on demandpay the same. - The paragraph clearly states the exceptions to thegeneral rule that insurance as to the interest of themortgagee, cannot be invalidated; namely: fraud, ormisrepresentation or arson. - Concealment of the aforecited co-insurances can easily be fraud, or in the very

least,misrepresentation. It is but fair and just that wherethe insured who is primarily entitled to receive theproceeds of the policy has by its fraud and/ormisrepresentation, forfeited said right, with morereason Pacific Banking which is merely claiming asindorsee of said insured, cannot be entitled to suchproceeds. - The fact of fraud was tried by express or at leastimplied consent of the parties. Pacific Banking did notonly object to the introduction of evidence but on thecontrary, presented the very evidence that proved itsexistence. - Be that as it may, SC has ample authority to givebeyond the pleadings where in the interest of justiceand the promotion of public policy, there is a need tomake its own finding to support its conclusion.Otherwise stated, the Court can consider a fact whichsurfaced only after trial proper. - Generally, the cause of action on the policy accrueswhen the loss occurs, but when the policy providesthat no action shall be brought unless the claim isfirst presented extrajudicially in the manner providedin the policy, the cause of action will accrue from thetime the insurer finally rejects the claim for payment.- In the case at bar, policy condition No. 11specifically provides that the insured shall on thehappening of any loss or damage give notice to thecompany and shall within fifteen (15) days after suchloss or damage deliver to the Oriental Assurance (a)a claim in writing giving particular account as to thearticles or goods destroyed and the amount of theloss or damage and (b) particulars of all other NSURANCE Page 61 insurances, if any. Likewise, insured was required "athis own expense to produce, procure and give to thecompany all such further particulars, plans,specifications, books, vouchers, invoices, duplicatesor copies thereof, documents, proofs and informationwith respect to the claim". - Evidence adduced shows that 24 days after the fire,Pacific Banking merely wrote letters to OrientalAssurance to serve as a notice of loss, thereafter, theformer did not furnish the latter whatever pertinentdocuments were necessary to prove and estimate itsloss. Instead, Pacific Banking shifted upon OrientalAssurance the burden of fishing out the necessaryinformation to ascertain the particular account of thearticles destroyed by fire as well as the amount of loss. - Oriental Assurance and its adjuster notified PacificBanking that insured had not yet filed a written claimnor submitted the supporting documents incompliance with the requirements set forth in thepolicy. Despite the notice, the latter remainedunheedful. Since the required claim by insured,together with the preliminary submittal of relevantdocuments had not been complied with, it followsthat Oriental Assurance could not be deemed to havefinally rejected Pacific Banking's claim and thereforethe latter's cause of action had not yet arisen.Compliance with condition No. 11 is a requirement sine qua non to the right to maintain an action as prior thereto no violation of Pacific Banking's rightcan be attributable to Oriental Assurance. As beforesuch final rejection, there was no real necessity forbringing suit. Pacific Banking should haveendeavored to file the formal claim and procure allthe documents, papers, inventory needed by OrientalAssurance or its adjuster to ascertain the amount ofloss and after compliance await the final rejection ofits claim. Indeed, the law does not encourageunnecessary litigation. - Pacific Banking prematurely filed the civil case anddismissal thereof was warranted under thecircumstances. While it is a cardinal principle ofinsurance law that a policy or contract of insurance isto be construed liberally in favor of the insured andstrictly as against the insurer company yet, contractsof insurance, like other contracts, are to beconstrued according to the sense and meaning of theterms which the parties themselves have used. Ifsuch terms are clear and unambiguous, they must betaken and understood in their plain, ordinary andpopular sense. - Contracts of insurance are contracts of indemnityupon the terms and conditions specified in the policy.The parties have a right to impose such reasonableconditions at the time of the making of the contractas they may deem wise and necessary. Theagreement has the force of law between the parties.The terms of the policy constitute the measure of theinsurer's liability, and in order to recover, the insuredmust show himself within those terms. Thecompliance of the insured with the terms of thepolicy is a condition precedent to the light of recovery. - It appearing that insured has violated or failed toperform the conditions under No. 3 and 11 of thecontract, and such violation or want of performancehas not been waived by the insurer, the insuredcannot recover, much less the herein Pacific Banking.Courts are not permitted to make contracts for theparties; the function and duty of the courts is simplyto enforce and carry out the contracts actually made. Disposition Petition dismissed. CA affirmed.

SUNLIFE ASSURANCE COMPANY v. CA (SPS. BACANI) 245 SCRA 268 QUIASON; June 22, 1995 NATURE A petition for review on certiorari. FACTS - April 15, 1986: Robert John B. Bacani procured alife insurance contract for himself from SUNLIFE(petitioner) valued at P100K. The designatedbeneficiary was his mother, Bernarda Bacani(respondent). - June 26, 1987: the insured died in a plane crash.Bernarda Bacani filed a claim with Sunlife, seekingthe benefits of the insurance policy taken by her son.Petitioner conducted an investigation and its findingsprompted it to reject the claim on the ground thatthe insured did not disclose facts material to theissuance of the policy. The insured gave falsestatements in the application when he answered inthe negative to the question have you ever had or sought advice for urine, kidney, bladder disorder? - Sunlife discovered that two weeks prior to theissuance, insured was diagnosed with renal failure,was confined, and underwent tests. - November 17, 1988: Bacani and her husband filed for specific performance against Sunlife. RTC granted the plea on the ground that that the facts concealedby the insured were made in good faith and underthe belief that they need not be disclosed, and thatthe disclosure was not material since the policy wasnon-medical. - Sunlife appealed to the CA, but the latter deniedthe appeal on the ground that the cause of deathwas unrelated to the facts concealed by the insured. Petitioners Claim > The insured did not disclose facts relevant to theissuance of the policy, thus rescission of the contractmay be invoked by the insurance company. Respondents Comments > The actual cause of death was not relevant to theconcealed information, and the policy was enteredinto by the insured in good faith. ISSUE WON the concealment renders the insurance policy rescissible HELD YES Ratio The terms of the contract are clear. The insured is specifically required to disclose to the insurer matters relating to his health. Reasoning SEC. 26 (IC) A neglect to communicate that which a partyknows and ought to communicate, is called aconcealment. SEC. 31 (IC) Materiality is to be determined not by the event,but solely by the probable and reasonable influenceof the facts upon the party to whom communicationis due, in forming his estimate of the disadvantagesof the proposed contract or in making his inquiries - The information which the insured failed to disclosewas material and relevant to the approval and theissuance of the insurance policy. The mattersconcealed would have definitely affected petitioner'saction on his application, either by approving it withthe corresponding adjustment for a higher premiumor rejecting the same. - Good faith is no defense in concealment. It appearsthat such concealment was deliberate on the part ofthe insured. - The waiver of a medical examination [in a non-medical insurance contract] renders even morematerial the information required of the applicantconcerning previous condition of health and diseases suffered, for such information necessarily constitutesan important factor which the insurer takes intoconsideration in deciding whether to issue the policyor not. - Anent the finding that the facts concealed had nobearing to the cause of death of the insured, it is wellsettled that the insured need not die of the diseasehe had failed to disclose to the insurer. It is

sufficientthat his non-disclosure misled the insurer in forminghis estimates of the risks of the proposed insurancepolicy or in making inquiries Disposition Petition is granted and the decision of CA is reversed and set aside.

EGUARAS v. GREAT EASTERN 33 PHIL. 263 TORRES.; January 24, 1916 NATURE Appeal filed through bill of exceptions from the judgment of the CFI FACTS - Francisca Eguaras filed a written complaint in court,alleging as a cause of action that her son-inlawDominador Albay had applied in writing to thedefendant insurance company to insure his life forthe sum of P5,000, naming as the beneficiary in caseof his death the plaintiff Francisca Eguaras; that aftercompliance with the requisites and the investigationcarried on by the defendant company, it accepted theapplication for insurance and issued the policy; that,said policy being in force, the insured died, anddespite the fact that the beneficiary submittedsatisfactory proofs of his death and that thedefendant company investigated the event, still itrefused and continues to refuse to pay to the plaintiffthe value of the policy. - Defendant set forth in special defense that theinsurance policy issued in the name of DominadorAlbay had been obtained through fraud and deceitknown and consented to by the interested partiesand is therefore completely illegal, void, andineffective. - A criminal case for frustrated estafa was filed bydefendant against Ponciano Remigio, Castor Garciaand Francisca Eguaras. They were acquitted, andclaim that the judgment produces the effect ofres judicata in the present suit. ISSUE WON the life insurance obtained by Dominador Albay was issued through fraud and deceit HELD YES Ratio In a contract where one of the contracting parties may have given his consent through error,violence, intimidation, or deceit, and in any of suchcases the contract is void, even though, despite thisnullity, no crime was committed. There may not havebeen estafa in the case at bar, but it was conclusivelydemonstrated by the trial that deceit entered into theinsurance contract, fulfillment whereof is claimed,and therefore the conclusions reached by the court inthe judgment it rendered in the criminal proceedingsfor estafa do not affect this suit, nor can theyproduce in the present suit the force ofres adjudicata. Reasoning - It is proven that the signatures on the insurance applications reading "Dominado Albay" are false andforged; that the person who presented himself to Dr.Vidal to be examined was not the real DominadorAlbay, but Castor Garcia who was positivelyidentified by Dr. Vidal; that at the time of theapplication for insurance and the issuance of thepolicy which is the subject matter of this suit the realDominador Albay was informed of all thosemachinations, wherefore it is plain that the insurancecontract between the defendant and DominadorAlbay is null and void because it is false, fraudulentand illegal. Disposition The judgment appealed from is reversed and the defendant absolved from the complaintwithout special finding as to the costs.

QUA CHEE GAN v. LAW UNION AND ROCK 98 PHIL 85 REYES; December 17, 1955 FACTS - Qua Chee Gan insured 4 of his bodegas with LawUnion & Rock Insurance Co in 1937. These bodegaswere used for the storage of stocks of copra and ofhemp, baled and loose. - Fire of undetermined origin that broke out in theearly morning of July 21, 1940, and lasted almostone week, gutted and completely destroyed BodegasNos. 1, 2 and 4, with the merchandise stored therein. - Qua Chee Gan informed the insurance company of the fire. Fire adjusters of the company conducted an extensive investigation. Qua Chee Gan submitted thecorresponding fire claims, totaling P398,562.81 (butreduced to the full amount of the insurance,P370,000), the Insurance Company resistedpayment, claiming violation of warranties andconditions, filing of fraudulent claims, and that thefire had been deliberately caused by the insured orby other persons in connivance with him. - Qua Chee Gan, his brother and his employees were tried for arson, where counsel of the insurancecompany acted as a private prosecutor. They wereacquitted. - This civil suit was then instituted to claim againstthe insurance company. The CFI ruled in favor of QuaChee Gan and ordered Law Union Rock Co. to pay. ISSUES 1. WON there was a breach of the fire hydrant warranty 2. WON the insured violated the Hemp warranty 3. WON Qua Chee Gan is guilty of overvaluation 4. WON Qua Chee Gan caused the fire 5. WON there was an error in the amount of copra and hemp lost 6. WON the claims contained false and fraudulent statements HELD 1. NO - It is argued that he should have 11 fire hydrants inthe compound, but he only had 2. We are inagreement with the trial Court that the appellant isbarred by waiver (or rather estoppel) to claimviolation of the so-called fire hydrants warranty, forthe reason that knowing fully all that the number ofhydrants demanded therein never existed from thevery beginning, the appellant nevertheless issued thepolicies in question subject to such warranty, andreceived the corresponding premiums. 2. NO - The insurance company avers that the insuredviolated the hemp warranty when it admitted that ithad 36 cans of gasoline in the building. It is well tonote that gasoline is not specifically mentionedamong the prohibited articles listed in the so-called"hemp warranty." The cause relied upon by theinsurer speaks of "oils (animal and/or vegetableand/or mineral and/or their liquid products having aflash point below 300o Fahrenheit", and is decidedlyambiguous and uncertain; for in ordinary parlance,"Oils" mean "lubricants" and not gasoline orkerosene. And how many insured, it may well be wondered, are in a position to understand ordetermine "flash point below 003o Fahrenheit. Here,again, by reason of the exclusive control of theinsurance company over the terms and phraseologyof the contract, the ambiguity must be held strictlyagainst the insurer and liberally in favor of theinsured, especially to avoid a forfeiture - Another point that is in favor of the insured is thatthe gasoline kept in Bodega No. 2 was only incidentalto his business, being no more than a customary 2day's supply for the five or six motor vehicles usedfor transporting of the stored merchandise). "It iswell settled that the keeping of inflammable oils onthe premises though prohibited by the policy doesnot void it if such keeping is incidental to thebusiness." (Bachrach vs. British American Ass. Co.,17 Phil. 555, 560) 3. NO - The charge that the insured failed or refused tosubmit to the examiners of the insurer the books,vouchers, etc. demanded by them was foundunsubstantiated by the trial Court, and no reasonhas been shown to alter this finding. - In view of the discrepancy in the valuationsbetween the insured and the adjuster Stewart for theinsurer, the Court referred the controversy to agovernment auditor, Apolonio Ramos; but the latterreached a different result from the other two. Notonly that, but Ramos reported two

differentvaluations that could be reached according to themethods employed. Clearly then, the charge offraudulent overvaluation cannot be seriouslyentertained. 4. NO - This defense is predicted on the assumption thatthe insured was in financial difficulties and set thefire to defraud the insurance company, presumably inorder to pay off the Philippine National Bank, towhich most of the insured hemp and copra waspledged. This defense is fatally undermined by theestablished fact that, notwithstanding the insurer'srefusal to pay the value of the policies the extensiveresources of the insured enabled him to pay off theNational Bank in a short time; and if he was able todo so, no motive appears for attempt to defraud theinsurer. While the acquittal of the insured in thearson case is not res judicata on the present civilaction, the insurer's evidence, to judge from thedecision in the criminal case, is practically identical inboth cases and must lead to the same result, sincethe proof to establish the defense of connivance atthe fire in order to defraud the insurer "cannot be materially less convincing than that required in order to convict the insured of the crime of arson. 5. NO - As to the defense that the burned bodegas couldnot possibly have contained the quantities of copraand hemp stated in the fire claims, the insurer's caserests almost exclusively on the estimates, inferencesand conclusions of its adjuster investigator,Alexander D. Stewart, who examined the premisesduring and after the fire. His testimony, however,was based on inferences from the photographs andtraces found after the fire, and must yield to thecontradictory testimony of engineer Andres Bolinas,and specially of the then Chief of the LoanDepartment of the National Bank's Legaspi branch,Porfirio Barrios, and of Bank Appraiser LoretoSamson, who actually saw the contents of thebodegas shortly before the fire, while inspectingthem for the mortgagee Bank 6. NO - Appellant insurance company also contends thatthe claims filed by the insured contained false andfraudulent statements that avoided the insurancepolicy. But the trial Court found that thediscrepancies were a result of the insured'serroneous interpretation of the provisions of theinsurance policies and claim forms, caused by hisimperfect knowledge of English, and that themisstatements were innocently made and withoutintent to defraud. The trial courts ruling must beupheld. - For example, the occurrence of previous fires in the premises insured in 1939, altho omitted in theclaims, Exhibits EE and FF, were neverthelessrevealed by the insured in his claims Exhibits Q (filedsimultaneously with them), KK and WW. Consideringthat all these claims were submitted to the smaeagent, and that this same agent had paid the losscaused by the 1939 fire, we find no error in the trialCourt's acceptance of the insured's explanation thatthe omission in Exhibits EE and FF was due toinadvertance, for the insured could hardly expectunder such circumstances, that the 1939 would passunnoticed by the insurance agents. Similarly, the 20per cent overclaim on 70 per cent of the hemo stock,was explained by the insured as caused by his beliefthat he was entitled to include in the claim hisexpected profit on the 70 per cent of the hemp,because the same was already contracted for andsold to other parties before the fire occurred.Compared with other cases of over-valuationrecorded in our judicial annals, the 20 per cent excess in the case of the insured is not by itselfsufficient to establish fraudulent intent. Certainly, theinsured's overclaim of 20 per cent in the case at bar,duly explained by him to the Court a quo, appearspuny by comparison (compared to other cases citedby the court), and can not be regarded as "morethan misstatement, more than inadvertence ofmistake, more than a mere error in opinion, morethan a slight exaggeration" that would entitle theinsurer to avoid the policy. It is well to note that theovercharge of 20 per cent was claimed only on apart(70 per cent) of the hemp stock; had the insuredacted with fraudulent intent, nothing prevented himfrom increasing the value of all of his copra, hempand buildings in the same proportion. This alsoapplies to the alleged fraudulent claim for burnedempty sacks, that was likewise explained to oursatisfaction and that of the trial Court. The rule isthat to avoid a policy, the false swearing must bewillful and with intent to defraud which was not thecause. Of course, the lack of fraudulent intent wouldnot authorize the collection of the expected profitunder the terms of the polices, and the trial Courtcorrectly deducted the same from its award. Disposition Decision affirmed

ARGENTE v. WEST COAST LIFE 51 PHIL 725 MALCOLM; March 19, 1928 FACTS - This is an action upon a joint life insurance policyfor P15,000 issued by the West Coast Life InsuranceCo., on May 15, 1925, in favor of Bernardo Argente,and his wife, Vicenta de Ocampo, the latter havingdied on November 18, 1925. Fraud in obtaining thepolicy was pleaded by way of special defense. On theissue thus suggested, the court adopted the theoryof the defendant, and held the insurance policy nulland void, with the result that the complaint wasdismissed, with costs. -Bernardo Argente signed an application for jointinsurance with his wife in the sum of P2,000. Thewife, Vicenta de Ocampo, signed a like application forthe same policy. - Bernardo Argente and his wife was examined by Dr.Cesareo Sta. Ana, a medical examiner for the WestCoast Life Insurance Co. which did not show previousand existing health problems. - A temporary policy for P15,000 was issued toBernardo Argente and his wife as of May 15, 1925.In view of the fact that more than thirty days had elapsed since the applicants were examined by thecompany's physician, each of them was required tofile a certificate of health before the policy wasdelivered to them. - On November 18, 1925, Vicenta de Ocampo died ofcerebral apoplexy. Thereafter Bernardo Argentepresented a claim. Following investigation conductedby the Manager of the Manila office of the insurancecompany, it was apparently disclosed that theanswers given by the insured in their medicalexaminations with regard to their health andprevious illnesses and medical attendance wereuntrue. West Coast Life Insurance Co. refused to paythe claim of Bernardo Argente, and wrote him to theeffect that the claim was rejected because theinsurance was obtained through fraud andmisrepresentation. - It is admitted that it appears in the MedicalExaminer's Report that Bernardo Argente gave falseresponses. As well as with the Medical Examiner'sReport that Vicenta de Ocampo. It is, however, notdisputed that Vicenta de Ocampo was taken by apatrolman, at the request of her husband, BernardoArgente, on May 19, 1924, to the Meisic policestation, and from there was transferred to the SanLazaro Hospital. In San Lazaro Hospital, her casewas diagnosed by the admitting physician as"alcoholism," but later Doctor Domingo made adiagnosis of probable "manicdepressive psychosis,"and still, later in Mary Chiles Hospital, made a finaldiagnosis of "phychoneurosis." - Bernardo Argente, while readily conceding most ofthe facts herein narrated, yet alleges that both heand his wife revealed to the company's physician,Doctor Sta. Ana, all the facts concerning theirprevious illnesses and medical attendance, but thatDoctor Sta. Ana, presumably acting in collusion withthe insurance agent, Jose Geronimo del Rosario,failed to record them in the medical reports. Theevidence on these points consists of the testimony ofthe plaintiff and his subordinate clerk, ApolonioEspiritu, on the one hand, and of the testimony ofDoctor Sta. Ana and Jose Geronimo del Rosario onthe other. This was rejected by the Trial Court. Trialjudge found with the insurance company with regardto the question of fact. SC agrees. There appears nomotive whatever on the part of Doctor Sta. Ana tofalsify the Medical Examiner's Reports and therebynot only jeopardize his career as a physician, butalso gravely implicate himself criminally. ISSUE WON the contract of insurance may be rescinded HELD YES - Bernardo Argente and his wife applications werefalse with respect to their state of health during theperiod of five years preceding the date of suchapplications and that they knew the representationsmade by them in their applications were false. Thequestion arises as to the state of the law in relationthereto. - One ground for the rescission of a contract ofinsurance under the Insurance Act is "aconcealment," which in section 25 is defined as "Aneglect to communicate that which a party knowsand ought to communicate." In an action on a lifeinsurance policy where the evidence conclusivelyshows that the answers to questions concerningdiseases were untrue, the truth or falsity of theanswers become the determining factor. If the policywas procured by fraudulent representations, thecontract of insurance apparently set forth thereinwas never legally existent. It can fairly be assumedthat had the true facts been disclosed by theassured, the insurance would never have beengranted. - In Joyce, The Law of Insurance, second edition, volume 3, Chapter LV, is found the following:

"The basis of the rule vitiating the contract incases of concealment is that it misleads ordeceives the insurer into accepting the risk, oraccepting it at the rate of premium agreed upon;The insurer, relying upon the belief that theassured will disclose every material fact within hisactual or presumed knowledge, is misled into abelief that the circumstance withheld does notexist, and he is thereby induced to estimate therisk upon a false basis that it does not exist. Theprincipal question, therefore, must be, Was theassurer misled or deceived into entering a contractobligation or in fixing the premium of insurance bya withholding of material information or factswithin the assured's knowledge or presumedknowledge? "It therefore follows that the assurer in assuming arisk is entitled to know every material fact of whichthe assured has exclusive or peculiar knowledge,as well as all material facts which directly tend toincrease the hazard or risk which are known by theassured, or which ought to be or are presumed to be known by him. And a concealment of such factsvitiates the policy. 'It does not seem to benecessary . . . that the . . . suppression of thetruth should have been willful.' If it were but aninadvertent omission, yet if it were material to therisk and such as the plaintiff should have known tobe so, it would render the policy void. But it is heldthat if untrue or false answers are given inresponse to inquiries and they relate to materialfacts the policy is avoided without regard to theknowledge or fraud of assured, although under thestatute statements are representations which mustbe fraudulent to avoid the policy. So under certaincodes the important inquiries are whether theconcealment was willful and related to a mattermaterial to the risk. xxx xxx xxx "If the assured has exclusive knowledge ofmaterial facts, he should fully and fairly disclosethe same, whether he believes them material ornot. But notwithstanding this general rule it willnot infrequently happen, especially in life risks,that the assured may have a knowledge actual orpresumed of material facts, and yet entertain anhonest belief that they are not material. . . . Thedetermination of the point whether there has orhas not been a material concealment must restlargely in all cases upon the form of the questionspropounded and the exact terms of the contract.Thus, where in addition to specifically nameddiseases the insured was asked whether he hadhad any sickness within ten years, to which heanswered 'No,' and it was proven that within thatperiod he had had a slight attack of pharyngitis, itwas held a question properly for the jury whethersuch an inflammation of the throat was a 'sickness'within the intent of the inquiry, and the courtremarked on the appeal decision that if it could beheld as a matter of law that the policy was therebyavoided, then it was a mere device on the part ofinsurance companies to obtain money withoutrendering themselves liable under the policy. . . . ". . . The question should be left to the jurywhether the assured truly represented the state ofhis health so as not to mislead or deceive theinsurer; and if he did not deal in good faith withthe insurer in that matter, then the inquiry shouldbe made, Did he know the state of his health so asto be able to furnish a proper answer to suchquestions as are propounded? A Massachusettscase, if construed as it is frequently cited, wouldbe opposed to the above conclusion; but, on the contrary, it sustains it, for the reason thatsymptoms of consumption had so far developedthemselves within a few months prior to effectingthe insurance as to induce a reasonable belief thatthe applicant had that fatal disease, and we shouldfurther construe this case as establishing the rulethat such a matter cannot rest alone upon theassured's belief irrespective of what is areasonable belief, but that it ought to be judged bythe criterion whether the belief is one fairlywarranted by the circumstances. A case in Indiana,however, holds that if the assured has someaffection or ailment of one or more of the organsinquired about so well defined and marked as tomaterially derange for a time the functions of suchorgan, as in the case of Bright's disease, the policywill be avoided by a nondisclosure, irrespective ofthe fact whether the assured knew of such ailmentor not. . . . " - Lastly, appellant contends that even if theinsurance company had a right to rescind thecontract, such right cannot now be enforced in viewof the provisions of section 47 of the Insurance Actproviding "Whenever a right to rescind a contract ofinsurance is given to the insurer by any provision ofthis chapter, such right must be exercised previousto the commencement of an action on the contract."This section was derived from section 2583 of theCalifornia Civil Code, but in contrast thereto, makesuse of the imperative "must" instead of thepermissive "may." Nevertheless, there are twoanswers to the problem as propounded. The first isthat the California law as construed by the codeexaminers, at whose recommendation it wasadopted, conceded that "A failure to exercise theright (of rescission), cannot, of course, prejudice anydefense to the action which the concealment mayfurnish." (Codes of California Annotated; Tan ChayHeng vs. West Coast Life Insurance Company[1927], p. 80, ante.) The second answer is that theinsurance company more than one

month previous tothe commencement of the present action wrote theplaintiff and informed him that the insurance contractwas void because it had been procured throughfraudulent representations, and offered to refund tothe plaintiff the premium which the latter had paidupon the return of the policy for cancellation. As heldin California as to a fire insurance policy, where anyof the material representations are false, theinsurer's tender of the premium and notice that thepolicy is canceled, before the commencement of suitthereon, operate to rescind the contract of insurance. (Rankin vs. Amazon Insurance Co. [1891], 89 Cal., 203.) Disposition Judgment affirmed, with the costs of this instance against the appellant.

GREAT PACIFIC LIFE v. CA (NGO HING) 89 SCRA 543 DE CASTRO, J; April 30, 1979 NATURE Petition for certiorari FACTS - On March 14, 1957, private respondent Ngo Hingfiled an application with the Great Pacific LifeAssurance Co. (Pacific Life) for a 20 year endowmentpolicy of P50k on the life of his 1 year old daughter,Helen. Ngo Hing supplied the essetntial data whichpetitioner Mondragon, branch manager of the PacificLife in Cebu, wrote on the corresponding form in hisown handwriting, later typing the data on anapplication form signed by Ngo Hing. The latter paidthe P1077.75 annual premium but retained P1,317as commission as he was also a duly authorizedagent of Pacific Life. The binding deposit receipt wasthen issued to Ngo Hing; Mondragon handwrote hisstrong recommendation for the approval of theapplication on the back of the form. - On April 30, Mondragon received a letter fromPacific Life which stated that the 20 year endowmentplan was not available for minors below 7, but thatPacific Life could consider the same under theJuvenile Triple Action Plan, advising that if the offerwas acceptable, the Juvenile NonMedical Declarationbe sent to the company. -Mondragon allegedly failed to inform Ngo Hing ofthe non-acceptance of the insurance plan, insteadwriting Pacific Life again, recommending the approvalof the endowment plan to children since customershad been asking for such coverage since 1954. -On May 28, 1957, Helen died of influenza. Ngo Hingsought the payment of the proceeds of theinsurance, but having failed to do so, filed an actionfor recovery with the CFI of Cebu. The Court orderedPacific Life to pay P50k with 6% interest, hence thispetition. ISSUE WON the binding deposit receipt constituted a temporary contract of the life insurance in question HELD NO - The binding deposit receipt is merely a provisionalcontract and only upon compliance with the ffconditions: (1) that the company be satisfied thatthe applicant was insurable on standard rates (2)that if the company does not accept the applicationand offers a different policy, the insurance contractshall not be binding until the applicant accepts thenew policy (3) that if the applicant is not found to beinsurable on standard rates and the application isdisapproved, the insurance shall not be in force atany time and the premium be returned to the applicant. -This implies the receipt is merely anacknowledgement, on behalf of the company, thatthe Cebu branch of Pacific Life had received thepremium and had accepted the application subject toprocessing by the insurance company, which willapprove or reject it depending on whether theapplicant is insurable on standard rates. As such, thereceipt was never in force it does not insureoutright. No liability attaches until the principalapproves the risk and a receipt is given by theagent; because private respondent failed to acceptPacific Lifes offer for the Juve nile Triple Action plan,there was no meeting of the minds and thus nocontract. Also, being an authorized agent of PacificLife, Ngo Hing must have known the company didnot offer the insurance applied for and merely took achance on Mondragons recommendation. Disposition the decision appealed from is set aside, absolving Pacific Life from their civil liabilities EDILLON v. MANILA BANKERS LIFE 117 SCRA 187 VASQUEZ; September 30, 1982 NATURE Appeal from a decision of the CFI FACTS - Sometime in April 1969, Carmen O, Lapuz appliedwith respondent insurance corporation for insurancecoverage against accident and injuries. In theapplication form which was dated April 15, 1969, shegave the date of her birth as July 11, 1904. On thesame date, she paid the sum of P20.00

representingthe premium for which she was issued thecorresponding receipt signed by an authorized agentof the respondent insurance corporation. Upon thefiling of said application and the payment of

thepremium on the policy applied for, the respondent INSURANCE Page 66 insurance corporation issued to Carmen O. Lapuz its Certificate of Insurance. The policy was to be effective for a period of 90 days. - On May 31, 1969 or during the effectivity of theInsurance, Carmen O. Lapuz died in a vehicularaccident. - On June 7, 1969, petitioner Regina L. Edillon, asister of the insured and who was the namedbeneficiary in the policy, filed her claim for theproceeds of the insurance, submitting all thenecessary papers and other requisites with theprivate respondent. Her claim having been denied,Regina L. Edillon instituted this action in the Court ofFirst Instance of Rizal. - In resisting the claim of the petitioner, therespondent insurance corporation relies on aprovision contained in the Certificate of Insurance,excluding its liability to pay claims under the policy inbehalf of "persons who are under the age of sixteen(16) years of age or over the age of sixty (60) years..." It is pointed out that the insured being over sixty(60) years of age when she applied for the insurancecoverage, the policy was null and void, and no riskon the part of the respondent insurance corporationhad arisen therefrom. - RTC dismissed the complaint. ISSUE WON the acceptance by the private respondentinsurance corporation of the premium and theissuance of the corresponding certificate of insuranceshould be deemed a waiver of the exclusionarycondition of overage stated in the said certificate ofinsurance HELD YES - The age of the insured Carmen 0. Lapuz was notconcealed to the insurance company. Her applicationfor insurance coverage which was on a printed formfurnished by private respondent and which containedvery few items of information clearly indicated herage of the time of filing the same to be almost 65years of age. Despite such information which couldhardly be overlooked in the application form,considering its prominence thereon and itsmateriality to the coverage applied for, therespondent insurance corporation received herpayment of premium and issued the correspondingcertificate of insurance without question. Theaccident which resulted in the death of the insured, arisk covered by the policy, occurred on May 31, 1969 or FORTY-FIVE (45) DAYS after the insurancecoverage was applied for. There was sufficient timefor the private respondent to process the applicationand to notice that the applicant was over 60 years ofage and thereby cancel the policy on that ground if itwas minded to do so. If the private respondent failedto act, it is either because it was willing to waivesuch disqualification; or, through the negligence orincompetence of its employees for which it has onlyitself to blame, it simply overlooked such fact. Underthe circumstances, the insurance corporation isalready deemed in estoppel. Its inaction to revokethe policy despite a departure from the exclusionarycondition contained in the said policy constituted awaiver of such condition. Disposition Judgment appealed from is REVERSED and SET ASIDE and respondent insurancecorporation is ordered to pay to the petitioner theproceeds of Insurance

HARDING v. COMMERCIAL UNION (supra p.36) TAN v. CA ( PHILIPPINE AMERICAN LIFE INSURANCE COMPANY) 174 SCRA 403 GUTIERREZ; June 29, 1989 NATURE Review oncertiorari of the decision of the Court ofAppeals affirming the decision of the InsuranceCommissioner FACTS - On September 23,1973, Tan Lee Siong, father ofherein petitioners, applied for life insurance in theamount of P 80,000.00 with respondent company.Said application was approved and was issuedeffective November 6, 1973 - On April 26,1975, Tan Lee Siong died of hepatoma(Exhibit B). Petitioners then filed with respondentcompany their claim for the proceeds of the lifeinsurance policy -respondent company denied petitioners' claim andrescinded the policy by reason of the allegedmisrepresentation and concealment of material factsmade by the deceased Tan Lee Siong in hisapplication for insurance. The premiums paid on thepolicy were thereupon refunded - Petitioners filed on November 27, 1975, acomplaint against the former with the Office of theInsurance Commissioner. Commissioner deniedpetition. CA affirmed Commissioners decision ISSUE WON according to Sec. 48of the Insurance Code,insurance company is barred from rescinding contract HELD - Section 48. Whenever a right to rescind a contract of insurance is given to the insurer by any provisionof this chapter, such right must be exercised previousto the commencement of an action on the contract.After a policy of life insurance made payable on thedeath of the insured shall have been in force duringthe lifetime of the insured for a period of two yearsfrom the date of its issue or of its last reinstatement,the insurer cannot prove that the policy is voidab initio or is rescindable by reason of the fraudulent concealment or misrepresentation of the insured or his agent. - According to the petitioners, the Insurance Law wasamended and the second paragraph of Section 48added to prevent the insurance company fromexercising a right to rescind after the death of theinsured - The so-called "incontestability clause" precludes theinsurer from raising the defenses of falserepresentations or concealment of material factsinsofar as health and previous diseases areconcerned if the insurance has been in force for atleast two years during the insured's lifetime. Thephrase "during the lifetime" found in Section 48simply means that the policy is no longer consideredin force after the insured has died. The key phrase inthe second paragraph of Section 48 is "for a periodof two years." - The policy was issued on November 6,1973 and theinsured died on April 26,1975. The policy was thus inforce for a period of only one year and five months.Considering that the insured died before the two-year period had lapsed, respondent company is not,therefore, barred from proving that the policy is void ab initio by reason of the insured's fraudulent concealment or misrepresentation. - The petitioners contend that there could have beenno concealment or misrepresentation by their latefather because Tan Lee Siong did not have to buyinsurance. He was only pressured by insistentsalesmen to do so -The legislative answer to the arguments posed bythe petitioners is the "incontestability clause" addedby the second paragraph of Section 48. The insurerhas two years from the date of issuance of

the INSURANCE Page 67

insurance contract or of its last reinstatement withinwhich to contest the policy, whether or not, theinsured still lives within such period. After two years,the defenses of concealment or misrepresentation,no matter how patent or well founded, no longer lie

TAN CHAY HENG v. WEST COAST LIFE INSURANCE 51 PHIL 80 JOHNS; November 21, 1927 FACTS - Plaintiff alleges that defendant accepted andapproved a life insurance policy of for the sum ofP10,000 in which the plaintiff was the solebeneficiary; that the policy was issued upon thepayment by the said Tan Ceang of the first year'spremium amounting to P936; that in and by itsterms, the defendant agreed to pay the plaintiff asbeneficiary the amount of the policy upon the receiptof the proofs of the death of the insured while thepolicy was in force; that without any premium beingdue or unpaid, Tan Ceang died on May 10, 1925;that in June, 1925, plaintiff submitted the proofs ofthe death of Tan Ceang with a claim for the paymentof the policy which the defendant refused to pay, forwhich he prays for a corresponding judgment, withlegal interest from the date of the policy, and costs. - Defendant alleges that the insurance policy on thelife of Tan Ceang, upon which plaintiff's action isbased, was obtained by the plaintiff in confabulationwith one Go Chulian, of Bacolod, Negros Occidental;Francisco Sanchez of the same place; and Dr. V. S.Locsin, of La Carlota, Negros Occidental, thru fraudand deceit perpetrated against this defendant in thefollowing manner, to wit: 1.Go, Sanchez and Locsin, caused Tan Caeng tosign an application for insurance with thedefendant in the sum of P10,000, in which it wassaid that Tan Ceang was single and was amerchant, and that the plaintiff Tan Chai Heng,the beneficiary, was his nephew, whereas intruth and in fact and as the plaintiff and his saidcoconspirators well knew, the said Tan Ceangwas not single but was married and had severalchildren; and was not a merchant but a mereemployee of Tan Quina from whom he receivedonly a meager salary, and that plaintiff was not anephew of the said Tan Ceang. 2.Tan Ceang was seriously ill, suffering frompulmonary tuberculosis of about three years'duration, which illness was incurable and was well known to the plaintiff and his said coconspirators. 3.Locsin, in his capacity as medical examiner forthe defendant, prepared and falsified thenecessary medical certificate, in which it wasmade to appear, among other things, that TanCeang had never used morphine, cocaine or anyother drug; that he was then in good health andhad never consulted any physician; that he hadnever spit blood; and that there was no sign ofeither present or past disease of his lungs;whereas in truth and in fact, plaintiff andcoconspirators well knew, Tan Ceang wasaddicted to morphine, cocaine, and opium andhad been convicted and imprisoned therefor, andfor about three year prior thereto had beensuffering from pulmonary tuberculosis. 4.Plaintiff caused a confidential report to thedefendant insurance company to be signed byone V. Sy Yock Kian, who was an employee of GoChulian, in which it was falsely represented thatTan Ceang was worth about P40,000, had anannual income of from eight to ten thousandpesos net, had the appearance of good health,and never had tuberculosis. 5.After said application for insurance, medicalcertificate and confidential report had beenprepared and falsified, plaintiff andcoconspirators caused the same to be forwardedto the defendant at its office in Manila, themedical certificate thru the said Dr. V. S. Locsinas medical examiner, and said application forinsurance and confidential report thru the saidFrancisco Sanchez in his capacity as one of theagents of the defendant insurance company inthe Province of Occidental Negros; that thedefendant, believing that the representationsmade in said document were true, and relyingthereon, provisionally accepted the saidapplication for insurance on the life of Tan Ceangin the sum of P10,000 and issued a temporarypolicy pending the final approval or disapprovalof said application by defendant's home-office inSan Francisco, California, where in case ofapproval a permanent policy was to be issued;that such permanent policy was never deliveredto the plaintiff because defendant discovered thefraud before its delivery. 6.That the first agreed annual premium on theinsurance in question of P936.50 not havingbeen paid within 60 days after medicalexamination of the applicant as required by the regulations of the defendant insurance company,plaintiff and coconspirators caused Tan Ceang tosign a health certificate for reinstatement; thatthe said temporary policy was delivered bydefendant to the insured on April 10, 1925, in the belief that said

statements andrepresentations were true and in reliancethereon. 7.2 months after the supposed medicalexamination above referred to, and exactly 1month after the date of the health certificate forreinstatement above set forth, Tan Ceang died inValladolid, Occidental Negros, of pulmonarytuberculosis, the same illness from whichsuffering at the time it is supposed he wasexamined by Dr. V. S. Locsin, but that theplaintiff coconspirators, pursuant to theirconspiracy, caused the said Dr. V. S. Locsin tostate falsely in the certificate of death that thesaid Tan Ceang had died of cerebral hemorrhage. - Defendant also alleges that plaintiff was, like V. SyYock Kian, an employee of Go Chulian; that the latterwas the ringleader of a gang of malefactors, who,during, and for some years previous to the datesabove mentioned, were engaged in the illicitenterprise of procuring fraudulent life insurancesfrom the present defendant, similar to the one inquestion, and which enterprise was capitalized byhim by furnishing the funds with which to pay thepremium on said fraudulent insurance; that the saidGo Chulian was the one who furnished the moneywith which to pay the first and only annual premiumon the insurance here in question, amounting toP936.50; that the said Go Chulian, on August 28,1926, was convicted by the Court of First Instance ofthe City of Manila, in criminal case No. 31425 of thatcourt, of the crime of falsification of privatedocuments in connection with an fraudulentinsurance, similar to the present, committed againstthis defendant in the month of September, 1924;that in the same case the said Francisco Sanchezwas one of the coaccused of the said Go Chulian butwas discharged from the complaint, because heoffered himself and was utilized as a state's witness;that there is another civil action now pending againstGo Chulian and Sanchez in the Court of FirstInstance of Manila (civil case No. 28680), in whichthe present defendant is the plaintiff, for therecovery of the amounts of two insurance policiesaggregating P19,000, fraudulently obtained by thesaid Go Chulian and Sanchez. - To this, plaintiff filed a demurrer which was granted. ISSUE WON defense is barred by Art. 47 HELD NO Ratio The word "rescind" has a well defined legal meaning, and as applied to contracts, it presupposes the existence of a contract to rescind. Reasoning - Plaintiff vigorously contends that section 47 of theInsurance Act should be applied, and that when soapplied, defendant is barred and estopped to pleadand set forth the matters alleged in its specialdefense. That section is as follows: Whenever a right to rescind a contract of insuranceis given to the insurer by any provision of thischapter, such right must be exercised previous tothe commencement of an action on the contract. - It will be noted that defendant does not seek tohave the alleged insurance contract rescinded. Itdenies that it ever made any contract of insurance onthe life of Tan Ceang or that any such a contract everexisted, and that is the question which it seeks tohave litigated by its special defense. In the verynature of things, if the defendant never made orentered into the contract in question, there is nocontract to rescind, and, hence, section 47 uponwhich the lower based its decision in sustaining thedemurrer does not apply. As stated, an action torescind a contract is founded upon and presupposesthe existence of the contract which is sought to berescinded. If all of the material matters set forth andalleged in the defendant's special plea are true, therewas no valid contract of insurance, for the simplereason that the minds of the parties never met andnever agreed upon the terms and conditions of thecontract. We are clearly of the opinion that, if suchmatters are known to exist by a preponderance ofthe evidence, they would constitute a valid defenseto plaintiff's cause of action. Upon the question as towhether or not they or are not true, we do not at thistime have or express any opinion, but we are clearthat section 47 does not apply to the allegationsmade in the answer, and that the trial court erred insustaining the demurrer. Disposition The judgment of the lower court is reversed and the case is remanded for such otherand further proceedings as are not inconsistent withthis opinion, with costs against the plaintiff. PIONEER INSURANCE

AND SURETY CORPORATION v. YAP 61 SCRA 426 FERNANDEZ; December 19, 1974 NATURE Appeal by certiorari from CA decision affirming a CFIdecision which declared plaintiff Yap entitled torecover from defendant Pioneer Insurance andSurety Corp, the full amount of the damage inquiredin Policy No. 4219 FACTS - Yap owned a store in a 2 storey building, where shesold shopping bags and footwear. Her son-inlaw wasin charge of the store - April 19, 1962- Yap took out Fire Insurance PolicyNo. 4216 from Pioneer with a face value of P25,000covering her stocks, office furniture, fixtures, etc. - among the conditions set forth: The Insured shall give notice to theCompany of any insurance or insurances alreadyeffected, or which may subsequently beeffected, covering any of the property herebyinsured, and unless such notice be given and the particulars of such insurance or insurancesbe stated in, or endorsed on this Policy by or onbehalf of the Company before the occurrence ofany loss or damage, all benefits under thisPolicy shall be forfeited. (emphasis supplied) It is understood that, except as may bestated on the face of this policy there is no otherinsurance on the property hereby covered andno other insurance is allowed except by theconsent of the Company endorsed hereon. Anyfalse declaration or breach or this condition willrender this policy null and void. - At the time of insurance of Policy 4219(April 19,1962), an insurance policy for P20,000 issued by theGreat American Insurance Company covering thesame properties was noted on said policy as coinsurance. - August 29, 1962 : parties executed an endorsement on Policy 4219 stating: It is hereby declared and agreed that the co-insurance existing at present under this policy is asfollows: P20,000.00 Northwest Ins., and not as originally stated. (emphasis supplied) Except as varied by this endorsement, all other terms and conditions remain unchanged. - September 26, 1962: Yap took out another fire insurance policy for P20,000 covering the sameproperties, from Federal Insurance Company. Thispolicy was procured without notice to and the written consent of Pioneer, and was therefore not noted as a co-insurance in Policy 4219. - December 19, 1962: Fire burned Yaps store ISSUE WON petitioner should be absolved from liability onFire insurance Policy No. 4219 on account of anyviolation by respondent Yap of the co-insuranceclause therein HELD YES - The petitioner should be absolved. Reasoning - There was a violation by Yap of the co-insurance clause contained in Policy No. 4219 which resulted in the avoidance of the petitioners liability. - By the plain terms of the policy, other insurancewithout the consent of petitioner would ipso factoavoid the contract. It required no affirmative act ofelection on the part of the company to makeoperative the clause avoiding the contract, whereverthe specified conditions should occur. Its obligationsceased, unless, being informed of the fact, itconsented to the additional insurance. - The obvious purpose of the aforesaid requirementin the policy is to prevent over-insurance and thusavert the perpetration of fraud. The public, as well asthe insurer, is interested in preventing the situationin which a fire would be profitable to the insured.According to Justice Story: "The insured has no rightto complain, for he assents to comply with all thestipulation on his side, in order to entitle himself tothe benefit of the contract, which, upon reason orprinciple, he has no right to ask the court

to dispensewith the performance of his own part of theagreement, and yet to bind the other party toobligations, which, but for those stipulation wouldnot have been entered into." Disposition the appealed judgment of the Court of Appeals is reversed and set aside, and the petitioner absolved from all liability under the policy.

NEW LIFE ENTERPRISES v. CA 207 SCRA 609 REGALADO; March 31, 1992 FACTS - Julian Sy and Jose Sy Bang are partners engaged inthe business of selling construction materials underthe business name New Life Enterprises. Julian Syinsured against fire the stocks in trade of New LifeEnterprises with Western Guaranty Corporation,Reliance Surety and Insurance Co. Inc., andEquitable Insurance Corporation in the aggregateamount of P1,550,000.00. When the building whereNew Life Enterprises was located, along with thestocks in trade therein, were gutted by fire,petitioners filed an insurance claim against the threecompanies. The insurance companies all deniedJulian Sys claim on the ground of breach of policycondition, (i.e., the other insurance clause whichrequired New Life Enterprises to inform each of theinsurance companies in case the former insures withanother company the same property already insuredby each of the insurance companies). - Because of the denial of their claims for paymentby the 3 insurance companies, petitioners filedseparate civil actions against the former before theRegional Trial Court of Lucena City, which cases wereconsolidated for trial. The trial court ruled in favor ofpetitioner. However, the Court of Appeals reversedthe trial courts decision, found petitioner to haveviolated Clauses 3 and 27 of the separate insurancepolicies issued by the 3 companies, and exoneratedthe insurance companies from liability. ISSUE WON petitioners violated the Other Insurance Clause of the insurance policies HELD YES - Petitioners admit that the respective insurancepolicies issued by private respondents did not stateor endorse thereon the other insurance coverageobtained or subsequently effected on the samestocks in trade for the loss of which compensation isclaimed by petitioners. It is further admitted bypetitioners that Equitable's policy stated "nil" in thespace thereon requiring indication of any co-insurance although there were 3 policies subsistingon the same stocks in trade at the time of the loss,namely, that of Western in the amount ofP350,000.00 and two 2 policies of Reliance in thetotal amount of P1,000,000.00. - The coverage by other insurance or co-insuranceeffected or subsequently arranged by petitionerswere neither stated nor endorsed in the policies ofthe 3 private respondents, warranting forfeiture of all benefits thereunder if we are to follow the express stipulation in Policy Condition No. 3. - The terms of the contract are clear andunambiguous. The insured is specifically required todisclose to the insurer any other insurance and itsparticulars which he may have effected on the samesubject matter. The knowledge of such insurance bythe insurer's agents, even assuming the acquisitionthereof by the former, is not the "notice" that wouldstop the insurers from denying the claim. Besides,the socalled theory of imputed knowledge, that is,knowledge of the agent is knowledge of the principal,aside from being of dubious applicability here haslikewise been roundly refuted by respondent courtwhose factual findings we find acceptable. The merefact that Yap Kam Chuan was an agent for bothReliance and Equitable does not justify the allegationthat the two are sister companies. Availment of theservices of the same agents and adjusters bydifferent companies is a common practice in theinsurance business and such facts do not warrant thespeculative conclusion of the trial court. - Considering the terms of the policy which requiredthe insured to declare other insurances, thestatement in question must be deemed to be astatement (warranty) binding on both insurer andinsured, that there were no other insurance on theproperty. The annotation then, must be deemed tobe a warranty that the property was not insured byany other policy. Violation thereof entitled the insurerto rescind. - The obvious purpose of the aforesaid requirementin the policy is to prevent over-insurance and thusavert the perpetration of fraud. The public, as well asthe insurer, is interested in preventing the situationin which a fire would be profitable to the insured. Theinsured has no right to complain, for he assents tocomply with all the stipulations on his side, in orderto entitle himself to the benefit of the contract,which, upon reason or principle, he has no right toask the court to dispense with the performance of hisown part of the agreement, and yet to bind the otherparty to obligations, which, but for those stipulations,would not nave been entered into.

- It is not disputed that the insured failed to revealbefore the loss three other insurances. By reason ofsaid unrevealed insurances, the insured had beenguilty of a false declaration; a clearmisrepresentation and a vital one because where theinsured had been asked to reveal but did not, thatwas deception. Otherwise stated, had the insurerknown that there were many co-insurances, it could have hesitated or plainly desisted from entering intosuch contract. Hence, the insured was guilty of clearfraud. - As the insurance policy against fire expresslyrequired that notice should be given by the insuredof other insurance upon the same property, the totalabsence of such notice nullifies the policy. - Additionally, insofar as the liability of respondentReliance is concerned, it is not denied that thecomplaint for recovery was filed in court bypetitioners only on January 31, 1984, or after morethan one (1) year had elapsed from petitioners'receipt of the insurers' letter of denial on November29, 1982. - The condition contained in an insurance policy thatclaims must be presented within one year afterrejection is not merely a procedural requirement butan important matter essential to a prompt settlementof claims against insurance companies as it demandsthat insurance suits be brought by the insured whilethe evidence as to the origin and cause ofdestruction have not yet disappeared.

QUA CHEE GAN v. LAW UNION (supra p.48) YOUNG v. MIDLAND TEXTILE INSURANCE CO. 30 PHIL 617 JOHNSON; March 31, 1915 FACTS - K.S. Young had a candy and fruit store on theEscolta, Manila, and occupied a building at 321 CalleClaveria, as a residence and bodega.The MidlandTextile Insurance Co. in consideration of the paymentof a premium of P60, entered into a contract ofinsurance with Young by the terms of which thecompany, upon certain conditions, promised to payYoung the sum of P3,000 in case said residence andbodega and contents should be destroyed by fire. - One of the conditions of the contract is: "WarrantyB It is hereby declared and agreed that during thependency of this policy no hazardous goods bestored or kept for sale, and no hazardous trade orprocess be carried on, in the building to which thisinsurance applies, or in any building connectedtherewith." - Young placed in the residence and bodega three boxes filled with fireworks. Said residence andbodega and the contents thereof were partiallydestroyed by fire. - The fireworks had been given to Young by the former owner of the Luneta Candy Store. Heintended to use them in the celebration of theChinese New Year.However, the authorities of thecity of Manila had prohibited the use of fireworks onsaid occasion, so Young then placed them in thebodega where they remained from the 4th or 5th ofFebruary, 1913 until after the fire of March 18, 1913.- Both parties agree that the fireworks come withinthe phrase "hazardous goods," mentioned in"Warranty B" of the policy; that the fireworks werefound in a part of the building not destroyed by thefire and that they in no way contributed to the fire,or to the loss that resulted. - The lower court rendered a judgment in favor of Young for the sum ofP2,708.78, and costs. ISSUE 1. WON the placing of the fireworks in the buildinginsured, they being "hazardous goods," was aviolation of the terms of the contract of insuranceand especially of "Warranty B." HELD 1. YES. Reasoning It is admitted by both parties that the fireworks are hazardous goods. The defendantalleged that they were "stored." The plaintiffcontends that under all the facts and circumstancesof the case, they were not stored in said building,and that the placing of them in the building was nota violation of the terms of the contract. - Whether a particular article is "stored" or not must,in some degree, depend upon the intention of theparties. Nearly all of the cases cited by the lowercourt are cases where the article was being put tosome reasonable and actual use, which might easilyhave been permitted by the terms of the policy, andwithin the intention of the parties, and excepted fromthe operation of the warranty, like the present. - (1) Where merchants have had or kept the"hazardous" articles in small quantities, and foractual daily use, for sale, such as gasoline,gunpowder, etc.; (2) Where such articles have beenbrought on the premises for actual use thereon, andin small quantities, such as oil, paints, etc; and (3)Where such articles or goods were used for lightingpurposes, and in small quantities. - In the present case no claim is made that the"hazardous goods" were placed in the bodega forpresent or daily use. It is admitted that they wereplaced in the bodega "for future use," or for futureconsumption, or for safe keeping. It seems clear tous that the "hazardous goods" in question were"stored" in the bodega, as that word is generallydefined. That being true, suppose the defendant hadmade an examination of the premises, even in theabsence of a fire, and had found the "hazardousgoods" there, would it not have been justified indeclaring the policy null and of no effect by reason ofa violation of its terms? If it might, then may it notrepudiate its liability, even after the fire? If the"warranty" is a term of the contract, will not itsviolation cause a breach and justify noncomplianceor repudiation? - Contracts of insurance are contracts of indemnity,upon the terms and conditions specified therein.Parties have a right to impose such reasonableconditions at the time of the making of the contractas they deem wise and necessary. The rate ofpremium is measured by the character of the riskassumed. The insurer, for a comparatively smallconsideration, undertakes to guarantee the

insuredagainst loss or damage, upon the terms andconditions agreed upon, and upon no other. Whenthe insurer is called upon to pay, in case of loss, hemay justly insist upon a fulfillment of the terms ofthe contract. If the insured, cannot bring himselfwithin the terms and conditions of the contract, he isnot entitled to recover for any loss suffered. Theterms of the contract constitute the measure of theinsurer's liability. If the contract has beenterminated, by a violation of its terms on the part ofthe insured, there can be no recovery. Compliancewith the terms of the contract is a conditionprecedent to the right of recovery. - Young argues that since the "storing" of thefireworks on the premises did not contribute in anyway to the damage occasioned by the fire, he shouldbe permitted to recover. That argument, however, isbeside the question, if the "storing" was a violationof the terms of the contract. The violation of theterms of the contract, by virtue of the provisions ofthe policy itself, terminated, at the election of eitherparty, the contractual relations. - Young paid a premium based upon the risk at thetime the policy was issued.Certainly, the placing ofthe firecrackers in the building insured increased therisk. Young had not paid a premium based upon theincreased risk, neither had the defendant issued a policy upon the theory of a different risk. He wasenjoying, if his contention may be allowed, thebenefits of an insurance policy upon one risk,whereas, as a matter of fact, it was issued upon anentirely different risk. The defendant had neitherbeen paid nor had issued a policy to cover theincreased risk. An increase of risk which issubstantial and which is continued for a considerableperiod of time, is a direct and certain injury to theinsurer, and changes the basis upon which thecontract of insurance rests. Disposition Decision of the lower court is REVERSED.

TAN v. CA (supra p.51) AREOLA v. CA (supra p.26) TAN CHAY v. WEST COAST (supra p.51) FILIPINAS LIFE ASSURANCE v. NAVA 17 SCRA 210 BAUTISTA ANGELO; May 20, 1966 NATURE Petition for review of a decision of the Court of Appeals FACTS - Before the war, Nava entered into a contract ofinsurance with Insular Life Assurance Co., Ltd. (facevalue of P5k), and 17 separate contracts of lifeinsurance with Filipinas Life Assurance Co. (total facevalue of P90k). Each and everyone of the 18 policiesissued by defendants to plaintiff contains a loanclause of the following tenor: Policy loans. After three full years'premiums have been paid upon this Policy, if nopremium payment is in default, the Company,subject to its then existing rules, will advance onproper assignment and delivery of this Policy andon the sole security thereof a sum equal to, or atthe option of the owner less than, the cash valuespecified in the Schedule of Policy Values, less anyexisting indebtedness on or secured by this Policyand any unpaid balance of the premium for thecurrent policy-year; provided interest at six percentum per annum on the whole amount of theloan is paid in advance to the end of the currentpolicy-year. At the end of the current policy-yearinterest at the same rate for one year in advancewill be due and payable, and annually thereafter, and if not so paid will be added to the principaland bear the same rate of interest. Failure torepay any such loan or interest shall not avoid thisPolicy unless the total indebtedness shall equal orexceed the full amount of the loan value availablehereunder. Any indebtedness on this Policy shall first bededucted from any money payable or in anysettlement under this Policy. - Nava had so far paid to Insular a total of P2,574; and to Filipinas Life, a total of P32,072.60. - April 28, 1948: Nava applied to the companies for aP5k loan in line with the loan clause, but theyrefused to grant it because certain regulations issuedby the Insurance Commissioner required theinsurance companies to withhold the payments onpremiums made during the Japanese occupationbecause the same shall be subject to futureadjustments " as soon as debtor-creditor relationshipis established" and because of such process of"withholding" plaintiff was not entitled to borrow anyamount until such adjustment has been made. - Sept 30, 1948: Nava called the attention of the insurance companies to the SC decision (Haw Pia v. China Banking Corporation) establishing and recognizing the relationship of debtor and creditorwith respect to payments in fiat currency madeduring the Japanese occupation on pre-warobligations. - Companies still refused saying that the SC decisionwas not applicable to transactions undertaken duringJapanese occupation when they relate to lifeinsurance policies. - Feb 4, 1949: Nava was again refused even if thetotal amount of the cash surrender values of the 18policies reached the sum of P9,468.29. - Feb 10, 1949: Nava brought case to the CFI Manilapraying for the rescission of the abovementioned 18policies and for the refund to him of all the premiumsso far paid by him to defendants in the amount ofP31,633.80, plus 6% interest thereon as damages - Nov 28, 1951: companies passed a resolutionwhich was approved by the Insurance Commissioner,giving full credit to all premium payments made bytheir policyholders in fiat currency during theJapanese occupation on account of pre-war policiesfor which reason they filed an amended answeroffering to pay plaintiff the amount of P9,468.29which represents the aggregate cash surrendervalues of all the policies in question as of February10, 1949, but apparently this offer was refused. - CFI:(1) rescinded the insurance contracts; (2)ordered defendant Filipinas Life Assurance Co. to payplaintiff the amount of P32,072.60; and (3) ordereddefendant Insular Life Assurance Co., Ltd. to payplaintiff the amount of P2,574.00

- CA affirmed. ISSUES 1. WON CA erred in ruling that as a consequence ofthe decision in the Haw Pia case petitioners violatedthe loan clause contained in the insurance policiesthereby entitling respondent to their rescission 2. WON CA erred in ruling that by virtue of Article1295 of the old Civil Code petitioners should refundto defendant all the premiums paid on his insurancepolicies as a consequence of their rescission 3. WON CA erred in not ruling that, even ifrespondent is entitled to the rescission of saidinsurance policies, he can only recover their cashsurrender value at the time the complaint was filed HELD 1. NO. - Even assuming the validity of the InsuranceCommissioners regulations, the fact however is thatsuch requirement has already lost its legal effect andvalue when our Supreme Court rendered its decisionin the Haw Pia case wherein it was declared, amongothers, that all payments made in fiat currencyduring the Japanese occupation in relation with anycontractual obligation executed before the war werevalid to all intents and purposes, and yet petitionersapparently did not give any importance to suchdecision for in their opinion it does not have anyapplication to transactions which have any relation topayment of premiums on life insurance policies. - It cannot be denied that a life insurance policyinvolves a contractual obligation wherein the insuredbecomes duty bound to pay the premiums agreedupon, lest he runs the risk of having his insurancepolicy lapse if he fails to pay such premiums. - The fact that if the insured had paid in full thepremiums corresponding to the first 3 years of thelife of his policy he cannot be considered delinquentthat would cause the lapse of his policy if the samecontains an automatic premium payment clausecannot divest such policy of its contractual nature,for the result of such failure would only be for him topay later the premium plus the correspondinginterest depending upon the condition of the policy.But certainly it does not cease to be a contractualliability insofar as the payment of that premium is concerned for whether he likes it or not thatpremium has to be paid lest he allows the lapse ofhis policy. Consequently, the payment of premiumson the life insurance policies made by Nava beforeand during the war up to the time he applied for theloan in question with petitioners should beconsidered likewise as valid payments upon thetheory that such insurance policies are in the natureof a contractual obligation within the meaning of thecivil law. In effect, therefore, those payments weremade by a debtor to a creditor within the meaning ofthe requirement of the regulations of the InsuranceCommissioner and as such they can offer no excuseto petitioners for refusing to grant the loan ascontemplated in the loan clause embodied in thepolicies in question. - It is clear from the foregoing that the petitionersviolated the loan clause embodied in each of the 18life insurance policies issued to respondent to rescindall said policies under Section 69 of the InsuranceAct, which provides: "The violation of a materialwarranty, or other material provision of a policy, onthe part of either party thereto, entitles the other torescind." - "The general rule is that a breach of the agreementto make the loan does not entitle the insured torescind the contract," is not controlling in thisjurisdiction. Firstly, it was not shown that theinsurance laws in the states where said rulingprevails contain a provision identical to Section 69 ofour Insurance Law we quoted above, and secondly,the rule cited by Vance is not a rule uniformlyfollowed by all states in the US, for on this matterthere is a marked divergence of opinion. 2. NO - Considering that our Insurance Law does notcontain an express provision as to what the courtshould do in cases of rescission of an insurancepolicy under Section 69, the provision that shouldapply is that embodied in Article 1225 of the old CivilCode, as postulated in Article 16 of the same Code,which provides that on matters which are notgoverned by special laws the provisions of said Codeshall supplement its deficiency. And said Article 1295provides: ART. 1295. Rescission makes necessary the returnof the things which were the subject-matter of thecontract, with their fruits, and of the price paid,with interest thereon. ...xxx - Said the petitioners: "Recovery of the full amountof the premium after the insurer has sustained forsometime the risk of the insurance and the insured has enjoyed the benefit of protection is obviouslyunjust and is so recognized by the betterauthorities." The ruling above quoted merelyrepresents the minority rule in the US, the majorityrule being that the insured can recover all premiumspaid, in some cases with interest in case of wrongfulcancellation, repudiation, termination or rescission ofthe contract of life insurance.

- Contention that because respondent cannot restoreto petitioners the "value of the benefit of protection"which he might have received under the 18 lifeinsurance policies in question he is not entitled torescind them under the provision of Article 1295 ofthe old Civil Code, is untenable because said articleonly contemplates a transaction whether materialthings are involved, and do not refer to intangibleones which cannot be the subject of restoration, forto interpret it otherwise would be to defeat the lawitself with the result that rescission can never be hadunder Section 69 of our Insurance Law. - It cannot be denied that petitioners had in turnalready derived material benefits from the use ofpremiums paid to them by respondent before, duringand after the last war from which they must haverealized huge profits, and in this light alonepetitioners cannot claim prejudice or unfairness ifthey are ordered to refund the premiums paid byrespondents. 3. NO. - Issue is corollary to preceding issue. No need to refute. Disposition Decision appealed from is AFFIRMED. Costs against petitioners

CHAPTER VII. RISKS AND COVERAGES VDA. DE BATACLAN v. MEDINA 102 PHIL 181 MONTEMAYOR; October 22, 1957 FACTS -Juan Bataclan rode Bus 30 of Medina Transport,driven by Saylon, shortly after midnight.The buswas running very fast.One of the front tires burst.Bus fell into canal and turned turtle.4 passengerscouldnt get out, including Bataclan.Gasoline beganto leak from the overturned bus.10 men came tohelp.1 carried a torch and when he approached thebus, fire started, killing the trapped passengers. -TC opined that proximate cause of Bataclansdeath was not the overturning of bus but the fire.Atthe time fire started, Bataclan, though injured, was still alive and damages were awarded, not for his death, but for physical injuries suffered. ISSUE/S WON the proximate cause is the overturning of the bus or the fire HELD -The proximate cause is the overturning of the bus.-Ordinarily, when a bus overturns and pins downpassenger, merely causing him injuries.If throughsome event, unexpected and extraordinary, the busis set on fire, and passenger is burned to death, onemight contend that the proximate cause was the fireand not the overturning of the vehicle. -But here, the proximate cause of Bataclans deathis the overturning of the bus, this for the reason thatwhen the vehicle turned not only on its side butcompletely on its back, leaking of gasoline from thetank was not unnatural or unexpected. -The coming of the men with the torch was inresponse to the call for help, made only not by thepassengers but even the driver and conductor, andbecause it was very dark, about 2:30 am, rescuershad to carry a light with them.Coming as they didfrom rural area where lanterns and flashlights werenot available, they had to use a torch.What wasmore natural than that said rescuers shouldinnocently approach the overturned vehicle to extendaid. -The coming of the men with the torch was to beexpected, and was a natural sequence of theoverturning of the bus, the trapping of some of itspassengers and the call for outside help. -The burning of bus can also in part be attributed tonegligence of carrier, through its driver andconductor.They, or at least the driver, should haveknown that in the position in which the overturnedbus was, gasoline could and must have leaked fromthe gasoline tank and soaked the area in and aroundthe bus.Gasoline can be smelt and detected evenfrom a distance, and yet neither the driver nor theconductor would appear to have cautioned or takensteps to warn rescuers not to bring the lighted torchtoo near the bus.

FINMAN GENERAL ASSURANCE CORPORATION v. CA (SURPOSA) 213 SCRA 493 NOCON; September 2, 1992 NATURE Certiorari FACTS - Oct. 22, 1986: Carlie Surposa was insured withFinman General Assurance Corporation under FinmanGeneral Teachers Protection Plan Master Policy No.2005 and Individual Policy No. 08924 with hisparents, spouses Julia and Carlos Surposa, andbrothers Christopher, Charles, Chester and Clifton, allsurnamed, Surposa, as beneficiaries. - While said insurance policy was in full force andeffect, the insured, Carlie Surposa, died on October18, 1988 as a result of a stab wound inflicted by oneof the 3 unidentified men without provocation andwarning on the part of the former as he and hiscousin, Winston Surposa, were waiting for a ride ontheir way home after attending the celebration of the"Maskarra Annual Festival." - Thereafter, Julia Surposa and the otherbeneficiaries of said insurance policy filed a writtennotice of claim with the FINMAN Corp which deniedsaid claim contending that murder and assault arenot within the scope of the coverage of the insurancepolicy. - Feb. 24, 1989: Surposa filed a complaint with theInsurance Commission which subsequently orderedFINMAN to pay Surposa the proceeds of the policywith interest. - CA affirmed said decision. ISSUE WON CA committed GAD in applying the principle of"expresso unius exclusio alterius" in a personalaccident insurance policy (since death resulting frommurder and/or assault are impliedly excluded in saidinsurance policy considering that the cause of deathof the insured was not accidental but rather adeliberate and intentional act of the assailant inkilling the former as indicated by the location of thelone stab wound on the insured) [TF they cannot bemade to indemnify the Surposa heirs] HELD NO - The record is barren of any circumstance showinghow the stab wound was inflicted. While the act maynot exempt the unknown perpetrator from criminalliability, the fact remains that the happening was apure accident on the part of the victim. The insureddied from an event that took place without hisforesight or expectation, an event that proceeded from an unusual effect of a known cause and, therefore, not expected. Reasoning - De la Cruz vs. Capital Insurance & Surety Co., Inc (1966)~ The terms "accident" and "accidental" as used in insurance contracts have not acquired any technical meaning, and are construed by the courtsin their ordinary and common acceptation. Thus, theterms have been taken to mean that which happenby chance or fortuitously, without intention anddesign, and which is unexpected, unusual, andunforeseen. An accident is an event that takes placewithout one's foresight or expectationan event thatproceeds from an unknown cause, or is an unusualeffect of a known cause and, therefore, not expected. Ratio The generally accepted rule is that, death or injury does not result from accident or accidentalmeans within the terms of an accident-policy if it isthe natural result of the insured's voluntary act,unaccompanied by anything unforeseen except thedeath or injury. There is no accident when adeliberate act is performed unless some additional,unexpected, independent, and unforeseen happeningoccurs which produces or brings about the result ofinjury or death.In other words, where the death orinjury is not the natural or probable result of theinsured's voluntary act, or if something unforeseenoccurs in the doing of the act which produces theinjury, the resulting death is within the protection ofthe policies insuring against death or injury fromaccident. - The personal accident insurance policy involvedherein specifically enumerated only 10 circumstanceswherein no liability attaches to FINMAN for anyinjury, disability or loss suffered by the insured as aresult of any of the stimulated causes. -The principle of " expresso unius exclusio alterius"the mention of one thing implies the exclusion ofanother thing is therefore applicable in the instantcase since murder and assault, not having

beenexpressly included in the enumeration of thecircumstances that would negate liability in saidinsurance policy: the failure of the FINMAN to include death resulting from murder or assault among theprohibited risks leads inevitably to the conclusionthat it did not intend to limit or exempt itself fromliability for such death. - A1377 NCC: The interpretation of obscure wordsor stipulations in a contract shall not favor the partywho caused the obscurity. - NPC vs. CA [1986]~ It is well settled thatcontracts of insurance are to be construed liberally infavor of the insured and strictly against the insurer.Thus ambiguity in the words of an insurance contractshould be interpreted in favor of its beneficiary. Disposition DENIED for lack of merit. CALANOC v. CA 98 PHIL 79 BAUTISTA; December 16, 1955 FACTS - Basilio was a watchman of the Manila Auto Supplylocated at the corner of Avenida Rizal and Zurbaran.He secured a life insurance policy from the PhilippineAmerican Life Insurance Company in the amount ofP2,000 to which was attached a supplementarycontract covering death by accident. On January 25,1951, he died of a gunshot wound on the occasion ofa robbery committed in the house of Atty. Ojeda atthe corner of Oroquieta and Zurbaran streets.Calanoc, the widow, was paid the sum of P2,000,face value of the policy, but when she demanded thepayment of the additional sum of P2,000representing the value of the supplemental policy,the company refused alleging, as main defense, thatthe deceased died because he was murdered by aperson who took part in the commission of therobbery and while making an arrest as an officer ofthe law which contingencies were expressly excludedin the contract and have the effect of exempting thecompany from liability. - It is contended in behalf of the company thatBasilio was killed which "making an arrest as anofficer of the law" or as a result of an "assault ormurder" committed in the place and therefore hisdeath was caused by one of the risks excluded by thesupplementary contract which exempts the companyfrom liability. This contention was upheld by theCourt of Appeals. Hence, this petition. ISSUE WON the death of the victim comes within thepurview of the exception clause of the supplementarypolicy and, hence, exempts the company from liability HELD NO - Basilio was a watchman of the Manila Auto Supplywhich was a block away from the house of Atty.Ojeda where something suspicious was happening which caused the latter to ask for help. While at firsthe declined the invitation of Atty. Ojeda to go withhim to his residence to inquire into what was goingon because he was not a regular policeman, he lateragreed to come along when prompted by the trafficpoliceman, and upon approaching the gate of theresidence he was shot and died. The circumstancethat he was a mere watchman and had no duty toheed the call of Atty. Ojeda should not be taken as acapricious desire on his part to expose his life todanger considering the fact that the place he was induty-bound to guard was only a block away. Involunteering to extend help under the situation, hemight have thought, rightly or wrongly, that to knowthe truth was in the interest of his employer it beinga matter that affects the security of theneighborhood. No doubt there was some risk comingto him in pursuing that errand, but that risk alwaysexisted it being inherent in the position he washolding. He cannot therefore be blamed solely fordoing what he believed was in keeping with his dutyas a watchman and as a citizen. And he cannot beconsidered as making an arrest as an officer of thelaw, as contended, simply because he went with thetraffic policeman, for certainly he did not go there forthat purpose nor was he asked to do so by thepoliceman. - Much less can it be pretended that Basilio died inthe course of an assault or murder considering thevery nature of these crimes. In the first place, thereis no proof that the death of Basilio is the result ofeither crime for the record is barren of anycircumstance showing how the fatal shot was fired.Perhaps this may be clarified in the criminal casenow pending in court as regards the incident butbefore that is done anything that might be said onthe point would be a mere conjecture. Nor can it besaid that the killing was intentional for there is thepossibility that the malefactor had fired the shotmerely to scare away the people around for his ownprotection and not necessarily to kill or hit the victim.In any event, while the act may not exempt thetriggerman from liability for the damage done, thefact remains that the happening was a pure accidenton the part of the victim. The victim could have beeneither the policeman or Atty. Ojeda for it cannot bepretended that the malefactor aimed at the deceasedprecisely because he wanted to take his life.

Disposition Decision set aside

BIAGTAN v. THE INSULAR LIFE ASSURANCE COMPANY, LTD. 44 SCRA 58 MAKALINTAL; March 29, 1972 NATURE Appeal from decision of CFI Pangasinan. FACTS -Juan Biagtan was insured with Insular for P5k anda supplementary contract Accidental Death Benefitclause for another P5kif "the death of the Insuredresulted directly from bodily injury effected solelythrough external and violent means sustained in anaccident . . . and independently of all other causes."The clause, however, expressly provided that it wouldnot apply where death resulted from an injury"intentionally inflicted by a third party." -One night, a band of robbers entered their house.Juan went out of his room and he was met with 9knife stabs. He died. The robbers were convicted ofrobbery with homicide. -The family was claiming the additional P5k fromInsular, under the Accidental Death Benefit clause.Insular refused on the ground that the death resultedfrom injuries intentionally inflicted by 3rd parties andwas therefore not covered. Biagtans filed againstInsular. CFI ruled in favor of Biagtans. ISSUE WON the injuries were intentionally inflicted HELD YES - Whether the robbers had the intent to kill or merelyto scare the victim or to ward off any defense hemight offer, it cannot be denied that the act itself ofinflicting the injuries was intentional. -The exception in the accidental benefit clauseinvoked by the appellant does not speak of thepurpose whether homicidal or not of a thirdparty in causing the injuries, but only of the fact thatsuch injuries have been "intentionally" inflicted this obviously to distinguish them from injurieswhich, although received at the hands of a thirdparty, are purely accidental. -Examples of unintentional: >>A gun which discharges while being cleaned and kills a bystander; >>a hunter who shoots at his prey and hits a person instead; >>an athlete in a competitive game involvingphysical effort who collides with an opponent andfatally injures him as a result. -In Calanoc vs. CA: Where a shot was fired and itturned out afterwards that the watchman was hit inthe abdomen, the wound causing his death, theCourt held that it could not be said that the killingwas intentional for there was the possibility that themalefactor had fired the shot to scare the peoplearound for his own protection and not necessarily tokill or hit the victim. A similar possibility is clearlyruled out by the facts in this case. For while a singleshot fired from a distance, and by a person who wasnot even seen aiming at the victim, could indeedhave been fired without intent to kill or injure, ninewounds inflicted with bladed weapons at close rangecannot conceivably be considered as innocent insofaras such intent is concerned. - In Hucthcraft's Ex'r vs. Travelers' Ins. Co. (UScase): where the insured was waylaid andassassinated for the purpose of robbery, the courtrendered judgment for the insurance company andheld that while the assassination of the insured wasas to him an unforeseen event and thereforeaccidental, "the clause of the proviso that excludesthe (insurer's) liability, in case death or injury isintentionally inflicted by any other person, applies tothis case." Disposition CFI decision reversed. SEPARATE OPINION TEEHANKEE [dissent] -Calanoc v. CA is controlling in this case becausethe insurance company wasnt able to prove that thekilling was intentional. (Burden of proof is with theinsurance company) -Insurance, being contracts of adhesion, must beconstrued strictly against insurance company incases of ambiguity. The supplementary contract

enumeratedexceptions. The only exception which is notsusceptible of classification is that provided inparagraph 5(e), the very exception herein involved,which would also except injuries "inflictedintentionally by a third party, either with or withoutprovocation on the part of the insured, and whetheror not the attack or the defense by the third partywas caused by a violation of the law by the insured."- This ambiguous clause conflicts with all the otherfour exceptions in the same paragraph 5 particularlythat immediately preceding it in item (d) whichexcepts injuries received where the insured hasviolated the law or provoked the injury, while thisclause, construed as the insurance company now claims, would seemingly except also all otherinjuries, intentionally inflicted by a third party,regardless of any violation of law or provocation bythe insured, and defeat the very purpose of thepolicy of giving the insured double indemnity in caseof accidental death by "external and violent means" in the very language of the policy.' - It is obvious from the very classification of theexceptions and applying the rule of noscitus a sociis,that the double-indemnity policy covers the insuredagainst accidental death, whether caused by fault,negligence or intent of a third party which isunforeseen and unexpected by the insured. All theassociated words and concepts in the policy plainlyexclude the accidental death from the coverage ofthe policy only where the injuries are self-inflicted orattended by some proscribed act of the insured orare incurred in some expressly excluded calamitysuch as riot, war or atomic explosion. - The untenability of insurer's claim that theinsured's death fell within the exception is furtherheightened by the stipulated fact that two otherinsurance companies which likewise covered theinsured for much larger sums under similaraccidental death benefit clauses promptly paid thebenefits thereof to plaintiffs beneficiaries.

SUN INSURANCE v. CA (LIM) 211 SCRA 554 CRUZ; July 17, 1992 FACTS -The petitioner issued Personal Accident Policy toFelix Lim, Jr. with a face value of P200,000.00. Twomonths later, he was dead with a bullet wound in hishead. As beneficiary, his wife Nerissa Lim soughtpayment on the policy but her claim was rejected.The petitioner agreed that there was no suicide. Itargued, however, that there was no accident either. Pilar Nalagon, Lim's secretary, was the onlyeyewitness to his death. According to Nalagon, Limwas in a happy mood (but not drunk) and wasplaying with his handgun, from which he hadpreviously removed the magazine. As she watchedthe television, he stood in front of her and pointedthe gun at her. She pushed it aside and said it mightbe loaded. He assured her it was not and thenpointed it to his temple. The next moment there wasan explosion and Lim slumped to the floor. He wasdead before he fell. - The term "accident" has been defined as follows: The words "accident" and "accidental" have neveracquired any technical signification in law, andwhen used in an insurance contract are to beconstrued and considered according to theordinary understanding and common usage andspeech of people generally. In substance, thecourts are practically agreed that the words"accident" and "accidental" mean that whichhappens by change or fortuitously, withoutintention or design, and which is unexpected,unusual, and unforeseen. The definition that hasusually been adopted by the courts is that anaccident is an event that takes place without one'sforesight or expectation an event that proceedsfrom an unknown cause, or is an unusual effect ofa known case, and therefore not expected. - An accident is an event which happens without anyhuman agency or, if happening through humanagency, an event which, under the circumstances, isunusual to and not expected by the person to whomit happens. It has also been defined as an injurywhich happens by reason of some violence orcasualty to the insured without his design, consent,or voluntary co-operation. ISSUE WON what happened was an accident HELD YES - The petitioner, however, cites one of the fourexceptions provided for in the insurance contract andcontends that the private petitioner's claim is barredby such provision. It is there stated: Exceptions The company shall not be liable in respect of. 1. Bodily injury. xxx xxx xxx b. consequent upon. i) The insured persons attempting to commitsuicide or wilfully exposing himself to needless perilexcept in an attempt to save human life. - To repeat, the parties agree that Lim did notcommit suicide. Nevertheless, the petitionercontends that the insured willfully exposed himself toneedless peril and thus removed himself from thecoverage of the insurance policy. That posture isarguable. But what is not is that, as the secretarytestified, Lim had removed the magazine from thegun and believed it was no longer dangerous. Heexpressed assured her that the gun was not loaded.It is submitted that Lim did not willfully expose himself to needless peril when he pointed the gun tohis temple because the fact is that he thought it wasnot unsafe to do so. The act was precisely intendedto assure Nalagon that the gun was indeed harmless. Disposition CA Affirmed

DE LA CRUZ v. CAPITAL INSURANCE 17 SCRA 554 BARRERA; June 30, 1966 NATURE Appeal from the decision of the CFI of Pangasinan FACTS - Eduardo de la Cruz, employed in the Itogon-SuyocMines, Inc., was the holder of an accident insurancepolicy underwritten by the Capital Insurance &Surety Co., Inc., for the period beginning November13, 1956 to November 12, 1957. - On January 1, 1957, the Itogon-Suyoc Mines, Inc.sponsored a boxing contest wherein the insuredEduardo de la Cruz participated. - In the course of his bout, Eduardo slipped and washit by his opponent on the left part of the back of thehead, causing Eduardo to fall, with his head hittingthe rope of the ring. - He was brought to the Baguio General Hospital, but he died as a result of hemorrhage, intracranial, left. - Simon de la Cruz, the father and named beneficiaryof the insured, filed a claim with the insurancecompany for payment of the indemnity, but it wasdenied. - He instituted the action in the CFI of Pangasinan for specific performance. - Defendant insurer set up the defense that thedeath of the insured, caused by his participation in aboxing contest, was not accidental and, therefore,not covered by insurance - The court rendered the decision in favor of the plaintiff, hence, the present appeal. ISSUE WON the death of the insured was not accidental and, therefore, not covered by insurance HELD NO - The terms "accident" and "accidental", as used ininsurance contracts, have not acquired any technicalmeaning, and are construed by the courts in theirordinary and common acceptation. Thus, the termshave been taken to mean that which happen bychance or fortuitously, without intention and design, and which is unexpected, unusual, and unforeseen.An accident is an event that takes place withoutone's foresight or expectation, an event thatproceeds from an unknown cause, or is an unusualeffect of a known cause and, therefore, not expected. - The generally accepted rule is that, death or injurydoes not result from accident or accidental meanswithin the terms of an accident-policy if it is thenatural result of the insured's voluntary act,unaccompanied by anything unforeseen except thedeath or injury. There is no accident when adeliberate act is performed unless some additional,unexpected, independent, and unforeseen happeningoccurs which produces or brings about the result ofinjury or death. In other words, where the death orinjury is not the natural or probable result of theinsured's voluntary act, or if something unforeseenoccurs in the doing of the act which produces theinjury, the resulting death is within the protection ofpolicies insuring against death or injury from accident. - In the present case, while the participation of theinsured in the boxing contest is voluntary, the injurywas sustained when he slid, giving occasion to theinfliction by his opponent of the blow that threw himto the ropes of the ring. - The fact that boxing is attended with some risks ofexternal injuries does not make any injuries receivedin the course of the game not accidental. In boxingas in other equally physically rigorous sports, such asbasketball or baseball, death is not ordinarilyanticipated to result. If, therefore, it ever does, theinjury or death can only be accidental or produced bysome unforeseen happening or event as whatoccurred in this case. - Furthermore, the policy involved herein specifically excluded from its coverage: (e) Death or disablement consequent upon theInsured engaging in football, hunting, pigsticking,steeplechasing, polo-playing, racing of any kind,mountaineering, or motorcycling. - Death or disablement resulting from engagement inboxing contests was not declared outside of theprotection of the insurance contract. Failure of thedefendant insurance company to include deathresulting from a boxing match or other sports amongthe prohibitive risks leads inevitably to the conclusionthat it did not intend to limit or exempt itself fromliability for such death. Disposition The decision appealed from is affirmed FORTUNE INSURANCE v. CA (supra p.7)

PHIL HOME ASSURANCE CORP v. CA (EASTERN SHIPPING) 257 SCRA 468 KAPUNAN; June 20, 1996 NATURE - Eastern Shipping Lines, Inc. loaded on board SSEastern Explorer in Kobe, Japan, the followingshipment for carriage to Manila and Cebu, freightpre-paid and in good order and condition:(a) 2boxes internal combustion engine parts, consigned toWilliam Lines, Inc.; (b) 10 metric tons (334 bags)ammonium chloride, consigned to Orca's Company;(c) 200 bags Glue 300, consigned to Pan OrientalMatch Company; and (d) garments, consigned toDing Velayo. All consignations were made by virtueof a Bill of Lading. - While the vessel was off Okinawa, a small flamewas detected on the acetylene cylinder located in theaccommodation area near the engine room.As thecrew was trying to extinguish the fire, the cylindersuddenly exploded, thus causing death and severeinjuries to the crew and instantly setting fire to thewhole vessel. - SS Eastern Explorer was then found to be aconstructive total loss and its voyage was declaredabandoned. - After the fire was extinguished, the cargoes whichwere saved were loaded to another vessel fordelivery to their original ports of destination. ESLIcharged the consignees several amountscorresponding to additional freight and salvagecharges. - The charges were all paid by Philippine Home Assurance Corporation (PHAC) under protest for and in behalf of the consignees. PHAC, as subrogee of the consignees, then filed a complaint before the RTCof Manila, against ESLI to recover the sum paidunder protest on the ground that the same wereactually damages directly brought about by the fault,negligence, illegal act and/or breach of contract of ESLI. - ESLI contended that it exercised the diligencerequired by law in the handling, custody and carriageof the shipment; that the fire was caused by anunforeseen event; that the additional freight chargesare due and demandable pursuant to the Bill ofLading; and that salvage charges are properlycollectible under Act No. 2616, known as the SalvageLaw. - RTC: dismissed PHAC's complaint and ruled in favor of ESLI. - The burning of the vessel was not the fault ornegligence of defendant but a natural disaster orcalamity. Salvage operations conducted by FukudaSalvage Company was perfectly a legal operation andcharges made on the goods recovered werelegitimate charges. Section 19 of Act No. 2616, theSalvage Law is applicable. With respect to theadditional freight charged by defendant from theconsignees of the goods, the same are also validlydemandable. - The burning of "EASTERN EXPLORER" while offOkinawa rendered it physically impossible fordefendant to comply with its obligation of deliveringthe goods to their port of destination pursuant to thecontract of carriage.Under Article 1266 of the CivilCode, the physical impossibility of the prestationextinguished defendant's obligation. - Note: The goods subject of the present controversy were neither lost nor damaged in transitby the fire that razed the carrier.In fact, these wereall delivered to the consignees, even if thetransshipment took longer than necessary.What isat issue therefore isnot whether or not the carrier isliable for the loss, damage, or deterioration of thegoods transported by them but who, among thecarrier, consignee or insurer of the goods, is liable forthe additional charges or expenses incurred by theowner of the ship in the salvage operations and inthe transshipment of the goodsvia a different carrier.In absolving respondent carrier of any liability, CAsustained the trial court's finding that the fire thatgutted the ship was a natural disaster or calamity. ISSUE WON the burning of the SS Eastern Explorerrendering it a constructive total loss was a naturaldisaster or calamity HELD NO Ratio In our jurisprudence, fire may not be considered a natural disaster or calamity since italmost always arises from some act of man or byhuman means.It cannot be an act of God unless

9 Section 1.When in case of shipwreck, the vessel or its cargo shall be beyond the control of the crew, or shall have been abandoned bythem, and picked up and conveyed to a safe place by other persons,the latter shall be entitled to a reward for the salvage. Those who, not being included in the above paragraph, assist in saving a vessel or its cargo from shipwreck, shall be entitled to like reward. caused by lightning or a natural disaster or casualty not attributable to human agency. Reasoning - There was no showing, and none was alleged by the parties, that the fire was caused by a naturaldisaster. On the contrary, there is strong evidenceindicating that the acetylene cylinder caught firebecause of the fault and negligence of respondentESLI, its captain and its crew: (1) The acetylene cylinder which was fully loadedshould not have been stored near the engine roomwhere the heat generated therefrom could cause theacetylene cylinder to explode by reason ofspontaneous combustion.ESLI should have easilyforeseen that the acetylene cylinder, containinghighly inflammable material, was in a real danger ofexploding. (2) ESLI should have known that by storing theacetylene cylinder in the accommodation areasupposed to be reserved for passengers, itunnecessarily exposed its passengers to gravedanger and injury. (3) The fact that the acetylene cylinder was checked,tested and examined and subsequently certified ashaving complied with the safety measures andstandards by qualified experts before it was loaded inthe vessel only shows to a great extent thatnegligence was present in the handling of theacetylene cylinderafter it was loaded and while itwas on board the ship. - From the foregoing premises, it indubitably followsthat the cargo consignees cannot be made liable torespondent carrier for additional freight and salvagecharges. Disposition Judgment appealed from is REVERSED and SET ASIDE.Respondent Eastern Shipping Lines,Inc. is ORDERED to return to petitioner PhilippineHome Assurance Corporation the amount it paidunder protest in behalf of the consignees herein.

PHILIPPINE AMERICAN LIFE INSURANCE COMPANY v. THE AUDITOR GENERAL 22 SCRA 135 SANCHEZ, JANUARY 18, 1968 NATURE PETITION FOR REVIEW of a ruling of the Auditor General. FACTS - Philamlife, a domestic life insurance corp., and American International Reinsurance Company(Airco), a corporation organized under the laws ofthe Republic of Panama, entered into aREINSURANCE TREATY wherein Philamlife agrees toreinsure with Airco on January 1950. Philamlifeagreed to pay premiums for all reinsurances on anannual premium basis. - In July 16, 1959, the Margin Law was approved andbecame effective, which exempts certain obligationsfrom payment of margin fees, particularly contractualobligations calling for payment of foreign exchangeissued, approved and outstanding as of the date thisAct takes place. - Central Bank of the Philippines collected P268,747.48 as foreign exchange margin on Philamlife remittances to Airco purportedly totalling$610,998.63 and made subsequent to July 16, 1959.Philamlife filed a claim for refund on the ground thatthe reinsurance premiums remitted were paid inpursuant to the January 1950 reinsurance treaty, andtherefore exempted. - Monetary Board exempted Philamlife from paymentof margin fee. However, Auditor of CB refused topass in audit Philamlifes claim for refund. Philamlifesought reconsideration but was denied, sayingreinsurance treaty NOT EXEMPTED. ISSUES 1. WON the premia remitted were in pursuance ofthe reinsurance treaty between Philamlife and Aircoof January 1959, a contract antedating the MarginLaw, and therefore, Philamlife exempted from payingmargin fee 2. WON Margin Law impairs the obligation of contract3. WON reinsurance contracts abroad would be madeimpractical by the imposition of the 25% margin fee HELD 1. NO - For an exemption to come into play, there must bea reinsurance policy or, as in the reinsurance treatyprovided, a "reinsurance cession" which may beautomatic or facultative. RatioA reinsurance policy is thus a contract of indemnity one insurer makes with another to protect the first insurer from a risk it has already assumed. . .. In contradistinction, a reinsurance treaty ismerely an agreement between two insurancecompanies whereby one agrees to cede and theother to accept reinsurance business pursuant toprovisions specified in the treaty. The practice ofissuing policies by insurance companies includes,among other things, the issuance of reinsurancepolicies on standard risks and also on substandardrisks under special arrangements. The lumping of thedifferent agreements under a contract has resulted inthe term known to the insurance world as 'treaties.'Such a treaty is, in fact, an agreement betweeninsurance companies to cover the different situationsdescribed. Reinsurance treaties and reinsurancepolicies are not synonymous. Treaties are contracts for insurance; reinsurance policies or cessions are contracts of insurance. Reasoning - Even if reinsurance treaty preceded the Margin Law by over nine years, nothing in the treaty obligatesPhilamlife to remit to Airco a fixed, certain, andobligatory sum by way of reinsurance premiums. Thereinsurance treaty per se cannot give rise to acontractual obligation for the payment of foreignexchange. Philamlifes obligation to remit reinsurancepremiums becomes fixed and definite upon the execution of the reinsurance cession. It is only

after a reinsurance cession is made that payment ofreinsurance premium may be exacted, as it is onlyafter Philamlife seeks to remit that reinsurancepremium that the obligation to pay the margin feearises. 2. NO Ratio. Existing laws form part of the contract "as the measure of the obligation to perform them by the one party and the right acquired by the other. If the obligation does not inhereand subsist in thecontract itself, propio vigore, but in the lawapplicable to the contract. Reasoning - . When petitioner entered into the reinsurance treaty of January 1, 1950 with Airco, it did so withthe understanding that the municipal laws of thePhilippines at the time said treaty was executed,became an unwritten condition thereof. Suchmunicipal laws constitute part of the obligation ofcontract. -Rationale of Margin Law: to reduce the excessive demand on and prevent further decline of our international reserves; to provide the Central Bankwith an additional instrument for effectively copingwith the problem and achieving domestic andinternational stability of our currency; to reduce theexcessive demand-for foreign exchange. - implementation of Margin Law in accordance with police power 3. NO Reasoning - First, there is no concrete evidence that such imposition of the 25% margin fee is unreasonable,Second, if really continuance of the existingreinsurance treaty becomes unbearable, that contract itself provides that petitioner maypotestatively write finis thereto on ninety days'written notice. Petitioner is not forced to continue its reinsurance treaty indefinitely with Airco. Disposition For the reasons given, the petition for review is hereby denied, and the ruling of the AuditorGeneral of October 24, 1961 denying refund ishereby affirmed. Costs against petitioner. So ordered.

FIELDMEN'S INSURANCE CO INC v. ASIAN SURETY & INSURANCE CO INC 34 SCRA 36 MAKALINTAL; July 31, 1970 FACTS - On various datesbetween April 11, 1960 and Jan.9, 1961the Asian Surety & Insurance Company, Inc.and the Fieldmen's Insurance Company, Inc. enteredinto 7 reinsurance agreements under which theformer, as the ceding company undertook to cede tothe latter, as the reinsuring company, a specifiedportion of the amount of insurance underwritten byASIAN upon payment to FIELDMEN'S of aproportionate share of the gross rate of the premiumapplicable with respect to each cession afterdeducting a commission. Said agreements were totake effect from certain specific dates and were to bein force until cancelled by either party upon previousnotice of at least 3 months by registered mail to theother party, the cancellation to take effect as of Dec.31 of the year in which the notice was given. - On Sep. 19, 1961 FIELDMEN'S, by means ofregistered mail, served notice to ASIAN of theformer's desire to be relieved from all participation inits various agreements with the latter effective Dec.31, 1961. This communication, although admittedlyreceived by ASIAN on Sep. 25, 1961, did not elicitany reply from ASIAN. - On Dec. 7, 1961 FIELDMEN'S sent another letter toASIAN expressing regrets at alleged violationscommitted by the latter with respect to the variousagreements between them and reiterated its positionthat it would consider itself "no longer at risk for anyreinsurance and/or cession" given by ASIAN whichmight be in force on Dec. 31, 1961. Not havingreceived any formal reply from ASIAN, FIELDMEN'Ssent a new a letter on Feb. 17, 1962 remindingASIAN of the cancellation of all the reinsurancetreaties and cessions as of Dec. 31, 1961 andrequested ASIAN to submit its final accounting of allcessions made to the former for the precedingmonths when the reinsurance agreements were inforce. - Meanwhile one of the risks reinsured withFIELDMEN'S issued in favor of the GSIS became aliability when the insured property was burned onFebruary 16, 1962. Since the policy was issued onJuly 1, 1961, it was supposed to expire on July 1,1962. 2 The next day, Feb. 17, ASIAN immediatelynotified FIELDMEN'S of said fire loss. - FIELDMEN'S, relying on the sufficiency of its noticeof termination dated September 19, 1961 andobviously bent on avoiding its liability under thereinsurance agreements with ASIAN, filed a petitionfor declaratory relief with the CFI of Manila to seek adeclaration that all the reinsurance contracts enteredinto between them had terminated as of December31, 1961 and to obtain an order directing ASIAN torender final accounting of the transactions betweenthem with respect to said reinsurance treaties as ofthe cut-off date. - In its answer below ASIAN denied having receivedFIELDMEN'S letter dated Sep 19, 1961, and arguedthat even assuming it did, FIELDMEN'S could nothave terminated the reinsurance treaties as of Dec31, 1961 because the letter was merely anexpression of FIELDMEN'S desire to cancel thetreaties and not a formal notice of cancellation ascontemplated in their reinsurance agreements. Byway of special defense Asian contended that even ifthe Sep. 19 letter were considered sufficient notice ofcancellation thereby rendering the reinsuranceagreements terminated as of December 31, 1961 the liability of FIELDMEN'S with respect to policies orcessions issued under two of the said agreementsprior to their cancellation continued to have full forceand effect until the stated expiry dates of suchpolicies or cessions. - On Dec. 4, 1962, the trial court declared 6 of the 7 reinsurance agreements in question cancelled as of Dec 31, 1961. At the same time, it upheld ASIAN'Sposition that all cessions of reinsurance made by it toFIELDMEN'S prior to the cancellation of thereinsurance treaties continued in full force and effectuntil expiry dates and ordered FIELDMEN'S to makean accounting of its business transactions with ASIANwithin 30 days. - On appeal to the CA, the decision of the trial courtwas substantially affirmed, with the slightmodification that the order for accounting waseliminated, without prejudice to the filing of a properaction between the parties for that purpose. ISSUE WON the cancellation as of Dec. 31, 1961 of thereinsurance treaties had the effect of terminatingalso the liability of FIELDMEN'S as reinsurer withrespect to policies or cessions issued prior to thetermination of the principal reinsurance contracts ortreaties HELD NO to the 2 reinsurance contracts

- Of the 6 reinsurance contracts, 2 containprovisions, which clearly and expressly recognize thecontinuing effectivity of policies ceded under themfor reinsurance notwithstanding the cancellation ofthe contracts themselves. The said treaties provide"that in the event of termination of this Agreement .. ., the liability of the Fieldmen's under currentcessions shall continue in full force and effect untiltheir natural expiry . . .;" and the 4th paragraph ofArticle VI of the Personal Accident Reinsurance Treatystates: "4.On the termination of this Agreement fromany cause whatever, the liability of the REINSURER(Fieldmen's) under any current cession includingany amounts due to be ceded under the terms ofthis Agreement and which are not cancelled in theordinary course of business shall continue in fullforce until their expiry unless the COMPANY(Asian) shall, prior to the thirty-first Decembernext following such notice, elect to withdraw theexisting cessions . . ." - Thus, insofar as the two reinsurance agreementsare concerned, there is clearly no merit inFIELDMEN'S claim that their cancellation carried withit ipso facto the termination of all reinsurancecessions thereunder. Such cessions continued to bein force until their respective dates of expiration.Since it was under one of said agreements that thereinsurance cession corresponding to the GSIS policy had been made, FIELDMEN'S cannot avoid liabilitywhich arose by reason of the burning of the insuredproperty. - With respect to the other 4 agreements, it wouldseem that the petition for declaratory relief is moot,and that no useful purpose would be served bydefining the respective rights and obligations of theparties thereunder. The said agreements have beencancelled, and it does not appear that any claim byor liability in favor of the insured has actually arisenunder any of the reinsurance cessions made prior tosuch cancellation. Future conflicts of the same natureas those which have motivated the present actioncan of course be obviated by using more precise anddefinite terminology in the reinsurance agreementswhich the parties may enter into henceforth. EQUITABLE INSURANCE v. RURAL INSURANCE 4 SCRA 343 BARRERA; January 31, 1962 FACTS - Plaintiff Equitable Insurance file a complaint withthe CFI of Manila against defendant Rural Insurancealleging, as first cause of action, that they enteredinto a reciprocal facultative reinsurance agreement,wherein they agreed to cede to each other. Pursuantto said agreement, plaintiff reinsured for P2k withdefendant the stock covered by fire insurance PolicyNo. 5880 issued by plaintiff which was later burned;the share of the loss of defendant as per insuranceagreement was computed at P2,024 for whichplaintiff sent to defendant a statement of account forpayment by the latter.Despite repeated demands byplaintiff, defendant refused to pay. - On the second cause of action, plaintiff reinsuredfor P2k with defendant stock covered by fireinsurance Policy No. 6062 which also burned. Again,defendant refused to pay its share of the loss ofP1,334 hence said complaint. - Defendant filed a motion to dismiss on the groundthat it states no cause of action, as pursuant to ArtVIII of the Reinsurance Agreement between theparties, before a court action can be brought, theparties agreed to submit all disputes to a board ofarbitrators. The Court denied the motion andrequired defendant to answer. - Defendant filed its answer, alleging that the natureof the ag reement is self-liquidating between theparties, the reinsurer becoming the reinsured andvice versa; and that said agreement has not yetbeen abrogated so the liability of either to the other is not yet known. Defendant prayed that thecomplaint be dismissed and plaintiff filed a motionfor judgment on the pleadings which the courtdenied. - Instead of going into a formal hearing, the partiessubmitted their case for decision stipulating the fffacts: defendant admits the allegations of thecomplaint and that plaintiff admits that the issues ofthe complaint were not submitted to a Board ofArbitrators as provided in par VIII of the complaint,but instead referred it to the InsuranceCommissioner. The CFI rendered judgment in favorof plaintiff. Hence this appeal. ISSUES 1. WON Equitable had no cause of action as thematter was not referred to the decision of arbitrators2. WON in a facultative obligation the right to choosean alternative remedy lies only with the debtor (herethe defendant) under Art 1206 HELD 1. NO

- The requirement of submission for decision to 2arbitrators or an umpire the matter of losses by fireor the liability of the parties thereto under Art VIII ofthe agreement arises only if the same is disputed byone of the parties. In the instant case, there is nodispute between the parties; in the stipulation offacts defendant admitted that plaintiff had paid itsliability and defendant likewise admitted that itignored plaintiffs demands for reimbursement fordefendants failure to pay its share as reinsurer. Asheld in Maligad v United Assurance Co., if in thecourse of the settlement of a loss, the action of thecompany or its agents amounts to refusal to pay, thecompany will be deemed to have waived thecondition precedent with reference to arbitration anda suit upon the policy will lie. 2. NO - There is no connection between Art 1206 NCC andthe agreement of this action. The term facultativeis used in reinsurance contracts, and it is so used inthis particular case, merely to define the right of thereinsurer to accept or not to accept participation inthe risk insured. But once the share is accepted, as itwas in the case at bar, the obligation is absolute andthe liability assumed thereunder can be dischargedby only one waythe payment of the share of thelosses. Disposition judgment appealed from the TC is affirmed

ARTEX DEVELOPMENT CO INC v. WELLINGTON INSURANCE CO INC 51 SCRA 352 TEEHANKEE; June 27, 1973 FACTS Wellington Insurance Co. Inc. insured forP24,346,509.00 the buildings, stocks and machineryof plaintiff Artex Development Co. Inc. against lossor damage by fire or lighting upon payment of theplaintiff of the corresponding premiums; that saidproperties were insured for an additional sum ofP883,034.00; that defendant insured plaintiff againstbusiness interruption (use and occupancy) forP5,200,000.00;Wellington entered into a contract ofreinsurance with Alexander and Alexander, Inc. ofNew York. USA. -The buildings, stocks and machineries of plaintiffs spinning department were burned. -Notice of the loss and damage was given thedefendant; that as per report of the adjusters, thetotal property loss of the plaintiff was the sum ofP10,106,554.40 and the total business interruptionloss was P3,000,000.00; - That defendant has paid to the plaintiff the sum ofP6,481,870.07 of the property loss suffered byplaintiff and P1,864,134.08 on its businessinterruption loss, leaving a balance of P3,624,683.43and P1,748,460.00, respectively. -The counsel for Artex filed a Manifestation sayingthat in view of the Deeds of Discharge and CollateralAgreement, the only remaining liability subject oflitigation shall be the proportion of the loss reinsuredwith or through Alexander and Alexander, Inc. ofNew York, USA, namely, P397,813.00. The document recited further that Artexacknowledges receipt of the sum of P3.6M paid bythe insurer in full and final settlement of all or anyclaims of Artex against its insurer. It discharges itsinsurer from all actions, proceedings, claims,demands, costs and expenses in respect thereof. With regard the balance unpaid, Wellingtoncontends that Artex should have been directedagainst the reinsurers to cover the liability and notagainst Wellington. ISSUE WON the insured (Artex) has a cause of action against the reinsurer HELD NO - Unless there is a specific grant in, or assignment of,the reinsurance contract in favor of the insured or amanifest intention of the contracting parties to thereinsurance contract to grant such benefit or favor tothe insured, the insured, not being privy to thereinsurance contract, has no cause of action againstthe reinsurer. It is expressly provided in Section 91the Insurance Act 1 that "(T)he original insured hasno interest in a contract of insurance." PERLA COMPANIA DE SEGUROS v. CA(LIM) 208 SCRA 487 NOCON; May 7, 1992 NATURE Petition for certiorari by Perla Compania de Segurosand FOC Credit Corporation seeking to annul and setaside CA decision revering the RTC decision forreplevin and damages. FACTS - Private respondents spouses Herminio and Evelyn Lim executed a promissory note in favor ofSupercars, Inc. in the sum of P77,940.00, payable inmonthly installments according to the schedule ofpayment indicated in said note,and secured by achattel mortgage over a brand new red Ford Laser,which is registered under the name of privaterespondent Herminio Limand insured with thepetitioner Perla Compania de Seguros, Inc. (Perla forbrevity) for comprehensive coverage. - On the same date, Supercars, Inc., with notice toprivate respondents spouses, assigned to petitionerFCP Credit Corporation (FCP for brevity) its rights,title and interest on said promissory note and chattelmortgage as shown by the Deed of Assignment. - At around 2:30pm Nov9, 1982, said vehicle wascarnapped while parked at the back of BroadwayCentrum. Evelyn Lim, who was driving said carbefore it was carnapped, immediately called up theAnti-Carnapping Unit of the Philippine Constabularyto report said incident and thereafter, went to thenearest police substation to make a police reportregarding said incident.

- On Nov10, 1982, Evelyn Lim reported said incidentto the LTO in compliance with the insurancerequirement. She also filed a complaint with theHeadquarters. Constabulary Highway Patrol Group. - On Nov11, 1982, private respondent filed a claimfor loss with the petitioner Perla but said claim wasdenied on Nov18, 1982 on the ground that Evelyn Lim, who was using the vehicle before it wascarnapped, was in possession of an expired driver'slicense at the time of the loss of said vehicle which isin violation of the authorized driver clause of theinsurance policy, which states, to wit: "AUTHORIZED DRIVER: Any of the following: (a) The Insured (b) Anyperson driving on the Insured's order, or with hispermission. Provided that the person driving ispermitted, in accordance with the licensing orother laws or regulations, to drive the ScheduledVehicle, or has been permitted and is notdisqualified by order of a Court of Law or byreason of any enactment or regulation in that behalf." - On Nov17, 1982, private respondents requestedfrom petitioner FCP for a suspension of payment onthe monthly amortization agreed upon due to theloss of the vehicle and, since the carnapped vehiclewas insured with petitioner Perla, said insurancecompany should be made to pay the remainingbalance of the promissory note and the chattelmortgage contract. - Perla, however, denied private respondents' claim.Consequently, petitioner FCP demanded that privaterespondents pay the whole balance of the promissorynote or to return the vehicle but the latter refused. - On July25, 1983, petitioner FCP filed a complaintagainst private respondents, who in turn filed anamended third party complaint against petitionerPerla on Dec8, 1983. After trial on the merits, TCordered sps Lim to pay jointly and severally, plaintiffthe sum of P55,055.93 plus interest thereon at therate of 24% per annum from July 2, 1983 until fullypaid; as well as the cost of suit. It also ordered thedismissal of the Third party complaint against Third-Party Defendant. - Upon appeal, CA reversed said decision - After petitioners' separate MFRs were denied by CA,petitioners filed these separate petitions for reviewon certiorari. ISSUE 1.WON there was grave abuse of discretion on thepart of the appellate court in holding that privaterespondents did not violate the insurance contractbecause the authorized driver clause is not applicableto the "Theft" clause of said Contract 2.WON the loss of the collateral exempted thedebtor from his admitted obligations under thepromissory note particularly the payment of interest,litigation expenses and attorney's fees HELD 1. NO - The comprehensive insurance policy issued bypetitioner Perla undertook to indemnify the privaterespondents against loss or damages to the car (a)by accidental collision or overturning, or collision oroverturning consequent upon mechanical breakdownor consequent upon wear and tear; (b) by fire,external explosion, self-ignition or lightning orburglary, housebreaking or theft; and (c) bymalicious act. - Where a car is unlawfully and wrongfully takenwithout the owner's consent or knowledge, suchtaking constitutes theft, and, therefore, it is the"THEFT" clause, and not the "AUTHORIZED DRIVER"clause, that should apply.The risk against accident isdistinct from the risk against theft. The 'authorizeddriver clause' in a typical insurance policy as incontemplation or anticipation of accident in the legalsense in which it should be understood, and not incontemplation or anticipation of an event such astheft. The distinction often seized upon by insurancecompanies in resisting claims from their assuredsbetween death occurring as a result of accident anddeath occurring as a result of intent may apply to thecase at bar. - If the insured vehicle had figured in an accident atthe time she drove it with an expired license, then,appellee Perla Compania could properly resistappellants' claim for indemnification for the loss ordestruction of the vehicle resulting from the accident.But in the present case, the loss of the insuredvehicle did not result from an accident where intentwas involved; the loss in the present case wascaused by theft, the commission of which wasattended by intent." - There is no causal connection between thepossession of a valid driver's license and the loss of avehicle. To rule otherwise would render car insurancepractically a sham since an insurance company caneasily escape liability by citing restrictions which arenot applicable or germane to the claim, therebyreducing indemnity to a shadow. 2.The court agrees with FCP that Lim spouses arenot relieved of their obligation to pay the former theinstallments due on the promissory note on accountof the loss of the automobile. The chattel

mortgageconstituted over the automobile is merely anaccessory contract to the promissory note. Being theprincipal contract, the promissory note is unaffected by whatever befalls the subject matter of the accessory contract. - The unpaid balance on the promissory note shouldbe paid, and not just the installments due andpayable before the automobile was carnapped, aserronously held by the CA - However, this does not mean that privaterespondents are bound to pay the interest, litigationexpenses and attorney's fees stipulated in thepromissory note. Because of the peculiar relationshipbetween the three contracts in this case, i. e., thepromissory note, the chattel mortgage contract andthe insurance policy, the Court is compelled toconstrue all three contracts as intimately interrelatedto each other, despite the fact that at first glancethere is no relationship whatsoever between theparties thereto. - Under the promissory note, Lim spouses areobliged to pay Supercars, Inc. the amount statedtherein in accordance with the schedule provided for.To secure said promissory note, private respondentsconstituted a chattel mortgage in favor of Supercars,Inc. over the automobile the former purchased fromthe latter. The chattel mortgage, in turn, requiredprivate respondents to insure the automobile and tomake the proceeds thereof payable to Supercars,Inc. The promissory note and chattel mortgage wereassigned by Supercars, Inc. to petitioner FCP, withthe knowledge of private respondents. Privaterespondents were able to secure an insurance policyfrom petitioner Perla, and the same was madespecifically payable to petitioner FCP. - From the abovementioned provision that upon theloss of the insured vehicle, the insurance companyPerla undertakes to pay directly to the mortgagor orto their assignee, FCP, the outstanding balance of themortgage at the time of said loss under themortgage contract. If the claim on the insurancepolicy had been approved by petitioner Perla, itwould have paid the proceeds thereof directly topetitioner FCP, and this would have had the effect ofextinguishing private respondents' obligation topetitioner FCP. Therefore, private respondents werejustified in asking petitioner FCP to demand theunpaid installments from petitioner Perla. - Because petitioner Perla had unreasonably deniedtheir valid claim, private respondents should not bemade to pay the interest, liquidated damages andattorney's fees as stipulated in the promissory note.As mentioned above, the contract of indemnity wasprocured to insure the return of the money loanedfrom petitioner FCP, and the unjustified refusal of petitioner Perla to recognize the valid claim of theprivate respondents should not in any way prejudicethe latter. - Private respondents can not be said to have unduly enriched themselves at the expense of FCP sincethey will be required to pay the latter the unpaidbalance of its obligation under the promissory note.- In view of the foregoing discussion, We hold thatthe Court of Appeals did not err in requiringpetitioner Perla to indemnify private respondents forthe loss of their insured vehicle. However, the lattershould be ordered to pay petitioner FCP the amountof P55,055.93, representing the unpaid installmentsfrom December 30, 1982 up to July 1, 1983, asshown in the statement of account prepared bypetitioner FCP, 18 plus legal interest from July 2,1983 until fully paid. - As to the award of moral damages, exemplarydamages and attorney's fees, private respondentsare legally entitled to the same since Perla had actedin bad faith by unreasonably refusing to honor theinsurance claim of the private respondents. Besides,awards for moral and exemplary damages, as well asattorney's fees are left to the sound discretion of theCourt. Such discretion, if well exercised, will not bedisturbed on appeal. Disposition the assailed decision of the CA ishereby MODIFIED to require private respondents topay petitioner FCP the amount of P55,055.93, withlegal interest from July 2, 1983 until fully paid. Thedecision appealed from is hereby affirmed as to allother respects. No pronouncement as to costs.

SHAFER v. JUDGE 167 SCRA 386 PADILLA; November 14, 1988 NATURE Petition for review on certiorari FACTS - Sherman Shafer obtained a private car policy overhis Ford Laser from Makati Insurance Company, Inc.,for third party liability.During the effectivity of thepolicy, an information for reckless imprudenceresulting in damage to property and serious physicalinjuries was filed against shafer.The informationsaid that on or about the 17th day of May 1985, inthe City of Olongapo. Shafer hit and bumped aVolkswagen car owned and driven by Felino llano yLegaspi, thereby causing damage in the total amountof P12,345.00 and as a result thereof one Jovencio Poblete, Sr. who was on board of the saidVolkswagen car sustained physical injuries whichinjuries causing deformity on the face.The owner ofthe damaged Volkswagen car filed a separate civilaction against petitioner for damages, while JovencioPoblete, Sr., who was a passenger in the Volkswagencar, did not reserve his right to file a separate civilaction for damages. Instead, in the course of the trialin the criminal case, Poblete, Sr. testified on his claimfor damages for the serious physical injuries whichhe claimed to have sustained as a result of the accident. - The court issued an order dismissing the third partycomplaint on the ground that it was premature,based on the premise that unless the accused(herein petitioner) is found guilty and sentenced topay the offended party (Poblete Sr.) indemnity ordamages, the third party complaint is without causeof action. The court further stated that the betterprocedure is for the accused (petitioner) to wait forthe outcome of the criminal aspect of the case todetermine whether or not the accused, also the thirdparty plaintiff, has a cause of action against the thirdparty defendant for the enforcement of its third partyliability (TPL) under the insurance contract. 6Petitioner moved for reconsideration of said order,but the motion was denied; hence, this petition. ISSUE WON the court a quo erred in dismissing petitioner'sthird party complaint on the ground that petitionerhad no cause of action yet against the insurancecompany HELD YES - There is no need on the part of the insured to waitfor the decision of the trial court finding him guilty ofreckless imprudence. The occurrence of the injury tothe third party immediately gave rise to the liabilityof the insurer under its policy.Respondent insurancecompany's contention that the third party complaintinvolves extraneous matter which will only clutter,complicate and delay the criminal case is withoutmerit.The civil aspect of the offense charged,i .e.,serious physical injuries allegedly suffered byJovencio Poblete, Sr., was impliedly instituted withthe criminal case. Petitioner may thus raise alldefenses available to him insofar as the criminal andcivil aspects of the case are concerned. The claim ofpetitioner for payment of indemnity to the injuredthird party, under the insurance policy, for the alleged bodily injuries caused to said third party,arose from the offense charged in the criminal case,from which the injured (Jovencio Poblete, Sr.) hassought to recover civil damages. Hence, such claimof petitioner against the insurance company cannotbe regarded as not related to the criminal action. - A third party complaint is a device allowed by therules of procedure by which the defendant can bringinto the original suit a party against whom he willhave a claim for indemnity or remuneration as aresult of a liability established against him in theoriginal suit. 13 Third party complaints are allowed tominimize the number of lawsuits and avoid thenecessity of bringing two (2) or more actionsinvolving the same subject matter. They arepredicated on the need for expediency and theavoidance of unnecessary lawsuits. If it appearsprobable that a second action will result if theplaintiff prevails, and that this result can be avoidedby allowing the third party complaint to remain, thenthe motion to dismiss the third party complaintshould be denied. - Compulsory Motor Vehicle Liability Insurance (thirdparty liability, or TPL) is primarily intended to providecompensation for the death or bodily injuriessuffered by innocent third parties or passengers as aresult of a negligent operation and use of motorvehicles. The victims and/or their dependents areassured of immediate financial assistance, regardlessof the financial capacity of motor vehicle owners. - The liability of the insurance company under theCompulsory Motor Vehicle Liability Insurance is forloss or damage. Where an insurance policy insuresdirectly against liability, the insurer's liability accruesimmediately upon the occurrence of the injury orevent upon which the liability depends, and does notdepend on the recovery of judgment by the injuredparty against the insured.

- The injured for whom the contract of insurance isintended can sue directly the insurer. The generalpurpose of statutes enabling an injured person toproceed directly against the insurer is to protectinjured persons against the insolvency of the insuredwho causes such injury, and to give such injuredperson a certain beneficial interest in the proceeds ofthe policy, and statutes are to be liberally construedso that their intended purpose may be accomplished.It has even been held that such a provision creates acontractual relation which inures to the benefit of anyand every person who may be negligently injured bythe named insured as if such injured person werespecifically named in the policy. - In the event that the injured fails or refuses toinclude the insurer as party defendant in his claim forindemnity against the insured, the latter is notprevented by law to avail of the procedural rulesintended to avoid multiplicity of suits. Not even a "noaction" clause under the policy-which requires that afinal judgment be first obtained against the insuredand that only thereafter can the person insuredrecover on the policy can prevail over the Rules ofCourt provisions aimed at avoiding multiplicity of suits. Disposition instant petition is GRANTED. Thequestioned order dated 24 April 1987 is SET ASIDEand a new one entered admitting petitioner's thirdparty complaint against the private respondentMakati Insurance Company, Inc.

VDA DE MAGLANA v. CONSOLACION 212 SCRA 268 ROMERO; August 6, 1992 NATURE Petition for certiorari FACTS - Lope Maglana was an employee of the Bureau ofCustoms whose work station was at Lasa, here inDavao City. One day, when he was on his way to hiswork, he met an accident that resulted in his death.He died on the spot. - The PUJ jeep that bumped the deceased was drivenby Pepito Into, operated and owned by defendantDestrajo. From the investigation conducted by thetraffic investigator, the PUJ jeep was overtakinganother passenger jeep that was going towards thecity poblacion. While overtaking, the PUJ jeep ofdefendant Destrajo running abreast with theovertaken jeep, bumped the motorcycle driven bythe deceased. The point of impact was on the lane ofthe motorcycle and the deceased was thrown fromthe road and met his untimely death. - Heirs of Lope Maglana, Sr. filed an action fordamages and attorney's fees against operatorPatricio Destrajo and the Afisco InsuranceCorporation (AFISCO).An information for homicidethru reckless imprudence was also filed againstPepito Into. - During the pendency of the civil case, Into wassentenced to suffer an indeterminate penalty, with allthe accessory penalties provided by law, and toindemnify the heirs of Lope Maglana, Sr. in theamount of twelve thousand pesos with subsidiary imprisonment in case of insolvency, plus fivethousand pesos in the concept of moral andexemplary damages with costs. No appeal wasinterposed by accused who later applied forprobation. - The lower court rendered a decision finding thatDestrajo had not exercised sufficient diligence as theoperator of the jeepney ordering him to pay plaintiffsthe sum for loss of income; funeral and burialexpenses of the deceased; moral damages, andattorney's fees and costs of suit.The defendantinsurance company is ordered to reimbursedefendant Destrajo whatever amounts the latter shallhave paid only up to the extent of its insurancecoverage. - Petitioners filed a motion for the reconsideration ofthe second paragraph of the decision contending thatAFISCO should not merely be held secondarily liablebecause the Insurance Code provides that theinsurer's liability is "direct and primary and/or jointlyand severally with the operator of the vehicle,although only up to the extent of the insurancecoverage." Hence, they argued that the P20,000.00coverage of the insurance policy issued by AFISCO,should have been awarded in their favor. -AFISCO argued that since the Insurance Code doesnot expressly provide for a solidary obligation, thepresumption is that the obligation is joint. - The lower court denied the motion forreconsideration ruling that since the insurancecontract "is in the nature of suretyship, then theliability of the insurer is secondary only up to theextent of the insurance coverage." - Petitioners filed a second motion for reconsiderationreiterating that the liability of the insurer is direct,primary and solidary with the jeepney operatorbecause the petitioners became direct beneficiariesunder the provision of the policy which, in effect, is astipulation pour autrui. This motion was likewisedenied for lack of merit. ISSUE WON AFISCO can be held directly liable HELD YES - As this Court ruled in Shafer vs. Judge, RTC of Olongapo City, Br. 75, "[w]here an insurance policy insures directly against liability, the insurer's liabilityaccrues immediately upon the occurrence of theinjury or even upon which the liability depends, and does not depend on the recovery of judgment by the injured party against the insured." - The underlying reason behind the third partyliability (TPL) of the Compulsory Motor VehicleLiability Insurance is "to protect injured personsagainst the insolvency of the insured who causessuch injury, and to give such injured person a certainbeneficial interest in the proceeds of the policy . . ."Since petitioners had received from AFISCO the sumof P5,000.00 under the no-fault clause, AFISCO'sliability is now limited to P15,000.00. - However, we cannot agree that AFISCO is likewise solidarily liable with Destrajo. In Malayan Insurance Co., Inc. v. Court of Appeals, this Court had the

opportunity to resolve the issue as to the nature ofthe liability of the insurer and the insuredvis-avisthe third party injured in an accident. Wecategorically ruled thus:While it is true that wherethe insurance contract provides for indemnity againstliability to third persons, such third persons candirectly sue the insurer, however, the direct liability of the insurer under indemnity contracts againstthird party liability does not mean that the insurercan be held solidarily liable with the insured and/orthe other parties found at fault. The liability of theinsurer is based on contract; that of the insured isbased on tort.In the case at bar, petitioner as insurer of Sio Choy, is liable to respondent Vallejos(the injured third party), but it cannot, as incorrectlyheld by the trial court, be made "solidarily" liablewith the two principal tortfeasors, namelyrespondents Sio Choy and San Leon Rice Mill, Inc. For if petitioner-insurer were solidarily liable withsaid, two (2) respondents by reason of the indemnitycontract against third party liability under which an insurer can be directly sued by a third party this will result in a violation of the principles underlying solidary obligation and insurance contracts. - While in solidary obligations, the creditor mayenforce the entire obligation against one of thesolidary debtors, in an insurance contract, theinsurer undertakes for a consideration to indemnifythe insured against loss, damage or liability arisingfrom an unknown or contingent event. - Similarly, petitioners herein cannot validly claimthat AFISCO, whose liability under the insurancepolicy is also P20,000.00, can be held solidarily liablewith Destrajo for the total amount of P53,901.70 inaccordance with the decision of the lower court.Since under both the law and the insurance policy,AFISCO's liability is only up to P20,000.00, thesecond paragraph of the dispositive portion of the decision in question may have unwittingly sownconfusion among the petitioners and their counsel.What should have been clearly stressed as to leaveno room for doubt was the liability of AFISCO underthe explicit terms of the insurance contract. Disposition present petition is hereby GRANTED. The award of P28,800.00 representing loss of incomeis INCREASED to P192,000.00 and the deathindemnity of P12,000.00 to P50,000.00.

FAR EASTERN SURETY v. MISA 25 SCRA 663 REYES; October 26, 1968 NATURE Appeal by petition for review from a CA judgment FACTS - Socorro Dancel Vda.de Misa and Araceli Pinto hireda taxi cab operated by La Mallorca on September 3,1957. The taxi they were riding in collided with agravel and sand truck resulting to injuries to bothMisa and Pinto. - The two passengers instituted a suit for damagesagainst La Mallorca who, while denying responsibility,instituted a third party complaint against Far EasternSurety to recoup from the latter any award fordamages that might be recovered by the passengers.- It would appear from the case that a sticker wasplaced in all the taxis of La Mallorca stating thatpassengers of the taxis were insured againstaccidents. This was done to entice the public intopatronizing La Mallorca. - The trial court awarded to Misa and Pinto actual,moral and exemplary damages, and attorneys feespayable by La Mallorca and sentenced Far Eastern topay La Mallorca P10,000. on its third party liabilityinsurance. - On appeal, the CA,while holding that the collisionwas due to the fault of the driver of the gravel andsand truck, found the taxi company liable fordamages to the passengers on the strength of itsrepresentation contained in the sticker above notedthat the passengers were insured against accidents.In so ruling, the CA overruled the defense of theinsurance company to the effect that it wasresponsible only if the insured, La Mallorca, was involved in accidents caused by, or arising out of, theuse of the motor vehicle. A motion forreconsideration was filed in and dismissed by the CA. ISSUE WON Far Eastern Surety is liable to the insured on its insurance policy HELD NO - The award for damages made to the passengerswas exclusively predicated on the representationmade by La Mallorca that its passengers wereinsured against accidents and not because it was atfault in causing the accident. Reasoning - In this case, the findings of the CA and the trialcourt that the causative factor of the mishap was thenegligence of the gravel and truck driver would havebeen sufficient to relieve the taxi company of anyliability arising from the accident. However, in view ofthe sticker in all of its taxicabs, La Mallorca hasinsured its passengers against accidents, whether itwas at fault or not. In other words, La Mallorcaaccepted the responsibility for damages or injuries topassengers even if it had no fault at all. - In the case of the insurance company, the SC ruledthat it neither authorized nor consentedto therepresentations made by the taxi company to itspassengers. As such, the liability of the saidinsurance company based on its insurance contractis limited to the recovery by the insured of all sums,cost and expenses which the insured shall becomelegally liable. The insurance company thereforecannot be held liable for the award. -The taxi company is adjudged to be the sole party responsible for the award. DispositionThe decision of the CA is modified by eliminating the award against Far Eastern.

PEZA v. ALIKPALA 160 SCRA 31 NARVASA; April 15, 1988 NATURE Motion praying that Judge Alikpala be declared guiltyof contempt of court for having decided the case onthe merits despite the pendency in this Court of thecertiorari action instituted by the plaintiffs FACTS - vehicular accident with 2 children running acrossthe path of a Chevrolet "Carry-All", belonging to apartnership known as Diman & Company driven byits driver, Perfecto Amar,as it was passing a nationalhighway at barrio Makiling Calamba, Laguna. Theywere killed. It was insured with the Empire InsuranceCo., Inc. under a so-called 'comprehensive coverage"policy, loss by theft excluded. The policy was in forceat the time of the accident. - Placida Peza, the managing partner of Diman & Co.filed a claim with Empire, for payment ofcompensation to the family of the 2 children whodied as a result of the accident. Empire refused topay on the ground that the driver had no authority tooperate the vehicle, a fact which it expresslyexcepted from liability under the policy. What Pezadid was to negotiate directly with the deceasedchildren father for an out-of-court settlement. Thefather agreed to accept P6,200.00 in fun settlementof the liability of the vehicles owner and driver, andPeza paid him this sum. -Peza thereafter sued Empire to recover this sum ofP6,200.00 as actual damages, as well as P20,000.00as moral damages, P10,000.00 as exemplarydamages, and P10,000.00 as attorney's fees. Sheamended her complaint shortly thereafter to includeDiman & Co. as alternative party plaintiff. - Empire's basic defense to the suit was anchoredon the explicit requirement in the policy limiting theoperation of the insured vehicle to the "authorizeddriver" therein defined, namely, (a) the insured, or(b) any person driving on the insured order or withhis permission, provided that... that the person driving is permited inaccordance with the licensing or other laws orregulations to drive the Motor vehicle or has beenso permitted and is not disqualified by order of theCourt of Law of by reason of any enactment orregulation in that behalf from driving such MotorVehicle.- driver Perfecto Amar, only having a temporaryoperator's permit (TVR) [already expired] his driverslicense having earlier been confiscated by an agentof the Land Transportation Commission for an allegedviolation of Land Transportation and Traffic Rules,was not permitted by law and was in truthdisqualified to operate any motor vehicle; Pezaattempted to neutralize that fact by(1) the issuanceof the TVR by the LTC officer to Amar; in proof of theproposition that there was no reason for confiscationof Amar's license (2) Amar's license had not expired,but had been renewed. - Judge Alikpala did not admit such evidence ISSUES 1.WON Judge Alikapala committed grave abuse of discretion in not admitting evidence 2.WON confiscation of license and expiration of TVRof the driver would serve as bar for Peza inrecovering from Empire HELD 1. NO - Even positing error in the Judge's analysis of theevidence attempted to be introduced and hisrejection thereof, it is clear that it was at most anerror of judgment, not such an error as may bebranded a grave abuse of discretion, i.e., suchcapricious and whimsical exercise of judgment as isequivalent to lack of jurisdiction, against which thewrit of certiorari will lie. In any event, the establishedprinciple is "that ruling of the trial court onprocedural questions and on admissibility of evidenceduring the course of the trial are interlocutory innature and may not be the subject of separateappeal or review oncertiorari, but are to be assignedas errors and reviewed in the appeal properly takenfrom the decision rendered by the trial court on themerits of the case. - In the meantime, Judge Alikpala renderedjudgment on the merits, since the case was thenalready ripe for adjudication. The judgment ordereddismissal of the case for failure on the part of theplaintiff to prove their cause of action againstEmpire. Notice of the judgment was served on theparties in due course. 2. YES - It would seem fairly obvious that whether the LTCagent was correct or not in his opinion that driverAmar had violated some traffic regulation warrantingconfiscation of his license and issuance of a TVR inlieu thereof, this would not alter the undisputed factthat Amar's licence had indeed been confiscated anda TVR issued to him, and the TVR had alreadyexpired at the time that the vehicle being

operatedby him killed two children by accident. Neither wouldproof of the renewal of Amar's license change thefact that it had really been earlier confiscated by theLTC agent. Dispositionpetition is DISMISSED for lack of merit

PERLA COMPANIA DE SEGUROS v. ANCHETA 164 SCRA 144 CORTES; August 8, 1988 NATURE Petition for certiorari and prohibition with prelim injunction to review orders of CFI Camarines Norte FACTS - Perla was the insurer of a Superlines bus whichfigured in a collision with a III Scout (its a kind ofvehicle). Injured passengers of the latter (andrespondents in this case) filed a complaint fordamages against Superlines, the bus driver, andPerla (as insurer of the bus). CFI Judge Anchetaordered that Perla should pay the respondentsimmediately the P5000 under the no fault clause asprovided in Sec. 378. Sec. 378: Any claim for death or injury to anypassenger or 3rd party pursuant to the provisions ofthis chapter shall be paid without the necessity ofproving fault or negligence of any kind. Provided,That for purposes of this section (i) The indemnity in respect of any one person shall not exceed P5,000; (ii) The following proofs of loss, when submittedunder oath, shall be sufficient evidence tosubstantiate the claim: (a) Police report of accident, and (b) Death certificateand evidence sufficient to establish, the proper payee, or (c) Medical report and evidence of medical orhospital disbursement in respect of which refund isclaimed; (iii) Claim may be made against one motor vehicle only. In the case of an occupant of a vehicle, claim shall lie against the insurer of the vehicle in whichthe occupant is riding, mounting or dismountingfrom. In any other case, claim shall lie against the insurer of the directly offending vehicle. In all cases,the right of the party paying the claim to recoveragainst the owner of the vehicle responsible for theaccident shall be maintained. Perla denied its liability under the above provisionand said that the insurer of the vehicle that therespondents were riding (Malayan Insurance in thiscase) should be liable.Its 2 MFRs denied, Perla filedthis action ISSUE WON Perla is the insurer liable to indemnify under Sec. 378 HELD NO Ratio The law is very clear the claim shall lie against the insurer of the vehicle in which theoccupant xxx is riding, and no other. The claimantis not free to choose from which insurer he will claimthe no fault indemnity, as the law, by using theword shall, makes it mandatory that the claim bemade against the insurer of the vehicle in which theoccupant is riding, mounting or dismounting from. Reasoning - the rules on claims under the no fault indemnityprovision, where proof of fault or negligence is notnecessary for payment of any claim for death orinjury to a passenger or to a 3rd party, are established: 1. A claim may be made against one motor vehicle only. 2. If the victim is an occupant of a vehicle, the claimshall lie against the insurer of the vehicle in which heis riding, mounting or dismounting from. 3. In any other case (i.e. if the victim is not anoccupant of a vehicle), the claim shall lie against theinsurer of the directly offending vehicle. 4. In all cases, the right of the party paying the claimto recover against the owner of the vehicleresponsible for the accident shall be maintained. -That the vehicle ridden might not be the one thatcaused the accident is of no moment since the lawitself provides that the party paying the claim underSec. 378 may recover against the owner of thevehicle responsible for the accident. This is preciselythe essence of no fault indemnity insurance whichwas introduced to and made part of our laws in orderto provide victims of vehicular accidents or theirheirs immediate compensation, although in a limitedamount, pending final determination of who

isresponsible for the accident and liable for the victims'injuries or death. In turn, the no fault indemnityprovision is part and parcel of the Code provisions oncompulsory motor vehicle liability insurance andshould be read together with the requirement forcompulsory passenger and/or 3rd party liabilityinsurance (Sec. 377) which was mandated in orderto ensure ready compensation for victims ofvehicular accidents. -Irrespective of whether or not fault or negligencelies with the driver of the Superlines bus, asrespondents were not occupants of the bus, theycannot claim the no fault indemnity provided inSec. 378 from Perla. The claim should be madeagainst the insurer of the vehicle they were riding. Disposition Petition GRANTED. Orders of CFI ordering Perla to pay respondents immediately P5000 ANNULLED and SET ASIDE

WESTERN GUARANTY CORPORATION v. CA (RODRIGUEZ & DE DIOS TRANS) 185 SCRA 652 FELICIANO; July 20, 1990 NATURE Petition for review the decision of CA affirming in totothe damages awarded to private respondent by thetrial court. FACTS - Respondent Priscilla E. Rodriguez was struck by aDe Dios passenger bus owned by respondent De DiosTransportation Co., Inc. Priscilla was thrown to theground, hitting her forehead. She was treated at theProtacio Emergency Hospital and later onhospitalized at the San Juan De Dios Hospital. Herface was permanently disfigured, causing her seriousanxiety and moral distress. Respondent bus companywas insured with petitioner Western GuarantyCorporation ("Western") under its Master Policywhich provided, among other things, for protectionagainst third party liability, the relevant sectionreading as follows: Section 1. Liability to the Public ? Company will,subject to the Limits of Liability, pay all sumsnecessary to discharge liability of the insured inrespect of ? (a) death of or bodily injury to or damage to property of any passenger as defined herein. (b) death of or bodily injury or damage to propertyof any THIRD PARTY as defined herein in anyaccident caused by or arising out of the use of theSchedule Vehicle, provided that the liability shallhave first been determined. In no case, however,shall the Company's total payment under bothSection I and Section 11 combined exceed theLimits of Liability set forth herein. With respect todeath of or bodily injury to any third party orpassenger, the company's payment per victim inany one accident shall not exceed the limitsindicated in the Schedule of indemnities provided for in this policy excluding the cost of additionalmedicines, and such other burial and funeralexpenses that might have been incurred. - Respondent Priscilla Rodriguez filed a complaint fordamages before the Regional Trial Court of Makatiagainst De Dios Transportation Co. and Walter A.Saga Respondent De Dios Transportation Co., in turn,filed a third-party complaint against its insurancecarrier, petitioner Western. On 6 August 1985, thetrial court rendered a decision in favor of respondentPriscilla E. Rodriguez, awarding moral damages,lossof earning and attorney's fees among others. - On appeal, the Court of Appeals affirmed in toto the decision of the trial court. - Petitioner contends that it cannot be held liable forloss of earnings, moral damages and attorney's feesbecause these items are not among those included inthe Schedule of Indemnities set forth in theinsurance policy. ISSUE WON petitioner can be held liable for loss of earnings, moral damages and attorney's fees HELD YES - The Schedule of Indemnities does not purport torestrict the kinds of damages that may be awardedagainst Western once liability has arisen. It wasmerely meant to set limits to the amounts themovant would be liable for in cases of claims fordeath, bodily injuries of, professional services andhospital charges, for services rendered to trafficaccident victims,' and not necessarily exclude claimsagainst the insurance policy for other kinds ofdamages, such as those in question. - Itwill be seen that the above quoted Schedule ofIndemnities establishes monetary limits whichWestern may invoke in case of occurrence of theparticular kinds of physical injury there listed. - It must be stressed, however, that the Schedule ofIndemnities does not purport to limit, or toenumerate exhaustively, the species of bodily injuryoccurrence of which generate liability for petitionerWestern. A car accident may, for instance, result ininjury to internal organs of a passenger or thirdparty, without any accompanying amputation or lossof an external member (e.g., a foot or an arm or aneye). But such internal injuries are surely covered bySection I of the Master Policy, since they certainlyconstitute bodily injuries. - The Schedule of Indemnities does not purport torestrict the kinds of damages that may be awardedagainst Western once liability has arisen. Section 1,quoted above, does refer to certain "Limits ofLiability" which in the case of the third party liabilitysection of the Master Policy, is apparently

P50,000.00per person per accident. Within this over-allquantitative limit, all kinds of damages allowable bylaw" actual or compensatory damages"; "moraldamages'; "nominal damages"; "temperate ormoderate damages"; "liquidated damages"; and"exemplary damages" ? may be awarded by acompetent court against the insurer once liability isshown to have arisen, and the essential requisites orconditions for grant of each species of damages arepresent. It appears to us self-evident that theSchedule of Indemnities was not intended to be anenumeration, much less a closed enumeration, of thespecific kinds of damages which may be awardedunder the Master Policy Western has issued. - The reading urged by Western of the Schedule ofIndemnities comes too close to working fraud uponboth the insured and the third party beneficiary ofSection 1, quoted above. For Western's readingwould drastically and without warning limit theotherwise unlimited and comprehensive scope ofliability assumed by the insurer Western underSection 1: "all sums necessary to discharge liabilityof the insured in respect of [bodily injury to a thirdparty]". This result- which is not essentially differentfrom taking away with the left hand what had beengiven with the right hand we must avoid as obviouslyrepugnant to public policy. If what Western nowurges is what Western intended to achieve by itsSchedule of Indemnities, it was incumbent uponWestern to use language far more specific andprecise than that used in fact by Western, so that theinsured, and potential purchasers of its Master Policy,and the Office of the Insurance Commissioner, maybe properly informed and act accordingly. - Moreover, an insurance contract is a contract ofadhesion. The rule is well entrenched in ourjurisprudence that the terms of such contract are tobe construed strictly against the party whichprepared the contract, which in this case happens tobe petitioner Western. Disposition Petition denied.

SUMMIT GUARANTY & INSURANCE COMPANY v. ARNALDO 158 SCRA 332 GANCAYCO; February 29, 1988 NATURE PETITION to review the order of the Insurance Commissioner. FACTS - On Nov. 26, 1976, a Ford Pick-up truck owned byMarcos Olasco was bumped by a cargo truck ownedby Floralde. FGU Insurance Corporation (FG U) by reason ofMotor Vehicle Insurance Policy No. IC-VF-07185 paidOlaso the sum of P2,817.50 as its share in the repaircost of the said Ford Pick-up. Having beensubrogated to the rights and causes of action ofOlaso in the said amount FGU formally demandedpayment of said amount from Floralde andattempted to verify Floralde's insurance carrier butfailed to do so. In 1978 FGU was able to ascertainthe identity of Floralde's insurance carrier to be theSummit Guaranty and Insurance Company, Inc.(Summit) and thus requested the insurancecommissioner for a conference with Summit anddemanded from Summit through counsel onFebruary 28, 1978 the payment of the damagessustained by the car of Olaso but to no avail. - Hence on May 22, 1978 FGU filed a case in theInsurance Commissioner's Office against Summit forrecovery of said amount. - Summit filed a motion to dismiss on the ground ofprescription under Section 384 of PD No. 612.Averring that the accident happened on November26, 1976 while the complaint was filed on May 22,1978 beyond the one-year period from the time ofthe accident provided for by the said provision. ISSUE WON the action must be dismissed on the ground of prescription under Section 384 of PD No. 612 HELD NO - The case do not fall within the meaning ofproper cases' as contemplated in Section 384 of the Insurance Code. Reasoning - Section 384 of PD 612 (Insurance Code) Any person having any claim upon the policyissued pursuant to this chapter shall, without anyunnecessary delay, present to the insurancecompany concerned a written notice of claim setting forth the amount of his loss, and/or thenature, extent and duration of the injuriessustained as certified by a duly licensed physician.Notice of claim must be filed within six monthsfrom date of the accident, otherwise, the claimshall be deemed waived. Action or suit for recoveryof damage due to loss or injury must be brought, in proper cases, with the Commissioner or the Courts within one year from date of accident,otherwise, the claimant's right of action shallprescribe. - It is very clear that the one-year period is onlyrequired In proper cases. Had the lawmakersintended it to be the way Petitioner Companyassumes it to be, then the phrase 'in proper cases'would not have been inserted. - in Aisporna. vs. Court of Appeals: 'Legislative intent must be ascertained from aconsideration of the statute as a whole. Theparticular words, clauses and phrases should notbe studied as detached and isolated expressions,but the whole and every part of the statute mustbe considered in fixing the meaning of any of itsparts and in order to produce a harmonious whole.A statute must be so construed as to harmonizeand give effect to all its provisions wheneverpossible.' - Petitioner company is trying to use Section 384 ofthe Insurance Code as a cloak to hide itself from itsliabilities. The facts of these cases evidently reflectthe deliberate efforts of petitioner company toprevent the filing of a formal action against it.Bearing in mind that if it succeeds in doing so untilone year lapses from the date of the accident it couldset up the defense of prescription, petitionercompany made private respondents believe that theirclaims would be settled in order that the latter willnot find it necessary to immediately bring suit. Inviolation of its duties to adopt and implementreasonable standards for the prompt investigation ofclaims and to effectuate prompt, fair and equitablesettlement of claims, and with manifest bad faith,petitioner company devised means and ways ofstalling the settlement proceedings. In G.R. No. L-50997, no steps were taken to process the claim andno rejection of said claim was ever made even ifprivate respondent had already complied with all therequirements.

- In G.R. No. L-48758-petitioner company evenprovided legal assistance to one of the privaterespondents in the criminal case filed against himleading Private respondents to believe that it was ready to pay. In the same case, petitioner companyadmits that it took no final action or adjudication ofthe claim. Worse still, in G.R. No. L-48679,assurances of payment were constantly given andpetitioner company even said that a check was readyfor release. This Court has made the observation thatsome insurance companies have been inventingexcuses to avoid their just obligations and it is onlythe State that can give the protection which theinsuring public needs from possible abuses of theinsurers. In view of the foregoing, - It is not denied that an extrajudicial demand forpayment was made by respondent FGU on petitionerbut petitioner failed to respond to the same.Nevertheless the complaint was filed even before adenial of the claim was made by petitioner. For alllegal purposes, the one-year prescriptive periodprovided for in Section 384 of the Insurance Codehas not begun to run.The cause of action arises onlyand starts to run upon the denial of the claim by theinsurance company.The court takes note of thedilatory tactics employed by petitioner in this as inthe several cases aforecited to avoid payment of itsliabilities.

VILLACORTA v. THE INSURANCE COMMISSION 100 SCRA 467 TEEHANKEE; October 30, 1980 FACTS - JEWEL VILLACORTA was the owner of a Colt Lancer,Model 1976, insured with respondent company forP35,000.00 - Own Damage; P30,000.00 - Theft; andP30,000.00 - Third Party Liability, effective May 16,1977 to May 16, 1978. - On May 9, 1978, the vehicle was brought to theSunday Machine Works, Inc., for general checkupand repairs. On May 11, 1978, while it was in thecustody of the Sunday Machine Works, the car wasallegedly taken by six (6) persons and driven out toMontalban, Rizal. While travelling along Mabini St.,Sitio Palyasan, Barrio Burgos, going North atMontalban, Rizal, the car figured in an accident,hitting and bumping a gravel and sand truck parkedat the right side of the road going south. As aconsequence, the gravel and sand truck veered tothe right side of the pavement going south and thecar veered to the right side of the pavement goingnorth. The driver, Benito Mabasa, and one of thepassengers died and the other four sustainedphysical injuries. The car, as well, suffered extensivedamage. Complainant, thereafter, filed a claim for total loss with the respondent company but claimwas denied. Hence, complainant was compelled toinstitute the present action." - The comprehensive motor car insurance policy forP35,000.00 issued by respondent Empire InsuranceCompany admittedly undertook to indemnify thepetitioner-insured against loss or damage to the car(a) by accidental collision or overturning, or collisionor overturning consequent upon mechanicalbreakdown or consequent upon wear and tear; (b)by fire, external explosion, self-ignition or lightningor burglary, housebreaking or theft; and (c) bymalicious act. - Respondent insurance commission, however,dismissed petitioner's complaint for recovery of thetotal loss of the vehicle against private respondent,sustaining respondent insurer's contention that theaccident did not fall within the provisions of thepolicy either for the Own Damage or Theft coverage,invoking the policy provision on "Authorized Driver"clause, which clause limits the use of the insuredvehicle to two (2) persons only, namely: the insuredhimself or any person on his (insured's) permission.Apparently, the Insurance commission sees theunauthorized taking of the vehicle for a joyride as aviolation of the 'Authorized Driver' clause of the policy." - Respondent commission likewise upheld privaterespondent's assertion that the car was not stolenand therefore not covered by the Theft clause, rulingthat "(T)he element of 'taking' in Article 308 of theRevised Penal Code means that the act of deprivinganother of the possession and dominion of a movablething is coupled . . . with the intention, at the time ofthe 'taking', of withholding it with the character ofpermanency ISSUE WON the Insurance commissions findings are in accord with law HELD NO - First, respondent commission's ruling that theperson who drove the vehicle in the person of BenitoMabasa, who, according to its own finding, was oneof the residents of the Sunday Machine Works, Inc.to whom the car had been entrusted for generalcheck-up and repairs was not an "authorized driver"of petitioner-complainant is too restrictive andcontrary to the established principle that insurancecontracts, being contracts of adhesion where the only participation of the other party is the signing of hissignature or his "adhesion" thereto, "obviously callfor greater strictness and vigilance on the part ofcourts of justice with a view of protecting the weakerparty from abuse and imposition, and prevent theirbecoming traps for the unwary." - The main purpose of the "authorized driver" clause,as may be seen from its text, supra, is that a personother than the insured owner, who drives the car onthe insured's order, such as his regular driver, or withhis permission, such as a friend or member of thefamily or the employees of a car service or repairshop must be duly licensed drivers and have nodisqualification to drive a motor vehicle. A car ownerwho entrusts his car to an established car serviceand repair shop necessarily entrusts his car key tothe shop owner and employees who are presumed tohave the insured's permission to drive the car forlegitimate purposes of checking or road-testing thecar. The mere happenstance that the employee(s) ofthe shop owner diverts the use of the car to his ownillicit or unauthorized purpose in violation of the trustreposed in the shop by the insured car owner doesnot mean that the "authorized driver" clause hasbeen violated such as to bar recovery, provided thatsuch employee is duly qualified to drive under a validdriver's license.

- Secondly, and independently of the foregoing (sincewhen a car is unlawfully taken, it is the theft clause,not the "authorized driver" clause, that applies),where a car is admittedly as in this case unlawfullyand wrongfully taken by some people, be theyemployees of the car shop or not to whom it hadbeen entrusted, and taken on a long trip toMontalban without the owner's consent orknowledge, such taking constitutes or partakes ofthe nature of theft as defined in Article 308 of theRevised Penal Code. - The Court rejects respondent commission'spremise that there must be an intent on the part ofthe taker of the car "permanently to deprive theinsured of his car" and that since the taking here wasfor a "joy ride" and "merely temporary in nature," a"temporary taking is held not a taking insured against." - The insurer must therefore indemnify the petitionerowner for the total loss of the insured car in the sumof P35,000.00 under the theft clause of the policy,subject to the filing of such claim for reimbursementor payment as it may have as subrogee against theSunday Machine Works, Inc.

CHAPTER VIII. MARINE INSURANCE MAGSAYSAY INC v. AGAN 96 PHIL 504 REYES; January 31, 1955 FACTS - The S S "San Antonio", vessel owned and operatedby plaintiff, left Manila on October 6, 1949, bound forBasco, Batanes, vis Aparri, Cagayan, with generalcargo belonging to different shippers, among themthe defendant. The vessel reached Aparri, but whilestill in the port, it ran aground at the mouth of theCagayan river, and, attempts to refloat it under itsown power having failed, plaintiff had it refloated bythe Luzon Stevedoring Co. at an agreedcompensation. Once afloat the vessel returned toManila to refuel and then proceeded to Basco, theport of destination. There the cargoes were deliveredto their respective owners or consignees, who, withthe exception of defendant, made a deposit or signeda bond to answer for their contribution to the average. - On the theory that the expenses incurred in floatingthe vessel constitute general average to which bothship and cargo should contribute, plaintiff broughtthe present action in the CFI of Manila to makedefendant pay his contribution, which, as determinedby the average adjuster, amounts to P841.40. - Defendant denies liability to his amount, alleging,among other things, that the stranding of the vesselwas due to the fault, negligence and lack of skill ofits master, that the expenses incurred in putting itafloat did not constitute general average, and thatthe liquidation of the average was not made inaccordance with law. - The lower court found for plaintiff ISSUE WON the expenses incurred in floating a vessel sostranded should be considered general average andshared by the cargo owners HELD NO Ratio The law on averages is contained in the Code of Commerce. Under that law, averages are classifiedinto simple or particular and general or gross.Generally speaking, simple or particular averagesinclude all expenses and damages caused to thevessel or cargo which have not inured to thecommon benefit (Art. 809), and are, therefore, to beborne only by the owner of the property gave rise tosame (Art. 810); while general or gross averagesinclude "all the damages and expenses which aredeliberately caused in order to save the vessel, itscargo, or both at the same time, from a real andknown risk" (Art. 811). Being for the commonbenefit, gross averages are to be borne by theowners of the articles saved (Art. 812). Reasoning - the stranding of plaintiff's vessel was due to the sudden shifting of the sandbars at the mouth of theriver which the port pilot did not anticipate. Thestanding may, therefore, be regarded as accidental.- Tolentino, in his commentaries on the Code ofCommerce, gives the following requisites for general average: First, there must be a common danger. This means, that both the ship and the cargo, after has beenloaded, are subject to the same danger, whetherduring the voyage, or in the port of loading orunloading; that the danger arises from the accidentsof the sea, dispositions of the authority, or faults ofmen, provided that the circumstances producing theperil should be ascertained and imminent or mayrationally be said to be certain and imminent. Thislast requirement exclude measures undertakenagainst a distant peril. Second, that for the common safety part of the vessel or of the cargo or both is sacrificed deliberately. Third, that from the expenses or damages caused follows the successful saving of the vessel and cargo. Fourth, that the expenses or damages should have been incurred or inflicted after taking proper legal steps and authority. - With respect to the first requisite, the evidencedoes not disclose that the expenses sought to berecovered from defendant were incurred to savevessel and cargo from a common danger...it is thesafety of the property, and not of the voyage, whichconstitutes the true foundation of the general average. - As to the second requisite, we need only repeat

that the expenses in question were not incurred for the common safety of vessel and cargo, since they, or at least the cargo, were not in imminent peril. - With respect to the third requisite, the salvageoperation, it is true, was a success. But as thesacrifice was for the benefit of the vessel to enable itto proceed to destination and not for the purpose ofsaving the cargo, the cargo owners are not in lawbound to contribute to the expenses. - The final requisite has not been proved, for it doesnot appear that the expenses here in question wereincurred after following the procedure laid down inarticle 813. Disposition Wherefore, the decision appealed from is reversed.

JARQUE v. SMITH, BELL & CO. 56 PHIL 758 OSTRAND; November11, 1932 NATURE Appeal from judgment of the lower court FACTS - Plaintiffs motorboat, Pandan was insured on amarine insurance policy with National Union FireInsurance Company (NUFIC) for P45K. According tothe provisions of a rider attached to the policy, theinsurance was against the absolute total loss of thevessel only. On Oct. 31, 1928, the ship ran into veryheavy sea and it became necessary to jettison aportion of the cargo. As a result of the jettison, theNUFIC was assessed P2,610.86 as its contribution tothe general average. - The insurance company, insisting that its obligationdid not extend beyond the insurance of the absolutetotal loss of the vessel only, and to pay proportionatesalvage of the declared value, refused to contributeto the settlement of the gen. ave. The present actionwas thereupon instituted, and after trial the courtbelow rendered judgment in favor of the plaintiff andordered the defendant to pay the plaintiff P2,610.86as its part of the indemnity for the gen. ave. broughtabout by the jettison of cargo. The insurancecompany then appealed to the SC. - The insurance contract is printed in the Englishcommon form of marine policies. One of the clausesof the document originally read as follows: Touching the Adventures and Perils which thesaid NUFIC is content to bear, and to take uponthem in this Voyage; they are of the Seas, Men-of-War, Fire, Pirates, Thieves, Jettison, Letters ofMart and Countermart, Surprisals, and Takings at Sea. Arrests, Restraints and Detainments, of allKings, Princes and People of what Nation,Condition or Quality soever; Barratry of the Masterand Marines, and of all other Perils, Losses andMisfortunes, that have or shall come to the Hurt,Detriment, or Damage of the said Vessel or anypart thereof; and in case of any Loss orMisfortunes, it shall be lawful for the Assured, hisor their Factors, Servants, or assigns, to sue,labour and travel for, in and about the Defence.Safeguard, and recovery of the said Vessel or anypart thereof, without Prejudice to this Insurance;to the Charges whereof the said Company, willcontribute, according to the rate and quantity ofthe sum herein assured... - Attached to the policy over and above the saidclause is a rider containing typewritten provisions,among which appears in capitalized type thefollowing clause: AGAINST THE ABSOLUTE TOTAL LOSS OF THEVESSEL ONLY, AND TO PAY PROPORTIONATESALVAGE CHARGES OF THE DECLARED VALUE. ISSUES 1. WON the lower court erred in disregarding thetypewritten clause endorsed upon the policy,expressly limiting insurer's liability thereunder of thetotal loss of the wooden vessel Pandan and toproportionate salvage charges 2. WON lower court erred in concluding thatdefendant and appellant, NUFIC is liable tocontribute to the general average resulting from thejettison of a part of said vessel's cargo HELD 1. NO Ratio In case repugnance exists between written and printed portions of a policy, the written portion prevails. Reasoning - Section 291 of the Code of Civil Procedure provides that when an instrument consists partly of writtenwords and partly of a printed form and the two areinconsistent, the former controls the latter. 2. NO Ratio The liability for contribution in general average is not based on the express terms of the policy, butrests upon the theory that from the relation of theparties and for their benefit, a quasi contract isimplied by law. Reasoning - In the absence of positive legislation to thecontrary, the liability of the defendant insurancecompany on its policy would, perhaps, be limited toabsolute loss of the vessel only, and to payproportionate salvage of the declared value. But thepolicy was executed in this jurisdiction andwarranted to trade within the waters of thePhilippine Archipelago only. Here, Art. 859 of theCode of Commerc e is still in force: ART. 859. The underwriters of the vessel, of thefreight, and of the cargo shall be obliged to pay forthe indemnity of the gross average in so far as isrequired of each one of these objects respectively.

- The article is mandatory in its terms, and theinsurers (whether for the vessel or for the freight orfor the cargo) are bound to contribute to theindemnity of the general average. The provisionsimply places the insurer on the same footing asother persons who have an interest in the vessel, orthe cargo therein, at the time of the occurrence ofthe general average and who are compelled tocontribute (Art. 812, Code of Commerce). - In the present case it is not disputed that the shipwas in grave peril and that the jettison of part of thecargo was necessary. If the cargo was in peril to theextent of call for general average, the ship must alsohave been in great danger, possibly sufficient tocause its absolute loss. The jettison was therefore asmuch to the benefit of the underwriter as to theowner of the cargo. The latter was compelled tocontribute to the indemnity; why should not theinsurer be required to do likewise? If no jettison hadtaken place and if the ship by reason thereof hadfoundered, the underwriter's loss would have beenmany times as large as the contribution nowdemanded. Disposition Appealed judgment is affirmed

GO TIACO v. UNION INSURANCE 40 PHIL 40 STREET; September 1, 1919 FACTS - Union Insurance Society of Canton, Ltd., issued amarine insurance policy upon a cargo of ricebelonging to the Go Tiaoco Brothers, which wastransported in the early days of May, 1915, on thesteamship Hondagua from the port of Saigon to Cebu. - On discharging the rice from one of thecompartments in the after hold, upon arrival atCebu, it was discovered that 1473 sacks had beendamaged by sea water. The loss was P3,875.25. - The trial court found that the inflow of the seawater during the voyage was due to a defect in oneof the drain pipes of the ship and concluded that theloss was not covered by the policy of insurance. Thetrial court made the ff findings: The drain pipe which served as a dischargefrom the water closet passed down throughthe compartment where the rice in questionwas stowed and thence out to sea through thewall of the compartment, which was a part ofthe wall of the ship. The joint or elbow wherethe pipe changed its direction was of castiron; and in course of time it had becomecorroded and abraded until a longitudinalopening had appeared in the pipe about oneinch in length. This hole had been in existencebefore the voyage was begun, and an attempthad been made to repair it by filling withcement and bolting over it a strip of iron. Theeffect of loading the boat was to submergethe vent, or orifice, of the pipe until it wasabout 18 inches or 2 feet below the level ofthe sea. As a consequence the sea water rosein the pipe. Navigation under these conditionsresulted in the washing out of the cement-filling from the action of the sea water, thuspermitting the continued flow of the salt waterinto the compartment of rice. - The court found in effect that the opening abovedescribed had resulted in course of time fromordinary wear and tear and not from the straining ofthe ship in rough weather on that voyage. The courtalso found that the repairs that had been made onthe pipe were slovenly and defective and that, byreason of the condition of this pipe, the ship was notproperly equipped to receive the rice at the time thevoyage was begun. For this reason the court heldthat the ship was unseaworthy. - The policy purports to insure the cargo from thefollowing among other risks: "Perils . . . of the seas,men, of war, fire, enemies, pirates, rovers, thieves,.jettisons, . . . barratry of the master and mariners,and of all other perils, losses, and misfortunes thathave or shall come to the hurt, detriment, or damageof the said goods and merchandise or any part thereof." ISSUE WON Union Insurance is liable for the loss of the Go Tiaco Brothers HELD NO - the words "all other perils, losses, and misfortunes"are to be interpreted as covering risks which are oflike kind (ejusdem generis) with the particular riskswhich are enumerated in the preceding part of thesame clause of the contract. ''According to theordinary rules of construction, these words must beinterpreted with reference to the words whichimmediately precede them. They were no doubtinserted in order to prevent disputes founded on nicedistinctions. X x x For example, if the expression'perils of the seas' is given its widest sense thegeneral words have little or no effect as applied tothat case. If on the other hand that expression is toreceive a limited construction, as apparently it did inCullen vs. Butler (5 M. & S., 461), and loss by perilsof the seas is to be confined to loss ex marinetempestatis discrimine, the general words becomemost important. X x x" (Thames and Mersey MarineInsurance Co. vs. Hamilton, Fraser & Co.) - a loss which, in the ordinary course of events,results from the natural and inevitable action of thesea, from the ordinary wear and tear of the ship, orfrom the negligent failure of the ship's owner toprovide the vessel with proper equipment to conveythe cargo under ordinary conditions, is not a peril of the sea. Such a loss is rather due to what has been aptly called the "peril of the ship." The insurerundertakes to insure against perils of the sea andsimilar perils, not against perils of the ship. Theremust, in order to make the insurer liable, be "somecasualty, something which could not be foreseen asone of the necessary incidents of the adventure. Thepurpose of the policy is to secure an indemnityagainst accidents which may happen, not againstevents which must happen." (Wilson, Sons & Co. vs.Owners of Cargo per the Xantho) - In the present case the entrance of the sea waterinto the ship's hold through the defective pipealready described was not due to any accident whichhappened during the voyage, but to the failure of theship's owner properly to repair a defect of theexistence of which he was apprised. The

loss wastherefore more analogous to that which directlyresults from simple unseaworthiness than to thatwhich results from perils of the sea. - there is no room to doubt the liability of theshipowner for such a loss as occurred in this case. Byparity of reasoning the insurer is not liable; for,generally speaking, the shipowner excepts the perilsof the sea from his engagement under the bill oflading, while this is the very peril against which theinsurer intends to give protection. As applied to thepresent case it results that the owners of thedamaged rice must look to the shipowner for redressand not to the insurer. The same conclusion must be reached if the questionbe discussed with reference to the seaworthiness ofthe ship. It is universally accepted that in everycontract of insurance upon anything which is thesubject of marine insurance, a warranty is impliedthat the ship shall be seaworthy at the time of theinception of the voyage. This rule is accepted in ourown Insurance Law (Act No. 2427, sec. 106). It isalso well settled that a ship which is seaworthy forthe purpose of insurance upon the ship may yet beunseaworthy for the purpose of insurance upon thecargo (Act No. 2427, sec. 106). Disposition Decision of trial court is affirmed

CATHAY INSURANCE CO. v. CA (REMINGTON INDUSTRIAL SALES CORP.) 151 SCRA 710 PARAS; June 30 1987 FACTS - Remington Industrial Sales Corp insured itsshipment of seamless steel pipes.It incurred lossesand damages (I gather the steel pipes rusted duringthe voyage from Japan to the Phils. on board vesselSS "Eastern Mariner) and filed complaint againstCathay Insurance Co seeking collection of the sum ofP868,339.15 - TC decided for Remington. Cathay filed MR, which was denied. CA affirmed. -CA said (among other things): 1. Coverage ofprivate respondent's loss under the insurance policyissued by petitioner is unmistakable; 2. Alleged contractual limitations contained ininsurance policies are regarded with extreme cautionby courts and are to be strictly construed against theinsurer; obscure phrases and exceptions should notbe allowed to defeat the very purpose for which thepolicy was procured; 3. Rust is not an inherent vice of the seamless steel pipes without interference of external factors -Cathay contend (among other things): 1. private respondent has admitted that the questioned shipment is not covered by a "square provision of thecontract," but private respondent claims impliedcoverage from the phrase "perils of the sea"mentioned in the opening sentence of the policy;2.The insistence of private respondent that rusting is aperil of the sea is erroneous;3.Rusting is not arisk insured against, since a risk to be insuredagainst should be a casualty or some casualty,something which could not be foreseen as one of thenecessary incidents of adventure;4. A fact capableof unquestionable demonstration or of publicknowledge needs no evidence. This fact ofunquestionable demonstration or of public knowledgeis that heavy rusting of steel or iron pipes cannotoccur within a period of a seven (7) day voyage.Besides, petitioner had introduced the clear cargoreceipts or tally sheets indicating that there was nodamage on the steel pipes during the voyage. ISSUE WON rusting is a peril of the sea HELD YES - There is no question that the rusting of steel pipesin the course of a voyage is a "peril of the sea" inview of the toll on the cargo of wind, water, and saltconditions. At any rate if the insurer cannot be heldaccountable therefor, We would fail to observe acardinal rule in the interpretation of contracts,namely, that any ambiguity therein should beconstrued against the maker/issuer/drafter thereof,namely, the insurer. Besides the precise purpose ofinsuring cargo during a voyage would be renderedfruitless. Disposition WHEREFORE, this petition is hereby DENIED, and the assailed decision of the Court of Appeals is hereby AFFIRMED. ROQUE v. IAC (PIONEER INSURANCE AND SURETY CORP.) 139 SCRA 596 GUTIERREZ; November 11, 1985 NATURE Petition for certiorari to review the decision of the IAC FACTS - February 19, 1972 Common carrier Manila BayLighterage Corp. entered into a contract with RoqueTimber Enterprises and Chiong.The contract stated that Manila Bay would carry 422.18 cu. meters oflogs on its vessel Mable 10 from Malampaya Sound,Palawan to Manila North Harbor.Roque insured thelogs with Pioneer Insurance for P100,000. - February 29, 1972 811 logs were loaded in Malampaya but en route to Manila, Mable 10 sank. - March 8,1972 Roque and Chiong wrote a letter toManila Bay, demanding payment of P150,000.00 forthe loss of the shipment plus P100,000.00 asunrealized profits but the latter ignored the demand.A letter was also sent to Pioneer, claiming the fullamount of P100,000.00 under the insurance

policybut Pioneer refused to pay on the ground that itsliability depended upon the "Total Loss by Total Lossof Vessel only". - After hearing, the trial court favored Roque.Pioneer and Manila Bay were ordered to pay RoqueP100,000.Pioneer appealed the decision. - January 30, 1984 Pioneer was absolved fromliability after finding that there was a breach ofimplied warranty of seaworthiness on the part of thepetitioners and that the loss of the insured cargo wascaused by the "perils of the ship" and not by the"perils of the sea". It ruled that the loss is notcovered by the marine insurance policy. - It was alleged that Mable 10 was not seaworthy and that it developed a leak - The IAC found that one of the hatches was leftopen, causing water to enter the barge and becausethe barge was not provided with the necessary coveror tarpaulin, the splash of sea waves brought morewater inside the barge. - Petitioners contend that the implied warranty ofseaworthiness provided for in the Insurance Coderefers only to the responsibility of the shipowner whomust see to it that his ship is reasonably fit to makein safety the contemplated voyage. - The petitioners state that a mere shipper of cargo,having no control over the ship, has nothing to dowith its seaworthiness. They argue that a cargoowner has no control over the structure of the ship,its cables, anchors, fuel and provisions, the mannerof loading his cargo and the cargo of other shippers,and the hiring of a sufficient number of competentofficers and seamen. ISSUE WON the loss should have been covered by the marine insurance policy HELD NO RatioIt is universally accepted that in every contract of insurance upon anything which is the subject ofmarine insurance, a warranty is implied that the shipshall be seaworthy at the time of the inception of thevoyage. In marine insurance, the risks insuredagainst are classified as 'perils of the sea, whichincludes such losses that are of extraordinary nature,or arise from some overwhelming power, whichcannot be guarded against by the ordinary exertionof human skill and prudence. Reasoning - Based on Sec. 113 and Sec. 99 of the InsuranceCode, the term "cargo" can be the subject of marineinsurance and that once it is so made, the impliedwarranty of seaworthiness immediately attaches towhoever is insuring the cargo whether he be theshipowner or not. - The fact that the un-seaworthiness of the ship wasunknown to the insured is immaterial in ordinarymarine insurance and may not be used by him as adefense in order to recover on the marine insurancepolicy. - Since the law provides for an implied warranty ofseaworthiness in every contract of ordinary marineinsurance, it becomes the obligation of a cargoowner to look for a reliable common carrier whichkeeps its vessels in seaworthy condition. The shipperof cargo my have no control over the vessel but hehas full control in the choice of the common carrierthat will transport his goods. - In marine cases, the risks insured against are'perils of the sea.The term extends only to lossescaused by sea damage, or by the violence of theelements, and does not embrace all losses happeningat sea. - It is quite unmistakable that the loss of the cargowas due to the perils of the ship rather than theperils of the sea. - Loss which, in the ordinary course of events,results from the natural and inevitable action of thesea, from the ordinary wear and tear of the ship, orfrom the negligent failure of the ship's owner toprovide the vessel with proper equipment to conveythe cargo under ordinary conditions, is not a peril ofthe sea but is called peril of the ship. Disposition Decision appealed from is affirmed.

LA RAZON v. UNION INSURANCE SOCIETY OF CANTON, LTD. 40 PHIL 40 STREET; September 1, 1919 FACTS - This is an action on a policy of marine insuranceissued by the Union Insurance Society of Canton,Ltd., upon a cargo of rice belonging to the plaintiffs,Go Tiaoco Brothers, which was transported on thesteamship Hondagua from the port of Saigon to Cebu. - On discharging the rice from one of thecompartments in the after hold, upon arrival atCebu, it was discovered that 1,473 sacks had beendamaged by sea water. - The loss so resulting to the owners of rice, afterproper deduction had been made for the portionsaved, was P3,875.25. - The trial court found that the inflow of the seawater during the voyage was due to a defect in oneof the drain pipes of the ship and concluded that theloss was not covered by the policy of insurance.Judgment was accordingly entered in favor of thedefendant and the plaintiffs appealed. - The court found in effect that the opening abovedescribed had resulted in course of time fromordinary wear and tear and not from the straining ofthe ship in rough weather on that voyage. The courtalso found that the repairs that had been made onthe pipe were slovenly and defective and that, byreason of the condition of this pipe, the ship was notproperly equipped to receive the rice at the time thevoyage was begun. For this reason the court heldthat the ship was unseaworthy. ISSUE WON the insurer is liable HELD - The question whether the insurer is liable on thispolicy for the loss caused in the manner above statedpresents two phases which are in a manner involvedwith each other. One has reference to the meaning ofthe expression "perils of the seas and all other perils,losses, and misfortunes," as used in the policy; theother has reference to the implied warranty, on thepart of the insured, as to the seaworthiness of the ship. - The meaning of the expression "perils * * * of theseas * * * and all other perils, losses, andmisfortunes," used in describing the risks covered bypolicies of marine insurance, has been the subject offrequent discussion; and certain propositions relativethereto are now so generally accepted as to beconsidered definitely settled. - The words "all other perils, losses, andmisfortunes" are to be interpreted as covering riskswhich are of like kind (ejusdem generis) with theparticular risks which are enumerated in thepreceding part of the same clause of the contract. - A loss which, in the ordinary course of events,results from the natural and inevitable action of thesea, from the ordinary wear and tear of the ship, orfrom the negligent failure of the ship's owner toprovide the vessel with proper equipment to conveythe cargo under ordinary conditions, is not a peril ofthe sea. Such a loss is rather due to what has beenaptly called the "peril of the ship." The insurerundertakes to insure against perils of the sea andsimilar perils, not against perils of the ship. - As was said by Lord Herschell in Wilson, Sons & Co.vs. Owners of Cargo per the Xantho, there must, inorder to make the insurer liable, be "some casualty,something which could not be foreseen as one of thenecessary incidents of the adventure. The purpose ofthe policy is to secure an indemnity against accidentswhich may happen, not against events which musthappen." - In the present case the entrance of the sea waterinto the ship's hold through the defective pipealready described was not due to any accident whichhappened during the voyage, but to the failure of theship's owner properly to repair a defect of theexistence of which, he was apprised. The loss wastherefore more analogous to that which directlyresults from simple unseaworthiness than to thatwhich results from perils of the sea. - It is universally accepted that in every contract ofinsurance upon anything which is the subject ofmarine insurance, a warranty is implied that the shipshall be seaworthy at the time of the inception of thevoyage. This rule is accepted in our own InsuranceLaw (Act No. 2427, see. 106). It is also well settled that a ship which isseaworthy for the purpose of insurance upon the shipmay yet be unseaworthy for the purpose of insuranceupon the cargo (Act No. 2427, see. 106). Disposition Jjudgment affirmed.

MALAYAN INSURANCE v. CA (supra p.10) FILIPINO MERCHANTS INS. CO. v. CA (supra p.19) COASTWISE LIGHTERAGE CORP v. CA (PHILIPPINE GENERAL INSURANCE COMPANY) 245 SCRA 796 FRANCISCO; July 12, 1995 NATURE Petition for review of CA Decision affirming decisionof RTC Manila holding that Coastwise is liable to payPhilGen Insurance the amount of P700thou plus legalinterest thereon, another sum of P100thou asattorney's fees and the cost of the suit. FACTS - Pag-asa Sales, Inc. entered into a contract totransport molasses from the province of Negros toManila with Coastwise, using the latter's dumbbarges. The barges were towed in tandem by thetugboat MT Marica, also owned by Coastwise. Uponreaching Manila Bay, while approaching Pier 18, oneof the barges struck an unknown sunken object. Theforward buoyancy compartment was damaged, andwater gushed in through a hole "two inches wide andtwenty-two inches long." - As a consequence, the molasses at the cargo tankswere contaminated and rendered unfit for the use itwas intended. This prompted consignee Pag-asaSales to reject the shipment of molasses as a totalloss. Thereafter, Pag-asa Sales filed a formal claimwith the insurer of its lost cargo (PhilGen) andagainst the carrier (Coastwise). Coastwise denied theclaim and it was PhilGen which paid Pagasa Salesthe amount of P700k representing the value of thedamaged cargo of molasses. - PhilGen then filed an action against Coastwisebefore the RTC Manila, seeking to recover the P700kwhich it paid to Pag-asa Sales for the latter's lostcargo. PhilGen now claims to be subrogated to all thecontractual rights and claims which the consigneemay have against the carrier, which is presumed tohave violated the contract of carriage. - RTC awarded the amount prayed for by PhilGen. CA affirmed. Hence, this petition. ISSUES 1. WON Coastwise Lighterage was transformed into aprivate carrier, by virtue of the contract ofaffreightment which it entered into with theconsignee, Pag-asa Sales, Inc. (Corollarily, if it werein fact transformed into a private carrier, did itexercise the ordinary diligence to which a privatecarrier is in turn bound?) 2. WON the insurer was subrogated into the rights ofthe consignee against the carrier, upon payment bythe insurer of the value of the consignee's goods lostwhile on board one of the carrier's vessels HELD 1. NO - The distinction between the two kinds of charterparties (i.e. bareboat or demise and contract ofaffreightment) is more clearly set out in the case of Puromines, Inc. vs. Court of Appeals, wherein SC ruled: Under the demise or bareboat charter of thevessel, the charterer will generally be regardedas the owner for the voyage or servicestipulated. The charterer mans the vessel withhis own people and becomes the owner pro hac vice, subject to liability to others for damages caused by negligence. To create a demise, theowner of a vessel must completely andexclusively relinquish possession, command andnavigation thereof to the charterer,anything short of such a complete transfer is a contract ofaffreightment (time or voyage charter party) ornot a charter party at all. On the other hand a contract of affreightment isone in which the owner of the vessel leases partor all of its space to haul goods for others. It is acontract for special service to be rendered by theowner of the vessel and under such contract thegeneral owner retains the possession, commandand navigation of the ship, the charterer orfreighter merely having use of the space in thevessel in return for his payment of the charterhire... An owner who retains possession of the shipthough the hold is the property of the charterer,remains liable as carrier and must answer forany breach of duty as to the care, loading andunloading of the cargo.

- Although a charter party may transform a commoncarrier into a private one, the same however is nottrue in a contract of affreightment on account of theaforementioned distinctions between the two. - SC agrees with Coastwise's admission that thecontract it entered into with the consignee was oneof affreightment. Pag-asa Sales, Inc. only leasedthree of petitioner's vessels, in order to carry cargofrom one point to another, but the possession,command and navigation of the vessels remainedwith Coastwise. As such, Coastwise, by the contractof affreightment, was not converted into a privatecarrier, but remained a common carrier and was stillliable as such. - Therefore, the mere proof of delivery of goods in good order to a carrier and the subsequent arrival of the same goods at the place of destination in badorder makes for a prima facie case against thecarrier. The presumption of negligence that attachesto common carriers, once the goods it transports arelost, destroyed or deteriorated, applies to Coastwise.This presumption, which is overcome only by proof ofthe exercise of extraordinary diligence, remainedunrebutted in this case. - The damage to the barge which carried the cargo ofmolasses was caused by its hitting an unknownsunken object as it was heading for Pier 18. Theobject turned out to be a submerged derelict vessel.The evidence on record appeared that far fromhaving rendered service with the greatest skill andutmost foresight, and being free from fault, thecarrier was culpably remiss in the observance of itsduties. - Jesus R. Constantino, the patron of the vessel"Coastwise 9" admitted that he was not licensed. TheCode of Commerce, which subsidiarily governscommon carriers (which are primarily governed bythe provisions of the Civil Code) provides: Art. 609.Captains, masters, or patrons of vessels must beFilipinos, have legal capacity to contract inaccordance with this code, and prove the skillcapacity and qualifications necessary to commandand direct the vessel, as established by marine andnavigation laws, ordinances or regulations, and mustnot be disqualified according to the same for thedischarge of the duties of the position. - Clearly, Coastwise Lighterage's embarking on avoyage with an unlicensed patron violates this rule.It cannot safely claim to have exercisedextraordinary diligence, by placing a person whosenavigational skills are questionable, at the helm ofthe vessel which eventually met the fateful accident.It may also logically, follow that a person withoutlicense to navigate, lacks not just the skill to do so,but also the utmost familiarity with the usual andsafe routes taken by seasoned and legally authorizedones. Had the patron been licensed, he could bepresumed to have both the skill and the knowledgethat would have prevented the vessel's hitting thesunken derelict ship that lay on their way to Pier 18.- As a common carrier, Coastwise is liable for breachof the contract of carriage, having failed to overcomethe presumption of negligence with the loss anddestruction of goods it transported, by proof of itsexercise of extraordinary diligence. 2. YES - Coastwise is liable for breach of the contract of carriage it entered into with Pag-asa Sales, Inc. However, for the damage sustained by the loss of thecargo which petitioner-carrier was transporting, itwas not the carrier which paid the value thereof toPag-asa Sales, Inc. but the latter's insurer, hereinprivate respondent PhilGen. - Article 2207 of the Civil Code: If the plaintiffs property has been insured, and he has received indemnity from the insurance company for the injuryor loss arising out of the wrong or breach of contractcomplained of, the insurance company shall besubrogated to the rights of the insured against thewrongdoer or the person who violated the contract.- This legal provision is founded on the wellsettledprinciple of subrogation. If the insured property isdestroyed or damaged through the fault ornegligence of a party other than the assured, thenthe insurer, upon payment to the assured will besubrogated to the rights of the assured to recoverfrom the wrongdoer to the extent that the insurerhas been obligated to pay. Payment by the insurer to the assured operated as an equitable assignment tothe former of all remedies which the latter may haveagainst the third party whose negligence or wrongfulact caused the loss. The right of subrogation is not dependent upon, nor does it grow out of, any privityof contract or upon written assignment of claim. Itaccrues simply upon payment of the insurance claimby the insurer. - Undoubtedly, upon payment by respondent insurerPhilGen of the amount of P700,000.00 to PagasaSales, Inc., the consignee of the cargo of molassestotally damaged while being transported bypetitioner Coastwise Lighterage, the former wassubrogated into all the rights which Pag-asa Sales,Inc. may have had against the carrier, hereinpetitioner Coastwise Lighterage. Disposition Petition denied. CA affrimed.

THE PHILIPPINE AMERICAN GENERALINSURANCE COMPANY INC v. CA (FELMANSHIPPING LINES) 273 SCRA 226 BELLOSILLO; June 11, 1997 FACTS - Coca-Cola Bottlers Philippines, Inc., loaded onboard MV Asilda, a vessel owned and operated byFelman 7,500 cases of 1-liter Coca-Cola softdrinkbottles to be transported from Zamboanga City toCebu for consignee Coca-Cola Bottlers Philippines,Inc.,Cebu. The shipment was insured with petitionerPhilippine American General under Marine Open Policy. - The vessel sank in the waters of Zamboanga delNorte bringing down her entire cargo with herincluding the subject 7,500 cases of 1-liter Coca-Colasoftdrink bottles. - The consignee filed a claim with respondentFELMAN for recovery of damages it sustained as aresult of the loss of its softdrink bottles that sankwithMV Asilda. Respondent denied the claimthus prompting the consignee to file an insuranceclaim with PHILAMGEN which paid its claim ofP755,250.00. - Claiming its right of subrogation PHILAMGENsought recourse against respondent FELMAN whichdisclaimed any liability for the loss.Consequently,PHILAMGEN sued the shipowner for sum of moneyand damages. - PHILAMGEN alleged that the sinking and total lossof MV Asilda and its cargo were due to the vesselsunseaworthiness as she was put to sea in anun stable condition.It further alleged thatthevessel was improperly manned and that itsofficers were grossly negligent in failing to takeappropriate measures to proceed to a nearby port orbeach after the vessel started to list. - FELMAN filed a motion to dismiss based on theaffirmative defense that no right of subrogation infavor of PHILAMGEN was transmitted by the shipper,and that, in any event, FELMAN had abandoned allits rights, interests and ownership over MV Asildatogether with her freight and appurtenances for thepurpose of limiting and extinguishing its liabilityunder Art. 587 of the Code of Commerce. - Trial court dismissed the complaint of PHILAMGEN.On appeal the Court of Appeals set aside thedismissal and remanded the case to the lower courtfor trial on the merits.FELMAN filed a petition for certiorari with this Court but it was subsequently denied on 13 February 1989. - Trial court rendered judgment in favor of FELMAN.It ruled that MV Asilda was seaworthy when it leftthe port of Zamboanga as confirmed by certificatesissued by the Philippine Coast Guard and theshipowners surveyor attesting to its seaworthiness.Thus the loss of the vessel and its entire shipmentcould only be attributed to either a fortuitous event,in which case, no liability should attach unless therewas a stipulation to the contrary, or to the negligenceof the captain and his crew, in which case, Art. 587of the Code of Commerce should apply. - CA ruled that MV Asilda was unseaworthy for being top- heavy as 2,500 cases of Coca-Cola softdrink bottles were improperly stowed on deck.Nonetheless, the appellate court denied the claim ofPHILAMGEN on the ground that the assureds impliedwarranty of seaworthiness was not complied with.Perfunctorily, PHILAMGEN was not properlysubrogated to the rights and interests of the shipper.Furthermore, respondent court held that the filing ofnotice of abandonment had absolved theshipowner/agent from liability under the limitedliability rule. ISSUES 1. WON MV Asilda was seaworthy when it left the port of Zamboanga 2. WON the limited liability under Art. 587 ofthe Code of Commerce should apply 3. WON PHILAMGEN was properly subrogated to therights and legal actions which the shipper hadagainst FELMAN, the shipowner HELD

1. YES - MV Asildawas unseaworthy when it left the port of Zamboanga.We subscribe to the findings of theElite Adjusters, Inc., and the Court of Appeals thatthe proximate cause of the sinking of MV Asildawas its being top-heavy. Contrary to the ship captains allegations, evidence shows thatapproximately 2,500 cases of softdrink bottles werestowed on deck.Several days after MV Asildasank, an estimated 2,500 empty CocaCola plasticcases were recovered near the vicinity of the sinking.Considering that the ships hatches were properlysecured, the empty Coca-Cola cases recovered couldhave come only from the vessels deck cargo.It issettled that carrying a deck cargo raises thepresumption of unseaworthiness unless it can beshown that the deck cargo will not interfere with theproper management of the ship.However, in thiscase it was established that MV Asilda was notdesigned to carry substantial amount of cargo ondeck.The inordinate loading of cargo deck resultedin the decrease of the vessels metacentric heightthus making it unstable.The strong winds andwaves encountered by the vessel are but theordinary vicissitudes of a sea voyage and as suchmerely contributed to its already unstable andunseaworthy condition. 2. NO - The ship agent is liable for the negligent acts of thecaptain in the care of goodsloadedonthevessel.This liability however can be limited through abandonment of the vessel, its equipment andfreightage as provided in Art. 587.Nonetheless,there are exceptional circumstances wherein theship agent could still be held answerable despite theabandonment, as where the loss or injury was due tothe fault of the shipowner and the captain. Theinternational rule is to the effect that the right ofabandonment of vessels, as a legal limitation of ashipowners liability, does not apply to cases wherethe injury or average was occasioned by theshipowners own fault. 3. YES - The doctrine of subrogation has its roots in equity.It is designed to promote and to accomplish justiceand is the mode which equity adopts to compel theultimate payment of a debt by one who in justice,equity and good conscience ought to pay. Therefore,the payment made by PHILAMGEN to Coca-ColaBottlers Philippines, Inc., gave the former the right tobring an action as subrogee against FELMAN.Havingfailed to rebut the presumption of fault, the liabilityof FELMAN for the loss of the 7,500 cases of 1-literCoca-Cola softdrink bottles is inevitable. - Sec. 113 of the Insurance Code provides that (i)nevery marine insurance upon a ship or freight, orfreightage, or upon anything which is the subject ofmarine insurance, a warranty is implied that the shipis seaworthy.Under Sec. 114, a ship is seaworthywhen reasonably fit to perform the service, and toencounter the ordinary perils of the voyage,contemplatedbythe parties to the policy.Th us itbecomes the obligation of the cargo owner to look fora reliable common carrier which keeps itsvesselsinseaworthy condition.He may have no control overthevesselbuthe has full control in the selection ofthe common carrier that will transport his goods.Healso has full discretion in the choice of assurer thatwill underwrite a particular venture. - In policies where the law will generally imply awarranty of seaworthiness, it can only be excludedby terms in writing in the policy in the clearestlanguage. And where the policy stipulates that theseaworthiness of the vessel as between the assuredand the assurer is admitted, the question ofseaworthiness cannot be raised by the assurerwithout showing concealment or misrepresentationby the assured. - PHILAMGENs action against FELMAN is squarelysanctioned by Art. 2207 of the Civil Code whichprovides: Art. 2207.If the plaintiffs property has been insured, and he has received indemnity from the insurance company for the injury or lossarising out of the wrong or breach of contractcomplained of, the insurance company shallbe subrogated to the rights of the insuredagainst the wrongdoer or the person who hasviolated the contract.If the amount paid bythe insurance company does not fully coverthe injury or loss, the aggrieved party shallbe entitled to recover the deficiency from theperson causing the loss or injury. Disposition Petition is GRANTED.

RespondentFELMAN SHIPPING LINES is ordered to pay petitionerPHILIPPINE AMERICANGENERAL INSURANCE CO.,INC., Seven Hundred Fifty-five Thousand TwoHundred and Fifty Pesos (P755,250.00) plus legalinterest thereon counted from 29 November 1983,the date ofjudicial demand, pursuant to Arts. 2212and 2213 of the Civil Code. PHILIPPINE MFTG. CO. v. UNION INSURANCE SOCIETY OF CANTON 42 PHIL 378 JOHNS; November 22, 1921 FACTS - The plaintiffs steel tank lighter was insured by defendant company for absolute total loss. As aresult of a typhoon, the lighter sunk in Manila Bay.The plaintiff demanded payment from the defendantinsurance company but the latter refused. Thecompany asked the plaintiff to salvage the ship,which it was able to do so. - With the plaintiff able to raise the lighter,reconstruct it and placed it in commission, thedefendant insurance company claims that it was onlyliable for a total absolute loss and that there was nototal destruction of the lighter. - The trial court decided in favor of the defendant,saying that the policy only covered an actual totalloss, not a constructive total loss. ISSUES 1. WON there was an absolute total loss that can be covered by the policy 2. WON the Marine Law of Great Britain applies HELD 1. YES - At the time that the lighter was at the bottom ofthe bay, it was of no value to the owner, thus therewas an actual total loss. - The ship was sunk in July 1, 1918. After severalfutile attempts, it was finally raised on Sept. 20,1918. It is faitr to assume that in its then conditionmuch further time would be required to make thenecessary repairs and install the new machinerybefore it could again be placed in commission.During that time the owner would be deprived of theuse of its vessel or the interest on its investment.When those questions are considered the testimonyis conclusive that the cost of salvage, repair andreconstruction was more than the original cost of theship at the time the policy was issued. As found bythe trial court, t is difficult to see how there couldhave been a more complete loss of the vessel thanthat which actually occurred. Upon the facts shownhere, any other construction would nullify the statuteand as applied to the conditions existing in theManila Bay, this kind of policy would be worthless,and there would not be any consideration for thepremium. 2. NO - The defendant argues that the policy contains theprovision that it shall be o f as force and effect asthe surest writing or policy of insurance made inLondon. However, for such law to apply to our courtsthe existence of such law must be proven. It cannotapply when such proof is lacking. Nevertheless, inthe English practice, a ship is a total loss when shehas sustained such extensive damages that it wouldnot be reasonably practical to repair her. DispositionDecision reversed

CHOA TIEK SENG v. CA (FILIPINO MERCHANTS INSURANCE) 183 SCRA 223 GANCAYO; March 15, 1990 NATURE Appeal from a decision of the Court of Appeals FACTS - Petitioner imported some lactose crystals from Holland. - The importation involved fifteen (15) metric tonspacked in 600 6-ply paper bags with polytheleneinner bags, each bag at 25 kilos net. The goods wereloaded at the port at Rotterdam in sea vans on boardthe vessel "MS Benalder' as the mother vessel, andthereafter aboard the feeder vessel "Wesser BrokerV-25" of respondent Ben Lines Container, Ltd. (BenLines for short). The goods were insured by therespondent Filipino Merchants' Insurance Co., Inc.(insurance company for short) for the sum ofP98,882.35, the equivalent of US$8,765.00 plus 50%mark-up or US $13,147.50, against all risks underthe terms of the insurance cargo policy. Upon arrivalat the port of Manila, the cargo was discharged intothe custody of the arrastre operator respondent E.Razon, Inc. (broker for short), prior to the delivery topetitioner through his broker. Of the 600 bagsdelivered to petitioner, 403 were in bad order. Thesurveys showed that the bad order bags sufferedspillage and loss later valued at P33,117.63.Petitioner filed a claim for said loss dated February16, 1977 against respondent insurance company inthe amount of P33,117.63 as the insured value ofthe loss. - Respondent insurance company rejected the claimalleging that assuming that spillage took place whilethe goods were in transit, petitioner and his agentfailed to avert or minimize the loss by failing torecover spillage from the sea van, thus violating theterms of the insurance policy sued upon; and thatassuming that the spillage did not occur while thecargo was in transit, the said 400 bags were loadedin bad order, and that in any case, the van did notcarry any evidence of spillage. - Petitioner filed a complaint in the RTC against theinsurance company seeking payment of the sum ofP33,117.63 as damages plus attorney's fees andexpenses of litigation. Insurance company denied allthe material allegations of the complaint and raisedseveral special defenses as well as a compulsorycounterclaim. Insurance company filed a third-partycomplaint against respondents Ben Lines and broker.- RTC dismissed the complaint, the counterclaim andthe third-party complaint with costs against thepetitioner. Appealed in CA but denied. MFR wasdenied as well. ISSUE WON insurance company should be held liable even ifthe technical meaning in marine insurance of aninsurance against all risk" is applied HELD YES -In Gloren Inc. vs. Filipinas Cia. de Seguros, 12 itwas held that an all risk insurance policy insuresagainst all causes of conceivable loss or damage,except as otherwise excluded in the policy or due tofraud or intentional misconduct on the part of theinsured. It covers all losses during the voyagewhether arising from a marine peril or not, includingpilferage losses during the war. - In the present case, the "all risks" clause of the policy sued upon reads as follows: "5.This insurance is against all risks of loss ordamage to the subject matter insured but shall inno case be deemed to extend to cover loss,damage, or expense proximately caused by delayor inherent vice or nature of the subject matterinsured. Claims recoverable hereunder shall bepayable irrespective of percentage." - The terms of the policy are so clear and require nointerpretation. The insurance policy covers all loss ordamage to the cargo except those caused by delayor inherent vice or nature of the cargo insured. It isthe duty of the respondent insurance company toestablish that said loss or damage falls within theexceptions provided for by law, otherwise it is liabletherefor. - An "all risks" provision of a marine policy creates aspecial type of insurance which extends coverage torisks not usually contemplated and avoids puttingupon the insured the burden of establishing that theloss was due to peril falling within the policy'scoverage. The insurer can avoid coverage upondemonstrating that a specific provision expresslyexcludes the loss from coverage. - In this case, the damage caused to the cargo hasnot been attributed to any of the exceptions providedfor nor is there any pretension to this effect. Thus,the liability of respondent insurance company is clear. Disposition the decision appealed from is hereby REVERSED AND SET ASIDE and another judgment ishereby rendered ordering the respondent FilipinasMerchants Insurance Company, Inc. to pay the sumof P33,117.63 as damages to petitioner

with legalinterest from the filing of the complaint, plusattorney's fees and expenses of litigation in theamount of P10,000.00 as well as the costs of the suit.

FILIPINO MERCHANTS INS. CO. v. CA (supra p.19) ABOITIZ SHIPPING v. PHILAMGEN INSURANCE 179 SCRA 357 GANCAYCO; October 5, 1989 NATURE Petition for review on certiorari FACTS Marinduque Mining Industrial Corporation(Marinduque) shipped on board SS Arthur Maerskfrom Boston, U.S.A. a shipment of 1 skid cartonparts for valves. The shipment was ordered fromJamesbury, Singapore PTE, LTD., which issued thecargo's packing list and Invoice number showing thecontents of the carton. The Philippine Consulate inSingapore issued invoice for the shipment showingthe contents and its total price of $39,419.60 andthe freight and other charges of $2,791.73. Whenthe cargo arrived in Manila, it was received anddeposited in the office of Aboitiz ShippingCorporation (Aboitiz) for transhipment to Nonoc Island. - In July 1980, Marinduque, as consignee of thecargo, made a report that said cargo was pilfered onJuly 3, 1980 due toheavy rain at the Aboitizterminal and that of the total value of the cargo of$42,209.33, only $7,412.00 worth remains of thecargo with the recommendation that the claim bemade against Aboitiz. - The services of the Manila Adjusters and SurveyorsCo. (Manila Adjusters) were engaged by the Phil-American General Insurance Co., Inc. (Phil Am)which came out with the report that the cargo inquestion, when inspected, showed that it waspilfered. A confirmatory report was submitted by theManila Adjusters. - On August 11, 1980 Marinduque then filed a claimagainst Aboitiz in the amount of P246,430.80representing the value of the pilfered cargo. On thesame day Marinduque filed a claim for the sameamount against the Phil-Am on the latter's policy.Phil-Am paid Marinduque the sum of P246,430.80 asinsurer of the cargo. - Phil-Am then filed a complaint in RTC Manilaagainst Aboitiz for recovery of same amount allegingthat it has been subrogated to the rights ofMarinduque. Complaint dismissed and MFR denied.CA reversed. MFR thereof was denied. Hence, thispetition. WON petitioner Aboitiz was properly held liable to the private respondent Phil-Am by the appellate court HELD YES - The questioned shipment is covered by a continuingopen insurance coverage (which took effect afterSept. 1, 1975, as contained in Marine Open PolicyNo. 100184) from the time it was loaded aboard theSS Arthur Maersk in Boston, U.S.A. to the time it wasdelivered to the possession of petitioner at its officesat Pier 4 in Manila until it was pilfered when the greatmajority of the cargo was lost on July 3, 1980.Hence, petitioner Aboitiz was properly held liable toPhil-Am. Reasoning [a] Records of the case show that Phil-Am executed a continuous and open insurance coverage coveringgoods of Marinduque imported into and exportedfrom the Philippines which took effect after Sept. 1,1975, as contained in Marine Open Policy No.100184.A similar insurance coverage was alsoexecuted by petitioner in favor of Marinduque for allits goods shipped or moved within the territoriallimits of the Philippines also effective after Sept. 1,1975 and contained in Marine Open Policy No. 100185. [b] TC in dismissing the complaint apparently relied on Marine Risk Note No. 017545 issued by privaterespondent Phil-Am only on July 28, 1980 after theshipment in question was already pilfered. ObviouslyTC mistook said Marine Risk Note as an insurancepolicy when it is NOT. It is only an acknowledgmentor declaration of the private respondent confirmingthe specific shipment covered by its Marine OpenPolicy, the evaluation of the cargo and thechargeable premium. [c] The contention of the Aboitiz that it could not be liable for the pilferage of the cargo as it was stoleneven before it was loaded on its vessel is untenable.Aboitiz received cargo when it arrived in Manila at itsoffices, and it was while in its possession and beforeloading it in its vessel that the cargo was pilfered. Itsliability is clear. Disposition Petition DISMISSED.

ORIENTAL ASSURANCE v. CA (PANAMA SAW MILL) 200 SCRA 459 MELENCIO-HERRERA; August 9, 1991 NATURE Petition for review on certiorari FACTS Sometime in January 1986, private respondent Panama Sawmill Co., Inc. (Panama) bought, inPalawan, 1,208 pieces of apitong logs, with a totalvolume of 2,000 cubic meters. It hired TranspacificTowage, Inc., to transport the logs by sea to Manilaand insured it against loss for P1-M with petitionerOriental Assurance Corporation (Oriental Assurance).- While the logs were being transported, rough seasand strong winds caused damage to one of the twobarges resulting in the loss of 497 pieces of logs outof the 598 pieces loaded thereon. - Panama demanded payment for the loss butOriental Assurance refuse on the ground that itscontracted liability was for "TOTAL LOSS ONLY." - Unable to convince Oriental Assurance to pay itsclaim, Panama filed a Complaint for Damages againstOriental Assurance before the Regional Trial Court. - RTC ordered Oriental Assurance to pay Panamawith the view that the insurance contract should beliberally construed in order to avoid a denial ofsubstantial justice; and that the logs loaded in thetwo barges should be treated separately such thatthe loss sustained by the shipment in one of themmay be considered as "constructive total loss" andcorrespondingly compensable. CA affirmed in toto. ISSUE WON Oriental Assurance can be held liable under itsmarine insurance policy based on the theory of adivisible contract of insurance and, consequently, aconstructive total loss HELD NO - The terms of the contract constitute the measure ofthe insurer liability and compliance therewith is acondition precedent to the insured's right to recoveryfrom the insurer. Whether a contract is entire orseverable is a question of intention to be determinedby the language employed by the parties. The policyin question shows that the subject matter insuredwas the entire shipment of 2,000 cubic meters ofapitong logs. The fact that the logs were loaded ontwo different barges did not make the contractseveral and divisible as to the items insured. Thelogs on the two barges were not separately valued orseparately insured. Only one premium was paid forthe entire shipment, making for only one cause or consideration. The insurance contract must, therefore, be considered indivisible. - More importantly, the insurer's liability was for"total loss only." A total loss may be either actual orconstructive (Sec. 129, Insurance Code). An actualtotal loss is caused by: (a) A total destruction of the thing insured; (b) The irretrievable loss of the thing by sinking, or by being broken up; (c) Any damage to the thing which renders itvalueless to the owner for the purpose for whichhe held it; or (d) Any other event which effectively deprives theowner of the possession, at the port of destination,of the thing insured. (Section 130, Insurance Code). - A constructive total loss is one which gives to aperson insured a right to abandon, under Section139 of the Insurance Code. This provision reads: SECTION 139. A person insured by a contract ofmarine insurance may abandon the thing insured,or any particular portion thereof separately valued by the policy, or otherwise separately insured, and recover for a total loss thereof, when the cause of the loss is a peril injured against, (a) If more than three-fourths thereof in value isactually lost, or would have to be expended torecover it from the peril; (b) If it is injured to such an extent as to reduce its value more than three-fourths; xxx xxx xxx - The requirements for the application of Section 139of the Insurance Code, quoted above, have not beenmet. The logs involved, although placed in twobarges, were not separately valued by the policy, norseparately insured. Resultantly, the logs lost in thedamaged barge in relation to the total number oflogs loaded on the same barge cannot be made thebasis for determining constructive total loss.

The logshaving been insured as one inseparable unit, thecorrect basis for determining the existence ofconstructive total loss is the totality of the shipmentof logs. Of the entirety of 1,208, pieces of logs, only497 pieces thereof were lost or 41.45% of the entireshipment. Since the cost of those 497 pieces doesnot exceed 75% of the value of all 1,208 pieces oflogs, the shipment cannot be said to have sustaineda constructive total loss under Section 139(a) of theInsurance Code. Disposition judgment under review is SET ASIDE

PAN MALAYAN INSURANCE v. CA (THE FOODAND AGRICULTURAL ORGANIZATION OF THEUNITED NATIONS) 201 SCRA 382 REGALADO; September 5, 1991 FACTS - The Food and Agricultural Organization of theUnited Nations (hereinafter referred to as FAO),ntended and made arrangements to send toKampuchea 1,500 metric petitions of IR-36 certifiedrice seeds to be distributed to the people for seedlingpurposes - LUZTEVECO was to ship the cargo amounting toUS$83,325.92 in respect of one lot of 1,500 metricpetitions winch is the subject of the present action.The cargo was loaded on board LUZTEVECO BargeNo. LC-3000 and consisted of 34,122 bags of IR-36certified rice seeds purchased by FAO from theBureau of Plant Industry for P4,602,270.00 - FAO secured insurance coverage in the amount ofP5,250,000.00 from petitioner, Pan MalayanInsurance Corporation - On June 16, 1980, FAO gave instructions toLUZTEVECO to leave for Vaung Tau, Vietnam todeliver the cargo which, by its nature, could notwithstand delay because of the inherent risks oftermination and/or spoilage. On the same date, theinsurance premiums on the shipment was paid byFAO petitioner - On June 26, 1980, FAO was advised of the sinkingof the barge in the China Sea, hence it informedpetitioner thereof and, later, formally filed its claimunder the marine insurance policy. On July 29, 1980,FAO was informed by LUSTEVECO of the recovery ofthe lost shipment, for which reason FAO formallyfiled its claim with LUZTEVECO for compensation ofdamage to its cargo - LUZTEVECO failed and refused to pay. Pan Malayanlikewise failed to pay for the losses and damagessustained by FAO by reason of its inability to recoverthe value of the shipment from LUZTEVECO - Pan Malayan claims that part of the cargo wasrecovered and thus the claim by FAO wasunwarranted. This is evidenced by two surveys uponthe cargo wherein it was found that only around 78%was lost. - FAO filed a civil case against both LUZTEVECO andPan Malayan. Trial court found in favor of FAO andordered both to pay jointly and severally the fullamount of the claim. This was affirmed by CA ISSUE 1. WON respondent court committed a reversibleerror in holding that the trial court is correct inholding that there is a total loss of the shipment HELD 1. NO - The law classifies loss into either total or partial.Total loss may be actual or absolute, or it mayotherwise be constructive or technical. Petitionersubmits that respondent court erred in ruling thatthere was total loss of the shipment despite the factthat only 27,922 bags of rice seeds out of 34,122bags were rendered valueless to FAO and theshipment sustained only a loss of 78%. FAO,however, claims that, for all intents and purposes, ithas practically lost itstotal or entire shipment in thiscase, inclusive of expenses, premium fees, and soforth, despite the alleged recovery by defendantLUZTEVECO. As found by the court below andreproduced with approval by respondent court, FAO"has never been compensated for thistotal loss ordamage, a fact which is not denied nor controverted - If there were some cargoes saved, by LUZTEVECO,private respondent abandoned it and the same wassold or used for the benefit of LUZTEVECO or PanMalayan Corporation. Under Sections 129 and 130 ofthe New Insurance Code, a total loss may either beactual or constructive. In case of total loss in Marine Insurance, the assured is entitled to recover from the underwriter the whole amount of his subscription - SEC. 130. An actual total loss is caused by: (c) Anydamage to the thing which renders it valueless to theowner for the purpose for which he held it; or (d) Any other event which effectively deprives theowner of the possession, at the port of destination ofthe thing insured. -as said and proven, the seeds were of fragilenature. And the wetting of said seeds affected thestate of seeds. Thus rendering them useless for FAO.Although there were bags which were recovered,these were stained and not in the same condition itwas brought in. in addition to this, FAO did notreceive any compensation for said recovered bags asthe same were distributed by LUZVETECO withoutauthorization of FAO

- the complete physical destruction of the subjectmatter is not essential to constitute an actual totalloss. Such a loss may exist where the form andspecie of the thing is destroyed, although thematerials of which it consisted still exist (GreatWestern Ins. Co. vs. Fogarty, N.Y., 19 Wall 640, 22 L. Ed. 216), as where the cargo by the process ofdecomposition or other chemical agency no longerremains the same kind of thing as before (Williamsvs. Cole, 16 Me. 207). - It is thus clear that FAO suffered actual total lossunder Section 130 of the Insurance Code, specificallyunder paragraphs (c) and (d) thereof, recompensefor which it has been denied up to the present -Section 135 of the Insurance Code explicitlyprovides that "(u)pon an actual total loss, a personinsured is entitled to payment without notice ofabandonment." This is a statutory adoption of a longstanding doctrine in maritime insurance law that incase of actual total loss, the right of the insured toclaim the whole insurance is absolute, without needof a notice of abandonment

PHILIPPINE AMERICAN LIFE INSURANCE COMPANY v. CA (ELIZA PULIDO) 344 SCRA 260 GONZAGA-REYES; November 15, 2000 NATURE This petition for review on certiorari seeks to reversethe Decision of the Special Second Division of theCourt of Appeals FACTS - On January 9, 1989, petitioner received from oneFlorence Pulido an application for life insurance,dated December 16, 1988, in the amount ofP100,000.00 which designated her sister, hereinprivate respondent, as its principal beneficiary.Because the insurance applied for was nonmedical,petitioner did not require a medical examination andissued a policy on the sole basis of the application onFebruary 11, 1989. On April 1992, petitionerreceived private respondents claim, which declaredthat the insured, Florence Pulido, died of acutepneumonia on September 10, 1991. - Petitioner withheld payment on the ground that thepolicy claimed under was void from the start forhaving been procured in fraud.It is petitionerscontention that even before they received privaterespondents claim for death benefits, theirinvestigation concerning the subject policy yieldedthe information that the insured, Florence Pulido,died in 1988, before the application for insurance onher life was made. While this was communicated toprivate respondent in a letter dated April 29, 1992,private respondent had already filed her claim earlierthat month. In another letter dated July 27, 1992, however, petitioner confirmed to private respondentreceipt of the claim papers and assured her that hercase was being given preferential attention andprompt action. - Following the filing by private respondent of herclaim, petitioner caused another investigationrespecting the subject policy. Pursuant to thefindings of this second investigation, petitioner stoodby its initial decision to treat the policy as void andnot to honor the claim.On November 9, 1992,private respondent enlisted the services of counsel inreiterating her claim for death benefits. Petitionerstill refused to make payment and thus, this action. -Petitioner: the results of its investigations havingindicated that the insured was already dead at thetime the policy was applied for. It alsocounterclaimed for attorneys fees.The first report,prepared by one Dr. Benedicto Briones, was datedApril 1, 1992, and had attached to it a questionnaire,responded to by one Ramon Piganto, whorepresented to be the brother-in-law of the insuredand the barangay chairman of Cardiz, Bagulin, LaUnion. To the question Where does [FlorencePulido] reside now?, Piganto had replied thatFlorence Pulido used to live in Cardiz, but was deadsince 1988.Pigantos statement was signed by him,and witnessed by his wife, Nenita Piganto. Thisreport was petitioners basis for treating the disputedpolicy as void since April 1992, even before receipt ofprivate respondents claim. ISSUE WON there was fraud (whether the insured, FlorencePulido, was in fact dead before the application forinsurance on her life was made) HELD NO - This the lower courts had effected ruled on, upon apreponderance of the evidence duly received fromboth parties.We see no reversible error in thefinding of both respondent court and the trial court infavor of the correctness of the entries in Certificateof Death, duly registered with the Local CivilRegistrar of Bagulin, La Union, which declared thatFlorence Pulido died of acute pneumonia onSeptember 10, 1991. Dr. Irineo Gutierrez, theMunicipal Health Officer of Bagulin, La Union whosesignature appeared in the death certificate, testifiedin addition that he ministered to the ailing FlorencePulido for two days immediately prior to her death.This fact is likewise noted in the death certificate. - Death certificates, and notes by a municipal healthofficer prepared in the regular performance of hisduties, are prima facie evidence of facts thereinstated. A duly-registered death certificate

isconsidered a public document and the entries foundtherein are presumed correct, unless the party whocontests its accuracy can produce positive evidenceestablishing otherwise. Petitioners contention thatthe death certificate is suspect because Dr. Gutierrezwas not present when Florence Pulido died, andknew of Florences death only through RamonPiganto, does not merit a conclusion of fraud.Nomotive was imputed to Dr. Gutierrez for seeking toperpetuate a falsity in public records.Petitioner waslikewise unable to make out any clear motive as towhy Ramon Piganto would purposely lie. Mereallegations of fraud could not substitute for the fulland convincing evidence that is required to prove it.A failure to do so would leave intact the presumptionof good faith and regularity in the performance ofpublic duties, which was the basis of both respondentcourt and the trial court in finding the date ofFlorence Pulid os death to be as plaintiffprivaterespondent maintained. - We cannot likewise give credence to petitionerssubmission that the inconsistencies in thetestimonies of the witnesses for plaintiff-privaterespondent are in themselves evidence of fraud.Such alleged inconsistencies are matters of credibilitywhich had been ably passed upon by the lower court.- The absence of fraud, as a factual finding of thelower court adopted by the Court of Appeals, entirelyconsistent with the evidence on record, will not bereversed and, hence, is final and conclusive upon thisCourt. DispositionThe instant petition is DENIED

CHAPTER IX. CLAIMS SETTLEMENT & SUBROGATION LONDRES v. NATIONAL LIFE INSURANCE 94 PHIL 627 BAUTISTA ANGELO; March 29, 1954 NATURE Appeal from a decision of the Court of First Instanceof Manila ordering defendant to pay to plaintiff thesum of P3,000, Philippine currency, plus legalinterest thereon from the time of the filing of thecomplaint until its full payment. FACTS - On April 14, 1943, the National Life InsuranceCompany of the Philippines issued a policy on the lifeof Jose C. Londres whereby it undertook to pay itsbeneficiary upon his death the sum of P3,000. All thepremiums due under the policy were actually paid ontheir dates of maturity and the policy was in forcewhen the insured died on February 7, 1945.Salvacion V. Londres, as beneficiary, demanded fromthe company the payment of the proceeds of thepolicy, and her demand having been refused, sheinstituted the present action against the company inthe Court of First Instance of Manila. - Defendant in its answer denied, for lack ofsufficient proof, the allegation that the insured diedon February 7, 1945, and set up the following specialdefenses: (a) that plaintiff's claim is covered by theMoratorium Law; (b) that the policy having beenissued during the Japanese occupation, it ispresumed that its face value should be paid inJapanese currency, there being no provision in thepolicy from which can be inferred that the partiescontemplated payment in any other currency; (c)that the money paid by the insured as premiums,together with the money received from other policyholders, was all deposited by the defendant in thePhilippine National Bank and said deposit wasdeclared without value by Executive Order No. 49 ofthe President of the Philippines; and (d) that thepolicy having been issued under abnormalcircumstances, it should be considered in the light ofequity which does not permit anyone to enrichhimself at the expense of another. Defendant,however, as a proof of good faith, offered to pay thevalue of the policy in accordance with the Ballantynescale of values, or the sum of P2,400, Philippinecurrency. - It appears that the deceased took up the policyunder consideration on April 15, 1943 for the sum ofP3,000. All the premiums due under the policy wereactually paid on their dates of maturity and thepolicy was in force when the insured died onFebruary 7, 1945. On said date, the battle of theliberation of the City of Manila was still raging. Whilethe northern part may have been liberated, not sothe southern part, as shown from the very affidavitssubmitted by appellee wherein it was stated that onthe aforesaid date, the insured, Jose Londres, andhis two sons were taken by the Japanese soldiersfrom their house at Singalong Street and weremassacred by their captors. It may therefore be saidthat the policy became due when the City of Manilawas still under the yoke of the enemy and became payable only after liberation which took place onMarch 10, 1945 when President Osmena issuedProclamation No. 6 following the restoration of thecivil government by General Douglas Mac Arthur. Andwe say that the policy became payable only afterliberation even if it matured sometime before,because before that eventuality the insurancecompany, appellant herein, was not yet in a positionto pay the value of the policy for the simple reasonthat it had not yet reopened. ISSUE WON the amount of P3,000 which appellant bounditself to pay to the insured under the policy upon hisdeath should be paid in accordance with the presentcurrency or should be adjusted under the Ballantynescale of values HELD YES, present currency. Reasoning - In the case of Rutter vs. Esteban, 93 Phil., 68, theMoratorium Law was declared invalid andunconstitutional. - During those days of liberation, while the peoplewere rejoicing because of the happy event, thebanks, the insurance companies, and for that matterother commercial and business firms, were stillfeeling the adverse effects of the sudden fall ofvalues and were uncertain and apprehensive as tothe manner the readjustment would be made by thenew Government. It is for this reason that

thebeneficiary, after realizing the truth about the deathof her husband, and after gathering evidence tosubstantiate his death, had difficulty in effecting thecollection of her claim from the insurance companybecause at that time it had not yet reopened forbusiness purposes. Although the record does notdisclose the exact date on which the insurancecompany reopened for this purpose, this Court cantake judicial notice that it only did so after liberation. At that time the legal tender was already the present currency. - As final plea, appellant invokes equity in its favor inview of the nullification of the deposits made by itwith the Philippine National Bank of all fiat moneyreceived from its policyholders, which money wasdeclared without value by Executive Order No. 49 ofthe President of the Philippines. Appellant claimsthat, considering the unexpected circumstances thatdeveloped, the indemnity to be paid by it should besuffered by it under Article 307 of the Code of Commerce which provides: "When the deposits areof cash, with a specification of the coins constitutingthem, . . . the increase or reduction which their valuemay suffer shall be for the account of the depositor." Appellant, by entering into an insurancecontract, cannot claim, if it suffers loss, thatthe beneficiary cannot enrich herself at itsexpense. This is a risk attendant to any wagering contract. One who gambles and loses cannot beheard to complain of his loss. To appellant, we canonly repeat the following admonition: "The parties herein gambled and speculated on thedate of the termination of the war and theliberation of the Philippines by the Americans. Thiscan be gleaned from the stipulation aboutredemption, particularly that portion to the effectthat redemption could be effected not before theexpiration of one year from June 24, 1944. Thiskind of agreement is permitted by law. We findnothing immoral or unlawful in it." (Gomezvs. Tabia) Disposition Wherefore, the decision appealed from is affirmed, with costs against appellant.

VDA. DE FERNANDEZ v. NATIONAL LIFE INSURANCE CO OF THE PHILS 105 PHIL 59 ENDENCIA; January 27, 1959 NATURE Appeal from CFI decision applying the Ballantynescale of values upon the proceeds of life insurancetaken and maturing during the Japanese occupationbut claimed after liberation FACTS - National Life Insurance Company (NLIC) insured J.Fernandezs life for P10,000 upon his payment ofP444 from July 15, 1944 to July 14, 1945 - The insured died on November 2, 1944, while the policy was in force - After more than 7 years, in 1952, Atty de la Torre,representing the benficiaries of the policy, informedthe company that Fernandez had died in 1944, andclaimed the proceeds of the policy. The company saidthat the status of the policies issued during theJapanese occupation was still pending considerationbefore the courts. NLIC said that because the policy matured upon the insureds death in November,19 44, they should compute the value of their claimunder the Ballantyne scale of values (which wouldamount only to P500) - beneficiaries commenced suit, and the lower courtsustained the stand of the company, dismissed thecomplaint. - beneficiaries maintain that the obligation of thecompany to pay accrued not upon the death ofFernandez, but only upon the receipt and approvalby the company, on proof of death of the insured,which was in 1954. The policy reads: National Life Insurance Company of the Philippine hereby agrees to pay at its Home Office, Manila,Ten Thousand Pesos to Juan D. Fernandez(hereinafter called the insured) on the 15th day of July, 1964, if the Insured is living and this Policy is in force, or upon receipt and approved at its Office of due proofs of the title of the claimant and of the prior death of the Insured while this Policy is in force to Teresa Duat Vda. De Fernandez, Maria T.and Manuela Fernandez, mother and sistersrespectively of the Insured (Hereinafter called theBeneficiary) subject to the right of the Insured tochange the beneficiary as stated on the secondpage of this Policy. - The above stipulation is apparently based on Sec.91-A of the Insurance Law which provides as follows:The proceeds of a life insurance policy shall be paidimmediately upon maturity of the policy, unless suchproceeds are made payable in installments or a as anannuity, in which case the installments or annuitiesshall be paid as they become due:Provided, however, That in case of a policy maturing by the death of the insured, the proceeds thereof shall be paid within sixty days after presentation of the claimand filing of the proof of the death of the insured.Refused to pay the claim within the time prescribedherein will entitle the beneficiary to collect intereston the proceeds of the policy for the duration of thedelay at the rate of six per centum per annum,unless such failure or refusal to pay is based on theground that the claim is fraudulent. . . . - Based on the foregoing provision of law and theaforequoted stipulation as well as on the allegationthat the filing of proof of death by the beneficiaries isa condition precedent of the demandability of theobligation of the insurer to pay the proceeds,appellants claim that they should be paid P10,000 inPhilippine currency and not under the Ballantynescale of values. ISSUE WON the policy matured upon the death of the insured HELD YES Ratio In life insurance, the policy matures either upon the expiration of the term set forth therein, orupon his death occuring at any time prior to theexpiration of such stipulated term, in which case, theproceeds are payable to his beneficiaries within sixtydays after their filing of proof of death. Reasoning - The sixty day period fixed by law within which to pay the proceeds after presentation of proof of deathIs merely procedural in nature, evidently todetermine the exact amount to be paid and theinterest thereon to which the beneficiaries may beentitled to collect in case of unwarranted refusal ofthe company to pay, and also to enable the

insurerto verify or check on the fact of death which it mayeven validly waive. It is the happening of thesuspensive condition of death that renders a lifepolicy matured, and not ht efiling of proof of deathwhich, as above stated, is merely procedural. Theinsured having died during the Japanese occupation,the proceeds of his policy should be adjustedaccordingly, for The rule is already settled thatwhere a debtor could have paid his obligation at anytime during the Japanese occupation, payment afterliberation must be adjusted in accordance with theBallantyne schedule (De Asis vs. Agdamag, amongother cases). (Collaboration is defined as the acts of working together in a joint project. DispositionJudgment affirmed

TIO KHE CHIO v. CA (EASTERN ASSURANCE & SURETY) 202 SCRA 119 FERNAN; September 30, 1991 FACTS - Petitioner Tio Khe Chio imported 1,000 bags offishmeal valued at $36,000.30 from Agro Impex,S.A. Dallas, Texas, U.S.A. The goods were insuredwith respondent EASCO and shipped on board theM/V Peskov, a vessel owned by Far Eastern ShippingCompany. When the goods reached Manila, theywere found to have been damaged by sea waterwhich rendered the fishmeal useless. Petitioner fileda claim with EASCO and Far Eastern Shipping. Both refused to pay. Whereupon, petitioner sued thembefore the then Court of First Instance of Cebu fordamages. EASCO, as the insurer, filed a counterclaimagainst the petitioner for the recovery of the unpaidinsurance premiums. - The trial court rendered judgment in favor ofpetitioner. The judgment became final as to EASCObut the shipping company appealed to the Court ofAppeals and was absolved from liability by the saidcourt. - The trial court, upon motion by petitioner, issued awrit of execution against EASCO. The sheriffenforcing the writ reportedly fixed the legal rate ofinterest at 12%. Respondent EASCO moved to quashthe writ alleging that the legal interest to becomputed should be 6% per cent per annum inaccordance with Article 2209 of the Civil Code. Thetrial court denied EASCO's motion. On appeal, theCourt of Appeals reversed the trial courts denial ofEASCOs motion and ruled that the applicableinterest is 6% per annum. Hence, this petition. ISSUE WON the applicable rate of interest is 12% per annum HELD NO - Sections 243 and 244 of the Insurance Code applyonly when there is an unjustified refusal orwithholding of payment on the insureds claim. Inthis case, EASCO's refusal to settle the claim to TioKhe Chio was based on some ground which, whilenot sufficient to free it from liability under its policy,nevertheless is sufficient to negate any assertion thatin refusing to pay, it acted unjustifiably. Simply put,the said provisions of the Insurance Code are notpertinent to the instant case. They apply only whenthe court finds an unreasonable delay or refusal inthe payment of the claims. - Circular No. 416 of the Central Bank, which raisedthe legal rate of interest from 6% to 12% per annumrefers only to loans or forbearances of money, goodsor credits and court judgments thereon but not tocourt judgments for damages arising from injury topersons and loss of property which does not involvea loan. Clearly, the applicable law is Article 2209 ofthe Civil Code. - And in the light of the fact that the contendingparties did not allege the rate of interest stipulated inthe insurance contract, the legal interest was properly pegged by the Appellate Court at 6% per cent. CATHAY INSURANCE v. CA(LUGAY) 174 SCRA 11 GRINO-AQUINO; June 5, 1989 FACTS - Petitioners are 6 insurance companies that issuedfire insurance policies for the total sum ofP4,000,000 to the Cebu Filipina Press owned byEmilia Chan Lugay.The fire policies described theinsured property as "stocks of Printing materials,papers and general merchandise usual to theAssured's trade" stored in a one-storey building ofstrong materials housing the Cebu Filipina Presslocated at UNNO Pres. Quirino cor.Don V. Sotto Sts.,Mabolo, Cebu City. The co-insurers were indicated ineach of the policies. All, except one policy(Paramount's), were renewals of earlier policiesissued for the same property. - On December 18, 1981, the Cebu Filipina Press wasrazed by electrical fire together with all the stocksand merchandise stored in the premises.On January15, 1982, Lugay submitted sworn Statements of Lossand Formal Claims to the insurers, through theiradjusters. She claimed a total loss of P4,595,000. - After nearly 10 months of waiting, she sued tocollect on December 15, 1982. The insurancecompanies denied liability, alleging violation ofcertain conditions of the policy, misdeclaration, andeven arson which was not seriously pressed for,come the pre-trial, the petitioners offered to pay50% of her claim, but she insisted on full recovery.

- Trial court rendered judgment in her favor orderingthe insurers to pay her a total of P4,000,000 asindemnity, P48,000 representing expenses of theplaintiff, a separate amount of 20% of theP4,000,000 representing fees of counsel, interests atthe rate of twice the ceiling being prescribed by theMonetary Board starting from the time when thecase was filed, and finally, with costs. CA affirmed. ISSUES 1. WON the insured's cause of action had already accrued before she filed her complaint 2. WON sufficient proofs of loss had been presented by the insured 3. WON the private respondents claim for loss was inflated 4. WON lower court erred in awarding damages to the private respondent in the form of interestequivalent to double the interest ceiling set by theMonetary Board 5. WON attorney's fees awarded were exorbitant HELD 1. YES - As the fire which destroyed the Cebu Filipina Pressoccurred on December 19, 1981 and the proofs ofloss were submitted from January 15, 1982 throughJune 21, 1982 in compliance with the adjusters'numerous requests for various documents, paymentshould have been made within 90 days thereafter(Sec 243), or on or before September 21, 1982.Hence, when the assured filed her complaint onDecember 15, 1982, her cause of action had alreadyaccrued. 2. YES - There is no merit in the petitioners' contention thatthe proofs of loss were insufficient because Lugayfailed to comply with the adjuster's request for thesubmission of her bank statements. Condition No. 13of the policy does not require the insured to produceher bank statements. Therefore, the insured was notobligated to produce them and the insurers had noright to ask for them. Condition No. 13 was preparedby the insurers themselves, hence, it should betaken most strongly against them. 3. NO - Both the trial court and the CA noted that theproofs were ample and more than enough fordefendant insurers to do a just assessmentsupporting the 1981 fire claim for an amountexceeding four million pesos. 4. NO - The award of double interest on the claim is lawfuland justified under Sections 243 and 244 of theInsurance Code which provide: Sec. 243 Refusal or failure to pay the loss ordamage within the time prescribed herein willentitle the assured to collect interest on theproceeds of the policy for the duration of the delayat the rate of twice the ceiling prescribed by theMonetary Board. Sec. 244 In case of any litigation for theenforcement of any policy or contract of insurance,it shall be the duty of the Commissioner or theCourt, as the case may be, to make a finding as towhether the payment of the claim of the insuredhas been unreasonably denied or withheld; and inthe affirmative case, the insurance company shall be adjudged to pay damages which shall consist ofattorney's fees and other expenses incurred by theinsured person by reason of such unreasonabledenial or withholding of payment plus interest oftwice the ceiling prescribed by the Monetary Boardof the amount of claim due the insured. - The petitioners' contention that the charging ofdouble interest was improper because nounreasonable delay in the processing of the fire claimwas proven is refuted by the trial court's explicitfinding that "there was a delay that was notreasonable in processing the claim and doingpayments". Under Section 244, a prima facieevidence of unreasonable delay in payment of theclaim is created by the failure of the insurer to paythe claim within the time fixed in both Sec. 242 and243 of the IC. - In view of the not insubstantial value of the privaterespondent's claims and the considerable time andeffort expended by them and their counsel inprosecuting these claims for the past 8 years,attorney's fees were properly awarded to the privaterespondents. 5. YES - An award equivalent to 10% of the proceeds of thepolicies would be more reasonable than the 20%awarded by the trial court and the CA. Disposition Decision of the CA AFFIRMED with MODIFICATION. NODA v. CRUZ

151 SCRA 227 FERNAN; June 22, 1987 NATURE Petition to review decision of the Insurance Commissioner FACTS - In 1977, Noda obtained from Zenith InsuranceCorporation 2 fire insurance policies: [1] No. F03724with a face value of P30k covering the goods andstocks in trade in his business establishment at themarket site in Mangagoy, Bislig, Surigao del Sur and[2] No. F-03734 with a face value in the aggregateamount of P100k and consisting of Item 1 for P40kon household furniture, fixtures, fittings and otherpersonal effects, and Item 2 for P60k on stocks intrade usual to petitioner's retail business situated in a two-storey building at 039 Barreda St., Mangagoy, Bislig, Surigao del Sur. - While both policies were in force, fire destroyedpetitioner's insured properties at the market site onSeptember 5, 1977 and at Barreda St. on November9, 1977. - When petitioner failed to obtain indemnity on hisclaims from Zenith, he filed a complaint with theInsurance Commission praying that Zenith beordered to pay him P130kj representing the value ofthe 2 policies insured by respondent with interest at12% per annum, plus damages, attorney's fees andother expenses of litigation. ... - Zenith interposed that petitioner had no cause ofaction; that Policy No. F-03724 was not in full forceand effect at the time of the fire because thepremium on the policy was not paid; that Zenith'sliability under Policy No. F-03734, if any, was limitedto P15,472.50 in view of the coinsurance; and thatpetitioner failed to substantiate his claim as to thevalue of the goods reputedly destroyed by fire. - While the case was pending, Zenith settledpetitioners fire loss claim under Item 1 of Policy No.03734 in the amount of P15,472.50. - Insurance Commissioner allowed petitioner torecover under said policy and ordered Zenith to payhim the amount of P20k with legal interest from thedate the complaint was filed, including P1k asattorney's fees but excluding the actual, moral andexemplary damages prayed for. As for petitioner'sclaim under Policy No. F-03734, she held that in viewof the payment of P15,472.50 to petitioner, Zenithhad fully discharged its liability under said policywhich covered furniture, fixtures, fittings and otherpersonal belongings of petitioner. - In allowing recovery under Policy No. F-03734,Commissioner placed much weight on the final reportprepared by Dela Merced Adjustment Corporation, anindependent fire, marine and casualty adjustercontracted by Zenith to investigate the claims of itsvarious policyholders. Said report concluded that"the sound value of P26,666.67 represented thewhole loss and damage" incurred by petitioner, butwith the application of the three-fourths loss clause,Zenith's liability was reduced to P20k. ISSUES 1. WON Insurance Commissioner erred in denyingpetitioner's demand for P60k under Item 2 of PolicyNo. F-03734 WON Insurance Commissioner erred in notawarding in favor of petitioner exemplary damagesfor Zenith's unjustified and wanton refusal to paypetitioner's claim under the said two insurancecontracts HELD 1. YES - To prove the existence of the stocks in tradecovered by Policy No. F-03734, petitioner offered histestimony and that of his wife as well asdocumentary exhibits. The foregoing evidence forpetitioner preponderantly showed the presence ofsome P590k worth of goods in his retail store duringthe fire of November 9, 1977. - While the insurer, and the Insurance Commissionerfor that matter, have the right to reject proofs of lossif they are unsatisfactory, they may not set up forthemselves an arbitrary standard of satisfaction.Substantial compliance with the requirements willalways be deemed sufficient. - Zenith introduced in evidence the final report onPolicy No. F-03734 submitted by its own adjuster,Dela Merced Adjustment Corporation. RespondentCommissioner however ignored such report,reasoning that with regard to Item 2 of Policy No. F-03734 the claim for loss of the stocks in trade wasnot successfully proven in view of petitioner's failureto present evidence; that the adjuster's report

deserved scant consideration since the allegationstherein were not substantiated, and that said reportdid not even make a recommendation for payment.- A scrutiny of the abovementioned adjuster's reportreveals that together with the formal demand for fullindemnity, petitioner submitted his income tax returnfor 1978, purchase invoices, certification from hissuppliers as to his purchases, and other supportingpapers. The report even took into account theappraisals of the other adjusters and concluded thatthe total loss sustained by petitioner in his householdeffectsand stocks in trade reached P379,302.12. Butafter apportioning said amount among petitioner'ssix different insurers [the co-insurance being knownto Zenith], the liability of Zenith was placed atP60,592.10. It therefore recommended that Zenithpay the petitioner the amount of P60, 592.10. - Said document was offered as evidence by Zenithitself and could very well be considered as anadmission of its liability up to the amountrecommended. Being in the nature of an admissionagainst interest, it is the best evidence which affords the greatest certainty of the facts in dispute. Respondent Commissioner should not haveperfunctorily dismissed that particular evidence as aworthless piece of paper. 2. NO - There is no showing that Zenith, in contestingpayment, had acted in a wanton, oppressive ormalevolent manner to warrant the imposition ofcorrective damages. Disposition Zenith Insurance Corporation ordered to pay petitioner Norman R. Noda the sum ofP60,592.10 with legal interest from the filing of thecomplaint until full payment, but deductingtherefrom the amount of P15,472.50 which it hadearlier paid to petitioner.

DELSAN TRANSPORT, INC. v. CA (AMERICAN HOME ASSURANCE) 369 SCRA 24 DE LEON, JR; November 15, 2001 NATURE A petition for review oncertiorari of the decision of CA. FACTS - Caltex entered into a contract of affreightment withthe petitioner, Delsan Transport Lines, Inc.(petitioner), for a period of one year whereby thesaid common carrier agreed to transport Caltexsindustrial fuel oil from the Batangas-Bataan Refineryto different parts of the country. Delsan took onboard its vessel, MT Maysun, 2,277.314 kiloliters ofindustrial fuel oil of Caltex to be delivered to theCaltex Oil Terminal in Zamboanga City.Theshipment was insured by American Home AssuranceCorporation (respondent). - August 14, 1986: MT Maysun set sail fromBatangas for Zamboanga City.The vessel sank inthe early morning of August 16, 1986 near PanayGulf in the Visayas taking with it the entire cargo offuel oil. Respondent paid Caltex P5,096,635.57representing the insured value of the lost cargo.Exercising its right of subrogation under Article 2207of the New Civil Code, the private respondentdemanded of the petitioner the same amount it paidto Caltex. Delsan refused to pay, forcing Americanhome to file a case for collection in the RTC. - RTC found that the vessel, MT Maysun, was seaworthy to undertake the voyage, and that the incident was caused by an unexpected inclementweather condition or force majeure, thus exemptingthe common carrier from liability for the loss of itscargo. - CA reversed RTC decision on the basis of evidencefrom PAG-ASA that there were no 20 ft. waves in thearea. CA ruled that the petitioner is liable on itsobligation as common carrier to respondentinsurance company as subrogee of Caltex. Petitioners Claim > In every marine insurance upon a ship or freight,or freightage, or upon any thing which is the subjectof marine insurance there is an implied warranty bythe shipper that the ship is seaworthy.10 Whenprivate respondent paid Caltex the value of its lostcargo, the act of the private respondent is equivalentto a tacit recognition that the ill-fated vessel wasseaworthy. Respondents Comment > American Home Assurance is entitled to payment by its right of subrogation. ISSUES 1. WON payment made by American Home to Caltexfor the insured value of the lost cargo amounted toan admission that the vessel was seaworthy, thusprecluding any action for recovery against thepetitioner 2. WON MT Maysun was seaworthy at the time of the voyage (outline topic) 3. WON non-presentation of the marine insurancepolicy bars the complaint for recovery of sum ofmoney for lack of cause of action HELD 1. NO Ratio The fact of payment grants American Home the subrogatory right which enables it to exerciselegal remedies that would otherwise be available toCaltex as owner of the lost cargo against thepetitioner common carrier. Reasoning Art. 2207. (Civil Code) If the plaintiffs property has been insured, and hehas received indemnity from the insurancecompany for the injury or loss arising out of thewrong or breach of contract complained of, theinsurance company shall be subrogated to the rights of the insured against the wrongdoer or theperson who has violated the contract.If theamount paid by the insurance company does notfully cover the injury or loss, the aggrieved partyshall be entitled to recover the deficiency from theperson causing the loss or injury.

- The right of subrogation is designed to promoteand to accomplish justice and is the mode whichequity adopts to compel the ultimate payment of adebt by one who in justice and good conscienceought to pay. It is not dependent upon, nor does itgrow out of, any privity of contract or upon writtenassignment of claim.It accrues simply uponpayment by the insurance company of the insuranceclaim. 2. NO Ratio Seaworthiness relates to a vessels actual condition.Neither the granting of classification or the issuance of certificates establishes seaworthiness. Reasoning - Common carriers are bound to observeextraordinary diligence in the vigilance over thegoods and for the safety of passengers transportedby them, according to all the circumstances of eachcase. There is no liability if the loss, destruction ordeterioration is by force majeure. - The tale of strong winds and big waves by the saidofficers of the petitioner however, was effectivelyrebutted and belied by the weather report from PAG-ASA. MT Maysun sank with its entire cargo for thereason that it was not seaworthy.There was nosquall or bad weather or extremely poor seacondition in the vicinity when the said vessel sank. - Petitioner may not escape liability by presenting inevidence certificates that tend to show that at thetime of dry-docking and inspection by the PhilippineCoast Guard MT Maysun, was fit for voyage.Thesepieces of evidence do not necessarily take intoaccount the actual condition of the vessel at the timeof the commencement of the voyage. At the time ofdry-docking and inspection, the ship may haveappeared fit.The certificates issued, however, donot negate the presumption of unseaworthinesstriggered by an unexplained sinking. - Authorities are clear that diligence in securingcertificates of seaworthiness does not satisfy thevessel owners obligation.Also securing the approvalof the shipper of the cargo, or his surveyor, of thecondition of the vessel or her stowage does notestablish due diligence if the vessel was in fact unseaworthy, for the cargo owner has no obligation in relation to seaworthiness. 3. NO Ratio The presentation in evidence of the marine insurance policy is not indispensable in this casebefore the insurer may recover from the commoncarrier the insured value of the lost cargo in theexercise of its subrogatory right.The subrogationreceipt, by itself, is sufficient to establish not only therelationship of respondent as insurer and Caltex, asthe assured shipper of the lost cargo of industrial fueloil, but also the amount paid to settle the insuranceclaim.The right of subrogation accrues simply uponpayment by the insurance company of the insuranceclaim. DispositionPetition is denied, and the decision of the CA is affirmed.

FINMAN GENERAL ASSURANCE CORP v. INOCENCIO 179 SCRA 480 FELICIANO; November 15, 1989 FACTS - Pan Pacific is a recruitment and employmentagency.It posted surety bond issued by FinmanGeneral Assurance and was granted license tooperate by POEA. - Inocencio, Palero, Cardones, Hernandez filed withPOEA complaints against Pan Pacific for violation ofLabor Code and for refund of placement fees.POEAAdministrator motu propio impleaded Finman assurety for Pan Pacific. - Pan Pacific moved out and no notice of transfer wasfurnished to POEA as required.POEA considered thatconstructive service of complaints had been effected.- Finman denied liability and said that -POEA had no jurisdiction over surety bonds; jurisdiction is vested in Insurance Commission -Finman had not violated Labor Code -Complainants have no cause of action against Finman -Amounts claimed were paid as deposits and not as placement fees. - POEA Administrator issued Order that respondentsshould pay.Finman appealed to Secretary of Labor.Secretary upheld the POEA order. ISSUE WON Finman can be held liable for complainants claims against Pan Pacific HELD YES -Under Insurance Code, liability of surety in asurety bond is joint and several with the principalobligor. -Conditions of a bond specified and required in theprovisions of a statute providing for submission ofthe bond, are incorporated into all bonds tenderedunder that statute even though not set out inprinters ink. -POEA held and Secretary of Labor affirmed thatPan Pacific had violated Labor Code, and at least oneof the conditions for the grant and continued use ofthe recruitment license.POEA and Secretary ofLabor can require Pan Pacific to refund the placementfees and to impose the fine. -If Pan Pacific is liable, and if Finman is solidarilyliable with Pan Pacific, then Finman is liable both toprivate respondents and to POEA. Cash and surety bonds are required fromrecruitment companies as means of ensuring promptand effective recourse against such companies whenheld liable.Public policy will be effectively negated ifPOEA and the DoLE were held powerless to compel asurety company to make good on its solidaryundertaking.

EAGLE STAR INSURANCE CO LTD v. CHIA YU 96 PHIL 696 REYES; March 31, 1955 NATURE Certiorari FACTS Atkin, Kroll & Co., loaded on the S. S. Roeph Silverlight owned and operated by Leigh Hoegh &Co., A/S, of San Francisco California, 14 bales ofassorted underwear valued at P8,085.23 consignedto Chia Yu in the City of Manila. - The shipment was insured against all risks by EagleStar Ins. Co. of San Francisco, California, under apolicy issued to the shipper and by the latterassigned to the consignee. - The vessel arrived in Manila but of the 14 bales(a.k.a. freights =p) consigned to Chia Yu only 10were delivered to him as the remaining 3 could not be found.3 of those delivered were also found damaged to the extent of 50 per cent. -Chia Yu claimed indemnity for the missing anddamaged bales. But the claim was declined, first, bythe carrier and afterward by the insurer, whereuponChia Yu brought the present action against both,including their respective agents in the Philippines. - An action was filed at the CFI after more than 2years after delivery of the damaged bales and thedate when the missing bales should have beendelivered, the action was resisted by the Atkins andEagle Star principally on the ground of prescription. -TC favored Chia Yu and CA affirmed. *** CARRIERs defense of prescription is made to rest on the following stipulation of the bill of lading: In any event the carrier and the ship shall bedischarged from all liability in respect of loss ordamage unless suit is brought within one yearafter the delivery of the goods or the date whenthe goods should have been delivered.(This stipulation is but a repetition of a provision in theCA 65 which says that bills of lading coveringshipments from the US to the Phils should bebrought w/in one year after the delivery of thegoods or the date when the goods should havebeen delivered to hold the carrier liable.) *** INSURERs claim of prescription is founded uponthe terms of the policy and not upon the bill oflading. (But in our jurisdiction, as per A1144, prescription is 10 years after action accrues.) No suit action on this Policy, for the recovery ofany claim, shall be sustainable in any Court of lawor equity unless the insured shall have fullycomplied with all the terms and conditions of thisPolicy nor unless commenced with twelve (12)months next after the happening of the loss . . . ISSUE WON ATKIN s action has prescribed HELD NO - Being contrary to the law of the forum, thestipulation in the policy cannot be given effect as itwould reduce the period allowed the insured forbringing his action to less than one year(becausethe prescription period begins from the happeningof the loss and that before any suit could besustained the insured shall have to comply with theterms and conditions of the policy first TF lesseningthe period to less than a year. ) - Insular Governmentvs. Frank(13 Phil. 236)~"matters respecting a remedy, such as the bringingof suit, admissibility of evidence, and statute oflimitations, depend upon the law of the place wherethe suit is brought" TF any policy clause repugnant tothis amendment to the Insurance Act cannot begiven effect in an action in our courts. SEC. 61-A. (Insurance Code) ~ Any condition,stipulation or agreement in any policy of insurance,limiting the time for commencing an actionthereunder to a period of less than one year from the time when the cause of action accrues, is void.- The prescription clause could be harmonized withsection 61-A of the Insurance Act by taking it tomean that the time given the insured for bringing hissuit is twelve months after the cause of action accrues. - If so, when did the cause of action accrue? ChiaYus action did not accrue until his claim was finallyrejected by the insurance company. This is because,before such final rejection, there was no realnecessity for bringing suit. - As the policy provides that the insured should filehis claim, first, with the carrier and then with theinsurer, he had a right to wait for his claim to befinally decided before going to court. - Furthermore, there is nothing in the record to showthat the claim was rejected in the year 1947, eitherby the insurance company in London or its settlingagents in the Philippines.

- For the purpose of this action, Chia Yu's claim wasconsidered to have been finally rejected by theinsurer on April 22, 1948. Having been filed withintwelve months form that date, the action cannot bedeemed to have prescribed even on the suppositionthat the period given the insured for bringing suitunder the prescriptive clause of the policy is twelvemonths after the accrual of the cause of action. - Contractual limitations contained in insurance policies are regarded with extreme jealousy bycourts and will be strictly construed against theinsurer and should not be permitted to prevent arecovery when their just and honest applicationwould not produce that result. (46 C. J. S. 273.) Disposition Judgment appealed from isREVERSED with respect to the carrier and its agents but AFFIRMED with respect to the insurance company and its agents.

ACCFA v. ALPHA INSURANCE 24 SCRA 151 REYES; July 29, 1968 FACTS - In order to guarantee the Asingan Farmers'Cooperative Marketing Association, Inc. (FACOMA)against loss on account of "personal dishonesty,amounting to larceny or estafa of its Secretary-Treasurer, Ladines, the appellee, Alpha Insurance &Surety Company had issued, on 14 February 1958,its bond, No. P-FID-15-58, for the sum of P5,000with said Ladines as principal and the appellee assolidary surety. On the same date, the AsinganFACOMA assigned its rights to the appellant,Agricultural Credit Cooperative and FinancingAdministration (ACCFA for short), with approval ofthe principal and the surety. - During the effectivity of the bond, Ladinesconverted and misappropriated, to his personalbenefit, some P11,513.22 of the FACOMA funds, ofwhich P6,307.33 belonged to the ACCFA. Upondiscovery of the loss, ACCFA immediately notified inwriting the survey company on 10 October 1958, andpresented the proof of loss within the period fixed inthe bond; but despite repeated demands the suretycompany refused and failed to pay. Whereupon,ACCFA filed suit against appellee on 30 May 1960. - Defendant Alpha Insurance & Surety Co., Inc., (nowappellee) moved to dismiss the complaint for failureto state a cause of action, giving as reason that (1)the same was filed more than one year after plaintiffmade claim for loss, contrary to the eighth conditionof the bond, providing as follows: EIGHT LIMITATION OF ACTION: No action, suit orproceeding shall be had or maintained upon thisBond unless the same be commenced within oneyear from the time of making claim for the loss uponwhich such action, suit or proceeding, is based, inaccordance with the fourth section hereof. (2) the complaint failed to show that plaintiff hadfiled civil or criminal action against Ladines, asrequired by conditions 4 and 11 of the bond; and (3)that Ladines was a necessary and indispensableparty but had not been joined as such. - At first, the Court of First Instance denieddismissal; but, upon reconsideration, the courtreversed its original stand, and dismissed thecomplaint on the ground that the action was filed beyond the contractual limitation period. Hence, this appeal. ISSUE WON the provision of a fidelity bond that no actionshall be had or maintained thereon unlesscommenced within one year from the making of aclaim for the loss upon which the action is based, isvalid, in view of Section 61-A of the Insurance Actinvalidating stipulations limiting the time forcommencing an action thereon to less than one yearfrom the time the cause of action accrues HELD NO - A fidelity bond is, in effect, in the nature of acontract of insurance against loss from misconduct,and is governed by the same principles ofinterpretation. Consequently, the condition of thebond in question, limiting the period for bringingaction thereon, is subject to the provisions of Section61-A of the Insurance Act (No. 2427), as amendedby Act 4101 of the pre-Commonwealth PhilippineLegislature, prescribing that: SEC. 61-A: A condition, stipulation or agreement inany policy of insurance, limiting the time forcommencing an action thereunder to a period ofless than one year from the time when the causeof action accrues is void. - Since a "cause of action" requires, as essentialelements, not only a legal right of the plaintiff and acorrelative obligation of the defendant but also "anact or omission of the defendant in violation of saidlegal right," the cause of action does not accrue untilthe party obligated refuses, expressly or impliedly, tocomply with its duty (in this case, to pay the amountof the bond). The year for instituting action in courtmust be reckoned, therefore, from the time ofappellee's refusal to comply with its bond; it can notbe counted from the creditor's filing of the claim ofloss, for that does not import that the suretycompany will refuse to pay. In so far, therefore, ascondition eight of the bond requires action to be filedwithin one year from the filing of the claim for loss,such stipulation contradicts the public policyexpressed in Section 61-A of the Philippine InsuranceAct. - Condition eight of the bond, therefore, is null andvoid, and the appellant is not bound to comply withits provisions. The discouraging of unnecessarylitigation must be deemed a rule of public policy, considering the unrelieved congestion in the courts.As a consequence of the foregoing, action may bebrought within the statutory period of limitation forwritten contracts (New Civil Code, Article 1144). ANG v. FULTON FIRE INSURANCE CO. 2 SCRA 945

LABRADOR; July 31, 1961 NATURE Appeal from judgment of the CFI ordering thedefendant Fulton Fire Insurance Co. to pay theplaintiffs the sum of P10,000.00, with interest, andan additional sum of P2,000.00 as attorney's fees,and costs. FACTS - The stocks of general merchandise in the store ofthe Ang spouses are insured with Fulton. While theinsurance was in force, fire destroyed the goods. TheAngs filed their first claim immediately after the fire.- Their claim was denied on April 6, 1956. Theyreceived notice on April 19, 1956. - The Angs brought an action against the agent onMay 11, 1956. The court denied the suit and the mfron Sept. 3 and 12, 1957. - The Angs filed against Fulton on May 26, 1958. - There was a clause in the policy: 13.If the claim be in any respect fraudulent, or ifany false declaration is made or used in supportthereof, or if any fraudulent means or devices areused by the Insured or any one acting on hisbehalf to obtain any benefit under this Policy, or, ifthe loss or damage be occasioned by the wilful actor with the connivance of the Insured, or, if the claim be made and rejected and an action or suitbe not commenced within twelve months aftersuch rejection or (in case of arbitration taking place in pursuance of the 18th condition of thisPolicy) within twelve months after the arbitrator orarbitrators or umpire shall have made their award all benefit under this Policy shall be forfeited." ISSUE WON the suit against the agent tolled theprescription period, such that the filing against Fultonwas only 9 months after the claim was rejected HELD NO - The bringing of the action against the ParamountSurety & Insurance Company, the agent of thedefendant company, cannot have any legal effectexcept that of notifying the agent of the claim.Beyond such notification, the filing of the action canserve no other purpose. There is no law giving anyeffect to such action upon the principal. Besides,there is no condition in the policy that the actionmust be filed against the agent, and the Court cannot by interpretation extend the clear scope of theagreement beyond what is agreed upon by the parties. - Their contract is the law between the parties, andtheir agreement that an action on a claim denied bythe insurer must be brought within one year from thedenial, governs, not the rules on the prescription ofactions. Disposition The judgment appealed from is hereby set aside and the case dismissed, with costs against plaintiffs-appellees.

TRAVELLERS INSURANCE & SURETY CORP. v. CA (MENDOZA) 272 SCRA 536 HERMOSISIMA, JR; May 22, 1997 NATURE The petition herein seeks the review and reversal ofthe decision of respondent Court of Appeals affirmingin toto the judgment of the Regional Trial Court in anaction for damages filed by private respondentVicente Mendoza, Jr. as heir of his mother who waskilled in a vehicular accident. FACTS -an old lady was hit by a taxicab. The taxicab waslater identified and a case was filed against the driverand owner. Later, an amendment was filed to includethe insurance company. RTC and CA ordered that theowner, driver as well as the insurance company beheld solidarily liable. ISSUE WON RTC and CA erred HELD YES - Where the contract provides for indemnity againstliability to third persons, then third persons to whomthe insured is liable can sue the insurer. Where thecontract is for indemnity against actual loss orpayment, then third persons cannot proceed against the insurer, the contract being solely to reimbursethe insured for liability actually discharged by himthru payment to third persons, said third persons'recourse being thus limited to the insured alone. Butin the case at bar, there was no contract shown. What then was the basis of the RTC and the CAto say that the insurance contract was a thirdparty liability insurance policy? Consequently, the trial court was confused as it did not distinguishbetween the private respondent's cause of actionagainst the owner and the driver of the Lady Lovetaxicab and his cause of action against petitioner.The former is based on torts and quasi-delicts whilethe latter is based on contract. - Even assuming arguendo that there was such acontract, private respondent's cause of action cannot prevail because he failed to file the written claim mandated by the Insurance Code (beforeit was amended-action must be brought withinsix months from date of the accident (this iswhats applicable here) ; after amendment"action or suit for recovery of damage due to loss orinjury must be brought in proper cases, with theCommissioner or the Courts within one year fromdenial of the claim, otherwise the claimant's right ofaction shall prescribe" ). He is deemed, under thislegal provision, to have waived his rights as againstpetitioner-insurer. Disposition petition granted

SUN INSURANCE v. CA (supra p.57) COASTWISE v. CA (supra p.70) CEBU SHIPYARD v. WILLIAM LINES (supra p.3) MANILA MAHOGANY MANUFACTURING CORP v. CA (ZENITH INSURANCE CORP) 154 SCRA 652 PADILLA; October 12, 1987 NATURE Petition to review CA decision ordering ManilaMahogany Manufacturing Corporation to pay ZenithInsurance Corporation P5,000 with 6% annualinterest, attorney's fees, and costs of suit FACTS - From 6 March 1970 to 6 March 1971, MLAMAHOGANY insured its Mercedes Benz 4-door sedanwith ZENITH. - On 4 May 1970, the insured vehicle was bumpedand damaged by a truck owned by San MiguelCorporation. For the damage caused, ZENITH paidMLA MAHOGANY P5,000 in amicable settlement. MLAMAHOGANY's general manager executed a Release ofClaim, subrogating respondent company to all itsright to action against San Miguel Corporation. - On 11 Dec 1972, ZENITH wrote InsuranceAdjusters, Inc. to demand reimbursement from SanMiguel. Insurance Adjusters, Inc. refusedreimbursement, alleging that San Miguel had alreadypaid petitioner P4,500, as evidenced by a cashvoucher and a Release of Claim executed by theGeneral Manager of petitioner. - ZENITH thus demanded from petitionerreimbursement of the sum of P4,500 paid by SanMiguel. - City Court ordered petitioner to pay respondent P4,500. - CFI affirmed the City Court's decision in toto. - CA affirned CFI, with the modification thatpetitioner was to pay the total amount of P5,000 ithad earlier received from ZENITH. Petitioners Claims >It is not bound to pay P4,500, and much more,P5,000 to ZENITH as the subrogation in the Releaseof Claim it executed in favor of respondent wasconditioned on recovery of the total amount ofdamages petitioner had sustained. Since totaldamages were valued by petitioner at P9,486.43 andonly P5,000 was received by petitioner, MLAMAHOGANY argues that it was entitled to go afterSan Miguel to claim the additional P4,500. > It cites Art. 220711 and Art. 130412 of the CivilCode, and claims a preferred right to retain theamount coming from San Miguel, despite thesubrogation in favor of ZENITH. Respondents Arguments > There was no qualification to its right of subrogation under the Release of Claim executed by petitioner, the contents having expressed all intents and purposes of the parties. ISSUE WON the insurer may recover the sum of P5,000 HELD YES Ratio Since the insurer can be subrogated to only such rights as the insured may have, should theinsured, after receiving payment from the insurer,release the wrongdoer who caused the loss, theinsurer loses his rights against the latter. But in sucha case, the insurer will be entitled to recover fromthe insured whatever it has paid to the latter, unlessthe release was made with the consent of the insurer. Reasoning - Although petitioners right to file a deficiency claim against San Miguel is with legal basis, withoutprejudice to the insurer's right of subrogation,nevertheless, when Manila Mahogany executedanother release claim discharging San

Miguel from"all actions, claims, demands and rights of actionthat now exist or hereafter arising out of or as aconsequence of the accident" after the insurer hadpaid the proceeds of the policy - the compromiseagreement of P5,000 being based on the insurancepolicy - the insurer is entitled to recover from theinsured the amount of insurance money paid. Sincepetitioner by its own acts released San Miguel,thereby defeating private respondents right ofsubrogation, the right of action of petitioner againstthe insurer was also nullified. - As held in Phil. Air Lines v. Heald Lumber Co., under Art. 2207, the real party in interest with regard to the portion of the indemnity paid is the insurer and not the insured. SUBROGATION: The right of subrogation can only exist after the insurer has paid the insured,otherwise the insured will be deprived of his right tofull indemnity. If the insurance proceeds are notsufficient to cover the damages suffered by theinsured, then he may sue the party responsible forthe damage for the remainder. To the extent of theamount he has already received from the insurerenjoys the right of subrogation. Disposition Petition DENIED. Judgment appealed from is AFFIRMED with costs against petitioner.

PIONEER INSURANCE v. CA ( BORDER MACHINERY & HEAVY EQUIPMENT INC) 175 SCRA 668 GUTIERREZ, JR.; July 28, 1989 NATURE Petitions for review on certiorari of a decision of the CA FACTS - In 1965, Jacob S. Lim was engaged in the airlinebusiness as owner-operator of Southern Air Lines(SAL), a single proprietorship. -On May 17, 1965, at Tokyo, Japan, Japan DomesticAirlines (JDA) and Lim entered into and executed asales contract for the sale and purchase of two (2)DC-3A Type aircrafts and one (1) set of necessaryspare parts for the total agreed price of US$109,000.00 to be paid in installments. - On May 22, 1965, Pioneer Insurance and SuretyCorporation, as surety, executed and issued itsSurety Bond No. 6639 in favor of JDA, in behalf of itsprincipal, Lim, for the balance price of the aircraftsand spare parts. -Border Machinery and Heavy Equipment Company,Inc. (Bormaheco), Francisco and Modesto Cervantes(Cervanteses) and Constancio Maglana contributedsome funds used in the purchase of the aboveaircrafts and spare parts. They executed two (2)separate indemnity agreements in favor of Pioneer,one signed by Maglana and the other jointly signedby Lim for SAL, Bormaheco and the Cervanteses. - On June 10, 1965, Lim doing business under thename and style of SAL executed in favor of Pioneeras deed of chattel mortgage as security for thelatter's suretyship in favor of the former. It wasstipulated therein that Lim transfer and convey tothe surety the two aircrafts. - Lim defaulted on his subsequent installmentpayments prompting JDA to request payments fromthe surety. - Pioneer paid a total sum of P298,626.12. - Pioneer then filed a petition for the extrajudicialforeclosure of the said chattel mortgage before theSheriff of Davao City. - The Cervanteses and Maglana, however, filed athird party claim alleging that they are co-owners ofthe aircrafts, - On July 19, 1966, Pioneer filed an action for judicial foreclosure with an application for a writ of preliminary attachment against Lim and respondents, the Cervanteses, Bormaheco and Maglana. **Maglana, Bormaheco and the Cervanteses filedcross-claims against Lim alleging that they were notprivies to the contracts signed by Lim and, by way ofcounterclaim, sought for damages for being exposedto litigation and for recovery of the sums of moneythey advanced to Lim for the purchase of theaircrafts in question. (this constitutes the second petition but will no longer be discussed becauseit is not relevant to the topic) - After trial on the merits, a decision was rendered holding Lim liable to pay Pioneer but dismissed Pioneer's complaint against all other defendants. - CA modified the trial court's decision in thatthe plaintiffs complaint against all the defendants (including Lim) was dismissed. ISSUE WON the petition of Pioneer Insurance and SuretyCorporation against all defendants was rightlydismissed HELD YES

- Both the TC and CA made the finding that Pioneerreinsured its risk of liability under the surety bond ithad executed in favor of JDA, collected the proceedsof such reinsurance in the sum of P295,000, and paidwith the said amount the bulk of its alleged liabilityto JDA under the said surety bond. The total amountpaid by Pioneer to JDA is P299,666.29. Since Pioneerhas collected P295,000.00 from the reinsurers, theuninsured portion of what it paid to JDA is thedifference between the two amounts, or P3,666.28.This is the amount for which Pioneer may suedefendants, assuming that the indemnity agreementis still valid and effective. But since the amountrealized from the sale of the mortgaged chattels areP35,000.00 for one of the airplanes and P2,050.00for a spare engine, or a total of P37,050.00, Pioneeris still overpaid by P33,383.72. Therefore, Pioneerhas no more claim against defendants. - The payment to the petitioner made by thereinsurers was not disputed. Considering thisadmitted payment, the only question was the effectof payment made by the reinsurers to the petitioner- In general a reinsurer, on payment of a lossacquires the same rights by subrogation as areacquired in similar cases where the original insurer pays a loss (Universal Ins. Co. v. Old Time Molasses Co.). - The rules of practice in actions on original insurancepolicies are in general applicable to actions orcontracts of reinsurance (Delaware, Ins. Co. v.Pennsylvania Fire Ins. Co.). - Hence the applicable law is Article 2207 of the new Civil Code, to wit: Art. 2207. If the plaintiffsproperty has been insured, and he has receivedindemnity from the insurance company for the injuryor loss arising out of the wrong or breach of contractcomplained of, the insurance company shall besubrogated to the rights of the insured against thewrongdoer or the person who has violated thecontract. If the amount paid by the insurancecompany does not fully cover the injury or loss, theaggrieved party shall be entitled to recover thedeficiency from the person causing the loss or injury- If a property is insured and the owner receives theindemnity from the insurer, it is provided in saidarticle that the insurer is deemed subrogated to therights of the insured against the wrongdoer and ifthe amount paid by the insurer does not fully coverthe loss, then the aggrieved party is the one entitledto recover the deficiency. Evidently, under this legal provision, the real party in interest with regard tothe portion of the indemnity paid is the insurer andnot the insured (. Air Lines, Inc. v. Heald Lumber Co ., and Manila Mahogany Manufacturing Corporation v. Court of Appeals) - It is clear from the records that Pioneer sued in its own name and not as an attorney-in-fact of thereinsurer. Accordingly, the appellate court did notcommit a reversible error in dismissing thepetitioner's complaint as against the respondents forthe reason that the petitioner was not the real partyin interest in the complaint and, therefore, has nocause of action against the respondents. Disposition Petitions dismissed. Questioned decision of CA affirmed.

PAN MALAYAN INSURANCE CORPORATION v. CA (FABIE, HER UNKNOWN DRIVER) 184 SCRA 54 CORTES, April 3, 1990 NATURE PETITION to review the decision of the Court of Appeals FACTS - Pan Malayan Insurance Company (Panmalay)insured the Mitsubishi Colt Lancer car registered inthe name of Canlubang Automotive ResourcesCorporation (Canlubang) under its motor vehicleinsurance policy. Among the provisions of the policywas a own -damage clause whereby Panmalayagrees to indemnify Canlubang in cases of damagecaused by accidental collision or overturning, or collision or overturning consequent upon mechanical breakdown or consequent upon wear and tear. - On 1985, the insured car was sideswept anddamaged by a car owned by Erlinda Fabie, driven byan unknown driver who fled the scene. Panmalay, inaccordance with the policy, defrayed the cost ofrepair of the insured car and was subrogated to therights of Canlubang against the driver and owner ofthe pick-up. Panmalay then filed a complaint fordamages with RTC Makati against Erlinda Fabie andher driver on the grounds of subrogation, with thelatter failing and refusing to pay their claim. Fabiefiled a Motion for Bill of Particulars. - RTC: dismissed complaint for lack of cause ofaction (payment by PANMALAY of CANLUBANG'sclaim under the "own damage" clause of theinsurance policy was an admission by the insurerthat the damage was caused by the assured and/orits representatives) Panmalay appealed - CA: dismissed appeal, affirmed RTC (applying theejusdem generis rule held that Section III-1 of thepolicy, which was the basis for settlement ofCANLUBANG's claim, did not cover damage arisingfrom collision or overturning due to the negligence ofthird parties as one of the insurable risk) ISSUE WON the insurer PANMALAY may institute an actionto recover the amount it had paid its assured insettlement of an insurance claim against privaterespondents as the parties allegedly responsible forthe damage caused to the insured vehicle, inaccordance with A2207, NCC HELD YES Ratio Article 2207 of the Civil Code is founded on the well-settled principle of subrogation. If theinsured property is destroyed or damaged throughthe fault or negligence of a party other than theassured, then the insurer, upon payment to theassured, will be subrogated to the rights of the assured to recover from the wrongdoer to the extentthat the insurer has been obligated to pay. Paymentby the insurer to the assured operates as anequitable assignment to the former of all remedieswhich the latter may have against the third partywhose negligence or wrongful act caused the loss.The right of subrogation is not dependent upon, nordoes it grow out of, any privity of contract or uponwritten assignment of claim. It accrues simply uponpayment of the insurance claim by the insurer Exceptions (1) if the assured by his own act releases thewrongdoer or third party liable for the loss ordamage, from liability, the insurer's right ofsubrogation is defeated; (2) where the insurer pays the assured the value ofthe lost goods without notifying the carrier who hasin good faith settled the assured's claim for loss, thesettlement is binding on both the assured and theinsurer, and the latter cannot bring an action againstthe carrier on his right of subrogation; (3) where the insurer pays the assured for a losswhich is not a risk covered by the policy, therebyeffecting "voluntary payment", the former has noright of subrogation against the third party liable forthe loss Reasoning - Both TC and CA are incorrect. ON TC: Own damage (not found in the insurance policy) simply meant that Panmalay had assumed toreimburse the cost for repairing the damage to theinsured vehicle. Its different from Third PartyLiability coverage (liabilities arising from the deathof or bodily injuries suffered by 3rd parties) and fromProperty Damage coverage (liabilities from damagecaused by insured vehicle to properties of 3rd parties) ON CA: the terms of a contract are to be construed according to the sense and meaning of the termswhich the parties thereto have used. In the case ofproperty insurance policies, the evident intention ofthe contracting parties, i.e., the insurer and

theassured, determine the import of the various termsand provisions embodied in the policy. It is onlywhen the terms of the policy are ambiguous,equivocal or uncertain, such that the partiesthemselves disagree about the meaning of particularprovisions, that the courts will intervene. In such anevent, the policy will be construed by the courtsliberally in favor of the assured and strictly againstthe insurer - Both Panmalay and Canlubang had the sameinterpretation regarding the coverage of insured riskregarding accidental collision or overturning toinclude damages caused by 3rd party to Canlubang soit was improper for CA to ascribe meaning contraryto the clear intention and understanding of the parties. - Court on several occasions defined accident oraccidental as taking place without ones foresightor expectation, an event that proceeds from anunknown cause, or is an unusual effect of a knowncause and, therefore, not expected [Dela Cruz v.Capital Insurance & Surety Co.] The concept"accident" is not necessarily synonymous with theconcept of "no fault". It may be utilized simply todistinguish intentional or malicious acts fromnegligent or careless acts of man. - damage/loss to insured vehicle due to negligence of3rd parties not listed as exceptions to coverage in theinsurance policy - Interpretation given by Panmalay is more inkeeping with rationale behind rules on interpretationof insurance contracts in favor of assured orbeneficiary: indemnity or payment - EVEN if voluntarily indemnified Canlubang, asinterpreted by TC: the insurer who may have norights of subrogation due to "voluntary" paymentmay never. theless recover from the third partyresponsible for the damage to the insured propertyunder Article 1236 of the Civil Code. [SverigesAngfartygs Assurans Forening v. Qua Chee Gan] Disposition the present petition is GRANTED. Petitioner's complaint for damages against privaterespondents is hereby REINSTATED. Let the case beremanded to the lower court for trial on the merits.

FIREMAN'S FUND INSURANCE COMPANY v. JAMILA & COMPANY, INC. 70 SCRA 323 AQUINO; April 1976 FACTS Jamila & Co., Inc. or the Veterans PhilippineScouts Security Agency contracted to supply securityguards to Firestone.Jamila assumed responsibilityfor the acts of its security guards. The First QuezonCity Insurance Co., Inc. executed a bond in the sumof P20,000 to guarantee Jamila's obligations underthat contract. -On May 18, 1963 properties of Firestone valued at P11,925 were lost allegedly due to the acts of its employees who connived with Jamila's securityguard. Fireman's Fund, as insurer, paid to Firestonethe amount of the loss. Fireman's Fund wassubrogated to Firestone's right to get reimbursementfrom Jamila.Jamila and its surety, First Quezon Cityfailed to pay the amount of the loss in spite ofrepeated demands. Upon defendant's motions, the lower courtdismissed the complaint as to Jamila on the groundthat there was no allegation that it had consented tothe subrogation and, therefore, Fireman's Fund hadno cause of action against it. It also dismissed thecomplaint as to First Quezon City on the ground ofres judicata. It appears that the same action waspreviously filed in a civil case which was dismissedbecause of the failure of the same plaintiffs and theircounsel to appear at the pre-trial. -Upon an MR, the lower court set aside its order ofdismissal and sustained plaintiff's contention thatthere was no res judicata as to First Quezon Citybecause the civil case was dismissed withoutprejudice. However, due to inadvertence, the lowercourt did not state in its order why it set aside itsprior order dismissing the complaint with respect toJamila. Jamilla had originally moved for the dismissalof the complaint on the ground of lack of cause ofaction. Its basis for its contention were: (1) that thecomplaint did not allege that Firestone, pursuant tothe contractual stipulation quoted in the complaint,had investigated the loss and that Jamila wasrepresented in the investigation and (2) that Jamiladid not consent to the subrogation of Fireman's Fundto Firestone's right to get reimbursement from Jamilaand its surety. The lower court in its order ofdismissal had sustained the second ground. -Jamila in its MR invoked the first ground whichhad never been passed upon by the lower court. Butthe lower court in its order granting Jamila's motionfor reconsideration, completely ignored that firstground. It reverted to the second ground which wasrelied upon in its order previous order. The lowercourt reiterated its order, stating that Fireman's Fundhad no cause of action against Jamila because Jamiladid not consent to the subrogation. The court did notmention Firestone, the co-plaintiff of Fireman's Fund.-Firestone and Fireman's Fund filed an MR on theground that Fireman's Fund was suing on the basis oflegal subrogation whereas the lower courterroneously predicated its dismissal order on thetheory that there was no conventional subrogationbecause the debtor's consent was lacking. -The plaintiffs cited article 2207 of the Civil Codewhich provides that "if the plaintiff's property hasbeen insured, and he has received indemnity fromthe insurance company for the injury or loss arisingout of the wrong or breach of contract complained of,the insurance company shall be subrogated to therights of the insured against the wrongdoer or theperson who has violated the contract". The lower court denied plaintiff's motion. Theyfiled a second MR, calling the lower court's attentionto the fact that the issue of subrogation was of nomoment because Firestone, the subrogor, is a party-plaintiff and could sue directly Jamila in its own right.Without resolving that contention, the lower courtdenied plaintiffs' second MR. ISSUE WON the complaint of Firestone and Fireman's Fund states a cause of action against Jamila HELD YES -Fireman's Fund's action against Jamila is squarelysanctioned by article 2207. As the insurer, Fireman'sFund is entitled to go after the person or entity thatviolated its contractual commitment to answer forthe loss insured against.

-The trial court erred in applying to this case therules on novation. The plaintiffs in alleging in theircomplaint that Fireman's Fund "became a party ininterest in this case by virtue of a subrogation rightgiven in its favor by" Firestone, were not relying onthe novation by change of creditors as contemplatedin articles 1291 and 1300 to 1303 of the Civil Codebut rather on article 2207. -Article 2207 is a restatement of a settled principleof American jurisprudence. Subrogation has beenreferred to as the doctrine of substitution. It is anarm of equity that may guide or even force one topay a debt for which an obligation was incurred butwhich was in whole or in part paid by another. -Subrogation is founded on principles of justice andequity, and its operation is governed by principles ofequity. It rests on the principle that substantialjustice should be attained regardless of form, that is,its basis is the doing of complete, essential, andperfect justice between all the parties without regardto form. Subrogation is a normal incident of indemnityinsurance. Upon payment of the loss, the insurer isentitled to be subrogated pro tanto to any right of action which the insured may have against the thirdperson whose negligence or wrongful act caused theloss. The right of subrogation is of the highest equity.The loss in the first instance is that of the insuredbut after reimbursement or compensation, itbecomes the loss of the insurer. Although many policies including policies in thestandard form, now provide for subrogation, andthus determine the rights of the insurer in thisrespect, the equitable right of subrogation as thelegal effect of payment inures to the insurer withoutany formal assignment or any express stipulation tothat effect in the policy. Stated otherwise, when theinsurance company pays for the loss, such paymentoperates as an equitable assignment to the insurer ofthe property and all remedies which the insured mayhave for the recovery thereof. That right is notdependent upon, nor does it grow out of, any privityof contract, or upon written assignment of claim, andpayment to the insured makes the insurer anassignee in equity. -On the other hand, Firestone is really a nominalparty in this case. It had already been indemnifiedfor the loss which it had sustained. Obviously, itjoined as a party-plaintiff in order to help Fireman'sFund to recover the amount of the loss from Jamilaand First Quezon City. Firestone had tacitly assignedto Fireman's Fund its cause of action against Jamilafor breach of contract. Sufficient ultimate facts arealleged in the complaint to sustain that cause of action. TABACALERA v. NORTH FRONT SHIPPING 272 SCRA 527 BELLOSILLO; May 16, 1997 FACTS - 20,234 sacks of corn grains valued at P3.5M wereshipped on board North Front 777, defendantsvessel. The cargo was consigned to Republic FlourMills Corp. under Bill of Lading No. 001 and insuredwith Tabacalera, Prudential Guarantee & Assurance,and New Zealand Insurance. - Republic Flour was advised of the vessels arrival inManila, but did not immediately commence theunloading operations. Unloading was sometimesstopped due to varying weather and sometimes forno apparent reason. Unloading was only completed20 days after the arrival of the barge; by then, thecargo was short 26.333 metric tons and the rest wasalready moldy and deteriorating. - Analyses showed that the deterioration was causedby moisture content from salt water, which could bearrested by drying. However, Republic Flour rejectedthe entire cargo and demanded that defendant NorthFront Shipping pay the damages suffered by it. Thedemands were unheeded and the insurancecompanies were obliged to pay Republic FlourP2,189,433 - By virtue of the insurance companies payment,they were subrogated to the rights of Republic Flour.Petitioners filed a complaint against North FrontShipping, claiming the loss was exclusivelyattributable to the latters fault and negligence.Having surveyed the vessel, it was found that thebarge had cracks in its bodega. The hatches on thecrates of grain were not sealed and the tarpaulinsused in covering them were not new, contrary toNorth Front Shippings claims. North Front Shippingreiterated that the barge was inspected prior toloading and found seaworthy and were issued apermit to sail by the Coast Guard. They furtheraverred that the grains were farm wet and notproperly dried before loading. - The court dismissed the complaint, ruling that thecontract entered into was a charterpartyagreement; as such, only ordinary diligence in thecare of the goods was required of North FrontShipping. ISSUE WON defendant is required to observe extraordinary diligence in its vigilance over the goods it transports

HELD YES - As a corporation engaged in the business of transporting cargo offering its servicesindiscriminately to the public, it is without a doubt acommon carrier. As such, it has the burden ofproving that it observed extraordinary diligence toavoid responsibility for the lost cargo. The clean billof lading it issued disprove the master of the vesselsclaim that the grains were farm wet when loaded. Ifthey were wet, the master of the vessel should haveknown that the grains would eventually deterioratewhen sealed in hot compartments in hatches of aship and should have undertaken precautionarymeasures to avoid this. The arrival of the goods atthe place of destination in bad order makes a primafacie case against the common carrier, which mustprove its non-liability. - While petitioners presented evidence of the vesselsbad shape and a laboratory analysis revealing thatthe grains were contaminated with salt water,defendants failed to rebut said arguments or evenendeavor to establish that the loss, destruction ordeterioration was due to a fortuitous event; anact/omission of the owner of the goods; thecharacter of the goods or defects in their packing; oran order or act of a competent public authority. - However, Republic Flour is also found to be guilty ofcontributory negligence for not immediately staringthe unloading operations and for providing noexplanation for the delay. As such, it should share atleast 40% of the loss. Disposition The decision of the CA is REVERSED and SET ASIDE

PHILIPPINE AMERICAN LIFE INSURANCE COMPANY v. CA (ELIZA PULIDO) 344 SCRA 360 GONZAGA-REYES; November 15, 2000 NATURE This petition for review on certiorari seeks to reversethe Decision of the Special Second Division of theCourt of Appeals FACTS - On January 9, 1989, petitioner received from oneFlorence Pulido an application for life insurance,dated December 16, 1988, in the amount ofP100,000.00 which designated her sister, hereinprivate respondent, as its principal beneficiary.Because the insurance applied for was nonmedical,petitioner did not require a medical examination andissued a policy on the sole basis of the application onFebruary 11, 1989. On April 1992, petitionerreceived private respondents claim, which declaredthat the insured, Florence Pulido, died of acutepneumonia on September 10, 1991. - Petitioner withheld payment on the ground that thepolicy claimed under was void from the start forhaving been procured in fraud.It is petitionerscontention that even before they received privaterespondents claim for death benefits, theirinvestigation concerning the subject policy yieldedthe information that the insured, Florence Pulido,died in 1988, before the application for insurance onher life was made. While this was communicated toprivate respondent in a letter, private respondent hadalready filed her claim earlier that month. In another letter, however, petitioner confirmed to privaterespondent receipt of the claim papers and assuredher that her case was being given preferentialattention and prompt action. - Petitioner caused another investigation respectingthe subject policy.Pursuant to the findings of thissecond investigation, petitioner stood by its initialdecision to treat the policy as void and not to honorthe claim. On November 9, 1992, privaterespondent enlisted the services of counsel inreiterating her claim for death benefitsPetitioner stillrefused to make payment and thus, this action. ISSUE WON lower court erred in holding that there was no fraud HELD - The records bear out that since the onset of thiscase, the main issue has always been whether therewas fraud in the obtainment of the disputed policy, orput differently, whether the insured, Florence Pulido,was in fact dead before the application for insuranceon her life was made.This the lower courts hadeffected ruled on, upon a preponderance of theevidence duly received from both parties.We see noreversible error in the finding of both respondentcourt and the trial court in favor of the correctness ofthe entries in Certificate of Death, duly registeredwith the Local Civil Registrar of Bagulin, La Union,which declared that Florence Pulido died of acutepneumonia on September 10, 1991. Dr. IrineoGutierrez, the Municipal Health Officer of Bagulin, LaUnion whose signature appeared in the deathcertificate, testified in addition that he ministered tothe ailing Florence Pulido for two days immediatelyprior to her death.This fact is likewise noted in thedeath certificate. - Death certificates, and notes by a municipal healthofficer prepared in the regular performance of hisduties, are prima facie evidence of facts thereinstated. A duly-registered death certificate isconsidered a public document and the entries foundtherein are presumed correct, unless the party whocontests its accuracy can produce positive evidenceestablishing otherwise. Petitioners contention thatthe death certificate is suspect because Dr. Gutierrezwas not present when Florence Pulido died, andknew of Florences death only through RamonPiganto, does not merit a conclusion o f fraud.Nomotive was imputed to Dr. Gutierrez for seeking to perpetuate a falsity in public records.Petitioner waslikewise unable to make out any clear motive as towhy Ramon Piganto would purposely lie. Mereallegations of fraud could not substitute for the fulland convincing evidence that is required to prove it.A failure to do so would leave intact the presumptionof good faith and regularity in the performance ofpublic duties, which was the basis of

both respondentcourt and the trial court in finding the date ofFlorence Pulidos death to be as plaintiff privaterespondent maintained. - We cannot likewise give credence to petitionerssubmission that the inconsistencies in thetestimonies of the witnesses for plaintiff-privaterespondent are in themselves evidence of fraud.Such alleged inconsistencies are matters of credibilitywhich had been ably passed upon by the lower court. Dispositionthe instant petition is DENIED

ST.PAUL FIRE & MARINE INSURANCE CO v. MACONDRAY & CO INC 70 SCRA 122 ANTONIO; March 25, 1976 FACTS -Winthrop Products, Inc. shipped aboard the SS "TaiPing", owned and operated by Wilhelm Wilhelmsen,218 cartons and drums of drugs and medicine, withthe freight prepaid, which were consigned toWinthrop-Stearns, Inc. Barber Steamship Lines, Inc.,agent of Wilhelm Wilhelmsen issued Bill of Lading No.34, in the name of Winthrop Products, Inc. asshipper, with arrival notice inManila to consigneeWinthrop-Stearns, Inc. The shipment was insured bythe shipper against loss and/or damage with the St.Paul Fire & Marine Insurance Company. -The SS "Tai Ping" arrived at the Port of Manila anddischarged its aforesaid shipment into the custody ofManila Port Service, the arrastre contractor for thePort of Manila. The said shipment was dischargedcomplete and in good order with the exception of one(1) drum and several cartons which were in badorder condition. Because consignee failed to receivethe whole shipment and as several cartons ofmedicine were received in bad order condition, theconsignee filed the corresponding claim in theamount of P1,109.67 representing the C.I.F. value ofthe damaged drum and cartons of medicine with thecarrier and theManila Port Service. However, bothrefused to pay such claim. Consequently, theconsignee filed its claim with the insurer, St. Paul Fire & Marine Insurance Co., the insurance company, onthe basis of such claim, paid to the consignee theinsured value of the lost and damagcd goods,including other expenses in connection therewith, inthe total amount of $1,134.46 U.S. currency. -As subrogee of the rights of' the shipper and/orconsignee, the insurer, St. Paul Fire & MarineInsurance Co., instituted an action against thedefendants for the recovery of said amount of$1,134.46, plus costs. -The defendants resisted the action. However, forthe purpose only of avoiding litigation withoutadmitting liability to the consignee, the defendantsoffered to settle the latters claim in full by payingthe C.I.F. value of the damaged cargo, but this offerwas declined by theplaintiff. The LC rendered judgment ordering thedefendants to pay the plaintiff the sum of P300.00.The plaintiff filed a MFR contending that it shouldrecover the amount of $1,134.46 or its equivalent inpesos at the rate of P3.90, instead of P2.00, but thiswas denied.Hence, this appeal. ISSUES 1. WON in case of loss or damage, the liability of thecarrier to the consignee is limited to the C.I.F. valueof the goods which were lost or damaged 2. WON the insurer who has paid the claim in dollarsto the consignee should be reimbursed in its pesoequivalent on the date of discharge of the cargo oron the date of the decision HELD 1. YES RatioThe purpose of the bill of lading is to provide for the rights and liabilities of the parties in referenceto the contract to carry. The stipulation in the bill ofladinglimiting the common carrier's liability to thevalue of the goods appearing in the bill, unless theshipper or owner declares a greater value, is validand binding. This limitation of the carrier's liability issanctioned by the freedom of the contracting partiesto establish such stipulations, clauses, terms, orconditions as they may deem convenient, providedthey are not contrary to law, morals, good customsand public policy. A stipulation fixing or limiting thesum that may be recovered from the carrier on theloss or deterioration of the goods is valid, provided itis (a) reasonable and just under the circumstances,and (b) has been fairly and freely agreed upon. Inthe case at bar, the liabilities of the defendantsappellees with respect to the lost or damagedshipments are expressly limited to the C.I.F. value ofthe goods as per contract of sea carriage embodiedin the bill of lading. - The plaintiff-appellant, as insurer, after paying theclaim of the insured for damages under theinsurance, is subrogated merely to the rights of theassured. As subrogee, it can recover only the amountthat is recoverable by the latter. Since the right ofthe assured, in case of loss or damage to the goods,is limited or restricted by the provisions in the bill oflading, a suit by the insurer as subrogee necessarilyis subject to like limitations and restrictions. 2.On the date of the discharge of the cargo.Thepeso equivalent was based by the consignee on theexchange rate of P2.015 to $1.00 which was the rateexisting at that time.

PHILAM v. AUDITOR (supra p.59) FIELDMENS v. ASIAN SURETY (supra p.60) EQUITABLE v. RURAL INSURANCE (supra p.60) COQUIA v. FIELDMEN'S INSURANCE CO. INC. 26 SCRA 178 CONCEPCION; November 29, 1968 NATURE Appeal from the decision of the CFI certified by CA FACTS - December 1, 1961, appellant Fieldmen's InsuranceCompany, Inc.issued, in favor of the Manila YellowTaxicab Co., Inc. a common carrier accidentinsurance policy, covering the period from December1, 1961 to December ,1962. It was stipulated in saidpolicy that: "The Company will, subject to the Limits of Liabilityand under the Terms of this Policy, indemnify theInsured in the event of accident caused by or arisingout of the use of Motor Vehicle against all sumswhich the Insured will become legally liable to pay inrespect of: Death or bodily injury to any fare-payingpassenger including the Driver, Conductor and/orInspector who is riding in the Motor Vehicle insuredat the time of accident or injury." - While the policy was in force, or on February 10,1962, a taxicab of the Insured, driven by CarlitoCoquia, met a vehicular accident to which he died.The Insured filed therefor a claim for P5,000.00 to which the Company replied with an offer to payP2,000.00, by way of compromise. The Insuredrejected it and made a counter-offer for P4,000.00,but the Company did not accept it. - On September 18, 1962, the Insured and Carlito'sparents filed a complaint against the Company tocollect the proceeds of the policy. In its answer, theCompany admitted the existence thereof, butpleaded lack of cause of action on the part of theplaintiffs. - TC rendered a decision sentencing the Company topay to the plaintiffs the sum of P4,000.00 and thecosts. Hence, this appeal by the Company, whichcontends that plaintiffs have no cause of actionbecause: 1) the Coquias have no contractual relationwith the Company; and 2) the Insured has notcomplied with the provisions of the policy concerningarbitration. ISSUES 1. WON there was contractual relations between the Coquias and the Company 2. WON the insured has not complied with the provisions of the policy concerning arbitration HELD 1. Although, in general, only parties to a contractmay bring an action based thereon, this rule issubject to exceptions, one of which is found in theArt 1311 CC, reading: "If a contract should contain some stipulation infavor of a third person, he may demand itsfulfillment provided he communicated his acceptanceof the obligor before its revocation. A mere incidentalbenefit or interest of a person is not sufficient. Thecontracting parties must have clearly anddeliberately conferred a favor upon a third person." - Does the policy in question belong to such class of contracts pour autrui? In this connection, said policy provides, inter alia: "Section I Liability to Passengers. 1. The Company will, subject to the Limits of Liability andunder the Terms of this Policy, indemnify the Insuredin the event of accident caused by or arising out ofthe use of Motor Vehicle against all sums which theInsured will become legally liable to pay in respectof: Death or bodily injury to any farepayingpassenger including the Driver. . . who is riding in theMotor Vehicle insured at the time of accident or injury. "Section II. Liability to the Public "3. In terms of and subject to the limitations ofand for the purposes of this Section, the Companywill indemnify any authorized Driver who is drivingthe Motor Vehicle . . . " "Conditions

"7. In the event of death of any person entitledto indemnify under this Policy, the Company will, inrespect of the liability incurred by such person,indemnify his personal representatives in terms ofand subject to the limitations of this Policy, provided,that such representatives shall, as though they werethe Insured, observe, fulfill and be subject to theTerms of this Policy insofar as they can apply. "8. The Company may, at its option, makeindemnity payable directly to the claimants or heirsof claimants, with or without securing the consent ofor prior notification to the Insured, it being the trueintention of this Policy to protect, to the extentherein specified and subject always to the Terms ofthis Policy, the liabilities of the Insured towards thepassengers of the Motor Vehicle and the Public." - Thus, the policy under consideration is typical ofcontracts pour autrui, this character being mademore manifest by the fact that the deceased driverpaid fifty percent (50%) of the correspondingpremiums, which were deducted from his weeklycommissions. Under these conditions, it is clear thatthe Coquias who, admittedly, are the sole heirs ofthe deceased have a direct cause of action againstthe Company, and, since they could have maintainedthis action by themselves, without the assistance ofthe Insured, it goes without saying that they couldand did properly join the latter in filing the complaintherein. 2. Based upon Section 17 of the policy: "If any difference or dispute shall arise withrespect to the amount of the Company's liabilityunder this Policy, the same shall be referred to thedecision of a single arbitrator to be agreed upon byboth parties or failing such agreement of a singlearbitrator, to the decision of two arbitrators, one tobe appointed in writing by each of the partieswithin one calendar month after having beenrequired in writing so to do by either of the partiesand in case of disagreement between thearbitrators, to the decision of an umpire who shallhave been appointed in writing by the arbitratorsbefore entering on the reference and the costs ofand incidental to the reference shall be dealt within the Award. And it is hereby expressly stipulatedand declared that it shall be a condition precedent to any right of action or suit upon this Policy thatthe award by such arbitrator, arbitrators or umpireof the amount of the Company's liability hereunderif disputed shall be first obtained." - The record showsthat none of the parties to thecontract invoked this section, or made any referenceto arbitration, during the negotiations preceding theinstitution of the present case. In fact, counsel forboth parties stipulated, in the trial court, that none ofthem had, at any time during said negotiations, evensuggested the settlement of the issue between themby arbitration, as provided in said section. Theiraforementioned acts or omissions had the effect of awaiver of their respective right to demand anarbitration. DispositionThe decision appealed from should be as it is hereby affirmed in toto, with costs against theherein defendant-appellant, Fieldmen's InsuranceCo., Inc.

COUNTRY BANKERS INSURANCE CORP v. LIANGA BAY DE LEON; January 25, 2002 NATURE Petition for review on certiorari FACTS - Lianga Bay is a duly registered cooperativejudicially declared insolvent and is here representedby, Cornelio Jamero.Country Bankers Insurance andLianga Bay entered into a contract of fire insurance.Country Bankers insured the respondents stocks-in-trade against fire loss, damage or liability during theperiod starting from June 20, 1989 at 4:00 p.m. toJune 20, 1990 at 4:00 p.m., for the sum ofP200,000.00. - On July 1, 1989, at or about 12:40 a.m., therespondents building at Barangay Diatagon, Lianga,Surigao del Sur was gutted by fire, resulting in thetotal loss of the respondents stocks -intrade, piecesof furnitures and fixtures, equipments and records. - Due to the loss, the respondent filed an insuranceclaim with the petitioner under its Fire InsurancePolicy, submitting: (a) the Spot Report of Pfc. ArturoV. Juarbal, INP Investigator, dated July 1, 1989; (b)the Sworn Statement of Jose Lomocso; and (c) theSworn Statement of Ernesto Urbiztondo. - The petitioner, however, denied the insurance claimon the ground that, based on the submitteddocuments, the building was set on fire by 2 NPA rebels who wanted to obtain canned goods, rice andmedicines as provisions for their comrades in theforest, and that such loss was an excepted risk underparagraph No. 6 of the policy conditions of FireInsurance Policy No. F-1397, which provides: This insurance does not cover any loss or damageoccasioned by or through or in consequence,directly or indirectly, of any of the followingoccurrences, namely: (d) Mutiny, riot, military or popular uprising,insurrection, rebellion, revolution, military orusurped power. Any loss or damage happening during theexistence of abnormal conditions (whetherphysical or otherwise) which are occasioned by orthrough or in consequence, directly or indirectly,of any of said occurrences shall be deemed to beloss or damage which is not covered by thisinsurance, except to the extent that the Insuredshall prove that such loss or damage happenedindependently of the existence of such abnormalconditions. - Finding the denial of its claim unacceptable, LiangaBay then instituted in the trial court the complaint forrecovery of "loss, damage or liability" againstCountry Bankers. - RTC ruled in favor of the cooperative.CA affirmed. ISSUE WON the cause of the loss was an excepted risk under the terms of the fire insurance policy HELD - Where a risk is excepted by the terms of a policywhich insures against other perils or hazards, lossfrom such a risk constitutes a defense which theinsurer may urge, since it has not assumed that risk,and from this it follows that an insurer seeking todefeat a claim because of an exception or limitationin the policy has the burden of proving that the losscomes within the purview of the exception orlimitation set up. If a proof is made of a lossapparently within a contract of insurance, the burdenis upon the insurer to prove that the loss arose froma cause of loss which is excepted or for which it isnot liable, or from a cause which limits its liability.Stated else wise, since Country bank here isdefending on the ground of non-coverage and relyingupon an exemption or exception clause in the fireinsurance policyit has the burden of proving thefacts upon which such excepted risk is based, by a preponderance of evidence. But petitioner failed to do so. - The petitioner relies on the Sworn Statements ofJose Lomocso and Ernesto Urbiztondo as well as onthe Spot Report of Pfc. Arturo V. Juarbal.A witnesscan testify only to those facts which he knows of hispersonal knowledge, which means those facts whichare derived from his perception.Consequently, awitness may not testify as to what he merely learnedfrom others either because he was told or read orheard the same. Such testimony is consideredhearsay and may not be received as proof of thetruth of what he has learned. Dispositionthe appealed Decision isMODIFIED. The rate of interest on the adjudged principal amount of Two

Hundred Thousand Pesos(P200,000.00) shall be six percent (6%) per annumcomputed from the date of filing of the Complaint inthe trial court. The awards in the amounts of FiftyThousand Pesos (P50,000.00) as actual damages,Fifty Thousand Pesos (P50,000.00) as exemplarydamages, Five Thousand Pesos (P5,000.00) aslitigation expenses, and Ten Thousand Pesos(P10,000.00) as attorney?s fees are hereby DELETED.

DBP POOL OF ACCREDITED INSURANCE v. RADIO MINDANAO NETWORK 480 SCRA 314 MARTINEZ; January 27, 2006 NATURE Petition for certiorari FACTS - In the evening of July 27, 1988, the radio station ofRadio Mindanao Network located at the SSS Buildingin Bacolod City was burned down causing damage inthe amount of over one million pesos. Respondentsought to recover under two insurance policies butthe claims were denied on the basis that the case ofthe loss was an excepted risk under condition no. 6(c) and (d), to wit: 6. This insurance does not cover any loss or damageoccasioned by or through or in consequence, directlyor indirectly, of any of the following consequences,namely: (c) War, invasion, act of foreign enemies, hostilities,or warlike operations (whether war be declared ornot), civic war. (d) Mutiny, riot, military or popular uprising,insurrection, rebellion, revolution, military or usurpedpower. - The insurers maintained that based on witnessesand evidence gathered at the site, the fire wascaused by the members of the Communist Party ofthe Philippines/New Peoples Army. Hence the refusalto honor their obligations. - The trial court and the CA found in favor of therespondent. In its findings, both courts mentionedthe fact that there was no credible evidencepresented that the CCP/NPA did in fact cause the firethat gutted the radio station in Bacolod. ISSUE WON the insurance companies are liable to pay Radio Mindanao Network under the insurance policies HELD YES - The Court will not disturb the factual findings of theappellant and trial courts absent compelling reason.Under this mode of review, the jurisdiction of thecourt is limited to reviewing only errors of law. - Particularly in cases of insurance disputes withregard to excepted risks, it is the insurancecompanies which have the burden to prove that theloss comes within the purview of the exception orlimitation set up. It is sufficient for the insured toprove the fact of damage or loss. Once the insuredmakes out a prima facie case in its favor, the duty orburden of evidence shifts to the insurer to controvertsaid prima facie case. Disposition Petition dismissed. Decision of the CA is affirmed.

LEA MER INDUSTRIES v. MALAYAN INSURANCE 471 SCRA 698 PANGANIBAN; September 30, 2005 NATURE Petition for Review FACTS - Ilian Silica Mining entered into a contract ofcarriage with Lea Mer Industries for the shipment of900 metric tons of silica sand valued at P565,000.Consigned to Vulcan Industrial and MiningCorporation, the cargo was to be transported fromPalawan to Manila.The silica sand was placed on board Judy VII, a barge leased by Lea Mer, the vessel sank, resulting in the loss of the cargo. ISSUE WON Lea Mer is liable for the loss of the cargo HELD YES - Common carriers are bound to observe extraordinary diligence in their vigilance over the goods and the safety of the passengers theytransport, as required by the nature of their businessand for reasons of public policy.Extraordinarydiligence requires rendering service with the greatestskill and foresight to avoid damage and destructionto the goods entrusted for carriage and delivery. - Common carriers are presumed to have been atfault or to have acted negligently for loss or damageto the goods that they have transported.Thispresumption can be rebutted only by proof that theyobserved extraordinary diligence, or that the loss ordamage was occasioned by any of the followingcauses: (1)Flood, storm, earthquake, lightning, or other natural disaster or calamity; (2) Act of the public enemy in war, whether international or civil; (3)Act or omission of the shipper or owner of the goods; (4)The character of the goods or defects in the packing or in the containers; (5)Order or act of competent public authority. - To excuse the common carrier fully of any liability,the fortuitous event must have been the proximateand only cause of the loss.It should have exerciseddue diligence to prevent or minimize the loss before,during and after the occurrence of the event. - Petitioner bore the burden of proving that it had exercised extraordinary diligence to avoid the loss, orthat the loss had been occasioned by a fortuitousevent -- an exempting circumstance. - The evidence presented by petitioner in support ofits defense of fortuitous event was sorelyinsufficient.It was not enough for the commoncarrier to show that there was an unforeseen orunexpected occurrence. DispositionPetition is DENIED and the assailed Decision and Resolution are AFFIRMED. Costs against petitioner.

LOADSTAR SHIPPING CO INC v. PIONEER ASIA INSURANCE CORP GR No. 157481 QUISUMBING; January 24, 2006 NATURE Review on certiorari (1) theDecis ion dated October15, 2002 and (2) theResolut ion dated February 27,2003 of CA FACTS - June 6, 1984 - Petitioner Loadstar Shipping Co.,Inc. (LOADSTAR), registered owner and operator ofthe vessel M/V Weasel, entered into a voyage-charter with Northern Mindanao Transport Company,Inc. for the carriage of 65,000 bags of cement fromIligan City to Manila.The shipper was Iligan CementCorporation, while the consignee in Manila wasMarket Developers, Inc. (MARKET) - June 24, 1984 - 67,500 bags of cement wereloaded on board M/V Weasel and stowed in the cargoholds for delivery to the consignee.The shipmentwas covered by petitioners Bill of Lading dated June23, 1984. - Prior to the voyage, the consignee insured theshipment of cement with respondent Pioneer AsiaInsurance Corporation (PIONEER) for P1,400,000, forwhich it issued Marine Open Policy No. MOP-006dated September 17, 1980, covering all shipmentsmade on or after September 30, 1980 - June 25, 1984 - Captain Montera of M/V Weaselordered the vessel to be forced aground whichrendered the entire shipment of cement as good asgone due to exposure to sea water.LOASTAR thusfailed to deliver the goods to MARKET in Manila. - MARKET demanded from LOADSTAR fullreimbursement of the cost of the lost shipment.LOADSTAR refused to reimburse the MARKET despiterepeated demands. - March 11, 1985 PIONEER paid the MARKETP1,400,000 plus an additional amount of P500,000,the value of the lost shipment of cement.In return,the MARKET executed a Loss and SubrogationReceipt in favor of PIONEER concerning the latterssubrogation rights against LOADSTAR. - October 15, 1986 PIONEER filed a complaintagainst LOADSTAR with the RTC Manila allegingthat:(1) the M/V Weasel was not seaworthy at thecommencement of the voyage; (2) the weather andsea conditions then prevailing were usual andexpected for that time of the year and as such, was an ordinary peril of the voyage for which theM/V Weasel should have been normally able to cope with; and (3) LOADSTAR was negligent in the selection andsupervision of its agents and employees thenmanning the M/V Weasel. - LOADSTAR alleged that no fault nor negligencecould be attributed to it because it exercised duediligence to make the ship seaworthy, as well asproperly manned and equipped and failure to deliverwas due to force majeure. - February 15, 1993 - RTC decided in favor ofPIONEER and that LOADSTAR , as a common carrier,bears the burden of proving that it exercisedextraordinary diligence in its vigilance over the goodsit transported.The trial court explained that in caseof loss or destruction of the goods, a statutorypresumption arises that the common carrier wasnegligent unless it could prove that it had observedextraordinary diligence. LOADSTARS defense of force majeure was found bereft of factual basis as a PAG-ASA report that at the time of the incident,tropical storm Asiang had moved away from thePhilippines was presented. - October 15, 2002 CA affirmed RTC Decision with modification ISSUES 1.WON LOADSTAR is a common carrier under Article 1732 CC 2.Assuming it is a common carrier, WON proximatecause of the loss of cargo was not a fortuitous eventbut was allegedly due to the failure of petitioner toexercise extraordinary diligence

HELD1. YES - A1732 CC defines a common carrier as follows: Common carriers are persons, corporations, firmsor associations engaged in the business of carryingor transporting passengers or goods or both, byland, water, or air, for compensation, offering theirservices to the public. - LOADSTAR is a corporation engaged in the businessof transporting cargo by water and for compensation,offering its services indiscriminately to the public.Thus, without doubt, it is a common carrier.Even ifit entered into a voyage-charter agreement withNorthern Mindanao Transport Company, Inc, it didnot in any way convert the common carrier into aprivate carrier. > Planters Products, Inc. v. CA - public carrier shall remain as such, notwithstanding the charter ofthe whole or portion of a vessel by one or morepersons, provided the charter is limited to the shiponly, as in the case of a time-charter or voyage-charter.It is only when the charter includes both thevessel and its crew, as in a bareboat or demise that acommon carrier becomes private, at least insofar asthe particular voyage covering the charter-party isconcerned. 2. YES - As a common carrier, LOADSTAR is required toobserve extraordinary diligence in the vigilance overthe goods it transports. When the goods placed in itscare are lost, LOADSTAR is presumed to have beenat fault or to have acted negligently.LOADSTAR hasthe burden of proving that it observed extraordinarydiligence in order to avoid responsibility for the lostcargo. - Compania Maritima V CA - It requires commoncarriers to render service with the greatest skill andforesight and to use all reasonable means toascertain the na ture and characteristics of goodstendered for shipment, and to exercise due care inthe handling and stowage, including such methods astheir nature requires. - A1734 CC enumerates the instances when a carriermight be exempt from liability for the loss of the goods. (1)Flood, storm, earthquake, lightning, or other natural disaster or calamity; (2)Act of the public enemy in war, whether international or civil; (3)Act or omission of the shipper or owner of the goods; (4)The character of the goods or defects in the packing or in the containers; and (5)Order or act of competent public authority - LOADSTAR claims that the loss of the goods wasdue to a fortuitous event under paragraph 1.Yet, itsclaim is not substantiated.It is supported byevidence that the loss of the entire shipment ofcement was due to the gross negligence of LOADSTAR - Records show that in the evening of June 24, 1984,the sea and weather conditions in the vicinity ofNegros Occidental were calm.The records revealthat LOADSTAR took a shortcut route, instead of theusual route, which exposed the voyage to unexpected hazard.LOADSTAR has only itself to blame for its misjudgment. Dispositionpetition is DENIED

S-ar putea să vă placă și